Вы находитесь на странице: 1из 590

SUR IAS ACEDAMY , GUJARAT

IASbaba Prelims in 60 Days [DAY 1]

Q.1) Consider the following statements with regard to Constitution of India and select the
incorrect statement/s from the codes given below:
1. Constitution authorises the Parliament to alter the areas or boundaries of the existing
states without the consent of concerned state legislature or union territory.
2. Constitution does not guarantee any state of its territorial integrity or continued
existence
3. Constitution declares that laws made for admission or establishment of new states can
be passed by a simple majority
Choose the appropriate code
a) 1 only
b) 2 only
c) 3 only
d) None
Q.1) Solution (d)

Constitution authorises the Parliament to form new states or alter the areas, boundaries or
names of the existing states without the consent of concerned state legislature or union
territory. In other words, Parliament can redraw the political map of India according to its
will. Hence, the territorial integrity or continued existence of any state is not guaranteed by
the Constitution.
Constitution (Article 4) itself declares that laws made for admission or establishment of new
states (under Article 2) and formation of new states and alteration of areas, boundaries or
names of existing states (under Articles 3) are not to be considered as amendments of the
Constitution under Article 368. This means that such laws can be passed by a simple
majority and by the ordinary legislative process.

Source: Refer Chapter 5: Union and its Territory, Indian Polity by Laxmikanth

Q.2) Which among the following are the features of Government of India Act of 1935?

1. It introduced responsible governments in provinces


www.iasbaba.com

Page 1

IASbaba Prelims in 60 Days [DAY 1]

2. It created a new office, Secretary of State for India


3. It provided for the establishment of a Reserve Bank of India
4. It provided for the establishment of Supreme Court, which was set up in 1937
Choose the correct code
a)
b)
c)
d)

1 and 2
1 and 3
1, 2 and 4
1, 3 and 4

Q.2) Solution (b)

GoI Act, 1935 introduced responsible governments in provinces, that is, the governor
was required to act with the advice of ministers responsible to the provincial legislature.
It was GoI Act, 1858 which created a new office, Secretary of State for India, who was
vested with complete authority and control over Indian administration.
GoI Act, 1935 provided for the establishment of a Reserve Bank of India to control the
currency and credit of the country.
GoI Act, 1935 provided for the establishment of Federal Court (not Supreme Court),
which was set up in 1937. It was Regulating Act of 1773 which provided for the
establishment of Supreme Court at Calcutta (1774)

Source: Refer Chapter 1: Historical Background, Indian Polity by Laxmikanth

Q.3) According to Dr B R Ambedkar, which among the following is a novel feature of the Indian
Constitution?
a)
b)
c)
d)

Directive Principles of State Policy


Fundamental Rights
Preamble
Judicial Review

www.iasbaba.com

Page 2

IASbaba Prelims in 60 Days [DAY 1]

Q.3) Solution (a)

According to Dr B R Ambedkar, the Directive Principles of State Policy is a novel feature of


the Indian Constitution.
Dr. Ambedkar said The directive principles are like instruments of instructions which were
issued to the Governor in General and Governors of colonies and to those of India by the
British Government under the 1935 Act under the Draft Constitution. It is proposed to issue
such instructions to the president and governors. The text of these instruments of the
instructions shall be found in scheduled IV to the Constitution of India. What are called
directive principles is that they are instructions to the Legislature and the Executive. Such a
thing is, to my mind, to be welcomed.
The Directive Principles commit the State to promote the welfare of the people by affirming
social, economic and political justice, as well as to fight economic inequality.

Source: Refer Chapter 3: Salient Features of the Constitution, Indian Polity by Laxmikanth

Q.4) In which of the following points is the Indian Constitution similar to that of Canadian
Constitution?
1.
2.
3.
4.

Federation with a strong Centre


Office of Governor
Advisory jurisdiction of the Supreme Court
Vesting of residuary powers in the Centre

Choose the correct codes


a)
b)
c)
d)

1, 2 and 3
2, 3 and 4
1, 3 and 4
All of the above

Q.4) Solution (c)


The following are the features borrowed by Indian Constitution from the Canadian Constitution
www.iasbaba.com

Page 3

IASbaba Prelims in 60 Days [DAY 1]

1) Federation with a strong Centre


2) Vesting of residuary powers in the Centre
3) Appointment of state governors by the Centre (not Office of Governor)
4) Advisory jurisdiction of the Supreme Court
Office of Governor was borrowed from Government of India Act of 1935

Source: Refer Chapter 3: Salient Features of the Constitution, Indian Polity by Laxmikanth

Q.5) Which one of the following Acts of British India established a system of double
government?
a)
b)
c)
d)

Government of India Act of 1919


Indian Councils Act of 1909
Government of India Act of 1935
Pitts India Act of 1784

Q.5) Solution (d)

The Pitts India Act of 1784 distinguished between the commercial and political functions of
the Company.
It allowed the Court of Directors to manage the commercial affairs but created a new body
called Board of Control to manage the political affairs. Thus, it established a system of
double government.
It empowered the Board of Control to supervise and direct all operations of the civil and
military government or revenues of the British possessions in India.

Source: Refer Chapter 1: Historical Background, Indian Polity by Laxmikanth

Q.6) Consider the following statements with regard to States and Union Territories. Which of
the following statement is/are incorrect?

www.iasbaba.com

Page 4

IASbaba Prelims in 60 Days [DAY 1]

1. Union of India is a wider expression than the Territory of India.


2. The states are the members of the federal system and share a distribution of powers
with the Centre.
3. The union territories and the acquired territories, on the other hand, are directly
administered by the Central government.
Choose the correct codes
a)
b)
c)
d)

1 only
3 only
2 and 3 only
None

Q.6) Solution (a)

Territory of India is a wider expression than the Union of India because the latter includes
only states while the former includes not only the states but also union territories and
territories that may be acquired by the Government of India at any future time.
The states are the members of the federal system and share a distribution of powers with
the Centre.
The union territories and the acquired territories, on the other hand, are directly
administered by the Central government.

Source: Refer Chapter 5: Union and its Territory, Indian Polity by Laxmikanth
Q.7) In which case, the Supreme Court held that the Indian Constitution is founded on the
bedrock of the balance between the Fundamental Rights and the Directive Principles?
a)
b)
c)
d)

Kesavananda Bharati case


Berubari case
Golaknath case
Minerva Mills case

www.iasbaba.com

Page 5

IASbaba Prelims in 60 Days [DAY 1]

Q.7) Solution (d)

It was in the Minerva Mills case (1980), the Supreme Court held that the Indian
Constitution is founded on the bedrock of the balance between the Fundamental Rights and
the Directive Principles.

Source: Refer Chapter 3: Salient Features of the Constitution, Indian Polity by Laxmikanth
Q.8) Which Schedule of the Constitution deals with Acts and Regulations of the state
legislatures dealing with land reforms and abolition of the zamindari system?
a)
b)
c)
d)

Schedule Eight
Schedule Nine
Schedule Eleventh
Schedule Twelfth

Q.8) Solution (b)

Ninth Schedule deals with Acts and Regulations of the state legislatures dealing with land
reforms and abolition of the zamindari system and of the Parliament dealing with other
matters.
This schedule was added by the 1st Amendment (1951) to protect the laws included in it
from judicial scrutiny on the ground of violation of fundamental rights.
However, in 2007, the Supreme Court ruled that the laws included in this schedule after
April 24, 1973, are now open to judicial review.

Source: Refer Chapter 3: Salient Features of the Constitution, Indian Polity by Laxmikanth

Q.9) Which one of the following Acts of British India introduced for the first time
bicameralism and direct elections in the country?
a) Indian Council Act of 1892
www.iasbaba.com

Page 6

IASbaba Prelims in 60 Days [DAY 1]

b) Government of India Act of 1919


c) Government of India Act of 1935
d) Indian Independence Act of 1947

Q.9) Solution (b)

Government of India Act of 1919 which is also known as Montagu-Chelmsford Reforms


introduced for the first time bicameralism and direct elections in the country.
Thus, the Indian Legislative Council was replaced by a bicameral legislature consisting of an
Upper House (Council of State) and a Lower House (Legislative Assembly).
The majority of members of both the Houses were chosen by direct election.

Source: Refer Chapter 1: Historical Background, Indian Polity by Laxmikanth

Q.10) Match the following


Type of Writ
1. Habeas Corpus
2. Certiorari
3. Mandamus
4. Quo-Warranto

Meaning
A. we command
B. to be certified
C. you may have the body
D. what is your authority

Select the correct code


a)
b)
c)
d)

1-A, 2-B, 3-C, 4-D


1-C, 2-A, 3-B, 4-D
1-C, 2-B, 3-A, 4-D
1-D, 2-B, 3-C, 4-A

www.iasbaba.com

Page 7

IASbaba Prelims in 60 Days [DAY 1]

Q.10) Solution (c)


Types of Writs
There are five types of writs in the Indian Constitution - Habeas Corpus, Certiorari, QuoWarranto, Mandamus and Prohibition.
1. Writ of Habeas Corpus This writ literally means you may have the body. This writ is issued to produce a person
physically before the court who has been imprisoned or detained by the law and to set him free
if there is no legal justification of his detention.
2. Writ of Certiorari
This writ literally means 'to be certified. This writ is issued by the higher court to the lower
court for quashing the order already passed by the lower court or removing a suit from the
lower court to higher court for speedy disposal.
3. Writ of Quo- Warranto
This writ literally means by what warrants? or what is your authority. It is a writ issued by the
Supreme Court or High Court with a view to restrain a person or authority from holding a public
office to which he is not entitled. The writ requires the concerned person or authority to
explain to the Court by what authority he/it holds the office.
4. Writ of Mandamus
This writ literally means we command. It is a a judicial writ issued by the Supreme Court or a
High Court as a command to an interior court or tribunal or a person to perform a public or
statutory duty.
5. Writ of Prohibition

www.iasbaba.com

Page 8

IASbaba Prelims in 60 Days [DAY 1]

This writ is popularly known as Stay Order. This writ is issued by a higher court to a lower court
to stop proceeding in a case on the ground of over-stepping of jurisdiction or absence of
jurisdiction. It is issued before the judgement or order is made in such cases.
Source: Refer Chapter 3 and Chapter 7: Salient Features of the Constitution and Fundamental
Rights, Indian Polity by Laxmikanth

Q.11) Which one of the following Acts of British India strengthened the Viceroys authority over
his executive council by substituting Portfolio or departmental system for corporate
functioning?
a)
b)
c)
d)

Indian Council Act, 1861


Government of India Act, 1858
Indian Council Act, 1892
Indian Council Act, 1909

Q.11) Solution (a)

Indian Council Act, 1861 empowered the Viceroy to make rules and orders for the more
convenient transaction of business in the council.
It also gave a recognition to the portfolio system, introduced by Lord Canning in 1859.
Under this, a member of the Viceroys council was made in-charge of one or more
departments of the government and was authorised to issue final orders on behalf of the
council on matters of his department(s).

Source: Refer Chapter 1: Historical Background, Indian Polity by Laxmikanth

Q.12) Consider the following statements with regard to the Constitution


1. The Constitution prohibits discrimination against any citizen on grounds of religion, race,
caste, sex or place of birth but not on the ground of residence.

www.iasbaba.com

Page 9

IASbaba Prelims in 60 Days [DAY 1]

2. The Constitution of India has introduced the system of single citizenship and provided
uniform rights for the people of India to promote the feeling of fraternity and unity
among them and to build an integrated Indian nation.
Which among the above statements is/are true?
a)
b)
c)
d)

1 only
2only
Both 1 and 2
Nether 1 nor 2

Q.12) Solution (c)

The Constitution (under Article 15) prohibits discrimination against any citizen on grounds
of religion, race, caste, sex or place of birth and not on the ground of residence. This means
that the state can provide special benefits or give preference to its residents in matters that
do not come within the purview of the rights given by the Constitution to the Indian
citizens. For example, a state may offer concession in fees for education to its residents.
The Constitution of India, like that of Canada, has introduced the system of single
citizenship and provided uniform rights (except in few cases) for the people of India to
promote the feeling of fraternity and unity among them and to build an integrated Indian
nation.
Despite this, India has been witnessing the communal riots, class conflicts, caste wars,
linguistic clashes and ethnic disputes. Thus, the cherished goal of the founding fathers and
the Constitution-makers to build an united and integrated Indian nation has not been fully
realized.

Source: Refer Chapter 6: Citizenship, Indian Polity by Laxmikanth


Q.13) The detailed provisions regarding acquisition and termination of Indian citizenship are
contained in the Citizenship Act which was passed by
a)
b)
c)
d)

The Indian Parliament in 1955


The Indian Parliament in 1950
The British Parliament in August 1948
The Constituent Assembly in 1949

www.iasbaba.com

Page 10

IASbaba Prelims in 60 Days [DAY 1]

Q.13) Solution (a)

The Constitution empowered the Parliament to enact a law to provide for acquisition and
loss of citizenship after the commencement of the Constitution.
Accordingly, the Parliament has enacted the Citizenship Act, 1955, which has been
amended in 1986, 1992, 2003 and 2005.

Source: Refer Chapter 6: Citizenship, Indian Polity by Laxmikanth

Q.14) Consider the statements regarding loss of citizenship:


1. If a person voluntarily renounces Indian citizenship, every minor child of that person
also loses his citizenship.
2. When a citizen voluntarily acquires the citizenship of another country, his Indian
citizenship automatically terminates. This provision does not apply during a war in which
India is engaged.
3. The citizenship is terminated if the citizen has unlawfully traded or communicated with
the enemy during a war.
Which of the above statements are correct?
a)
b)
c)
d)

1 and 2
2 and 3
1 and 3
All of the above

Q.14) Solution (d)


If a person voluntarily renounces Indian citizenship, every minor child of that person also loses
his citizenship. However, when such a child attains the age of eighteen, he may resume Indian
citizenship.
www.iasbaba.com

Page 11

IASbaba Prelims in 60 Days [DAY 1]

Refer Laxmikanth Loss of citizenship

Q.15) Consider the following statements regarding Constitutional provisions of Citizenship:


1. The Part II of the Indian Constitution deals with Citizenship.
2. Detailed provisions of acquisition and loss of citizenship are mentioned in the
Constitution.
3. Any law made regarding citizenship is considered as a Constitutional Amendment.
Which of the above statements are incorrect?
a)
b)
c)
d)

1 only
2 and 3
3 only
None of the above

Q.15) Solution (b)


The constitution neither contains any permanent nor elaborate provisions regarding citizenship.
It empowers the parliament to enact a law to deal with the problem of acquisition or loss of
citizenship. Laws made by the parliament regarding citizenship are not considered as
Constitutional amendment.

Q.16) Which of the statements regarding Overseas Citizen of India cardholder are correct?
1. They get a multiple entry, multipurpose lifelong visa for visiting India.
2. They get parity with NRIs in respect of economic, financial and education fields except
acquisition of agricultural lands.

www.iasbaba.com

Page 12

IASbaba Prelims in 60 Days [DAY 1]

3. They are required to register with local police authorities once their stay in India
exceeds 180 days for the first time.
Select the correct code from the following:
a)
b)
c)
d)

1 and 2
2 and 3
1 and 3
All of the above

Q.16) Solution (a)


OCIs dont need to get registered with local police station.

Q.17) Consider the following statements with respect to Asiatic Lion


1. The Gir Forest, in Gujarat, is the last natural range wild Asiatic lions.
2. It is critically endangered as per IUCN red list

Select the correct answer


a)
b)
c)
d)

1 only
2 only
Both
None

Q.17) Solution (a)

It is endangered as per IUCN red list


Click here

Q.18) India successfully test-fires advanced interceptor missile from Abdul kalam Island.
www.iasbaba.com

Page 13

IASbaba Prelims in 60 Days [DAY 1]

Select the correct option with respect to Abdul kalam Island.


a)
b)
c)
d)

It is the new name of Sri Hari Kota located in Andhra Pradesh.


It is located in French Guinea
It is an island located off Odisha coast.
It is located in Andaman and Nicobar islands.

Q.18) Solution (c)

Click here
Abdul Kalam Island, formerly known as Wheeler Island, is an island off the coast of
Odisha, India,
approximately 150
kilometres (93 mi) from the
state
capital Bhubaneswar. The Integrated Test Range maintaining a missile testing facility is
located in the island.

Q.19) Consider the following statements with respect to Porpoises


1.
2.
3.
4.

Porpoises are group of Marine amphibians


Vaquita marina is the worlds smallest porpoise
They are critically endangered
They are endemic to Gulf of California and Mexico

Select the correct option


a)
b)
c)
d)

1, 2 and 3 only
2, 3 and 4 only
1, 3 and 4 only
1,2,3 and 4 only

Q.19) Solution (b)

Porpoises are a group of fully aquatic marine mammals.


Click here

www.iasbaba.com

Page 14

IASbaba Prelims in 60 Days [DAY 1]

Q.20) Chinnar Wildlife Sanctuary accounted for a large number of fatalities over the past six
months. Chinnar Wild life sanctuary is located in which of the following state/UT of India
a)
b)
c)
d)

Karnataka
Tamil Nadu
Puducherry
Kerala

Q.20) Solution (d)

Click here

Q.21) Consider the following statements about the Fiscal Policy.


1. Fiscal policy is the use of government spending and taxation to influence the economy
2. The most immediate effect of fiscal policy is to change the aggregate demand for goods
and services.
3. Fiscal policy is said to be loose or expansionary when revenue is higher than spending.

Which of the above statements are correct?


a)
b)
c)
d)

1 and 2 only
2 and 3 only
1 and 3 only
All

Q.21) Solution (a)

Fiscal policy is said to be tight or contractionary when revenue is higher than spending
(i.e., the government budget is in surplus).
Click here

www.iasbaba.com

Page 15

IASbaba Prelims in 60 Days [DAY 1]

Q.22) The High Court of Rajasthan has declared the practice of Santhara as illegal. Which of
the statements correctly explains Santhara?
a)
b)
c)
d)

It is a ritual of sacrificing Bulls in front of Goddess Kali in local annual festival.


It is a Jain custom of voluntary death by performing fast until death.
It is the name given to the performance of Child marriage in Rajasthan.
It is a practice of honour killing of a couple if a girl and a boy of same village gets
married to each other.

Q.22) Solution (b)


Santhara is a Jain custom of embracing voluntary death it involves practitioners taking an oath
to stop eating until they die of starvation. According to Jains, this is a way to purge oneself of
bad karma and attain moksha.
Every year a question comes from indigenous ancient religions. Since Santhara was in news it
becomes important. A related question on Jainism can be asked like- which famous mauryan
ruler is believed to have performed Santhara. (Chandragupta Maurya)

Q.23) Consider the following statements regarding the scheme Sakhi:


1. The scheme is meant to help women suffering from sexual, mental or emotional
harassments.
2. It is an initiative of Union Government under the Nirbhaya Fund for women safety.
3. Women will be provided vocational training to make them feel confident and earn
livelihood as a part of Skill India initiative.
Which of the above statements are correct?
a)
b)
c)
d)

1 only
1 and 2
3 only
All of the above

www.iasbaba.com

Page 16

IASbaba Prelims in 60 Days [DAY 1]

Q.23) Solution (b)


A scheme called Sakhi, was inaugurated to help the women, who are suffering from sexual,
mental and emotional harassments(Including eve-teasing, domestic violence and rape). It is an
initiative of Union Government under the Nirbhaya Fund for women safety pertaining to
strategic areas of prevention, protection and rehabilitation. The Objective of this scheme is to
provide necessary help to women victims and others, who are suffering from any kind of
harassment.

Q.24) The National Aeronautics and Space Administration (NASA) and the US Agency for
International Development (USAID) have launched a new joint project named SERVIR-Mekong.
Which of the following statements regarding this project are correct?
1. The SERVIR program helps governments and development stakeholders in incorporating
Earth observations and geospatial technologies into natural disaster response to
improve food security, safeguard human health and manage water and natural
resources.
2. It is recently launched for five South Asian countries namely Burma, Cambodia, Laos,
Thailand and Vietnam.
Which of the above statements are correct?
a)
b)
c)
d)

1 only
2 only
Both 1 and 2
Neither 1 nor 2

Q.24) Solution (a)


The National Aeronautics and Space Administration (NASA) and the US Agency for International
Development (USAID) have launched a new joint project named SERVIR-Mekong to strengthen
regional environmental monitoring in five countries of Southeast Asia namely Burma,
Cambodia, Laos, Thailand and Vietnam.

www.iasbaba.com

Page 17

IASbaba Prelims in 60 Days [DAY 1]

It was launched in the Asian Disaster Preparedness Center in Bangkok, Thailand. Under this
project, researchers will draw from a continuous stream of space-based climate, weather and
other Earth observation data from Nasa and its partners. The main focus of this project is to
host vital issues in the Mekong river basin region in those five countries which is also called as
Rice Bowl of Asia.

Q.25) Consider the following statements:


1. Bank of Baroda has introduced Maharaja Sayyaji Rao Gaekwad award.
2. The award will be conferred to such personality every year which has made significant
contribution in creating awareness and spreading Marathi language amongst people.
Which of the above statements are incorrect?
a)
b)
c)
d)

1 only
2 only
Both 1 and 2
Neither 1 nor 2

Q.25) Solution (b)


Bank of Baroda has introduced Maharaja Sayajirao Bhasha Samman an award in the
memory of its founder Maharaja Sir Sayajirao Gaekwad III.
Sayajirao Gaekwad III the erstwhile ruler of Baroda state had played a vital role in economic
growth of the country and propagating the use of Hindi.
"The award will be conferred upon such a personality every year who has made significant
contribution for creating awareness and spreading Hindi among public at large,"

Q.26) Consider the following statements regarding Sugamya Bharat Abhiyan:


1. The campaign has been launched by Ministry of Social Justice and Empowerment.
www.iasbaba.com

Page 18

IASbaba Prelims in 60 Days [DAY 1]

2. The campaign focuses on making public buildings accessible to Persons with Disabilities
(PWDs).
3. The campaign also focuses on enhancing the accessibility and usability of Public
Transport, and information & communication.
Which of the above statements are correct?
a)
b)
c)
d)

1 and 2
2 and 3
1 and 3
All of the above

Q.26) Solution (d)


The Ministry of Social Justice and Empowerment launched the Accessible India Campaign
(Sugamya Bharat Abhiyan) that focuses on building accessible government buildings for persons
with disabilities (PwDs).
The campaign was initiated by the government to increase the accessibility of the physical
environment in the country. In addition, the campaign focuses on enhancing the accessibility
and usability of Public Transport, and information & communication.

www.iasbaba.com

Page 19

IASbaba '60 Day Plan' - Prelims Test 2016 [Day 2]

Q.1) Article 13 declares that all laws that are inconsistent with or in derogation of any of the
fundamental rights shall be void. Which among the following laws can be challenged in
courts as violating FR?
1. Ordinances issued by the president or the state governors.
2. Delegated legislations.
3. Constitutional amendments.
Choose the correct codes
a)
b)
c)
d)

1 and 2 only
1 and 3 only
3 only
All the above

Q.1) Solution (d)


The term law in Article 13 has been given a wide connotation so as to include the
following:
(a) Permanent laws enacted by the Parliament or the state legislatures;
(b) Temporary laws like ordinances issued by the president or the state governors;
(c) Statutory instruments in the nature of delegated legislation (executive legislation) like
order, bye-law, rule, regulation or notification; and
(d) Non-legislative sources of law, that is, custom or usage having the force of law.
Further, Article 13 declares that a constitutional amendment is not a law and hence cannot
be challenged. However, the Supreme Court held in the Kesavananda Bharati case2 (1973)
that a Constitutional amendment can be challenged on the ground that it violates a
fundamental right that forms a part of the basic structure of the Constitution and hence,
can be declared as void.

Q.2) The term State has been used in different provisions concerning the fundamental
rights. According to Article 12, the State includes which among the following entities
1.
2.
3.
4.

Executive organs of the Union and state governments.


Legislative organs of the Union and state governments.
Judiciary at Union and state levels.
Local Self-governments created under 73rd and 74th amendments.

www.iasbaba.com

Page 1

IASbaba '60 Day Plan' - Prelims Test 2016 [Day 2]


5. Statutory authorities.
6. Non-statutory authorities.
Choose the correct codes
a)
b)
c)
d)

1, 2, 3, 4 and 5 only
1, 2, 4, 5 and 6 only
1, 2, 3 and 4 only
All the above

Q. 2) Solution (b)
According to Article 12, the State includes the following:
(a) Government and Parliament of India, that is, executive and legislative organs of the
Union government.
(b) Government and legislature of states, that is, executive and legislative organs of state
government.
(c) All local authorities that is, municipalities, panchayats, district boards, improvement
trusts, etc.
(d) All other authorities, that is, statutory or non-statutory authorities like LIC, ONGC, SAIL,
etc.
Thus, State has been defined in a wider sense so as to include all its agencies. It is the
actions of these agencies that can be challenged in the courts as violating the Fundamental
Rights. According to the Supreme Court, even a private body or an agency working as an
instrument of the State falls within the meaning of the State under Article 12.

Q.3) Which among the following FR does a foreign citizen can avail
1. Freedom to manage religious affairs.
2. Freedom of conscience and free profession, practice and propagation of religion.
3. Freedom from attending religious instruction or worship in certain educational
institutions.
4. Right to elementary education.
5. Protection of language, script and culture of minorities.
6. Right of minorities to establish and administer educational institutions.

www.iasbaba.com

Page 2

IASbaba '60 Day Plan' - Prelims Test 2016 [Day 2]


Choose the correct codes
a)
b)
c)
d)

1, 2, 3, 4 and 5 only
1, 2, 3, 5 and 6 only
1, 2, 3 and 4 only
All the above

Q.3) Solution (c)


FR available to both citizens and foreigners (except enemy aliens):
1. Equality before law and equal protection of Laws (Article 14).
2. Protection in respect of conviction for offences (Article 20).
3. Protection of life and personal liberty (Article 21).
4. Right to elementary education (Article 21A).
5. Protection against arrest and detention in certain cases (Article 22).
6. Prohibition of traffic in human beings and forced labour (Article 23).
7. Prohibition of employment of children in factories etc., (Article 24).
8. Freedom of conscience and free profession, practice and propagation of religion (Article
25).
9. Freedom to manage religious affairs (Article 26).
10. Freedom from payment of taxes for promotion of any religion (Article 27).
11. Freedom from attending religious instruction or worship in certain educational
institutions (Article 28).

Q.4) The concept "Equality before law" connotes which among the following
1. The absence of any special privileges in favour of any person.
2. The equality of treatment under equal circumstances.
3. The equal subjection of all persons to the ordinary law of the land.
Choose the correct codes
a) 1 and 2 only
www.iasbaba.com

Page 3

IASbaba '60 Day Plan' - Prelims Test 2016 [Day 2]


b) 1 and 3 only
c) 2 and 3 only
d) All the above

Q.4) Solution (b)


The concept "Equality before law" connotes:
(a) The absence of any special privileges in favour of any person,
(b) The equal subjection of all persons to the ordinary law of the land administered by
ordinary law courts, and
(c) No person (whether rich or poor, high or low, official or non-official) is above the law.
On the other hand, "Equal protection of laws" connotes:
(a) The equality of treatment under equal circumstances, both in the privileges conferred
and liabilities imposed by the laws,
(b) The similar application of the same laws to all persons who are similarly situated, and
(c) The like should be treated alike without any discrimination.

Q.5) Consider the following statements regarding Fundamental Duties enshrined in the
constitution of India
1. These are applicable to all the residents of India irrespective of their caste, creed, sex
and religion.
2. The parliament cannot impose legal sanction against the violation of Fundamental
duties.
Choose the correct codes
a)
b)
c)
d)

1 only
2 only
Both 1 and 2
Neither 1 nor 2

www.iasbaba.com

Page 4

IASbaba '60 Day Plan' - Prelims Test 2016 [Day 2]

Q.5) Solution (d)


Unlike some of the Fundamental Rights which extend to all persons whether citizens or
foreigners1, the Fundamental Duties are confined to citizens only and do not extend to
foreigners. So, statement 1 is wrong as residents of India may include foreign citizens also.
Like the Directive Principles, the fundamental duties are also non-justiciable. The
Constitution does not provide for their direct enforcement by the courts. Moreover, there is
not legal sanction against their violation. However, the Parliament is free to enforce them
by suitable legislation. So, statement 2 is also wrong.

Q.6) Which of the following statements can be considered as significance of Fundamental


Duties?
1. They help the courts in examining and determining the constitutional validity of a
law.
2. They are enforceable by law.
Choose the correct codes
a)
b)
c)
d)

1 only
2 only
Both 1 and 2
Neither 1 nor 2

Q.6) Solution (c)


They help the courts in examining and determining the constitutional validity of a law. In
1992, the Supreme Court ruled that in determining the constitutionality of any law, if a
court finds that the law in question seeks to give effect to a fundamental duty, it may
consider such law to be reasonable in relation to Article 14 (equality before law) or Article
19 (six freedoms) and thus save such law from unconstitutionality. Hence, statement 1 is
correct.
They are enforceable by law. Hence, the Parliament can provide for the imposition of
appropriate penalty or punishment for failure to fulfil any of them. So, statement 2 is also
correct.
www.iasbaba.com

Page 5

IASbaba '60 Day Plan' - Prelims Test 2016 [Day 2]

Q.7) Which among the following legal provisions are meant for the implementation of some
of the Fundamental Duties enshrined in the constitution of India?
1. Unlawful activities (Prevention) Act.
2. Representation of People Act (1951).
3. The Undisclosed Foreign Income and Assets (Imposition of Tax) Bill, 2015.
Choose the correct codes
a)
b)
c)
d)

1 and 2 only
1 and 3 only
2 and 3 only
All the above

Q.7) Solution (a)


The Verma Committee on Fundamental Duties of the Citizens (1999) identified the existence
of legal provisions for the implementation of some of the Fundamental Duties.
The Unlawful Activities (Prevention) Act of 1967 provides for the declaration of a communal
organisation as an unlawful association. Hence, statement 1 is correct.
The Representation of People Act (1951) provides for the disqualification of members of the
Parliament or a state legislature for indulging in corrupt practice that is, soliciting votes on
the ground of religion or promoting enmity between different sections of people on grounds
of caste, race, language, religion and so on. So, statement 2 is also correct.
Duty to pay taxes is not considered as Fundamental Duty. Hence, statement 3 is wrong.

Q.8) Which among the following is called as "Conscience of the constitution" by Granville
Austin?
1. Fundamental Rights.
2. Fundamental Duties.
3. Directive Principles of State Policy
Choose the correct codes

www.iasbaba.com

Page 6

IASbaba '60 Day Plan' - Prelims Test 2016 [Day 2]


a)
b)
c)
d)

1 and 2 only
1 only
1 and 3 only
All the above

Q.8) Solution (c)


The Directive Principles along with the Fundamental Rights contain the philosophy of the
Constitution and is the soul of the Constitution. Granville Austin has described the Directive
Principles and the Fundamental Rights as the Conscience of the Constitution

Q.9) DPSP are like the instrument of instructions. These instructions or recommendations to
the State in which among the following matters
1.
2.
3.
4.
5.

Legislative organs of the central and state governments.


Executive organs of the central and state governments.
Judiciary at Union and state levels.
All local authorities of the country.
All public authorities of the country.

Choose the correct codes


a)
b)
c)
d)

1, 2, 3 and 4 only
1, 2, 4 and 5 only
1, 2 and 3 only
All the above

Q. 9) Solution (b)
Directive Principles of State Policy are the constitutional instructions or recommendations
to the State in legislative, executive and administrative matters. According to Article 36, the
term State in Part IV has the same meaning as in Part III dealing with Fundamental Rights.
Therefore, it includes the legislative and executive organs of the central and state
governments, all local authorities and all other public authorities in the country.

www.iasbaba.com

Page 7

IASbaba '60 Day Plan' - Prelims Test 2016 [Day 2]

Q.10) In Minerva mills case (1980), according to Supreme Court which among the following
constitutes the core of commitment to social revolution
1. Fundamental Rights.
2. Fundamental Duties.
3. Directive Principles of State Policy
Choose the correct codes
a)
b)
c)
d)

1 and 2 only
1 only
1 and 3 only
All the above

Q.10) Solution (c)


In the Minerva Mills case (1980), the Supreme Court also held that the Indian Constitution is
founded on the bedrock of the balance between the Fundamental Rights and the Directive
Principles. They together constitute the core of commitment to social revolution.

Q.11) Consider the following statements regarding Quo-Warranto:


1. It prevents illegal usurpation of public office by a person.
2. It cannot be issued in case of Ministerial office or private office.
3. This can be sought by any interested person and not necessarily by the aggrieved
person.
Which of the above statements are correct?
a)
b)
c)
d)

1 and 2
1 and 3
2 and 3
All of the above

Q.11) Solution (d)


Refer Laxmikanth Fundamental Rights, Quo-Waranto.

www.iasbaba.com

Page 8

IASbaba '60 Day Plan' - Prelims Test 2016 [Day 2]


Q.12) Article 34 of the Indian Constitution provides for the restriction on fundamental rights
while Martial Law is in force in any area within the territory of India. Which of the following
statements regarding Martial Law are correct?
1. The concept of Martial law has been borrowed from the English Common Law.
2. It refers to a situation where civil administration is run by military authorities
according to their own rules and regulations.
3. The expression Martial Law has not been defined anywhere in the constitution.
Select the correct code from the following:
a)
b)
c)
d)

1 and 2
2 and 3
1 and 3
All of the above

Q.12) Solution (d)


All the statements are correct. During the operation of martial law, the military authorities
are vested with abnormal powers to take all necessary. They impose restrictions and
regulations on the rights of the civilians, can punish the civilians and can condemn them to
death.

Q.13) Which of the following Fundamental Rights are available only to citizens and not to
foreigners?
1.
2.
3.
4.

Prohibition of discrimination on grounds of religion, race, caste, sex or place of birth.


Equal opportunity in matters of public employment.
Freedom of speech and expression.
Protection of language, script and culture of minorities.

Select the correct code from the following:


a)
b)
c)
d)

4 only
2,3 and 4
1,2 and 3
All of the above

Q.13) Solution (d)


FRs available only to citizens and not to foreigners are:
www.iasbaba.com

Page 9

IASbaba '60 Day Plan' - Prelims Test 2016 [Day 2]


Article 15- prohibition of discrimination on the basis of religion, race, caste, sex or place of
birth.
Article 16- Equality of opportunity in matters of public employment.
Article 19- Six freedoms.
Article 29- Protection of language, script and culture of minorities.
Article 30- Right of minorities to establish and administer educational institutions.

Q.14) Consider the following statements regarding Uniform Civil Code:


1. It is a Directive Principle of State Policy given under article 45 of Indian Constitution.
2. It is a Gandhian directive principle.
3. Uniform Civil Code refers to the body of laws governing rights and duties pertaining
to property and personal matters like marriage, divorce, adoption and inheritance.
Which of the above statements are correct?
a)
b)
c)
d)

All of the above


2 and 3
1 and 2
3 only

Q.14) Solution (d)


It is a DPSP given in article-44
It is a Liberal- Intellectual principle.

Q.15) Which of the following statements are a correct comparison between Fundamental
Rights(FR) and Directive Principle of State Policy (DPSPs):
1. FRs are justiciable while DPSPs are not.
2. FRs are negative as they prohibit state from doing certain things while DPSPs are
positive as they require state to do certain things.
3. In a conflict between DPSPs and FRs, FRs always prevails.
Select the correct code from the following:
www.iasbaba.com

Page 10

IASbaba '60 Day Plan' - Prelims Test 2016 [Day 2]


a)
b)
c)
d)

1 and 2
2 and 3
1 and 3
All of the above

Q.15) Solution (a)


Directive Principles given in Article 39 (b) and (c) have been given superiority over Article 14
and Article 19.

Q.16) Consider the following regarding the Fundamental Duties:


1. They were added as Part IVA of the Constitution by 44th Amendment Act.
2. Initially there were only 10 Fundamental Duties.
3. 86th Constitution Amendment Act 2002, added the duty to provide education to
ones child or ward between the age of 6 and 14 years.
Which of the above statements are incorrect?
a)
b)
c)
d)

2 and 3
1 only
1 and 2
None of the above

Q.16) Solution (b)


Fundamental Duties were added by 42nd Constitutional Amendment Act.

Q.17) Consider the following


1.
2.
3.
4.
5.

AmritMahal
Khillari
Sahiwal
Hallikar
Gir

Which of the following are Indigenous drought breed cattles?


a) 1, 2, and 3 only
www.iasbaba.com

Page 11

IASbaba '60 Day Plan' - Prelims Test 2016 [Day 2]


b) 3 and 5 only
c) 1, 2 and 4 only
d) None of the above

Q.17) Solution (c)

AmritMahal, Khillari, Hallikar, Kangayam,Bargur,Umblacherry, Pullikulam, are some


of the indigenous drought breeds.
Gir, Sahiwal, Red sindhi, Deoni are some of the indigenous dairy breeds.
Tharparkar,Hariana, Kankrej,Ongole, Krishna valley are some of the dual purpose
breeds.
Jersey, Holstein Friesian,Brown swiss, Red dane, Ayrshire, Guernsey.
Murrah, Surthi, jaffarbandi,Bhadawari,Nili ravi, Mehsana, Nagpuri,Toda.

Q.18) Consider the following


1. Udaan is a Special Industry initiative that aims to provide an exposure to the
unemployed graduates to the best of Corporate India.
2. It is implemented by Ministry of skill development and Entrepreneurship.

Select the correct answer with respect to UDAAN scheme


a)
b)
c)
d)

1 only
2 only
Both
None

Q.18) Solution (c)

Click here
It is advised to the candidates that there is a similar scheme UDAAN by MHRD,
UDAAN by MHRD is an initiative of the Central Board of Secondary Education (CBSE)
to enable disadvantaged girl students and other students from SC/ST & minorities to
transit from school to post-school professional education specially in Science and
Math.

www.iasbaba.com

Page 12

IASbaba '60 Day Plan' - Prelims Test 2016 [Day 2]


Q.19) Consider the following
1. Lebanon
2. Croatia
3. Cyprus

Which of the following countries are surrounded by Mediterranean Sea?


a)
b)
c)
d)

1 only
1 and 3 only
2 and 3 only
2 only

Q.19) Solution (b)

Croatia is surrounded by Adriatic sea

Q.20) Consider the following statements with respect to Green bonds


1. Green bond is a debt instrument issued by an entity for raising funds from investors
2. The funds generated are used to fund both green and non-green projects
3. In India IDBI,EXIM, Yes bank have started issuing green bonds

Select the correct answer using the code given below


a)
b)
c)
d)

1 and 3 only
2 and 3 only
All the above
None of the above

Q.20) Solution (a)

Fund generated from bonds are used only to fund green projects
Click here to know more about Green Bonds

Q.21) Consider the following statements with respect to Ethanol blending.


1. Ethanol is derived only from Sugarcane.

www.iasbaba.com

Page 13

IASbaba '60 Day Plan' - Prelims Test 2016 [Day 2]


2. Recently Central Government has scaled up blending targets from 5% to 10% to
promote blending of ethanol with petrol and its use as alternative fuel
3. It reduces the carbon monoxide emissions by up to 30 per cent.
4. Normal cars can be used up 10% ethanol without any up gradation

Select the incorrect statement/s from the above


a)
b)
c)
d)

1 only
1 and 2 only
3 and 4 only
2 ,3 and 4 only

Q.21) Solution (a)

Ethanol is a renewable, domestically produced alcohol fuel made from plant


material, such as corn, sugar cane, or grasses. Using ethanol can reduce oil
dependence and greenhouse gas emissions
Motor vehicles can use ethanol up to 10 % without any up gradation. It also reduces
the carbon monoxide emissions by up to 30 percent
Click here

Q.22) An issue of updating the National Citizen Register (NCR) in Assam was in news.
Consider the following statements w.r.t NCR
1. The provisions governing NCR updation in Assam are Citizenship Act, 1951 and The
Citizenship (Registration of Citizens and Issue of National Identity cards) Rules, 2003
2. Policy decisions, guidelines, funds for NRC updation and implementation are
provided by and are done through the State Government machinery.
3. Persons whose names appear in NCR, 1955 are eligible for inclusion in NCR
Select the correct code
a)
b)
c)
d)

1 and 2
2 and 3
1, 2 and 3
None

Q.22) Solution (d)


www.iasbaba.com

Page 14

IASbaba '60 Day Plan' - Prelims Test 2016 [Day 2]


The provisions governing NCR updation in Assam are Citizenship Act, 1955 and The
Citizenship (Registration of Citizens and Issue of National Identity cards) Rules, 2003
Policy decisions, guidelines and funds for NRC updation are provided by the
Central Government of India, but its implementation is done through the State Government
machinery under the Registrar General of India
Eligibility criterion for inclusion in NCR

Persons whose names appear in NRC, 1951


Persons whose names appear in any of the Electoral Rolls up to 24 March (midnight),
1971
Descendants of the above-mentioned persons
Persons who came to Assam on or after 1 January 1966 but before 25 March 1971
and registered themselves in accordance to the rules made by the Central
Government with the Foreigners Registration Regional Officer (FRRO) and those who
have not been declared as illegal migrants or foreigners by the authority
Persons who are original inhabitants of Assam and their children & descendants,
who are citizens of India (provided the citizenship of such persons is ascertained)
D voters can apply for inclusion of their names in the updated NRC. However, their
names will. Be finally included only when the appropriate Foreigner Tribunal
declares them as non-foreigners.
All Indian Citizens including their children and descendants who have moved to
Assam post 24 March 1971 would be eligible for inclusion in the updated NRC on
adducing satisfactory proof of residence in any part of the country (outside Assam)
as on 24 March 1971.
Persons who can provide any one of the documents issued up to midnight of 24
March 1971 as mentioned in the list of documents admissible for citizenship.

Read these articles


http://indianexpress.com/article/india/india-others/national-register-of-citizens-67-lakhforms-filed-assam-gears-up-to-verify-citizenship-claims/
http://www.thehindu.com/opinion/lead/the-starkness-of-beingnowhere/article7633845.ece

Q.23) The Union Cabinet chaired by the Prime Minister has given its approval to the
National Offshore Wind Energy Policy (OWEP). Consider the following w.r.t Indias Wind
Energy Potential and policies
1. Indias onshore wind energy potential is more than offshore
www.iasbaba.com

Page 15

IASbaba '60 Day Plan' - Prelims Test 2016 [Day 2]


2. National Institute of Wind Energy (NIWE) has been authorized as the Nodal Agency
for development of offshore wind energy in the country
3. Offshore Wind Energy Policy allows wind energy development, research and
development activities upto seaward distance of territorial waters i.e 12 nautical
miles
Select the correct statement/s
a)
b)
c)
d)

Only 1
1 and 2
1, 2 and 3
2 and 3

Q.23) Solution (b)

The Union cabinet approved the National Offshore Wind Energy Policy, aiming to harness
wind power along Indias 7,600 km coastline. With this approval, the Ministry of New &
Renewable Energy (MNRE) has been authorized as the Nodal Ministry for use of offshore
areas within the Exclusive Economic Zone (EEZ) of the country and the National Institute of
Wind Energy (NIWE) has been authorized as the Nodal Agency for development of offshore
wind energy in the country and to carry out allocation of offshore wind energy blocks,
coordination and allied functions with related ministries and agencies.
The decision paves the way for offshore wind energy development, including projects and
research and development activities up to a seaward distance of 200 nautical miles, which
constitutes the exclusive economic zone (EEZ) of the country.
Of Indias 36,642 MW of installed renewable power capacity, around 23,864 MW is onshore
wind energy alone.
http://www.livemint.com/Politics/hjYjnG8v4SWLIPKr5yo2FJ/Cabinet-approves-nationalwind-energy-policy.html

Q.24) Consider the following statements


1. It is also referred as dancing deer
2. Its habitat is located along the largest fresh water lake in eastern India.
3. The dancing deer is found in its natural habitat over the floating biomass locally
called "phumdi"

www.iasbaba.com

Page 16

IASbaba '60 Day Plan' - Prelims Test 2016 [Day 2]


Identify the species based on given statements
a)
b)
c)
d)

Reindeer
Goral
Himalayan Tahr
Sangai

Q.24) Solution (d)


http://timesofindia.indiatimes.com/home/environment/flora-fauna/Wildlife-Institute-ofIndia-to-relocate-endangered-dancing-deer-of-Manipur/articleshow/52282276.cms

Q.25) Consider the following w.r.t Inactivated Polio Vaccine (IPV)


1. The inactivated polio vaccine produces antigens in the blood to all three types of
poliovirus.
2. IPV is not a 'live' vaccine
3. It is also called the Salk vaccine
Select the incorrect statement/s
a)
b)
c)
d)

2 and 3
Only 1
Only 3
None

Q.25) Solution (b)


Inactivated polio vaccine (IPV) was developed in 1955 by Dr Jonas Salk. Also called the Salk
vaccine, IPV consists of inactivated (killed) poliovirus strains of all three poliovirus types.
IPV is given by intramuscular injection and needs to be administered by a trained health
worker
The inactivated polio vaccine produces antibodies in the blood to all three types of
poliovirus. In the event of infection, these antibodies prevent the spread of the virus to the
central nervous system and protect against paralysis.
Advantages
As IPV is not a 'live' vaccine, it carries no risk of vaccine-associated polio paralysis.
IPV triggers an excellent protective immune response in most people.
www.iasbaba.com

Page 17

IASbaba '60 Day Plan' - Prelims Test 2016 [Day 2]


Disadvantages
IPV induces very low levels of immunity in the intestine. As a result, when a person
immunized with IPV is infected with wild poliovirus, the virus can still multiply inside the
intestines and be shed in the faeces, risking continued circulation.
IPV is over five times more expensive than oral polio vaccine.
Administering the vaccine requires trained health workers and sterile injection equipment
and procedures.

www.iasbaba.com

Page 18

IASbaba '60 Day Plan' - Prelims Test 2016 [Day 3]

Q.1) Which of the following statements correctly explains a Cordillera?


a)
b)
c)
d)

It is a massive mountain range with two or more parallel mountain ranges.


It is a huge volcanic depression formed due to collapse of a volcano crater.
It is lake formed in mountain valleys.
It is lake formed in depressions of rocky deserts.

Q.1) Solution (a)


A Cordillera is a system of mountain ranges often consisting of a number of more or less
parallel chains. Eg. Rockies of North America, Himalayas of India etc.

Q.2) Consider the following statements about the Aravallis?


1.
2.
3.
4.

They are the oldest mountain ranges of India.


Aravalli is an example of block mountains.
Guru Shikhar is the highest peak of Aravalli range.
Aravalli hills lie perpendicular to the direction of monsoon winds causing heavy rainfall
in eastern Rajasthan.

Which of the above statements are correct?


a)
b)
c)
d)

1 and 3
2 and 4
1,2 and 3
All of the above

Q.2) Solution (a)


Aravalli range is an old folded mountain range. They lie parallel to the monsoon winds causing
no interception hence very little rainfall.

www.iasbaba.com

Page 1

IASbaba '60 Day Plan' - Prelims Test 2016 [Day 3]


Q.3) Consider the following statements:
1. When an air parcel is pushed up, first dry adiabatic cooling take place and then wet
adiabatic cooling take place.
2. During dry adiabatic cooling, air contains no water vapour.
3. The rate of dry adiabatic cooling is constant across the world, while rate of wet adiabatic
cooling constantly changes.
Which of the above statements are correct?
a)
b)
c)
d)

1 and 2
2 and 3
1 and 3
All of the above

Q.3) Solution (c)


During dry adiabatic cooling air contains water vapour and during wet adiabatic cooling air
contains condensed water.
Dry adiabatic cooling is a physical phenomenon which occurs due to change in density as the air
parcel moves up. The rate is constant across the world. On contrary, during wet adiabatic
cooling, water starts condensing. Due to condensation, latent heat is released which reduces
the net rate of cooling. More the heat is released, slower the rate of cooling becomes. Thats
why it is not constant.

Q.4) Consider the following statements regarding Bhabhar plains:


1. It is the northern most tracts of Indian plains lying at the foothills of Siwaliks.
2. It contains fine alluvium deposited by the Himalayan streams.
3. It is very fertile and suitable for cultivation of crops.
Which of the above statements are incorrect?
a)
b)
c)
d)

All of the above


1 only
2 and 3
None of the above

www.iasbaba.com

Page 2

IASbaba '60 Day Plan' - Prelims Test 2016 [Day 3]

Q.4) Solution (c)


Bhabar region lies along the foot of the Siwaliks from the Indus to the Tista. It comprises of
pebble-studded rocks in the shape of porous beds. It is due to porosity of the rocks; the
streams disappear and flow underground. The underground streams of the Bhabar re-emerge
on the surface and give birth to marshy area. The region is not much suitable for agriculture.

Q.5) India and its neighboring countries are generally referred to as the Indian Subcontinent.
The reason behind this is:
a)
b)
c)
d)

Political dominance of India


Regions isolation from rest of the world
Vast size of the country
Typical trade links with the rest of the world

Q.5) Solution (b)


The region is isolated from rest of the world by Himalayan range in the north and Indian ocean
in the south. This isolation has evolved a unique culture of Indian subcontinent.

Q.6) Mt K2 is the highest peak of India. In which of the following mountain ranges does it lie:
a)
b)
c)
d)

Karakoram Range
Greater Himalayas
Central Himalayas
Siwaliks

Q.6) Solution (a)


The height of Mt. K2 is 8611m from the sea level. It is the second highest mountain peak of the
world.

www.iasbaba.com

Page 3

IASbaba '60 Day Plan' - Prelims Test 2016 [Day 3]


There is a controversy whether K2 is the highest peak of India or Kanchenjunga. K2 lies in POK,
but India consider it as Indian territory hence in all official records, K 2 is the highest peak of
India.

Q.7) Which of the following statements are the major causes of drought in India?
1.
2.
3.
4.

High variability of the rainfall


Absence of forests resulting in rapid runoff and hence droughts
Rainfall is not distributed throughout the year
Amount of rainfall is inadequate

Select the correct code from the following:


a)
b)
c)
d)

1,2 and 3
2,3 and 4
1,3 and 4
All of the above

Q.7) Solution (a)


The absolute amount of rainfall is high. Had it been distributed across the year it would have
been sufficient, but it is concentrated only in few months. Also the variability of rains at a place
is very high.

Q.8) Consider the following statements regarding Indias Shola forests:


1. They are evergreen mountain forests.
2. They are found only in high altitudes (>1500m) in tropical areas.
3. Trees are stunted and have low or no timber value.
Which of the above statements are correct?
a)
b)
c)
d)

1 and 2
2 and 3
1 and 3
All of the above

www.iasbaba.com

Page 4

IASbaba '60 Day Plan' - Prelims Test 2016 [Day 3]


Q.8) Solution (d)
The Sholas are a mosaic of mountane evergreen forests and grasslands. They are found only in
high altitude (>1500 metres asl) regions within the tropics, and are limited to the southern part
of the western ghats. They are characterised by undulating grassland patches, interspersed with
thickets of stunted evergreen tree species, and are home to a host of endemic and endangered
plants and animals. They are also vitally important in keeping water cycles alive. They retain
most of the rain they get over the monsoons, and release it slowly through the year via a
network of streams and rivers, that eventually serve the needs of a huge number of human
settlements across south India.

Q.9) In which of the following regions there is a possibility of finding laterite soil?
a)
b)
c)
d)

Hot and humid


Hot and dry
Cold and dry
Cold and humid

Q.9) Solution (a)


Laterite soils are found in regions of very high rainfall. Due to rains the minerals get leached to
lower horizons. These soils are found in hot and humid areas.

Q.10) Consider the following statements regarding Deccan Trap:


1. It is a vast area of basaltic lava plateau.
2. The rocks are stratified into different layers showing multiple events of volcanism.
3. It is an area of large volcanic cones.
Which of the above statements are incorrect?
a)
b)
c)
d)

1 and 2
3 only
1 and 3
None of the above

www.iasbaba.com

Page 5

IASbaba '60 Day Plan' - Prelims Test 2016 [Day 3]


Q.10) Solution (b)
Basaltic lava has low viscosity and flow rapidly covering large area. It does not make volcanic
cones but results into volcanic shields.

Q.11) In which of the following coal mines do we get Anthracite coal?


1. Kalakot, Jammu and Kashmir
2. Jharia, Jharkhand
3. Neivelli, Tamilnadu
Select the correct code from the following:
a)
b)
c)
d)

1 only
2 and 3
1 and 2
All of the above

Q.11) Solution (a)


Anthracite coal is found only at one place in India, Kalakot mine of J and K.
Jharia is famous for its bituminous coal while Neivelli has lignite

Q.12) The eastern and western coast of Indian peninsula has got contrasting features. Which
of the following contrasts are correct about the coastal plains?
1. Eastern coast is an example of submerging coast while western coast is an example of
emerging coast.
2. A number of rivers form deltas on the western coast, while it is not true for the eastern
coast.
3. As compared to eastern coastal plains, the western coastal plains are much broader.
Select the correct code from the following
a) 1 and 2
b) 2 and 3
c) 1 and 3
www.iasbaba.com

Page 6

IASbaba '60 Day Plan' - Prelims Test 2016 [Day 3]


d) None of the above

Q.12) Solution (d)


Western coast is submerging while eastern coast is emerging.
Eastern coast is much broader than the western coast.
Large rivers make deltas on the eastern coast while rivers flowing towards west forms
estuaries.

Q.13) Consider the following statements about the Indian Thar desert
1. The underlying rock structure of desert is the extension of Peninsular plateau.
2. Fluvial erosion is common making mushroom rocks and shifting dunes.
3. Rivers flow in short streams and disappears showing inland drainage.
Which of the above statements are correct?
a)
b)
c)
d)

1 and 2
2 and 3
1 and 3
All of the above

Q.13) Solution (c)


Aeolian (wind) erosion is dominant in this region.
Fluvial erosion is due to streams and rivers.

Q.14) Consider the following statements and identify the personality?


1. 2015 marks 108th Birth Anniversary of this historical personality
2. He founded India nationalist youth organization, Naujawan Bharat Sabha
3. He was inspired by Anarchism and Marxism
www.iasbaba.com

Page 7

IASbaba '60 Day Plan' - Prelims Test 2016 [Day 3]


Select the correct code
a)
b)
c)
d)

Udham Singh
Lala Lajpat Rai
Bhagat Singh
Chandra Shekhar Azad

Q.14) Solution (c)

Recently controversies like


1. Bipan Chandra book mentioning Bhagat Singh as Revolutionary Terrorist
2. Shashi Tharoor relating Kanhayiya and Rohit Vemula on the path of Bhagat Singh were
in news.
Prepare Bhagat Singh and his contributions in Freedom Struggle. Books written by him,
organizations, he was part of or established. His role models and ideological influences etc.
Bhagat Singh was born on 27 September 1907 at Banga in Lyallpur district (now Pakistan) to
Kishan Singh and Vidya Vati.

Q.15) ISRO has successfully launched ASTROSAT. Consider the following statements w.r.t it
1. It is a kind of telescope
2. It is Indias first Space Observatory
3. It is the first of its kind in the world
Select the correct code
a)
b)
c)
d)

1 and 2
Only 2
2 and 3
1, 2 and 3

Q.15) Solution (a)

www.iasbaba.com

Page 8

IASbaba '60 Day Plan' - Prelims Test 2016 [Day 3]


http://www.thehindu.com/sci-tech/astrosat-indias-unique-spaceobservatory/article7692305.ece
http://www.thehindu.com/sci-tech/science/isro-launches-astrosat-first-spaceobservatory/article7697707.ece

Q.16) Consider the following about Samvatsari


1. It is known as 'Introspection, is a process during which Jain repent for their sins
2. It is also known as International Forgiveness Day and is the holiest day of Jain Calendar
Chose the incorrect statement/s
a)
b)
c)
d)

Only 1
Only 2
Both
None

Q.16) Solution (a)


http://pib.nic.in/newsite/PrintRelease.aspx

Q.17) Consider the following


1.
2.
3.
4.

Nuakhai
Gudhi Parwa
Chavang Kut
Gawai Dayak

Which of the above is/are Agricultural/Harvest festivals of India?


a)
b)
c)
d)

Only 2
1, 2 and 4
1, 2 and 3
2, 3 and 4

Q.17) Solution (c)


www.iasbaba.com

Page 9

IASbaba '60 Day Plan' - Prelims Test 2016 [Day 3]

Nuakhai- Oddisa
Chavang Kut- celebrated by the Kuki-chin group in North-east India on 1 November
Gudhi Parwa- elebrated by the Marathi people in Maharashtra, Karnataka
Gawai Dayak- Sarawak, Malaysia and West Kalimantan, Indonesia

Q.18) Consider the following


1. The largest biomes are temperate grasslands, savannas and shrublands biomes
2. The region is colloquially referred as Stans
3. The region experiences very dry climatic conditions, and inadequate precipitation has
led to heavy dependence on the Syr Darya and Amu Darya for irrigation.
Identify the region based on given statements
a)
b)
c)
d)

Eastern Asia
Eastern Africa
Western Africa
Central Asia

Q.18) Solution (d)


Self-explanatory

Q.19) Consider the following statements with respect to LEGUMES


1. Legumes have an ability to fix nitrogen from the atmosphere
2. Legumes help in increasing soil fertility
3. Legumes are highly nutritive but also contain anti-nutrients that are harmful
Select the correct codes
a) 1 and 2
b) 2 and 3
c) 1, 2 and 3
www.iasbaba.com

Page 10

IASbaba '60 Day Plan' - Prelims Test 2016 [Day 3]


d) Only 1

Q.19) Solution (c)


The legume family consists of plants that produce a pod with seeds inside. Common edible
legumes include lentils, peas, chickpeas, beans, soybeans and peanuts.
Cultivation of all the crops including cereals and regular vegetables will get benefitted if the
legume vegetables are grown once a season.
Apart from fixing atmospheric nitrogen, the legume vegetables also help in increasing the
organic content of the soil as they produce huge quantum of foliage that gets added into the
soil. In addition to this, they also enhance the water-holding capacity of soil, he points out. They
have a positive impact on human health too as these protein-rich legume vegetables can help
fight protein and energy-deficiency induced malnutrition.
They also contain so-called anti-nutrients like Phytic Acid, Lectins and Saponins which are
substances that can interfere with digestion and the absorption of other nutrients.

Q.20) Bhama Kalapam, Parvathi Parinayam and Bhakta Prahlada are related to which of
the dance forms
a)
b)
c)
d)

Kathakali
Kuchipudi
Bharatnatyam
Odissi

Q.20) Solution (b)

Q.21) Consider the following


1.
2.
3.
4.

Atropa Belladona
Datura
Morphine
Opioids

www.iasbaba.com

Page 11

IASbaba '60 Day Plan' - Prelims Test 2016 [Day 3]


Which of the following does not belong to the category of Drugs?
a)
b)
c)
d)

1 and 3 only
1 only
2 and 3 only
None of these

Q.21) Solution (d)

Click here

Q.22) Rio Olympics is round the corner. Consider the following statements
1. The official mascot of Rio 2016 Olympics is Vinicius
2. The Olympics will be held in Capital city of Brazil known as Rio De Generio
Select the correct answer
a)
b)
c)
d)

1 only
2 only
Both
None

Q.22) Solution (a)

The capital city of Brazil is known as Brasilia

Q.23) Consider the following


1. Ringworms
2. Ascaris
3. Filariasis
Which of the following is caused by Fungi?
a) 1 only
b) 2 only
c) 1 and 3 only
www.iasbaba.com

Page 12

IASbaba '60 Day Plan' - Prelims Test 2016 [Day 3]


d) 2 and 3 only
Q.23) Solution (a)

Ringworms are caused by Fungi.


Ascaris and Filariasis is caused by parasites like round worms and tape worms etc

Q.24) Consider the following statements with respect to Rehabilitation of Bonded labour
scheme 2016
1. The quantum of financial assistance under this scheme is increased from Rs 20 thousand
to one lakh rupees.
2. The scheme aims to address new forms of bondage such as organised begging rings,
forced prostitution and child labour for which females are mercilessly used by the
powerful elements.
3. The scheme is implemented by Ministry of women and child development.
Select the correct option
a)
b)
c)
d)

1 only
2 and 3 only
1 and 2 only
All the above

Q.24) Solution (c)

The scheme is implemented by Ministry of Labour and

Employment.

Q.25) Emphasis to restore Jagannath temple at puri was in news recently. Consider the
following statements with respect to Puri Jagannath Temple
1. The idol of Jagannath temple at Puri is made of wood and is replaced once in twelve
years.
2. The temple was rebuilt in 12th century by Eastern Ganga dynasty King Anantavarman
Chodaganga Deva
3. The puri Jagannath temple constitutes one of the four Char Dham pilgrimages that a
Hindu is expected to visit in his lifetime.

www.iasbaba.com

Page 13

IASbaba '60 Day Plan' - Prelims Test 2016 [Day 3]


Select the correct option
a)
b)
c)
d)

1 only
2 and 3 only
1 and 2 only
All the above

Q.25) Solution (d)

Click here

www.iasbaba.com

Page 14

IASbaba Prelims 60 Days: [Day 4]


Q.1) Consider the following
1. International Day for Older Person (IDOP) is celebrated every year on 1 st October
2. The recent theme of IDOP was Leaving No One Behind: Promoting a Society for All
3. 2015, marks the golden jubilee of IDOP
Select the correct code
a)
b)
c)
d)

1 and 2
Only 1
1, 2 and 3
Only 3

Q.1) Solution (a)

The International Day of Older Persons is observed on October 1 each year.


On December 14, 1990 the United Nations General Assembly voted to establish October 1
as the International Day of Older Persons as recorded in Resolution 45/106. The holiday was
observed for the first time on October 1, 1991
Annual Themes
2011: The Growing Opportunities & Challenges of Global Ageing
2012: Longevity: Shaping the Future
2013: The future we want: what older persons are saying
2014: Leaving No One Behind: Promoting a Society for All
2015: Sustainability and Age Inclusiveness in the Urban Environment

Q.2) Consider the following regarding Trident Juncture and select the correct code?
a)
b)
c)
d)

It is the largest military exercise launched by NATO


It is a trilateral exercise of India, Indonesia and Japan
It is a bilateral military exercise between Srilanka and India
It is a trilateral military exercise of NATO, EU and USA

www.iasbaba.com

Page 1

IASbaba Prelims 60 Days: [Day 4]


Q.2) Solution (a)
North Atlantic Treaty Organization (NATO) has launched largest exercise Trident
Juncture since 2002 with troops from 30 states, including non-NATO in Italy, Spain and
Portugal.
30 nations will participate in the drills which are taking place at sea, in the air and across the
territory of three European states.

Q.3) Mission Indradhanush aims to cover which of the following diseases?


1.
2.
3.
4.
5.
6.

Measles
AIDS
Cancer
Polio
TB
Whooping Cough

Select the correct code


a)
b)
c)
d)

1, 3, 4 and 5
3, 4, 5 and 6
1, 4, 5 and 6
1, 2, 3, 4 and 5

Q.3) Solution (c)


Mission Indradhanush: for vaccination against 7 preventable disease by 2020.
The diseases are Diphtheria, Whooping cough, Tetanus, Polio, Tuberculosis, Measles and
Hepatitis B.
The aim is to cover all the children who have been left out or missed out for immunization.

Q.4) Consider the following about Nobel Prize, 2015


1. Nobel Prize in Physics has been given for the discovery of neutrino oscillations, which
shows that neutrinos have mass

www.iasbaba.com

Page 2

IASbaba Prelims 60 Days: [Day 4]


2. Nobel Prize in Chemistry has been given for the discoveries concerning a novel
therapy against infections caused by roundworm parasite.
3. Nobel Prize in Medicine has been given for mechanistic studies of DNA repair.
Select the correct code
a)
b)
c)
d)

1 and 2
Only 2
Only 1
1, 2 and 3

Q.4) Solution (c)


The Nobel Prize in Physics 2015
Takaaki Kajita and Arthur B. McDonald
"for the discovery of neutrino oscillations, which shows that neutrinos have mass"

The Nobel Prize in Chemistry 2015


Tomas Lindahl, Paul Modrich and Aziz Sancar
"for mechanistic studies of DNA repair"

The Nobel Prize in Physiology or Medicine 2015


William C. Campbell and Satoshi mura
"for their discoveries concerning a novel therapy against infections caused by roundworm
parasites"
Youyou Tu
"for her discoveries concerning a novel therapy against Malaria"

Q.5) Consider the following statements regarding Compost


1. Organic ingredients intended for composting can alternatively be used to
generate biogas through aerobic digestion
2. Compost is used to control soil erosion and sediment run off.
www.iasbaba.com

Page 3

IASbaba Prelims 60 Days: [Day 4]


Select the correct code
a)
b)
c)
d)

Only 1
Only 2
Both
None

Q.5) Solution (b)

Compost is rich in nutrients. It is used in gardens, landscaping, horticulture, and agriculture.


The compost itself is beneficial for the land in many ways, including as a soil conditioner, a
fertilizer, addition of vital humus or humic acids, and as a natural pesticide for soil. In
ecosystems, compost is useful for erosion control, land and stream reclamation, wetland
construction, and as landfill cover. Organic ingredients intended for composting can
alternatively be used to generate biogas through anaerobic digestion.
Topsoil loss is a serious ecological issue. The use of compost to control sediment run-off and
fight erosion is a relatively new technology, now being adopted by local authorities,
developers, farmers, and other major disturbers of soil as another tool to reduce topsoil
loss.
http://www.thehindu.com/todays-paper/tp-national/firms-must-buy-compost-frommunicipal-waste/article8230689.ece

Q.6) Consider the following regarding Biochar


1. It is made up of biomass by a process called combustion
2. It is an unstable solid formed by charcoal
3. It helps in increasing acidity of the soil when the PH of the soil is high
Select the correct code
a)
b)
c)
d)

1 and 2
1, 2 and 3
Only 3
None

6.) Solution (d)

www.iasbaba.com

Page 4

IASbaba Prelims 60 Days: [Day 4]


Bio-char reduces the acidity of the soil, protects the plants from diseases, promotes growth
of friendly micro organisms, and reduces the loss of micro nutrients apart from increasing
water retain-ability.
http://www.thehindu.com/business/agri-business/biochar-compost-tea-are-the-newrecipe/article7830057.ece
https://en.wikipedia.org/wiki/Biochar

Q.7) Consider the statements regarding the use of Antibiotics


1. Flu or cold can be treated by using antibiotics
2. Antibiotic resistance is caused when body becomes resistant to antibiotics
3. Antibiotic resistance is caused by genetic mutation and by acquiring resistance from
another bacterium
Select the correct code
a)
b)
c)
d)

1, 2 and 3
2 and 3
Only 3
1 and 2

Q.7) Solution (c)


Because cold or flu is caused by a virus, while antibiotics can only cure infections caused by
bacteria. (You might need antibiotics only if your flu gets worse and results in a secondary
bacterial infection like bronchitis or pneumonia.)
One should understand that it is the bacteria which become resistant to antibiotics, not the
body. Through indiscriminate antibiotics use, you are helping create drug-resistant bacteria,
which then cause difficult-to-treat infections in others.
Some bacteria are naturally resistant to certain types of antibiotics. However, bacteria may
also become resistant in two ways: 1) by a genetic mutation or 2) by acquiring resistance
from another bacterium.
http://www.thehindu.com/news/national/kerala/no-longer-a-cureall/article7907429.ece

Q.8) National Tribal Advisory Council to be set up is to be chaired by?

www.iasbaba.com

Page 5

IASbaba Prelims 60 Days: [Day 4]


a)
b)
c)
d)

President of India
Prime Minister of India
Ministry of Tribal Affairs
Prime Minister Office (PMO)

Q.8) Solution (b)


A National Tribal Advisory Council under the chairmanship of Prime Minister will be set up
for real time monitoring of the ongoing programmes and schemes aimed at welfare of tribal
people and development of tribal areas. The Council will meet once or twice a year

Q.9) Which of the following are correct about Karewas


1. Karewas are the lacustarine deposits found only in Kashmir valley and bhadarwa
valley of Jammu, in India.
2. It is suitable for growing one of the worlds best qualities of saffron.
3. The soil is peaty and contains a lot of carbon content.
Which of the above statements are correct?
a)
b)
c)
d)

1 and 2
2 and 3
1 and 3
All of the above

Q.9) Solution (d)


Karewas are lacustrine deposits [deposits in lake] in the Valley of Kashmir and in Bhadarwah
Valley of the Jammu Division. These are the flat topped mounds that border the Kashmir
Valley on all sides. They are characterized with fossils of mammals and at places by peat.

Q.10) Which of the following statements are correct about crop rotation?
1. Growing of different crops on a piece of land in a pre-planned succession is called
crop rotation.
2. The availability of moisture and irrigation facilities decide the choice of crops to be
grown.
3. Ideally staple grain crops are rotated with leguminous crops.

www.iasbaba.com

Page 6

IASbaba Prelims 60 Days: [Day 4]


Select the correct code from above:
a) 1 and 2
b) 2 and 3
c) 1 and 3
d) All of the above
Q.10) Solution (d)
Usually grain crops like rice, wheat etc, needs a lot of nutrition. Repeated growing of these
crops rob the land from its nutrients. Hence it is ideal to rotate them with leguminous crops
so that their fertility can be regained through nitrogen fixation.

Q.11) Which of the following statements correctly explains the process of Ratooning?
a) It is a method of harvesting a crop which leaves the roots and the lower parts of the
plant uncut to give stubble crop.
b) It is a method of preparing field by burning the stubs of harvested crops in the field,
to provide nutrition to the soil.
c) It is method of using burrowing animals to make the field porous before harvesting
of crop.
d) None of the above.

Q.11) Solution (a)


Ratooning is a method of harvesting a crop which leaves the roots and the lower parts of
the plant uncut to give the ratoon or the stubble crop. One of the important methods of the
intensive cropping, allowing the stubbles of the original crop to strike again after harvesting
and to raise another crop.The main benefit of ratooning is that the crop matures earlier in
the season. Ratooning can also decrease the cost of preparing the field and planting.
This method cannot be used endlessly as the yield of the ratoon crop decreases after each
cycle. Ratooning is most often used with crops which are known to give a steady yield for
three years under most conditions.

Q.12) Arrange the following rock systems of our country as classified by the Geological
Survey of India from oldest to recent:
1. The Purana Rock System
2. The Dravidian Rock System
www.iasbaba.com

Page 7

IASbaba Prelims 60 Days: [Day 4]


3. The Archaean Rock System
4. The Aryan Rock System
Select the correct code from the following:
a) 2, 3, 1, 4
b) 2, 3, 4, 1
c) 3, 1, 2, 4
d) 3, 1, 4, 2

Q.12) Solution (c)


Self explanatory

Q.13) Which amongst the following rivers has the largest drainage basin in India?
a) Indus
b) Brahmaputra
c) Krishna
d) Godavari
Q.13) Solution (a)
Amongst the given rivers Indus has the largest basin in India. It has the second largest basin
after river Ganga.
Ganga> Indua> Godavari> Krishna> Brahmaputra

Q.14) Consider the following statements:


1. Easterly jet stream over India is seasonal and exists only during summer.
2. Westerly jets are helpful in pushing western disturbances towards India, causing
winter rainfall.
3. Westerly jets are bifurcated by the Tibetan plateau during summers with one stream
flowing over northern India.
Which of the above statements are correct?
a) All of the above
www.iasbaba.com

Page 8

IASbaba Prelims 60 Days: [Day 4]


b) 1 and 2
c) 2 and 3
d) 1 and 3

Q.14) Solution (b)


During winters, as the ITCZ shifts downwards, the subtropical high pressure belt also shifts.
With this shift, a downward shift in westerly jet stream is also seen. This jet stream when
strikes Pamir knot and tibetan plateau, it is bifurcated. One stream goes over the plateau
and one stream is pushed downwards, which flows over India.
Q.15) Consider the following statements:
1. The deviation of actual rainfall of a place from its average rainfall is called variability.
2. The highest variability is found at places where average rainfall is lowest.
3. Variability of rainfall is measured in percentage.
Which of the above statements are correct about variability of rainfall?
a)
b)
c)
d)

1 only
1 and 3
2 and 3
All of the above

Q.15) Solution (d)


The actual rainfall of a place in a year deviates from its average rainfall by 10 to over 60
percent. The highest variability is found in areas which have less rainfall. For eg. In desert
area if average rainfall is only 20 cm, then even 10 cm increase in rain in a year would be
highly variable.
The variability of rainfall plays a significant role in agricultural and other economic activities.

Q.16) Which of the following statements are correct about black soils of India?
1.
2.
3.
4.

They have a volcanic origin.


They cover the largest reporting area of the country.
They are deficient in nitrogen, phosphorus and organic matter.
They have clayey texture and have high water holding capacity.

www.iasbaba.com

Page 9

IASbaba Prelims 60 Days: [Day 4]


Select the correct code from the following:
a)
b)
c)
d)

1 and 4
1,2 and 3
1,3 and 4
All of the above

Q.16) Solution (c)


Alluvial soils are most abundant covering 43.4% of reporting area. The second most
abundant soil is red soil with 18.5% area and then black soil with 15% area.

Q.17) Consider the following statements about the texture of soil:


1.
2.
3.
4.

The size of soil particles defines the texture of soil.


A soil having mixture of substantial amount of sand, silt and clay is called loam.
Soil texture decides the water holding capacity of the soil.
Finer the soil texture, the better it is for agriculture.

Which of the above statements are incorrect?


a)
b)
c)
d)

1,2 and 3
1 and 4
4 only
None of the above

Q.17) Solution (c)


Medium sized particles are ideal for agriculture. (Silty loam)
Very fine particles have high water holding capacity and lacks aeration. Also they are hard to
work upon as they become sticky when wet and very hard when dry.

Q.18) Soil salinity has emerged a major problem affecting the fertility of soil in India.
Almost 2.5% of countrys reporting area is adversely affected by alkali formations. Which
of the following statements are correct about saline soils?
1. Salinity is naturally found in arid and semi-arid areas where rate of evaporation is
higher than rate of precipitation.
www.iasbaba.com

Page 10

IASbaba Prelims 60 Days: [Day 4]


2. Saline soils develop in highly irrigated area as more salt comes up due to capillary
action of soil.
3. Salinity can be cured by adding gypsum to the soil.
Select the correct code from the following:
a)
b)
c)
d)

1 and 3
1 and 2
2 and 3
All of the above

Q.18) Solution (d)


Natural salinity is found in desert regions like Rann of Kutch. These natural salt formations
are commercially utilized.

Q.19) Consider the following statements:


1. Kerala has no east flowing rivers.
2. Madhya Pradesh has no west flowing rivers.
Which of the above statements are correct?
a)
b)
c)
d)

1 only
2 only
Both 1 and 2
Neither 1 nor 2

Q.19) Solution (d)


There are three rivers rise in Kerala and flow eastwards, Kabini into Karnataka and the other
two into Tamil Nadu.

Kabani
Bhavani
Pambar

Q.20) Which one of the following rivers do not originate in India?


a) Ravi
b) Beas
www.iasbaba.com

Page 11

IASbaba Prelims 60 Days: [Day 4]


c) Sutluj
d) Chenab

Q.20) Solution (c)


The origin of the Sutlej River is located close to Lake Rakhastal in Tibet in the vicinity of the
Kailash Mountain Range. The Sutlej River usually runs according to a west to southwest
itinerary moving into India via the Shipki La Pass located in Himachal Pradesh.

Q.21) Consider the following statements regarding equatorial climate:


1. Very heavy down pour, normally for a short duration in a year.
2. Convectional rainfall characterised by lightning and thunder.
3. There is a very high diurnal temperature range in equatorial regions.
Select the correct answer
a)
b)
c)
d)

1 and 2 only
2 only
2 and 3 only
None of the above

Q.21) Solution (b)


Equatorial regions have very low diurnal temperature range and rainfall occur throughout
the year.

Q.22) The recent advent of yellow fever in some parts of Africa was in news recently.
Consider the following statements with respect to Yellow fever
1. The disease is transmitted by the bite of a Mosquito.
2. It is curable.
3. The disease is characterised by high fever and Jaundice.
Select the correct answer
a) 1 and 3 only
b) 2 only
www.iasbaba.com

Page 12

IASbaba Prelims 60 Days: [Day 4]


c) 2 and 3 only
d) 1,2 and 3
Q.22) Solution (a)
Yellow fever is a serious, potentially deadly flu-like disease spread by mosquitoes.
Characterized by a high fever and jaundice, it is most prevalent in certain parts of Africa and
South America. The disease is not curable, but is preventable with the yellow fever vaccine.
Q.23) Recently ISRO test fired Prithvi 2 missile from a test range at Chandipur in Odisha.
Conisder the following statements with respect to Prithvi 2 missile
1.
2.
3.
4.

It is a surface to air missile


It has a strike range of 2000 km
It is the first missile developed under integrated missile development programme.
It has a single stage liquid fuelled propellant

Select the correct answer


a)
b)
c)
d)

1 and 2 only
2 and 3 only
3 and 4 only
1 and 3 only

Q.23) Solution (c)

Click here

Q.24) PM referred to Pragjyotishpur at the inaugural address of south Asian games.


Which of the following city today refers to Pragjyotishpur in ancient India?
a)
b)
c)
d)

Dibrugarh
Thimpu
Dispur
Guwahathi

Q.24) Solution (d)


Pragjyotishpur in Ancient India refers to Guwahati in Modern India

www.iasbaba.com

Page 13

IASbaba Prelims 60 Days: [Day 4]

Q.25) ARYA and Farmers first to revamp agricultural produce in India are proposed
initiatives by
a)
b)
c)
d)

Indian agricultural research institute


Ministry of agriculture in collaboration with ICRISAT
Indian council of agricultural research
Ministry of agriculture in collaboration with FAO

Q.25) Solution (c)


ARYA (Attracting and retaining youth in Agriculture), Mera Gaon Mera Gaurav and farmers
first initiatives by ICAR

www.iasbaba.com

Page 14

IASbaba Prelims 60 Days: [Day 5]


Q.1) Consider the following statements regarding Bio-fertilizers
1. The microorganisms in bio-fertilizers restore the soil's natural nutrient cycle and
build soil organic matter.
2. Bio-fertilizers directly increase soil fertility by adding nutrients
3. Rhizobium is a free living nitrogen fixing bacteria
Select the correct code
a)
b)
c)
d)

1 and 2
2 and 3
Only 1
1, 2 and 3

Q.1) Solution (c)


Picked from class 9th NCERT- Mention of bio-fertilizers is there in chapter 15
IMPROVEMENT IN FOOD RESOURCES
UPSC has asked questions based on such terms which are always in news. Try to know about
types of Bio-fertilizers also.
First statement is correct but second and third is not. How and Why?
First of all, bio-fertilizers are not fertilizers, which directly increase soil fertility. Rather Biofertilizers add nutrients through the natural processes of nitrogen fixation,
solubilizing phosphorus, and stimulating plant growth through the synthesis of growthpromoting substances.
There are various groups in which Bio-fertilizers are grouped like Free living, Symbiotic,
Phosphorous mobilizing, P Solubilizing etc
Symbiotic Nitrogen Fixing Bacteria:
They form a mutually beneficial association with the plants. The bacteria obtain food and
shelter from plants. In return, they give a part of their fixed nitrogen to the plants. The most
important of the symbiotic nitrogen fixing bacteria is Rhizobium (pi Rhizobia). It forms
nodules on the roots of legume plants. There are about a dozen species of Rhizobium which
form association with different legume roots,
Free Living Nitrogen Fixing Bacteria:

www.iasbaba.com

Page 1

IASbaba Prelims 60 Days: [Day 5]


They live freely in the soil and perform nitrogen fixation. Some of them are saprotrophic,
living on organic remains, e.g., Azoto- bacter, Bacillus polymyxa, Clostridium, Beijerinckia.
They are further distinguished into aerobic and anaerobic forms.
In case you wonder, how it is in news in recent times? See here
http://www.thehindu.com/news/cities/Coimbatore/tamil-nadu-tops-in-use-ofbiofertilizers/article8118408.ece
Also learn about differences between liquid and solid bio-fertilizers?

Q.2) Arrange the following in increasing order of their water content.


1.
2.
3.
4.

Egg
Milk
Meat
Fish

Select the correct code


a)
b)
c)
d)

1-4-3-2
2-1-4-3
1-3-4-2
4-1-3-2

Q.2) Solution (c)


NCERT- 9th Class, Chapter 15
Water Content
Milk (Cow)- 87.20
Egg - 74.00
Meat- 74.20
Fish- 77.20

Q.3) Consider the following regarding India Fishery Production


1. India fisheries is the second largest in the world
www.iasbaba.com

Page 2

IASbaba Prelims 60 Days: [Day 5]


2. Inland fish production is more than marine in India
3. Contribution of fishery to Indian agriculture is less than its contribution towards GDP
Select the correct code
a)
b)
c)
d)

1 and 2 only
Only 2
Only 3
2 and 3 only

Q.3) Solution (b)

Again picked from class 9th NCERT- chapter 15 IMPROVEMENT IN FOOD RESOURCES FISH
PRODUCTION
Refer this link of Government Website
http://nfdb.gov.in/html/aboutus.htm

Q.4) Consider the following statements regarding Vertebrates


1. The majority of animal species are vertebrates
2. Shark is an example of vertebrate
3. Vertebrates can be identifies by the presence of notochord and gill pouches
Select the correct code
a)
b)
c)
d)

1 and 2
2 and 3
1 and 3
1, 2 and 3

Q.4) Solution (b)


Again picked from class 9th NCERT- chapter 7 IMPROVEMENT IN FOOD RESOURCES FISH
PRODUCTION
The majority of animal species are invertebrates; one estimate puts the figure at 97%.
Vertebrates make up about 4% of all described animal species; the rest are invertebrates,
which lack vertebral columns.
www.iasbaba.com

Page 3

IASbaba Prelims 60 Days: [Day 5]


In case you think how vertebrate forms part of current affairs, have a look at this link
http://www.thehindu.com/news/national/kerala/high-vertebrate-diversity-recorded-instate/article7902615.ece
Q.5) Consider the following w.r.t Meristematic Tissues
1. Meristematic tissues are a group of young cells that are in a continuous state of
division
2. They are living and thin walled
3. They have few vacuoles and are small in size
Select the incorrect code
a)
b)
c)
d)

1, 2 and 3
Only 2
2 and 3
None

Q.5) Solution (d)


Picked from class 9th NCERT- chapter 6 TISSUES

Q.6) Consider the following statements regarding Chronic Diseases


1. Some of the common examples of chronic diseases are arthritis, cardiovascular
disease, AIDS and Cancer
2. Chronic diseases can be Non-Communicable Disease (NCD)
3. Diabetes is a type of chronic disease
Select the correct code
a)
b)
c)
d)

1 and 2
2 and 3
Only 1
1, 2 and 3

Q.6) Solution (d)


Picked from class 9th NCERT- chapter 13 WHY DO WE FALL ILL
In case you wonder it is not in current affairs, click below
www.iasbaba.com

Page 4

IASbaba Prelims 60 Days: [Day 5]


http://www.thehindu.com/news/cities/bangalore/not-everyone-can-afford-to-fightchronic-diseases/article8608998.ece
http://www.thehindubusinessline.com/news/blueprint-on-chronic-diseases-calls-forpreventive-care/article7829903.ece

Q.7) Recently scientists made a historic detection of Gravitational Waves. The discovery
would represent a scientific landmark, opening the door to an entirely new way to
observe the cosmos and unlock secrets about the early universe and mysterious objects
like black holes and neutron stars. Consider the following w.r.t Gravitational Waves
1. Gravitational waves can be produced by Humans, Cars and Aeroplanes.
2. The strongest gravitational waves are produced by catastrophic events such as
colliding black holes, the collapse of supernovae and the remnants of gravitational
radiation created by the birth of the Universe itself.
Select the correct code
a)
b)
c)
d)

Only 1
Only 2
Both
None

Q.7) Solution (c)


Gravitational waves are 'ripples' in the fabric of space-time caused by some of the most
violent and energetic processes in the Universe. Albert Einstein predicted the existence of
gravitational waves in 1916 in his general theory of relativity. Einstein's mathematics
showed that massive accelerating objects (such as neutron stars or black holes orbiting each
other) would disrupt space-time in such a way that 'waves' of distorted space would radiate
from the source (like the movement of waves away from a stone thrown into a pond).
Furthermore, these ripples would travel at the speed of light through the Universe, carrying
with them information about their cataclysmic origins, as well as invaluable clues to the
nature of gravity itself.
The strongest gravitational waves are produced by catastrophic events such as colliding
black holes, the collapse of stellar cores (supernovae), coalescing neutron stars or white
dwarf stars, the slightly wobbly rotation of neutron stars that are not perfect spheres, and
the remnants of gravitational radiation created by the birth of the Universe itself.

www.iasbaba.com

Page 5

IASbaba Prelims 60 Days: [Day 5]


Any object with mass that accelerates (which in science means changes position at a
variable rate, and includes spinning and orbiting objects) produces gravitational waves,
including humans and cars and airplanes etc. But the gravitational waves made by us here
on Earth are much too small to detect. In fact, it isnt even remotely possible to build a
machine that can spin an object fast enough to produce a detectible gravitational wave
even the worlds strongest materials would fly apart at the rotation speeds such a machine
would require.
http://www.thehindu.com/specials/in-depth/gravitational-wavesexplained/article8224119.ece
http://www.thehindu.com/sci-tech/listening-to-the-chirps-of-gravitationalwaves/article8233625.ece

Q.8) Consider the following regarding Light Emitting Diodes (LED)


1. They work on the principle of electroluminescence
2. White LEDs are used in sterilization and hygiene since it can kill harmful bacteria
3. Blue led can be used for flat screens and date storage since it is used in compact
disks to store data.
Select the correct code
a)
b)
c)
d)

1 and 2
2 and 3
1, 2 and 3
1 and 3

Q.8) Solution (d)


The applications of blue led and white led are different. In general, white led is usually used
in lighting applications. Well, blue led can be used in sterilization and hygiene since it can kill
harmful bacteria that can cause dental and skin problems. Blue led also can be used for flat
screens and date storage since it is used in compact disks to store data.
A light-emitting diode (LED) is a two-lead semiconductor light source. It is a pn
junction diode, which emits light when activated. When a suitable voltage is applied to the
leads, electrons are able to recombine with electron holes within the device, releasing
energy in the form of photons. This effect is called electroluminescence, and the color of the
light (corresponding to the energy of the photon) is determined by the energy band gap of
the semiconductor.

www.iasbaba.com

Page 6

IASbaba Prelims 60 Days: [Day 5]


http://www.thehindu.com/sci-tech/technology/internet/led-bulb-could-connect-you-tointernet/article8253782.ece
http://www.thehindu.com/business/Industry/switching-to-led-bulbs-could-save-you-morethan-rs4000-a-year/article8447542.ece
Q.9) Generic Drugs have gain lot of attention in recent times due to Indo-US spate over
IPR and Drug regime. Consider the following statements, to gain FDA approval, a generic
drug must:
1.
2.
3.
4.

Contain the same active ingredients as the innovator drug


Be identical in strength, dosage form, and route of administration
Be bioequivalent
Meet the same batch requirements for identity, strength, purity, and quality

Select the correct code


a)
b)
c)
d)

1, 2 and 3
2 and 4
1, 3 and 4
1, 2, 3 and 4

Q.9) Solution (d)


Health professionals and consumers can be assured that FDA approved generic drugs have
met the same rigid standards as the innovator drug. To gain FDA approval, a generic drug
must:

contain the same active ingredients as the innovator drug(inactive ingredients may
vary)
be identical in strength, dosage form, and route of administration
have the same use indications
be bioequivalent
meet the same batch requirements for identity, strength, purity, and quality
be manufactured under the same strict standards of FDA's good manufacturing
practice regulations required for innovator products

http://www.thehindu.com/news/cities/bangalore/even-doctors-feel-generic-drugs-aresubstandard/article8398116.ece
http://www.thehindu.com/opinion/editorial/editorial-on-ban-on-generic-medicines-moremedicine-for-less/article8528990.ece

www.iasbaba.com

Page 7

IASbaba Prelims 60 Days: [Day 5]

Q.10) The process of nitrogen-fixation by bacteria does not take place in the presence of
a)
b)
c)
d)

Molecular form of hydrogen


Elemental form of oxygen
Water
Elemental form of nitrogen

Q.10) Solution (b)


Picked from Class 9th Science- Chapter 14, Natural Resources

Q.11) Which of these options are not a function of Ribosomes?


1.
2.
3.
4.

It helps in manufacture of protein molecules


It helps in manufacture of enzymes
It helps in manufacture of hormones
It helps in manufacture of starch molecules

Select the correct code


a)
b)
c)
d)

1 and 2
2 and 3
3 and 4
1 and 4

Q.11) Solution (c)


Picked from Class 9th Science- Chapter 5, THE FUNDAMENTAL UNIT OF LIFE

Q.12) Which of the following are homogeneous in nature?


1.
2.
3.
4.

Ice
Wood
Soil
Air

Select the correct code

www.iasbaba.com

Page 8

IASbaba Prelims 60 Days: [Day 5]


a)
b)
c)
d)

1 and 3
2 and 4
1 and 4
3 and 4

Q.12) Solution (c)


Picked from Class 9th Science- Chapter 2, IS MATTER AROUND US PURE

Q.13) Consider the following statements regarding Lifi technology:


1. Li-Fi technology uses visible light for data transfer and communication.
2. Li-Fi uses common household LED (light emitting diodes) lightbulbs to enable data
transfer, boasting speeds of up to 224 gigabits per second.
3. On-off light signals are received as binary signals at very high speed by a receptor.
Which of the above statements are correct?
a)
b)
c)
d)

1 and 2
2 and 3
1 and 3
All of the above

Q.13) Solution (d)


Light Fidelity or Li-Fi is a Visible Light Communications (VLC) system running wireless
communications travelling at very high speeds.
Li-Fi uses common household LED (light emitting diodes) lightbulbs to enable data transfer,
boasting speeds of up to 224 gigabits per second.

Q.14) Rotavirus is the most common cause of severe vomiting and diarrhoea among
infants and young children. Which of the following statements are correct about rotavirus:
1. It is the biggest cause of death of children under 5 years of age in India.
2. India has recently launched an indigenous vaccine for rotavirus called Rotavac.
3. Rotavac has been added in Indias Universal Immunisation Program.
Select the correct code from the following:
a) 1 and 2
www.iasbaba.com

Page 9

IASbaba Prelims 60 Days: [Day 5]


b) 2 and 3
c) 1 and 3
d) All of the above

Q.14) Solution (d)


Diarrhoea caused by Rotavirus is one of the leading causes of severe diarrhoea and death
among children less than five years of age. In India, between 80,000 to one lakh children die
due to Rotavirus diarrhoea annually while nearly 9 lakh children are admitted to hospital
with severe diarrhoea.
The Rotavac is in addition to three new vaccines that have been introduced in Indias
Universal Immunisation Programme (UIP) including Inactivated Polio Vaccine (IPV), Measles,
Rubella (MR) vaccine, and Adult Japanese Encephalitis (JE) vaccine.

Q.15) The acoustic frequencies used in sonar systems


1.
2.
3.
4.

ultrasonic waves
infrasonic waves
radio waves
audible sound waves

Select the correct code


a)
b)
c)
d)

1 and 2
Only 1
1 and 3
1 and 4

Q.15) Solution (a)


The acoustic frequencies used in sonar systems vary from very low (infrasonic) to extremely
high (ultrasonic). The study of underwater sound is known as underwater
acoustics or hydroacoustics.
Concept picked from Class 9th Science- Chapter 12, SOUND

www.iasbaba.com

Page 10

IASbaba Prelims 60 Days: [Day 5]


Q.16) Which of the following reactions are irreversible chemical changes:
1.
2.
3.
4.

Rusting of Iron
Cooking of food
Fermentation of grapes
Formation of ozone

Select the correct code from the following:


a)
b)
c)
d)

All of the above


1,2 and 3
2,3 and 4
1,3 and 4

Q.16) Solution (b)


Formation of Ozone is a reversible process. Once an ozone molecule is formed, it breaks
again into oxygen gas(O2) and Oxygen molecule(O).

Q.17) Which of the following processes are exothermic i.e. they release energy?
1. Burning of coal
2. Respiration
3. Decomposition of vegetables to form compost
Select the code from the following:
a)
b)
c)
d)

1 and 2
2 and 3
1 and 3
All of the above

Q.17) Solution (d)


In the process of respiration, the food we eat reacts with the oxygen we breathe and
releases energy. Note: all the three examples are taken from class X NCERT.

Q.18) If someone is suffering from the problem of acidity. What is the ideal thing he
should take to have relief?
www.iasbaba.com

Page 11

IASbaba Prelims 60 Days: [Day 5]


a)
b)
c)
d)

Lemon juice
Vinegar
Baking soda solution
Aerated soft drink

Q.18) Solution (c)


Baking soda solution is a basic solution which helps in nutralising the acidity. Other three are
acidic which will aggravate the problem.

Q.19) Consider the following statements:


1. Distilled water does not conduct electricity.
2. The pH of distilled water is seven.
Which of the above statements are correct?
a)
b)
c)
d)

1 only
2 only
Both 1 and 2
Neither 1 nor 2

Q.19) solution (c)


Distilled water is pure water and does not contain any salt ions. Pure water is a bad
conductor of electricity.
Distilled water is neither acidic nor basic. The pH value is 7.

Q.20) Copper was the first metal to be used by mankind on a large scale. What was the
correct reason behind that?
a)
b)
c)
d)

Copper is less reactive and can be easily extracted from its ore.
Copper was the most abundant metal of the world.
Copper had higher conductivity than iron or aluminum.
Copper had religious significance.

www.iasbaba.com

Page 12

IASbaba Prelims 60 Days: [Day 5]

Q.20) Solution (a)


Copper was accidentally discovered by man. It was extensively used as it was easy to extract
it from its ore. (Just by heating)

Q.21) Consider the following statements regarding metal alloys:


1. An alloy is a homogenous mixture of two or more metals.
2. A non-metal can also be added in an alloy with base metal.
3. An alloy has improved qualities over base metal and also prevents corrosion.
Which of the above statements are correct?
a)
b)
c)
d)

1 and 2
2 and 3
1 and 3
All of the above

Q.21) Solution (d)


An alloy is a homogeneous mixture of two or more metals, or a metal and a non-metal. Eg.
Carbon is mixed with iron to get steel.

Q.22) Food cans are coated with tin and not with zinc because:
a)
b)
c)
d)

Zinc is costlier than tin.


Zinc has a higher melting point than tin.
Zinc is more reactive than tin.
Zinc is less reactive than tin.

Q.22) Solution (c)


Zinc can react with food items especially acidic things like vinegar, lemon juice etc. and can
contaminate the food.

Q.23) Consider the following statements:

www.iasbaba.com

Page 13

IASbaba Prelims 60 Days: [Day 5]


1. Sexual reproduction leads to higher biodiversity than asexual reproduction.
2. The genetic material is transferred to the progeny in sexual reproduction and not in
asexual reproduction.
3. Asexual reproduction results in almost identical progeny.
Which of the above statements are correct?
a)
b)
c)
d)

1 only
2 and 3
1 and 3
All of the above

Q.23) Solution (c)


Sexual reproduction leads to intermixing of jeans which causes more diversity.
In both sexual as well as asexual reproduction, genetic material is transferred.
Since in asexual reproduction, genetic material is copied, the progeny is almost an identical
copy.eg. binary fission of an amoeba.
Q.24) Consider the following statements regarding common eye defects:
1.
2.
3.
4.

Myopia is near sightedness i.e. a person is unable to clearly see nearby objects.
Myopia is corrected using concave lens of suitable power.
Hypermetropia is far sightedness i.e. a person is unable to clearly see distant object.
Hypermetropia is corrected using a convex lens.

Which of the above statements are incorrect?


a)
b)
c)
d)

1 and 3
2 and 4
None of the above
All of the above

Q.24) Solution (a)


1 and 3 are incorrect. Myopia is nearsightedness i.e. a person is unable to see the distant
objects. While hypermetropia is farsightedness i.e. a person is unable to clearly see the
nearby objects.

www.iasbaba.com

Page 14

IASbaba Prelims 60 Days: [Day 5]


Q.25) Interpol has issued a Red Corner Notice (RCN) against Shahid Latif, Pakistan-based
handler of Jaish-e-Mohammed terrorists who had attacked Pathankot Air Force base in
January. Consider the following statements with respect to INTERPOL
1. It is the second largest political organization after the United Nations in terms of
international representation.
2. INTERPOL is not an organ of UN
3. INTERPOL headquarters is located in Lyon, France
Select the correct option
a)
b)
c)
d)

1 and 2 only
2 and 3 only
3 only
1, 2 and 3

Q.25) Solution (d)

Click here

Q.26) The cyclonic storm ROANU has affected which of the following three states of India
a)
b)
c)
d)

Odisha, Andhra Pradesh, West Bengal


Odisha, Kerala, Tamil Nadu
West Bengal, Tamil Nadu, Karnataka
Andhra Pradesh, Tamil Nadu, Kerala

Q.26) Solution (a)

Click here

Q.27) Consider the following


1. Ammonia
2. Insecticides
3. Sulphur dioxide
Which of the following does not belong to the category of inorganic compound which
cause water pollution?
a) 1 and 3 only
www.iasbaba.com

Page 15

IASbaba Prelims 60 Days: [Day 5]


b) 2 only
c) 2 and 3 only
d) None of the above
Q.27) Solution (b)

Click here
News

Q.28) Consider the following


1. Fibrinogen
2. Globulins
3. Albumins
Which of the above are the components of Plasma?
a)
b)
c)
d)

1 only
2 and 3 only
3 only
All

Q.28) Solution (d)

Click here

Q.29) The indigenous people Pygmies are found in which of the following deserts
a)
b)
c)
d)

Kalahari Desert
Arabian Desert
Gobi Desert
Atacama Desert

Q.29) Solution (a)

www.iasbaba.com

Page 16

IASbaba Prelims 60 Days: [DAY 6]


Q.1) Consider the following about Global Environment Facility (GEF)
It serves as financial mechanism for?
1.
2.
3.
4.
5.
6.

Convention on Biological Diversity (CBD)


United Nations Framework Convention on Climate Change (UNFCCC)
Stockholm Convention on Persistent Organic Pollutants (POPs)
Montreal Protocol
UN Convention to Combat Desertification (UNCCD)
Minamata Convention on Mercury

Select the correct code


a)
b)
c)
d)

2, 3, 4, 5 and 6
2, 4 and 5
1, 2, 3, 4 and 5
1, 2, 3, 5 and 6

Q.1) Solution (d)


The GEF serves as financial mechanism for the following conventions:
Convention on Biological Diversity (CBD)
United Nations Framework Convention on Climate Change (UNFCCC)
Stockholm Convention on Persistent Organic Pollutants (POPs)
UN Convention to Combat Desertification (UNCCD)
Minamata Convention on Mercury

Source- Picked from India Year Book, Chapter on Environment (Global Environment Facility)

Q.2) Consider the following


1. International Day for Biological Diversity is celebrated on 29th December
2. It is held on December 29 to celebrate the day the Convention on Biological
Diversity went into effect.
www.iasbaba.com

Page 1

IASbaba Prelims 60 Days: [DAY 6]


Select the correct code
a)
b)
c)
d)

Only 1
Only 2
Both
None

Q.2) Solution (d)


It is now celebrated on 22nd May. Earlier it was celebrated on 29th December until 2000. Since
then it is celebrated on 22nd May to commemorate the adoption of the Convention on May 22,
1992 at the Rio Earth Summit.
http://www.business-standard.com/article/government-press-release/biodiversity-forsustainable-development-environment-minister-s-message-on-international-day115052200177_1.html

Q.3) Consider the following


1. Nagoya Protocol came into force before Cartagena Protocol
2. Nagoya Protocol comes under Convention on Biological Diversity
3. The Nagoya Protocol on Biosafety is an international agreement on biosafety as a
supplement to the Convention on Biological Diversity
Select the correct code
a)
b)
c)
d)

Only 1
Only 2
1 and 3
1, 2 and 3

Q.3) Solution (b)


Source- Picked from India Year Book, Chapter on Environment (CBD)
Nagoya Protocol came into force in 2014 while Cartagena in 2003

www.iasbaba.com

Page 2

IASbaba Prelims 60 Days: [DAY 6]

Q.4) National Biodiversity Authority (NBA) is


1. An autonomous organization
2. A statutory organization
3. A regulatory organization
Select the correct code
a)
b)
c)
d)

1 and 2
Only 2
2 and 3
1, 2 and 3

Q.4) Solution (d)


The Biological Diversity Act, 2002 came into force in 2003. The Act extents to the whole of
India. The objectives of the Act are conservation, sustainable utilization and fair and equitable
sharing of benefits arising out of the use of biological resources and associated knowledge. The
Act is being implemented in a three-tiered institutional structure. The NBA is a body corporate
established in accordance with the provisions of Section 8 of the Biological Diversity Act, 2002,
at Chennai in 2003. It is an autonomous, statutory and regulatory organization which is
intended to implement the provisions of Biological Diversity Act, 2002.
Source- Picked from India Year Book, Chapter on Environment (NBA)

Q.5) Consider the following statements regarding Mangroves


1. India accounts for nearly 10% of worlds mangrove vegetation
2. There has been a net decreases in the mangrove cover of India over last few years
3. Andhra Pradesh has more mangrove cover than Gujarat
Select the correct code
a) Only 1
b) 1 and 2
c) 1, 2 and 3
www.iasbaba.com

Page 3

IASbaba Prelims 60 Days: [DAY 6]


d) None

Q.5) Solution (d)


India accounts for nearly 3 % of worlds mangrove vegetation
There has been a net increase in the mangrove cover of India over last few years (latest
assessment in 2015 has more mangrove cover than assessment of 2013)
Andhra Pradesh (367 km2) has lesser mangrove cover than Gujarat ( 1107 km2)

Source- Picked from India Year Book, Chapter on Environment (Mangroves) and Indian State of
Forest Report, 2015
http://fsi.nic.in/isfr-2015/isfr-2015-mangrove-cover.pdf

Q.6) According to Ramsar Convention definition of Wetland includes


1. Area that is saturated with water permanently
2. Areas of marine water the depth of which at low tide does not exceed eight metres
Select the correct code
a)
b)
c)
d)

Only 1
Only 2
Both
None

Q.6) Solution (a)


Under the Ramsar international wetland conservation treaty, wetlands are defined as follows:
"...wetlands are areas of marsh, fen, peatland or water, whether natural or artificial, permanent
or temporary, with water that is static or flowing, fresh, brackish or salt, including areas of
marine water the depth of which at low tide does not exceed six metres."

www.iasbaba.com

Page 4

IASbaba Prelims 60 Days: [DAY 6]


"[Wetlands] may incorporate riparian and coastal zones adjacent to the wetlands,
and islands or bodies of marine water deeper than six metres at low tide lying within the
wetlands."

Q.7) Consider the following w.r.t Ecotones and Ecoclines


1. an ecocline describes a variation in species prevalence and is often not strictly
dependent on a major physical factor separating an ecosystem from another, with
resulting habitat variability.
2. an ecotone is a variation of the physicochemical environment dependent of one or two
physico-chemical factors of life, and thus presence/absence of certain species.
Select the correct code
a)
b)
c)
d)

Only 1
Only 2
Both
None

Q.7) Solution (d)


Definition is interchanged.

Q.8) Consider the following regarding Keystone Species


1. Keystone species have lower levels of biomass in the trophic pyramid relative to the
importance of their role.
2. Sea Otters are example of keystone species
Select the correct code
a)
b)
c)
d)

Only 1
Only 2
Both
None

www.iasbaba.com

Page 5

IASbaba Prelims 60 Days: [DAY 6]


Q.8) Solution (c)
http://www.thehindubusinessline.com/blink/explore/age-of-extinction/article7433560.ece
https://en.wikipedia.org/wiki/Keystone_species#Examples

Q.9) Consider the following regarding Fly Ash


1. It is used as a fertilizer
2. It can be used in soil stabilization
3. It is used as a replacement for Portland cement in concrete.
Select the correct code
a)
b)
c)
d)

2 and 3
1 and 2
Only 3
1, 2 and 3

Q.9) Solution (d)


All are correct
Source- Reference picked from India Year Book- Environment (Fly Ash Utilization)

Q.10) Electronic waste or e-waste describes discarded electrical or electronic devices.


Consider the following regarding E- waste
1.
2.
3.
4.
5.
6.

Beryllium
Cadmium
Gold
Lithium
Uranium
Silver

Which of the above are found in e-waste?


a) 1, 2, 3, 4 and 5
www.iasbaba.com

Page 6

IASbaba Prelims 60 Days: [DAY 6]


b) 1, 2, 3, 4 and 6
c) 1, 2, 3 and 4
d) 1, 2 and 4

Q.10) Solution (b)


Except Uranium which is radioactive, all of the above are found in e-waste.
The Ministry of Environment, Forest and Climate Change has recently notified the E-Waste
Management Rules, 2016 in supersession of the e-waste (Management & Handling) Rules,
2011.
http://pib.nic.in/newsite/PrintRelease.aspx?relid=138319
http://www.fastcoexist.com/3050542/the-gold-and-silver-and-more-in-our-e-waste-is-worthbillions-of-dollars
http://www.thehindu.com/news/national/other-states/ewaste-can-produce-more-gold-andsilver-than-mining/article8104059.ece

Q.11) Consider the following statements:


1. Eurythermal organisms can tolerate and thrive in wide range of temperature.
2. Warm blooded animals can maintain a constant body temperature even when the
surrounding temperature is changing.
3. Stenothermal organisms are restricted to a small range of temperature.
4. Cold blooded animals generally hibernate to protect themselves from extreme climatic
conditions.
Which of the above statements are correct:
a)
b)
c)
d)

1 and 3
2 and 4
All of the above
None of the above

Q.11) Solution (c)


www.iasbaba.com

Page 7

IASbaba Prelims 60 Days: [DAY 6]


Self explanatory. (Class XII NCERT)

Q.12) In Equatorial rainforest, the undergrowth on the floor is very less. The correct reason
behind this is:
a)
b)
c)
d)

Very less sunlight reaches the ground for survival of plants due to thick canopies.
The hardwood trees excrete toxic resins which do not allow the plants to grow.
Due to high population of herbivorous species.
The soil of rainforest is less fertile due to heavy leaching of minerals.

Q.12) Solution (a)


Equatorial rain forests have thick canopy cover which does not allow the sunlight to reach the
floor. There is stiff competition to between the trees to catch the maximum sunlight. Due to
this there is very little undergrowth.
Q.13) Which of the following statements are correct about life processes of plants.
1. The process of making of food with the help of photosynthesis takes place only during
the day.
2. The process of respiration takes place during day and night both.
Which of the above statements are incorrect:
a)
b)
c)
d)

1 only
2 only
Both 1 and 2
Neither 1 nor 2

Q.13) Solution (a)


The plants prepare their own food by the process of photosynthesis. The process has two
phases, the light reaction and the dark reaction. The light reaction takes place in the presence
of sunlight, while the dark reaction does not require light and continues both during day and
night. The byproduct of photosynthesis is Oxygen gas.
The plants burn their food to produce energy for their survival. This process is respiration. The
plants take in O2 and release CO2. Since light reaction does not take place during night, the
respiration dominates. Thats why it is not wise to sleep under a tree at night as it releases a lot
of Carbon dioxide.

www.iasbaba.com

Page 8

IASbaba Prelims 60 Days: [DAY 6]

Q.14) Consider the following statements:


1. Despite various external conditions, the organisms maintain constant internal
environment like temperature and osmotic pressure. This process is called Homeostasis.
2. During winters the human body shivers to generate heat and maintain the body
temperature of 37oC.
3. During summers the body sweats profusely to maintain the temperature.
4. Plants also have the same mechanism as animals to maintain homeostasis.
Which of the above statements are correct:
a)
b)
c)
d)

1 only
1,2 and 3
1, 3 and 4
All of the above

Q.14) Solution (b)


The organism should try to maintain the constancy of its internal environment (a process called
homeostasis) despite varying external environmental conditions that tend to upset its
homeostasis.
Some organisms are able to maintain homeostasis by physiological (sometimes behavioural
also) means which ensures constant body temperature, constant osmotic concentration, etc. All
birds and mammals, and a very few lower vertebrate and invertebrate species are indeed
capable of such regulation (thermoregulation and osmoregulation). Evolutionary biologists
believe that the success of mammals is largely due to their ability to maintain a constant body
temperature and thrive whether they live in Antarctica or in the Sahara desert.
The mechanisms used by most mammals to regulate their body temperature are similar to the
ones that we humans use. We maintain a constant body temperature of 370C. In summer,
when outside temperature is more than our body temperature, we sweat profusely. The
resulting evaporative cooling, similar to what happens with a desert cooler in operation, brings
down the body temperature. In winter when the temperature is much lower than 370C, we
start to shiver, a kind of exercise which produces heat and raises the body temperature.
Plants, on the other hand, do not have such mechanisms to maintain internal temperatures.

Q.15) Which of the following statements are correct about Keoladeo National Park.

www.iasbaba.com

Page 9

IASbaba Prelims 60 Days: [DAY 6]


1. It is a man-made and man-managed wetland.
2. It is famous for migratory birds which come to visit the park during winters.
3. It is a World Heritage Site.
Select the code from the following:
a)
b)
c)
d)

1 and 2
2 and 3
1 and 3
All of the above

Q.15) Solution (d)


Keoladeo Ghana National Park formerly known as the Bharatpur Bird Sanctuary in Bharatpur,
Rajasthan, India is a famous avifauna sanctuary that hosts thousands of birds, especially during
the winter season. It is a man-made and man-managed wetland and also a World Heritage Site.

Q.16) Consider the following statements:


1. The exchange of air (breathing) in plants takes place only through stomata present in
the leaves.
2. Many desert plants have their stomata in deep pits to minimize water loss by
evaporation.
3. Desert plants have a mechanism that their stomata remain closed during day time.
Which of the above statements are incorrect:
a)
b)
c)
d)

1 and 2
2 and 3
1 and 3
None of the above

Q.16) Solution (a)


Apart from stomata plants and trees also breathe through lenticles, which are small pores
present on their stem.
Many desert plants have their stomata in deep pits to minimize water loss by transpiration and
not evaporation.

www.iasbaba.com

Page 10

IASbaba Prelims 60 Days: [DAY 6]


Q.17) At higher elevations, people generally have breathing problems due to low pressure
and lack of oxygen. This is called altitude sickness. Soon the body acclimatise this situation
and and breathing becomes normal. Which of the following functions does the body take to
adapt to this situation:
1. The body increases red blood cell production.
2. Body decreases the binding capacity of hemoglobin.
3. Body increases the breathing rate.
Select the correct code from the following:
a)
b)
c)
d)

1 only
2 and 3
1 and 3
All of the above

Q.17) Solution (d)


At higher altitude people experience altitude sickness. Its symptoms include nausea, fatigue
and heart palpitations. This is because of low atmospheric pressure, the body doesnt get
enough oxygen. But gradually one gets acclimatized as body compensates low oxygen
availability by producing more red blood cells, decreasing the binding capacity of hemoglobin
and by increasing breathing rate. (NCERT XII Biology)

Q.18) Consider the following statements:


1. Detrivores like earthworm, breakdown detritus into smaller particles. This process is
called fragmentation.
2. Bacterial and fungal enzymes degrade detritus into simpler inorganic substances. This
process is called as catabolism.
Which of the above statements are correct:
a)
b)
c)
d)

1 only
2 only
Both 1 and 2
Neither 1 nor 2

Q.18) Solution (c)

www.iasbaba.com

Page 11

IASbaba Prelims 60 Days: [DAY 6]


Detritivores (e.g., earthworm) break down detritus into smaller particles. This process is called
fragmentation. By the process of leaching, watersoluble inorganic nutrients go down into the
soil horizon and get precipitated as unavailable salts. Bacterial and fungal enzymes degrade
detritus into simpler inorganic substances. This process is called as catabolism.

Q.19) Consider the following statements regarding Humus:


1. Humus is a finely divided partially decomposed organic material in soil, whose origin
cannot be traced by looking at it.
2. Humus is colloidal and helps in binding minerals in the top soil and making them
available to the roots of trees.
3. It is highly resistant to microbial action and decomposes at a very slow rate.
Which of the above statements are correct:
a)
b)
c)
d)

1 and 2
2 and 3
1 and 3
All of the above

Q.19) Solution (d)


Humification leads to accumulation of a dark coloured amorphous substance called humus that
is highly resistant to microbial action and undergoes decomposition at an extremely slow rate.
Being colloidal in nature it serves as a reservoir of nutrients. The humus is further degraded by
some microbes and release of inorganic nutrients occur by the process known as
mineralisation.
Q.20) Consider the following statements regarding energy flow from one tropic level to other
tropic level:
1. The energy flow is always unidirectional, i.e. from lower tropic level to higher tropic
level.
2. The energy transfer in tropic levels follows a 10% law, i.e. only 10 percent energy is
transferred from lower tropic level to the next tropic level.
Which of the above statements are correct:
a)
b)
c)
d)

1 only
2 only
Both 1 and 2
Neither 1 nor 2

www.iasbaba.com

Page 12

IASbaba Prelims 60 Days: [DAY 6]


Q.20) Solution (c)
Refer Biology Class XII NCERT, chapter 14

Q.21) The ordinance making power of president lies with which of the following articles of
Indian constitution

a)
b)
c)
d)

Article 122
Article 123
Article 124
Article 125

Q.21) Solution (b)

Self explanatory

Q.22) Anuradhapura is an ancient Buddhist town found in which of the following country

a)
b)
c)
d)

Thailand
Sri Lanka
Cambodia
Nepal

Q.22) Solution (b)

Anuradhapura is a ancient Buddhist site in Sri Lanka which was famous during the time
of Ashoka. Ashoka sent his emissarys to propagate Buddhism in Sri Lanka. Buddhas
relics can also be found in this place. PM modi visited Anuradhapura when he made a
trip to Sri Lanka last time.

www.iasbaba.com

Page 13

IASbaba Prelims 60 Days: [DAY 6]

Q.23) Consider the following

1. Algae prepare food by the process of photosynthesis


2. Plants can synthesise not only carbohydrates but also proteins and fats
3. All plants are autotrophs

Which of the statements is true with respect to plants and algae

a)
b)
c)
d)

1 and 2 only
2 and 3 only
1 and 3 only
All the above

Q.23) solution (a)

Not all plants are autotrophs. Plants like nephanthesis, picture plants are dependent on
other organisms for food and their survival.

Q.24) Consider the following

1. Copper Anaemia
2. Vitamin B1 Beri Beri
3. Vitamin C Scurvy

Which of the above mineral deficiency and its disorder is rightly matched

a) 1 and 2 only
b) 2 and 3 only
www.iasbaba.com

Page 14

IASbaba Prelims 60 Days: [DAY 6]


c) 1 and 3 only
d) All the above

Q.24) Solution (b)

Anaemia is caused by deficiency of iron and not by copper.

Q.25) Which of the following statements with respect to e Sahayog is correct

a) It is launched by the ministry of heavy industries to facilitate production of capital goods


b) It is launched by ministry of consumer affairs to facilitate new start ups in the area of
supply chain management
c) Launched by ministry of finance to facilitate taxpayers
d) Launched by law ministry to facilitate e filing of cases

Q.25) Solution (c)

www.iasbaba.com

Page 15

IASBABA 60 Day Plan Day 7

Q.1) Consider the following statements with respect to Reusable launch vehicle(RLV)
1. It is co developed by ISRO and DRDO
2. The RLV will be a dummy flight, i.e. The RLV will not be a powered flight

Select the correct option


a)
b)
c)
d)

1 only
2 only
Both
None

Q.1) Solution (b)

Click here

Q.2) Consider the following


1. Gwadar port is the only port in Iran that has the direct access to ocean located in close
proximity to Gulf of Oman
2. The Gwadar port will help India to bypass Pakistan and open up a route to land-locked
Afghanistan and Central Asia
3. With Gwadar port India hopes to compete with the Chinese, who are building Chabahar
port, in Pakistani Baluchistan.
Select the correct answer
a)
b)
c)
d)

1 and 2 only
2 and 3 only
1 and 3 only
None of the above

Q.2) Solution (d)

www.iasbaba.com

Page 1

IASBABA 60 Day Plan Day 7

Gwadar port is in Pakistan and it is developed by Chinese and Chabahar port is in Iran
and it is developed in India.

Q.3) Consider the following statements with respect to Pradhan Mantri Jhan Dhan Yojana
1. The scheme is run by Dept of Financial services Ministry of Finance
2. Anybody above the age of 10 can open the bank account with zero balance
3. The account holder can take loan benefit of Rs 50000 from the bank after six months
from opening the account.

Select the correct answer


a)
b)
c)
d)

1 and 2 only
2 and 3 only
1 and 3 only
All the above

Q.3) Solution (a)

Click here
The account holder can take loan benefit(over draft) of Rs 5000 from the bank after six
months from opening the account.

Q.4) Consider the following statements with respect to Kalaripayattu


1. It is a type of dance form originated in Kerala
2. It got developed during the time of Sangam age.

Select the correct one

a) 1 only
www.iasbaba.com

Page 2

IASBABA 60 Day Plan Day 7

b) 2 only
c) Both
d) None

Q.4) Solution (b)

Kalaripayattu is a martial art form

Q.5)Jagor is a folk dance observed in which of the following states of India


a)
b)
c)
d)

Gujarat
Chattisgarh
Jharkhand
Goa

Q.5) Solution (d)

Jagor, the traditional folk dance-drama, is performed by the Hindu Kunbi and Christian
Gauda community of Goa, to seek the Devine Grace for protection and prosperity of the
crop. Literal meaning of Jagor is jagran or wakeful nights. The strong belief is that the
night long performance awakens the deities once a year and they continue to remain
awake throughout the year guarding the village.

Q.6) Consider the following statements


1. Oligotropic lakes have more oxygen present in the hypolimnion (Bottom layer) than
eutrophic lakes
2. The total number of plants and animal species in eutrophic lakes is more than
oligotropic lakes.

Select the correct answer using the codes given below


a) 1 only
b) 2 only
www.iasbaba.com

Page 3

IASBABA 60 Day Plan Day 7

c) Both
d) None

Q.6) Solution (a)

The total number of plants and animal species in eutrophic lakes is less than oligotropic
lakes cos of lack of oxygen.
Oxygen is required by all respiring plants and animals and it is replenished by
photosynthesis of green plants, as the oxygen levels are already low because of
population explosion and further oxygen taken up by micro organism which feed off the
dead algae during decomposition process.

Q.7) Consider the following statements with respect to National youth policy 2014
1. The National youth policy 2014 replaces National youth policy 2003 to reap the benefits
of Demographic dividend.
2. The National youth policy defines the age of youth from 18 to 29 years.

Select the correct answer using the codes given below.


a)
b)
c)
d)

1 only
2 only
Both
None

Q.7) Solution (a)

The national youth policy defines the age of youth from 15 to 29 years.
President of India mentioned this in a speech Youth and Nation-Building on Jan 19th
2016.

www.iasbaba.com

Page 4

IASBABA 60 Day Plan Day 7

Q.8) Consider the following statements with respect to Conference of Parties


1. It is organised by United Nations framework convention on climate change.
2. 2015 edition of COP was held in Nairobi Kenya.
3. It is conducted annually.

Select the correct answer using the codes given below.

a)
b)
c)
d)

1 and 3 only
2 only
2 and 3 only
3 only

Q.8) Solution (a)

The COP 21st edition was held in Paris.


The WTO ministerial conference was held in Nairobi Kenya.
The conference negotiated the Paris Agreement, a global agreement on the reduction
ofclimate change, the text of which represented a consensus of the representatives of
the 196 parties attending it.

Q.9) Consider the following statements with respect to Sukanya samridhi yojana
1. It was launched as a part of Beti Bachao Beti padhao campaign.
2. The scheme is related to Girls health.
3. The account will remain operative for 21 years from the date of opening or marriage of
the girl child attaining 18 years of age, whichever is earlier.

Select the correct answer using the code given below.


a) 1 only
b) 2 and 3 only
www.iasbaba.com

Page 5

IASBABA 60 Day Plan Day 7

c) 1 and 3 only
d) All the above

Q.9) Solution (c)


Sukanya Samruddi Yojana

The Scheme is a small savings instrument that focuses on making girls financially secure.
It was launched on Jan 2015 by PM as part of Beti Bhacho,Beti Padhao
Sukanya samridhi account can be opened in the name of the girl child any time from her
birth till she attains the age of 10 with minimum deposited of RS 1000.
A maximum deposit of 1,50,000 can be deposited in a financial year and the account
holder girl child will be able to operate the account herself on attaining the age of 10
years.
The account will remain operative for 21 years from the date of opening or marriage of
the girl child after attaining the age 18 years of age, whichever is earlier.
The provision of not allowing withdrawal from the account till the girl child attains the
age of 18 years has been kept to prevent early marriage.

Q.10) Satyameva Jayate is a slogan given by which of the following freedom fighter
a)
b)
c)
d)

Madan Mohan Malviya


Bal Gangadar Tilak.
Mahatma Gandhi
Gopala Krishna Gokale

Q.10) Solution (a)

It was stated by Pandit Madan Mohan Malaviya, the founder of Benares Hindu
University, who went door-to-door to collect donations for this university.
He was a simple living, high thinking politician, notable for his role in the freedom
struggle and his espousal of Hindu nationalism.
Malaviya popularised the famous slogan Satyameva Jayate (Truth alone will win).

www.iasbaba.com

Page 6

IASBABA 60 Day Plan Day 7

Q.11) Consider the following statements:


1. Mughal Emperor Shah Alam was the first to grant the English the exclusive rights to
reside and establish factories in parts of India
2. Battle of Buxar paved the way for English to obtain diwani rights to collect and
manage the revenues
Which of the statements given above is/are correct?
a)
b)
c)
d)

1 only
2 only
Both 1 and 2
Neither 1 nor 2

Q.11) Solution (b)

Mughal Emperor Jahangir was the first to grant the English the exclusive rights to reside and
establish factories in Surat and other areas
The Mughal Emperor, Shah Alam, granted Diwani to the Company after its victory in the
Battle of Buxar (1764).

Q.12) The message, the name Ezhava does not denote a caste or a religion and he made
temple rights to everyone. Therefore people can be admitted to this organization without
paying heed to differences of caste said in relation to Temple Entry Movement, was by
a)
b)
c)
d)

Chattambi Swamikal
T. K. Madhavan
Rajaram Mohan Roy
Narayana Guru

Q.12) Solution (d)

www.iasbaba.com

Page 7

IASBABA 60 Day Plan Day 7

In various parts of South India, particularly Kerala, the disabilities imposed on the avarnas or
people of depressed classes were most inhuman and degrading, against which the struggle
was being waged since the end of the 19th century under the leadership of several
reformers and intellectuals such as Sri Narayana Guru, N. Kumaran Assan and T.K.Madhavan
The temple entry movement used all the techniques developed in the course of the
nationalist struggle. In November 1936, the Maharaja of Travancore issued a proclamation
throwing open all Government controlled temples to all Hindus irrespective of caste.
Sri Narayana Guru gave the message - the name Ezhava does not denote a caste or a
religion and he made temple rights to everyone. Therefore people can be admitted to this
organization without paying heed to differences of caste

Q.13) Which of the following are the establishments by Pandita Ramabai Saraswati?
1. Arya Mahila Samaj in Pune
2. Sharda Sadan in Bombay
3. Brahmo Samaj in Belgaum
Select the correct answer
a)
b)
c)
d)

1 and 2
Only 1
Only 2
All of the above

Q.13) Solution (a)

Pandita Ramabai was an Indian social reformer, a champion for the emancipation of
women, and a pioneer in education. She acquired a reputation as a Sanskrit scholar.
Ramabai founded Arya Mahila Samaj in Pune, which is Sanskrit for "Noble Women's
Society."
The purpose of the society was to promote the cause of women's education and
deliverance from the oppression of child marriage.
She estabished Sharada Sadan in Bombay during a severe famine in 1896. Ramabai toured
the villages of Maharashtra with a caravan of bullock carts and rescued thousands of
outcast children, child widows, orphans, and other destitute women and brought them to
the shelter of Mukti and Sharada Sadan

www.iasbaba.com

Page 8

IASBABA 60 Day Plan Day 7

Q.14) Consider the following statements with regard to Young Bengal group of students:
1. They were influenced by Henry Vivian Derozio and were responsible for the
establishment of the Society for Translating European Sciences
2. They posed an intellectual challenge to the religious and social orthodoxy of Hinduism
3. They had complete faith in everything British and Western learning which alienated
them from the masses
Which of the statements given above is/are correct?
a)
b)
c)
d)

1 and 2 only
1 and 3 only
2 and 3 only
All of the above

Q.14) Solution (c)

Young Bengal, a controversial group of students, influenced by a Eurasian teacher of Hindu


College in Calcutta, were responsible for the establishment of Society for Acquisition of
General Knowledge
This group became infamous for their social rebellion. Their rebellion extended to the
religious sphere and posed an intellectual challenge to the religious and social orthodoxy of
Hinduism
Their very radicalism and complete faith in everything British and Western learning
alienated them from the masses

Q.15) Consider the following statements, in regard to Raja Rammohan Roy


1. His emphasis was on the ethico-religious thought common to all religions of the world.
2. He accepted British rule as a fait accompli (irreversible accomplishment) and believed
that it would work as a regenerative force for the advancement of India.
Select the correct answer from the following options:
www.iasbaba.com

Page 9

IASBABA 60 Day Plan Day 7

a)
b)
c)
d)

Only 1 is correct
Only 2 is correct
Both 1 and 2 are correct
Neither 1 nor 2 is correct

Q.15) Solution (c)

Raja Rammohan Roy believed in the modern scientific approach and principles of human
dignity and social equity. (Rest is self explanatory both statements are true)

Q.16) Which among the following were the characteristics of early nineteenth century socioreligious reformers?
1. These reformers gained from western education but were not West-sponsored
2. The reforms promoted by these reformers were not meant to structurally reorganize
the whole society
3. Issues dealt by these reformers were applicable and common to the higher classes of
the Hindu society and had no meaning for the lower classes
Which of the statements given above is/are correct?
a)
b)
c)
d)

1 and 2 only
1 and 3 only
2 and 3 only
All of the above

Q.16) Solution (d)

The early nineteenth century was marked by the transformation of the Indian society and
the initiative for such reforms came from individuals who revolted against the prescribed
rules of the society.
These reformers though gained from western education were not West-sponsored but they
themselves created the reforms and promoted them.

www.iasbaba.com

Page 10

IASBABA 60 Day Plan Day 7

It is also undeniable that the reform movements had a strong intellectual base which kept
them aloof from the masses and were confined to the educated sections of the society.
The reforms were not meant to structurally reorganize the whole society for the benefit of
the underprivileged or the depressed sections of the society. The aim of the reforms was to
infuse a new life into the existing social structure.
It has been noted by most historians that the issues dealt with by the social reformers in the
early part of the nineteenth century were applicable and common to the higher classes of
the Hindu society and had no meaning for the lower classes.
The movements of the later part of the nineteenth century along with those of the
twentieth century were the ones that reached across to the wider society including the
lower and depressed sections of the society.

Q.17) What was the purpose with which Lord Ripon was sent to India as Viceroy by British
Prime Minister William Gladstone?
a) To reform the structure of the Indian Government and take steps to promote local selfgovernment
b) To facilitate a discussion on Indias Independence in the British Parliament
c) To campaign for the entry of Indians into the Imperial Judiciary
d) To abolish the office of the secretary of state and transfer his functions to the secretary
of state for Commonwealth Affairs
Q.17) Solution (a)

In 1880, there was a change of Government in England and the Liberal Party under William
Gladstone came into power. He was known for his liberal ideas and was a believer in moral
principles.
He sent Lord Ripon, a close friend and follower, to reform the structure of the Indian
Government. He repealed the Vernacular Press Act of 1878, commenced industrial
legislation by passing the first Factories Act and took steps to promote local selfgovernment in big cities and towns.
Lord Ripon is known to have granted the Indians first taste of freedom by introducing the
Local Self Government in 1882.

www.iasbaba.com

Page 11

IASBABA 60 Day Plan Day 7

Q.18) Which of the following are the ones associated with Dadabhai Naoroji?
1.
2.
3.
4.

Gyan Prasarak Mandali


Poverty and un-British Rule in India
Bombay Association
Bombay Gazette

Select the correct answer from the codes given below:


a)
b)
c)
d)

1 and 2
1, 2 and 4
1, 2 and 3
2 and 3

Q.18) Solution (c)

Bombay Herald was the first English newspaper in Bombay. It was started in 1789. Bombay
Courier was started in the following year 1790. In 1791, the name of Bombay Herald was
changed to Bombay Gazette. Bombay Courier was published on Saturdays and Gazette on
Wednesdays.
Bombay Courier continued to exist for fifty-six years, after which it was merged with
Bombay Telegraph. Bombay Gazette stopped its operation in 1842.
Both these papers contained almost entirely parts from English papers and occasional law
reports. The first native newspaper in Bombay was Bombay Samachar in 1822. This paper
was started as a weekly. It became a bi-weekly in 1833 and daily in 1860. Mumbai Karkman
was started in 1830.
The Bombay Samachar is the oldest continuously published newspaper in India. Established
in 1822, it is published in Gujarati and English

Q.19) The Deoband School, a revivalist school of 19th century, has been started with the
objective
www.iasbaba.com

Page 12

IASBABA 60 Day Plan Day 7

1. To extend contacts with Muslims and organize them in accordance with original
principles of Islam.
2. To keep alive the spirit of jihad against the foreign rulers.
Select the correct answer from the following options:
a)
b)
c)
d)

Only 1 is correct
Only 2 is correct
Both 1 and 2 are correct
Neither 1 nor 2 is correct

Q.19) Solution (c)

The objectives of the Deoband School To propagate among the Muslims the pure teachings of the Koran and the Hadis (original
principles of Islam)
To exalt the word God, unaffected by any temptation, patronage, pressure or favour
To follow traditional educational curriculum prescribed by Dar-esi- Nizami.
Hindus and Muslims belong to one nation. They were convinced that the Muslims could,
jointly with the Hindus, create a state in which both could live honourably and happily.

Q.20) Consider the following statements:


1. The Wahabi Movement named after its founder Abdul Wahab originated with a view of
restoring Islam to its pristine purity and order.
2. In India, the Wahabi movement was started by Vilayet Ali.
3. It was a revivalist movement which held that the return to the true spirit of Islam was
the only way to get rid of the socio- political oppression.
4. It offered a serious threat to the British supremacy in India.
Select the correct answer from the following:
a) 1, 3 and 4
www.iasbaba.com

Page 13

IASBABA 60 Day Plan Day 7

b) 1, 2 and 3
c) Only 1
d) All are correct

Q.20) Solution (a)

The Wahabi Movement was a part of the Indian freedom struggle as it offered a serious
threat to British supremacy in India in the 19th century.
The movement was led by Syed Ahmed Barelvi (not Vilayet Ali), who was greatly influenced
by the teaching of Abdul Wahab of Arabia and the preaching of Delhi saint Shah Walliullah.
The Wahabi Movement essentially condemned all changes and innovations to Islam. It was
a revivalist movement which held that the return to the true spirit of Islam was the only way
to get rid of the socio- political oppression.
In 1847, the Wahabis started full preparation for an absolute war against the British rule in
India from their base camp in Sitana. Vilayet Ali became the undisputed leader.
The British took up brutal measures and the period between 1863-65, witnessed a series of
trials by which all the principal leaders of the Wahabi movement were arrested.

Q.21) Consider the following statements regarding Permanent Settlement introduced by


British Government
1. The zamindars of Bengal were not recognised as the owners of land
2. The amount of revenue that the zamindars had to pay to the Company was firmly fixed
and would not be raised under any circumstances.
3. The ryots became tenants since they were considered the tillers of the soil.
4. This settlement took away the administrative and judicial functions of the zamindars.
Which of the above were the main features of Permanent Settlement?
a) 1, 3 and 4
b) 2, 3 and 4
c) 1, 2 and 3
www.iasbaba.com

Page 14

IASBABA 60 Day Plan Day 7

d) 1, 2, 3 and 4

Q.21) Solution (b)


Cornwallis at the time of his appointment was instructed by the Directors to find a satisfactory
and permanent solution to the problems of the land revenue system in order to protect the
interests of both the Company and the cultivators. It obliged the Governor- General to make a
thorough enquiry into the usages, tenures and rents prevalent in Bengal. The whole problem
occupied Lord Cornwallis for over three years and after a prolonged discussion with his
colleagues like Sir John Shore and James Grant he decided to abolish the annual lease system
and introduce a decennial (Ten years) settlement which was subsequently declared to be
continuous. The main features of the Permanent Settlement were as follows:

The zamindars of Bengal were recognised as the owners of land as long as they paid the
revenue to the East India Company regularly.

The amount of revenue that the zamindars had to pay to the Company was firmly fixed
and would not be raised under any circumstances.

The ryots became tenants since they were considered the tillers of the soil.

This settlement took away the administrative and judicial functions of the zamindars.

Q.22) Consider the following


1. Permanent Settlement
2. Ryotwari System
3. Mahalwari System
Which of the above removed concept of middle men?
a) 1 and 3
b) 1 and 2
c) Only 2
d) 2 and 3
www.iasbaba.com

Page 15

IASBABA 60 Day Plan Day 7

Q.22) Solution (d)

The Ryotwari settlement was introduced mainly in Madras, Berar, Bombay and Assam. Sir
Thomas Munro introduced this system in the Madras Presidency. Under this settlement, the
peasant was recognised as the proprietor of land. There was no intermediary like a Zamindar
between the peasant and the government.

In 1833, the Mahalwari settlement was introduced in the Punjab, the Central Provinces and
parts of North Western Provinces. Under this system the basic unit of revenue settlement was
the village or the Mahal. As the village lands belonged jointly to the village community, the
responsibility of paying the revenue rested with the entire Mahal or the village community. So
the entire land of the village was measured at the time of fixing the revenue. Though the
Mahalwari system eliminated middlemen between the government and the village community
and brought about improvement in irrigation facility, yet its benefit was largely enjoyed by the
government.

Q.23) Which among the following was one of the causes for the Vellore Mutiny?
a) Doctrine of lapse
b) Collection of Tributes
c) Introduction of new army regulations
d) Economic exploitation of the British rule.

Q.23) Solution (c)


www.iasbaba.com

Page 16

IASBABA 60 Day Plan Day 7

Several causes are attributed to the Vellore Mutiny. Indian sepoys had to experience numerous
difficulties when they went to serve in the Companys army. The sepoys were forced to serve
under the Company since their earlier patrons (the native chieftains) were all disappearing from
the scene. The strict discipline, practice, new weapons, new methods and uniforms were all
new to the sepoys. Anything new appears to be difficult and wrong for a man who is wellsettled in the old way of life for a long-time.
Sir John Cradock, the commander-in-chief, with the approval of Lord-William Bentinck, the
Governor of Madras, introduced a new form of turban, resembling a European hat. Wearing ear
rings and caste marks were also prohibited. The sepoys were asked to shave the chin and to
trim the moustache. The sepoys felt that these were designed to insult them and their religious
and social traditions. There was also a popular belief that this was the beginning of a process by
which all of them would be converted to Christianity.

Q.24) Consider the following statements regarding The Regulating Act of 1773
1. The Governor General became very powerful
2. The act put an end to the arbitrary rule of the Company
3. This Act promoted the servants of the Company including the Governor-General,
members of his council and the judges of the Supreme Court from receiving directly or
indirectly any gifts in kind or cash
Select the incorrect code
a) 1 and 2
b) 1 and 3
c) 2 and 3
d) 1, 2 and 3

Q.24) Solution (b)

www.iasbaba.com

Page 17

IASBABA 60 Day Plan Day 7

Merits and Demerits of the Act


The significance of the Regulating Act is that it brought the affairs of the Company under the
control of the Parliament. Besides, it proved that the Parliament of England was concerned
about the welfare of Indians. The greatest merit of this Act is that it put an end to the arbitrary
rule of the Company and provided a framework for all future enactments relating to the
governing of India. The main defect of the Act was that the Governor-General was made
powerless because the council which was given supreme power often created deadlocks by
over-ruling his decision. However, many of these defects were rectified by the Pitts India Act of
1784.
This Act prevented the servants of the Company including the Governor-General, members of
his council and the judges of the Supreme Court from receiving directly or indirectly any gifts in
kind or cash.

Q.25) Match the following


1. Tripartite Alliance

a. 1784

2. Treaty of Salbai

b. 1782

3. Treaty of Srirangapattinam

c. 1786

4. Treaty of Mangalore

d. 1783

Select the correct match


a) 1, 2, 3
b) 1, 3 and 4
c) 2 and 4
d) 1 and 3

Q.25) Solution (c)


Tripartite Alliance
www.iasbaba.com

a. 1783
Page 18

IASBABA 60 Day Plan Day 7

Treaty of Salbai

b. 1782

Treaty of Srirangapattinam

c. 1786

Treaty of Mangalore

d. 1784

In 1789, the British concluded a tripartite alliance with the Nizam and the Marathas against
Tipu.
1782, the Treaty of Salbai was signed between Warren Hastings and Mahadaji Scindia.
The Treaty of Seringapatam (also called Srirangapatinam), signed 18 March 1792, ended
the Third Anglo-Mysore War. Its signatories included Lord Cornwallis on behalf of the British
East India Company, representatives of the Nizam of Hyderabad and the Mahratta Empire,
and Tipu Sultan, the ruler of Mysore.
The Treaty of Mangalore was signed between Tipu Sultan and the British East India
Company on 11 March 1784. It was signed in Mangalore and brought an end to the Second
Anglo-Mysore War.

www.iasbaba.com

Page 19

IASbaba Prelims 60 Days [Day 8]


Q.1) Kalileh-wa-Dimneh is a Persian translation of which of the following Indian works
a)
b)
c)
d)

Mahabharata
Ramayana
Panchtantra
Bhagavatgeeta

Q.1) Solution (c)

Kalileh-wa-Dimneh is a Persian translation of Panchatantra. PM Modi mentioned this


at a speech when he visited Tehran two days back.
Click here

Q.2) Consider the following


1. Diesel vehicles emit more carbon monoxide then petrol cars.
2. Diesel cars emit more nitrogen oxide air pollutants than petrol cars
Which of the following is/are the prime reason for banning of diesel cars?
a)
b)
c)
d)

1 only
2 only
Both
None

Q.2) Solution (b)


Diesel cars produce more nitrogen oxide air pollutants than petrol cars. The physical and
chemical conditions that exist inside any suchdiesel engines under any conditions differ
considerably from spark-ignition engines, because, by design, diesel engine power is not
controlled by the air/fuel mixture (as in most gasoline engines), but rather it is directly
controlled by the fuel supply. For instance, diesel engines generally produce 28 times less
carbon monoxide than gasoline engines, as diesels burn their fuel in excess air even at full
load. However, the lean-burning nature of diesel engines and the high temperatures and
pressures of the combustion process result in significant production of gaseous nitrogen
oxides (NOx), an air pollutant that constitutes a unique challenge with regard to their
reduction.[not verified in body] Total nitrogen oxides from petrol cars have decreased by
around 96% through adoption of exhaust catalytic converters as of 2012, while diesel cars
still produce nitrogen oxides at a similar level to those bought a decade and a half ago under
real world tests; hence, diesel cars emit around 20 times more nitrogen oxides than petrol
cars.

IASbaba Prelims 60 Days [Day 8]

Q.3) The conservation of Galapagos Islands was in news recently. Consider the following
statements with respect to Galapagos Islands
1. Galapagos Islands are located off coast of Peru
2. It is a UNESCO world heritage site
3. Galapagos Islands passes through Equator
Select the correct option
a)
b)
c)
d)

1 and 2 only
2 and 3 only
1 and 3 only
All

Q.3) Solution (b)

It is located off coast of Ecuador and it passes through equator. It was recognised as
the world heritage site in 1979.

Q.4) Recently a Cancer-causing chemical found was in bread samples from Delhi has
alarmed the authorities. Consider the following statements
1. Potassium bromate / Iodate are carcinogenic chemicals that are used while making
confectionary items.
2. Potassium Iodate/Bromate was found to cause tumour of the kidney and thyroid,
and cancer of the abdominal lining in laboratory animals.
Select the correct option
a)
b)
c)
d)

1 only
2 only
Both
None

Q.4) Solution (c)

Click here

IASbaba Prelims 60 Days [Day 8]

Q.5) Consider the following


1.
2.
3.
4.

Mahanadi and Godavari


Krishna and Pennar
Ken and Betwa
Narmada and Krishna

Which of the following are the proposed inter basin water transfer links
a)
b)
c)
d)

2, 3 and 4 only
1, 3, and 4 only
1, 2 and 3 only
All

Q.5) Solution (c)

Click here

Q.6) Consider the following statements regarding Indias Foreign Trade Policy 2015-20
1. It aims at increasing Indias exports in goods and services to 900 billion by 2019-20.
2. Previous reward schemes of goods and services for export have been merged into
two schemes- Merchandise Export from India Scheme (MEIS) and Service Export
from India Scheme (SEIS).
3. It seeks to raise Indias share in World exports from 2% to 3.5%
Select the correct option
a) 1 only
b) 2 only
c) 1 and 2 only
d) All
Q.6) Solution (d)

Click here

Q.7) Map of Africa

IASbaba Prelims 60 Days [Day 8]

Identify the countries located in the Map


a)
b)
c)
d)

1-Niger ,2-Somalia, 3-Burkino Faso ,4-Alegeria


1-Somalia ,2-Niger, 3-Algeria, 4-Burkino Faso
1-Algeria, 2-Niger,3-Somalia,4-Burkino Faso
1-Burkino Faso,2-Somalia,3-Algeria,4-Niger

Q.7) Solution (c)

Refer Atlas

Q.8) Consider the following statements regarding Amaravathi.


1. It was the capital of Satavahanas who ruled from 2nd century BCE to 3rd century CE
2. It is located on the banks of river Godavari.
3. Amaravati has been chosen as one of the sites for Heritage City Development and
Augmentation Yojana (HRIDAY) scheme of Government of India.
Choose the correct answer using the codes given below
a) 1 and 2 only
b) 1 and 3 only

IASbaba Prelims 60 Days [Day 8]


c) 2 and 3 only
d) All the above
Q.8) Solution (b)

Amaravathi is an important historic town, and served as the capital of the


Satavahana kingdom in ancient days.

Sri Amaralingeswara Swamy temple is located at the Amararama Pancharama


Kshetra site in this village, which makes it a holy town for the Hindus. It is also a
historic Buddhist site, and the Amaravati Mahachaitya stupa was built here between
the 2nd century BCE and the 3rd century CE.

Designated as a historic site by the Government of India, Amaravati has been chosen
as one of the sites for Heritage City Development and Augmentation Yojana
(HRIDAY) scheme of Government of India.

It is located on the banks of river Krishna.

Q.9) As per Ease of Doing Business Rank 2015, India ranked 130 out of 189 countries.
Which of the following parameters are used in calculating ease of doing business rank?
1.
2.
3.
4.
5.

Starting a business
Getting electricity
Trading across borders
Getting credit
Protecting minority investors

Select the correct answer using the code given below.


a)
b)
c)
d)

1, 2, 3 and 4 only
1, 3 and 4 only
1, 4 and 5 only
All the above

Q.9) Solution (d)

Starting a business
Dealing with construction permits
Getting electricity
Registering property

IASbaba Prelims 60 Days [Day 8]

Getting credit
Protecting minority investors
Paying taxes
Trading across borders
Enforcing contracts

10) Match list 1 with list 2 and choose the correct answer using the codes given below
1.
2.
3.
4.

BrahMosAkashPrithviNAG-

A. Anti tank guided missile


B. Stealth cruise missile
C. Surface to air missile
D. Short range ballistic missile

Code

a)
b)
c)
d)

B
A
B
D

A
B
C
B

C
D
D
C

D
C
A
A

Q.10) Solution (C)

Self explanatory

Q.11) Consider the following statements


1. Lord Mayo became the first Viceroy of India
2. He gave the policy of financial decentralization
3. The first ever Census conducted in India was under his patronage
Select the correct code
a)
b)
c)
d)

Only 1
1 and 2
2 and 3
None

IASbaba Prelims 60 Days [Day 8]


Q.11) Solution (c)
Richard Southwell Bourke or Lord Mayo was the fourth Viceroy of India who held office
from 1869-1872. The administrative policies of Lord Mayo have further received
prominence with regards to the first ever Census that was conducted in India in 1871 under
his patronage. He was also instrumental in arranging a Statistical Survey of India during his
tenure. The Resolution of 1870 or the policy of financial decentralization promulgated by
Lord Mayo is a glorious example of his proficiency in the execution of economic reforms.

Q.12) Identify the personality based on given statements


1. He believed that self-government is the highest and noblest principles of politics.
2. He abolished Vernacular Press Act
3. Hunter commission was appointed under his rule
Select the correct option
a)
b)
c)
d)

Lord Canning
Lord Rippon
Lord Mayo
Lord Lytton

Q.12) Solution (b)


Lord Ripon was a staunch Liberal democrat with faith in self-government.He was appointed
as the Viceroy of India by Gladstone, the Liberal Party Prime Minister of England. Ripon was
instructed to reverse the Afghan policy of Lytton.Therefore, as soon as he came to India,
peace wasmade with Afghanistan without affecting the Britishprestige. The proposal of
appointing a Resident in Kabul was dropped. He was also responsible for the rendition of
Mysore to its Hindu ruler. Moreover, he repealed the Vernacular Press Act and earned much
popularity among Indians. Then, he devoted himself to task of liberalising the Indian
administration.
Like Lord William Bentinck, Lord Ripon was a champion ofeducation of the Indians. Ripon
wanted to review the working of the educational system on the basis of the
recommendations of the Woods Despatch. For further improvement of the system Ripon
appointed a Commission in 1882 under the chairmanship of Sir William Hunter.
Lord Ripon was the most popular Viceroy that England ever sentto India. The Indians by and
large hailed him as Ripon the Good, because he was the only Viceroy who handled the

IASbaba Prelims 60 Days [Day 8]


Indian problems with compassion and sympathy. His attempt to remove racial distinction in
the judiciary, the repeal of the Vernacular Press Act, the rendition of Mysore and the
introduction of the Local-Self Government increased his popularity among Indians. His
resignation was deeply regretted by Indians who cherished his memory with gratitude.

Q.13) Identify the personality associated with the institution of the following
act/regulations.
1.
2.
3.
4.

Indian Universities Act, 1904


Ancient Monuments Act, 1904
Calcutta Corporation Act, 1899
Police Commission in 1902

Select the correct code


a)
b)
c)
d)

Warren Hasting
Lord Wellesley
LordDalhousie
Lord Curzon

Q.13) Solution (d)

Q.14) Which of the following were formed before 1857 Revolt?


1.
2.
3.
4.
5.
6.

The British Indian Association


The Bombay Association
East India Association
Madras Native Association
Poona Sarvojanik Sabha
The Madras Mahajana Sabha

Select the correct code


a)
b)
c)
d)

1, 2, 3, 5 and 6
2, 3, 4, 5 and 6
1, 2, 3 and 4
1, 3, 4 and 5

Q.14) Solution (c)

IASbaba Prelims 60 Days [Day 8]

The British Indian Association 1851 Bengal

The Bombay Association 1852 DadabhaiNaoroji

East India Association 1856 London

Madras Native Association 1852

Poona Sarvojanik Sabha1870

The Madras Mahajana Sabha1884

Q.15) The Indian Factory Act, 1881 dealt primarily with the problem of child labour. Its
significant provisions were:
1. Increased the minimum age (from 7 to 9 years) and the maximum (from 12 to 14
years) for children,
2. Reduced maximum working hours for children to 7 hours a day,
3. Fixed maximum working hours for women at 11 hours per day with an one-and-ahalf hour interval (working hours for men were left unregulated),
4. Provided weekly holiday for all
Select the correct code
a)
b)
c)
d)

1, 2, 3 and 4
2 and 3 Only
1, 2 and 3
None

Q.15) Solution (d)


These were the provisions of The Indian Factory Act, 1891.
The Indian Factory Act, 1881 dealt primarily with the problem of child labour (between 7
and 12 years of age).
Its significant provisions were:
I.
II.
III.
IV.

Employment of children under 7 years of age


prohibited,
Working hours restricted to 9 hours per day for
children,
Children to get four holidays in a month,
Hazardous machinery to be properly fenced off.

IASbaba Prelims 60 Days [Day 8]


Q.16) Which among the following statements were associated with Aligarh Movement?
1) It was the foremost movement for social reforms among the Indian Muslims
2) The aim of the movement was to organize the Muslim peasants against Hindu
landlords and British indigo planters
3) All-India Muslim League was the outcome of this movement
4) The movement offered a serious threat to the British supremacy in India
Choose the appropriate answer
a)
b)
c)
d)

1 and 2 only
1 and 3 only
1, 2 and 4 only
All the above

Q.16) Solution (b)

The Aligarh Movement of Sir Syed Ahmad Khan was the foremost movement for social
reforms among the Indian Muslims.

Aligarh Movement was a cultural movement with the objective of regeneration of


liberal values in literature, social life, education and religion. It main objective were the
popularization of education and rationalization of religious tenets.

It was Titu Mirs Movement (not Aligarh Movement) who organized the Muslim
peasants against Hindu landlords and British indigo planters

The objective of the Aligarh movement was to ensure continued British support for its
programmes. It assiduously supported all government legislation with a view to
stamping out the stigma of disloyalty with which the entire Muslim community had been
branded since the Rebellion of 1857 (therefore, the movement did not offer any serious
threat to the British supremacy in India)

All-India Muslim League was the outcome of the Aligarh movement, so were many leaders
of the League and the community
Q.17) Bankim Chandra Chatterjees work Anandmath, one of the most important novels in
the history of Bengali and Indian literature, was set in the background of
a)
b)
c)
d)

Ramosi Uprising
Santhal Rebellion
Sannyasi-Faqir Rebellion
Sawantwadi Revolt

Q.17) Solution (c)

IASbaba Prelims 60 Days [Day 8]

Anandamath is a Bengali novel, written by Bankim Chandra Chattopadhyay and


published in 1882.

It is considered one of the most important novels in the history of Bengali and Indian
literature whose work was set in background of the cause of Faqir-Sannyasis Rebellion

Its importance is heightened by the fact that it became synonymous with the struggle
for Indian independence from the British Empire. The novel was banned by the British.
The ban was lifted later by the Government of India after independence.
The national song of India, Vande Mataram, was first published in this novel.

Q.18) Which one of the following observations is true about the Polygar Rebellions?
1) Polygars were the offshoots of the Nayankara system prevalent in the Vijayanagar
administration
2) Polygars were traditional classes, quite similar to Rajputs of North India, often acted
as sovereigns and extracted taxes from the people
3) First Polygar war was lead by a confederacy consisting of Marudu Pandian, Gopal
Nayak, Kerala Verma and Dhoondaji
4) Kattabomma Nayak was considered as the main leader of the rebellion during
Secong Polygar war, also known as South Indian Rebellion
Choose the appropriate answer
a)
b)
c)
d)

1 and 2 only
1, 2 and 3 only
1, 3 and 4 only
All the above

Q.18) Solution (a)

Polygars were the offshoots of the Nayankara system prevalent in the Vijayanagar
administration. The Polygars were quite similar to the Rajputs of North India and
were given land in exchange for military service when called upon.

However, their influence and power increased beyond the traditional lines and they
often acted as sovereigns, even to extent of extracting taxes from the people.

But as the Companys Government wanted to augment its own sources of revenues,
it sought to control the Polygars.

Kattabomma Nayak was considered as the main leader of the rebellion during the
First Polygar war (not Second Polygar War). He was publicly hanged by the British in
front of other polygars as a warning.

The Secong Polygar war, also known as South Indian Rebellion, was lead by a

IASbaba Prelims 60 Days [Day 8]

confederacy consisting of Marudu Pandian of Shivaganaga, Gopal Nayak of Dundigal,


Kerala Verma of Malabar and Krishnappa Nayak and Dhoondaji of Mysore.

The British succeeded in suppressing the rebellion and was followed by signing of the
Carnatic Treaty, whereby the British assumed direct control over Tamil Nadu.

The Polygar system, which had flourished for two and a half centuries, came to
violent end and the Company introduced the Zamindari settlement in its place.

Q.19) What were the causes that triggered tribal movements?


1)
2)
3)
4)
5)

Imposition of land revenue settlement


Expansion of non-tribals to tribal areas
Eminent Domain
Work of Christian missionaries
Reactions against Dikus

Choose the appropriate answer


a)
b)
c)
d)

1, 2 and 5 only
1, 2, 4 and 5 only
1, 3 and 4 only
All the above

Q.19) Solution (d)


Causes of Tribal Movements

Imposition of land revenue settlement, expansion of agriculture by the non-tribals to


tribal areas or over forest covers led to erosion of the tribal traditions of joint ownership
and increased socio-economic differentiation in the egalitarian structure of the tribal
society.

Work of Christian missionaries brought about further changes in the socio-economic and
cultural equation of the tribals and the mainstream society. Plus, in turbulent times, the
tendency of the missionaries to refuse to take up arms or in discouraging people from
rising against the Government made the missionaries to be viewed as extensions of
colonialism and were often attacked by the rebels.
The word dikus means outsiders or who come from outside like moneylenders,
traders, zamindars, contractors, British etc. There are a number of reasons for anger of
the tribals against the dikus:
The tribals practiced shifting cultivation but the British forced them to follow settled
agriculture and also introduced land settlements.
Traders and moneylenders were coming into the forest, wanting to buy forest
produce at a very cheap rate, luring them to take cash loans at high interests etc. The
innocent and poor people initially fell in the trap of these moneylenders and traders and

IASbaba Prelims 60 Days [Day 8]


remained indebted throughout their lives. So the tribals considered the traders,
moneylenders as evil outsiders.
Under British rule the tribal chiefs lost their authorities they had enjoyed earlier
amongst their people, were unable to fulfill their traditional functions. Rather they had
to pay tribute to the British.
By the introduction of forest laws, the British evacuated them from their own lands.
As a result they became homeless and went in search of work and livelihood.
Q.20) Match the following (Part I) with (Part II) from the codes given below
Part I
(Revolt/Movement)
A. Pagal Panthis

Part II
(Leader associated with it)
1. Karam Shah

B. Kuka Revolt

2. Bhagwat Jawar Mal

C. Sawantwadi

3. Anna Sahib

D. Ramosi

4. Chittur Singh

A-B-C-D
a) 3-2-1-4
b) 2-4-1-3
c) 2-1-3-4
d) 1-2-3-4

Q.20) Solution (d)

Self-explanatory

Q.21) Consider the following with regard to Telegraph in India:


1) The first director general of telegraphs in India was William Brooke O'Shaughnessy
2) Lord Dalhousie had authorised him to construct an experimental line and in 1852 he
was appointed director general of telegraphs
3) It was "telegraph" that saved India in mutiny of 1857
4) Telegraph communication between India and England was opened in 1865 by the
Persian Gulf line
Which of the statements given above is/are correct?

IASbaba Prelims 60 Days [Day 8]


a)
b)
c)
d)

1 and 2 only
1, 2 and 3 only
1, 3 and 4 only
All the above

Q.21) Solution (d)


Telegraphs began in India in 1854

The first director general of telegraphs in India was William Brooke O'Shaughnessy. Lord
Dalhousie had authorised him to construct an experimental line and in 1852 he was
appointed director general of telegraphs and authorised to construct an extensive
system.

The 800-mile line from Calcutta to Agra was opened in March, 1854, and two years later
4000 miles were in operation, including lines to Bombay and Madras.

It was "telegraph" that saved India in mutiny of 1857. Telegraph communication


between India and England was opened in 1865 by the Persian Gulf line

Q.22) What were the causes for failure of Revolt of 1857?


1)
2)
3)
4)

Lack of complete nationalism


Lack of coordination between sepoys, peasants and zamindars and other classes
Disunity of Indians and poor organisation especially between Hindu and Muslim
Many had different motives for participating in the revolt

Choose the appropriate answer


a)
b)
c)
d)

2, 3 and 4 only
1, 2 and 3 only
1, 2 and 4 only
All the above

Q.22) Solution (c)

The Revolt of 1857 was an unsuccessful but heroic effort to eliminate foreign rule. The
main causes of failure were:

Disunity of Indians and poor organisation, lack of complete nationalism Scindias,


Holkars, Nizams and others actively helped the British, lack of coordination between
sepoys, peasants and zamindars and other classes and many had different motives for
participating in the revolt

The important element in the Revolt lay in Hindu-Muslim unity. People exhibited
patriotic sentiment without any touch of communal feelings. (therefore, statement 3 is

IASbaba Prelims 60 Days [Day 8]


wrong)

Q.23) Who among the following was the founder of the organisation - Indian Home Rule
Society (India House), London established in 1904?
a)
b)
c)
d)

Veer Savarkar
Tarak Nath Das
Shyamji Krishna Verma
Lala Hardayal

Q.23) Solution (c)


Shyamji Krishna Verma
Shyamji Krishna Verma, an Indian revolutionary fighter, was the founder of Indian
Home Rule Society, India House and The Indian Sociologist in London.

In 1905 he founded the India House and The Indian Sociologist, which rapidly developed
as an organised meeting point for radical nationalists among Indian students in Britain at
the time and one of the most prominent centres for revolutionary Indian nationalism
outside India. Most famous among the members
of this organisation was Veer Savarkar.

Krishna Varma moved to Paris in 1907, avoiding prosecution. He died in 1930.


Vikram Damodar Savarkar
V D Savarkar or Veer Savarkar (commonly known as Swatantryaveer Savarkar) was an
Indian pro-independence activist.

Savarkar's revolutionary activities began while studying in India and England, where he
was associated with the India House and founded student societies including Abhinav
Bharat Society and the Free India Society.

Taraknath Das

Taraknath Das was an anti-British Bengali Indian revolutionary and internationalist


scholar. He was a pioneering immigrant in the west coast of North America and
discussed his plans with Tolstoy, while organising the Asian Indian immigrants in favour
of the Indian freedom movement.

With Panduranga Khankoje (B.G. Tilak's emissary), Tarak founded the Indian
Independence League.

Lala Har Dayal

IASbaba Prelims 60 Days [Day 8]

Lala Har Dayal was an Indian nationalist revolutionary who founded the Ghadar Party in
America.

He was a polymath who turned down a career in the Indian Civil Service. His simple
living and intellectual acumen inspired many expatriate Indians living in Canada and the
USA to fight against British Imperialism during the First World War.

Q.24) Which among the following characterizes the important events/works during
Viceroy Lord Curzon period?
1)
2)
3)
4)

Swadeshi movement
Foundation of Muslim League
Idea to build Victoria Memorial
Indian Coinage and Paper Currency Act

Choose the appropriate answer


a)
b)
c)
d)

2 and 4 only
3 and 4 only
1, 3 and 4 only
All the above

Q.24) Solution (b)


Viceroy Lord Curzon (1899-1905)
appointed Police Commission in 1902 under Andrew Frazer;

Set up the Universities Commission and accordingly the Indian Universities Act of 1904
was passed;

Set up of Department of Commerce and Industry;

Passed the Indian Coinage and Paper Currency Act (in 1899) and put India on a global
standard;

Partition of Bengal took place in 1905 (It was


cardinal blunder of Curzon);
The idea to build Victoria Memorial (Calcutta) was conceived by Lord Curzon
Swadeshi Movement (1905-08) and Foundation of Muslim League(1906) was during Lord
Minto-II period (not Lord Curzon)

Q.25) Who among the following Viceroys introduced financial decentralization in India?
He was also known for introduction of State Railways.
a) Lord Lytton
b) Lord Minto

IASbaba Prelims 60 Days [Day 8]


c) Lord Mayo
d) Lord Hardinge
Q.25) Solution (c)
Lord Mayo (1869-72) Introduced financial decentralization in India; Established Mayo
College at Ajmer for the Princes; Organised the Statistical Survey of India; Established the
department of Agriculture and Commerce, Introduction of State Railways

IASbaba Prelims 60 Days: [Day 9]

Q.1) Which among the following is/are relative merits of GDP?


1. It enables central banks and policymakers to evaluate whether the economy is in recession
or inflation
2. GDP is available for practically every country in the world and allows crude comparisons
between the standard of living in different countries
3. GDP has also held significance as a universal metric, as it includes voluntary market
transactions
Choose the correct code
a)
b)
c)
d)

1 and 2 only
2 only
2 and 3 only
1, 2 and 3

Q.1) Solution (a)


Explanation:
Relative merits of GDP includes:

GDP growth over time enables central banks and policymakers to evaluate whether the
economy is in recession or inflation.
GDP has also held significance as a universal metric over the years, to measure the
economic performance of a nation.
GDP excludes (not includes) voluntary market transactions: GDP fails to account for
productive non-market activities, like a mother taking care of her child, a homemaker doing
household chores, a homeowner doing maintenance of his house, leisure (paid vacation,
holidays, leave time), improvement in product quality, etc.

Q.2) Macroeconomics sees an economy as a combination of 4 sectors. They are


a)
b)
c)
d)

Land, labour, capital, entrepreneur


Households, firms, government, external sector
Consumer, entrepreneur, firms, government
Consumer, firms, government, external sector

www.iasbaba.com

Page 1

IASbaba Prelims 60 Days: [Day 9]

Q.2) Solution (b)


Explanation:
Macroeconomics deals with the aggregate economic variables of an economy. It also takes
into account various interlinkages which may exist between the different sectors of an
economy. This is what distinguishes it from microeconomics; which mostly examines the
functioning of the particular sectors of the economy, assuming that the rest of the economy
remains the same.

Macroeconomics emerged as a separate subject in the 1930s due to Keynes.


Macroeconomics sees an economy as a combination of four sectors, namely households,
firms, government and external sector.

Q.3) Consider the following statements about Nominal and Real GDP:
1. Nominal GDP refers to the current year production of financial goods and services
valued at base year prices
2. Real GDP refers to the current year production of financial goods and services valued at
current year prices
Which of the above statements is/are not correct?
a)
b)
c)
d)

Only 1
Only 2
Both 1 and 2
Neither 1 nor 2

Q.3) Solution (c)


Explanation:
Nominal GDP refers to the current year production of financial goods and services valued at
current year prices
Real GDP refers to the current year production of financial goods and services valued at
base year prices
Q.4) Consider the following statements with regard to GDP and GNP:

www.iasbaba.com

Page 2

IASbaba Prelims 60 Days: [Day 9]

1. Gross Domestic Product (GDP), measures the output of goods and services produced
within the geographical boundary of a nation regardless of the residence of that labour
or owner of capital.
2. Gross National Product (GNP) measures the total output of goods and services produced
by residents of a nation regardless of where they live and work or where they own
capital.
Which of the above statements is/are correct?
a)
b)
c)
d)

Only 1
Only 2
Both 1 and 2
Neither 1 nor 2

Q.4) Solution (c)


Explanation:
Gross Domestic Product
An estimated value of the total worth of a countrys production and services, within its
boundary, by its nationals and foreigners, calculated over the course on one year.
Total value of products & Services produced within the territorial boundary of a country.
GDP is essentially about where production takes place.
Gross National Product
An estimated value of the total worth of production and services, by citizens of a country,
on its land or on foreign land, calculated over the course on one year.
Total value of Goods and Services produced by all nationals of a country (whether within or
outside the country).
GNP is about who produces.
Q.5) Which of the following provides correct definition of GDP deflator?
a)
b)
c)
d)

The ratio of GDP at current prices to the GDP at constant prices.


GDP at constant prices divided by Wholesale Price Index.
The ratio of GDP at constant prices to the GDP at current prices.
None of the above.

www.iasbaba.com

Page 3

IASbaba Prelims 60 Days: [Day 9]

Q.5) Solution (a)


Explanation:

In economics, the GDP deflator (implicit price deflator) is a measure of the level of prices of
all new, domestically produced, final goods and services in an economy. GDP stands for
gross domestic product, the total value of all final goods and services produced within that
economy during a specified period.

Formula
Where, Nominal GDP is GDP at current prices and Real GDP is GDP at constant prices
(Link: https://upload.wikimedia.org/math/8/1/8/818ffd5a7c33b377f1c98c4f2220c53d.png)

Q.6) Which among the following is correct in regard to Social Progress Index (SPI)?
1. SPI goes beyond the traditional measure of GDP and has most parameters that are
required to fulfill SDGs
2. SPI is based on three fundamental pillars: basic needs for survival; access to the building
blocks to improve living conditions, and access to opportunity to pursue goals and
ambitions
3. SPI focuses on outcomes rather than inputs that are used in GDP
4. SPI can best be described as a complementary index to GDP and can be used along with
GDP to achieve social progress
Choose the correct code
a)
b)
c)
d)

1, 2 and 4 only
2 and 4 only
1, 2 and 3 only
All the above

Q.6) Solution (d)


Explanation:
About Social Progress Index (SPI)
www.iasbaba.com

Page 4

IASbaba Prelims 60 Days: [Day 9]

The Social Progress Index measures the extent to which countries provide for the social and
environmental needs of their citizens.
Fifty-four indicators in the areas of basic human needs, foundations of wellbeing, and
opportunity to progress show the relative performance of nations.
SPI is based on three fundamental pillars: basic needs for survival; access to the building
blocks to improve living conditions, and access to opportunity to pursue goals and
ambitions.

Difference between GDP and SPI:


SPI focuses on outcomes rather than inputs that are used in GDP.
For example, the quality of life and longevity are measured instead of spending on health
care, and peoples experience of discrimination is looked at instead of focusing on whether
there is a law against discrimination.
SPI takes into consideration not just GDP but also inclusive, sustainable growth that will lead
to a significant improvement in peoples lives.
SPI can best be described as a complementary index to GDP and can be used along with
GDP to achieve social progress.
Q.7) Recently union government made some changes to the GDP calculation method. Which
of the following is/are correct with regards to these changes?
1. The base year of estimating national income data is changed from 2004-05 to 2011-12
2. GDP is estimated at market prices, which excludes indirect taxes but includes subsidies
3. The change in base year has pushed up the countrys economic growth as compared to
older series data
Choose the correct code
a)
b)
c)
d)

1 and 2 only
2 only
1 and 3 only
1, 2 and 3

Q.7) Solution (c)


Explanation:
CSO has revised the base year of estimating national income data from 2004-05 to 2011-12
www.iasbaba.com

Page 5

IASbaba Prelims 60 Days: [Day 9]

With the revised definition of GDP with new base rate 2011-12, countrys economic growth
got pushed up as compared to older series data
In new definition of the economic growth with new base rate 2011-12, GDP is estimated at
market process, which includes indirect taxes and exclude subsidies
Earlier, GDP growth was estimated at factor cost, which excludes indirect taxes but includes
subsidies

Q.8) The most appropriate measure of a countrys economic growth is its


a)
b)
c)
d)

Gross Domestic Product


Net Domestic Product
Net National Product
Per Capita Real Income

Q.8) Solution (d)


Explanation:
Per Capita Real Income is the most appropriate measure of a countrys economic growth
Q.9) Consider the following statements regarding purchasing power parity.
1. It is a technique used to determine the relative value of different currencies.
2. This concept works on the assumption that markets work on the law of one price.
3. This is a popular method used by the IMF and WB in studying the living standards of
people in different economies.
Which of the following is/are correct?
a)
b)
c)
d)

1 and 2 only
1 and 3 only
1 only
All of the above

Q.9) Solution (d)


Explanation:
Law of one price, i.e., identical goods and services (in quantity as well as quality) must have
the same price in different markets when measured in a common currency.
www.iasbaba.com

Page 6

IASbaba Prelims 60 Days: [Day 9]

Q.10) Which among the following are a part of core industries?


1)
2)
3)
4)
5)
6)

Fertilizers
Refinery Products
Natural Gas
Iron
Cement
Electricity

Choose the correct code:


a)
b)
c)
d)

1,2,4,5 and 6
1,2,3,4 and 6
1,2,3,5 and 6
1,2,3,4,5 and 6

Q.10) Solution (c)


Explanation:
There are eight core sector industries - Coal, Crude Oil, Natural Gas, Refinery Products,
Fertilizers, Steel (Note: No Iron or Iron ore), Cement and Electricity.

Q.11) International Solar Alliance (ISA) will have membership from countries
a)
b)
c)
d)

Lying above the Tropic of Cancer


Lying between the Tropic of Cancer and the Tropic of Capricorn
Lying below the Tropic of Cancer and above the Tropic of Capricorn
Along Equator and Tropic of Cancer

Q.11) Solution (b)


India and France have launched an International Solar Alliance to boost solar energy in
developing countries.
www.iasbaba.com

Page 7

IASbaba Prelims 60 Days: [Day 9]

The initiative was launched at the UN Climate Change Conference in Paris on 30 November by
Indian Prime Minister Narendera Modi and French President Francois Hollande.
The alliance includes around 121 countries that support the Declaration on the occasion to
launch the international solar alliance of countries dedicated to the promotion of solar energy.
Sourced from- Economic Survey, State of Economy- An Overview
http://www.thehindu.com/sci-tech/energy-and-environment/modi-launches-internationalsolar-alliance/article7934560.ece

Q.12) Sustainable Development Goals (SDGs) and Millennium Development Goals (MDGs)
commonly mentions about
1.
2.
3.
4.
5.

Infrastructure
Poverty and Hunger
Inequality
Peace and Justice
Water and Sanitation

Select themes common to both, SDGs and MDGs?


a)
b)
c)
d)

1, 2, 3 and 5
2, 3, 4 and 5
2, 3 and 5
1, 2 and 3

Q.12) Solution (a)

Sourced from- Economic Survey, State of Economy- An Overview

www.iasbaba.com

Page 8

IASbaba Prelims 60 Days: [Day 9]

Peace and Justice, 16th Goal of SDG- Promote peaceful and inclusive societies for sustainable
development, provide access to justice for all and build effective, accountable and inclusive
institutions at all levels.
MDGs
Goal 1: Eradicate extreme poverty and hunger
Goal 2: Achieve universal primary education
Goal 3: Promote gender equality and empower women
Goal 4: Reduce child mortality rates
Goal 5: Improve maternal health
Goal 6: Combat HIV/AIDS, malaria, and other diseases
Goal 7: Ensure environmental sustainability
Goal 8:Develop a global partnership for development

Q.13) Select the correct statement


a) The Labour Force Participation Rate (LFPR) of women is significantly lower than that of
males in both rural and urban areas.
b) The Labour Force Participation Rate (LFPR) of males is significantly lower than that of
women in both rural and urban areas.
c) The Labour Force Participation Rate (LFPR) of women is significantly lower than that of
males in rural areas but higher in urban areas.
d) The Labour Force Participation Rate (LFPR) of women is significantly lower than that of
males urban areas but higher in rural areas.

Q.13) Solution (a)


www.iasbaba.com

Page 9

IASbaba Prelims 60 Days: [Day 9]

As per the fourth Annual Employment-Unemployment Survey conducted by the Labour Bureau
during the period January 2014 to July 2014, the Labour Force Participation Rate (LFPR) (usual
principal status) is 52.5 for all persons. The LFPR of women is significantly lower than that of
males in both rural and urban areas. The Worker Population Ratio (WPR) reflects similar
patterns.
Sourced from- Economic Survey, State of Economy- An Overview

Q.14) Consider the following statements regarding Turkey


1. Istanbul is the capital of Turkey
2. Syria, Bulgaria and Georgia are few nations that border Turkey
Choose the correct answer using the code given below
a)
b)
c)
d)

1 only
2 only
Both
None

Q.14) Solution (b)

Refer Altas

Q.15) Consider the following statements regarding Moscow declaration.


1. It was signed during third BRICS ministerial conference held at Moscow
2. It is concerned with ending gender discrimination, and providing Equal access to
women in Education ,Employment etc
Choose the correct answer using the code given below
a)
b)
c)
d)

1 only
2 only
Both
None

www.iasbaba.com

Page 10

IASbaba Prelims 60 Days: [Day 9]

Q.15) Solution (a)

BRICS nations signed Moscow Declaration for supporting Multilateral Science Projects
Purpose-For co-investment of resources for supporting multilateral Research and
Development (R&D) Projects in mutually agreed areas.
It calls for collaboration in:
Co-operation for large research infrastructures, including mega-science projects along
with coordination of the existing large-scale national programme of member countries.
Implementation and development of a BRICS Framework Programme for funding joint
multilateral research projects, innovation and technology commercialization.
Establishment of BRICS Research and Innovation Networking Platform

Q.16) Puga valley in J&k is famous for availability of which of the following energy reserve
a)
b)
c)
d)

Coal
Geo Thermal
Hydro
Wind

Q.16) Solution (b)

www.iasbaba.com

Page 11

IASbaba Prelims 60 Days: [Day 9]

Q.17) The government of India scheme known as Anubhav refers to


a)
b)
c)
d)

Speedy redressal of Infrastructural projects


A platform for retiring employees
A new communication satellite on the anvil
None of the above

Q.17) Solution (b)

Click here

www.iasbaba.com

Page 12

IASbaba Prelims 60 Days: [Day 9]

Q.18) Recently Ministry of Water resources and Ganga rejuvenation appointed Ganga task
force for which of the following purposes?
a)
b)
c)
d)

To lay down guidelines to promote tourism in areas surrounding Ganga river


To ensure that industry and civilians do not pollute the river
To remove silt and other pollutants from Ganga river
To protect Ganges river dolphins

Q.18) Solution (b)

Ganga Task force is a bunch of Jawans who will be deployed on the banks of river Ganga
to ensure that industry and civilians do not pollute the river.

Q.19) Consider the following statements with respect to Indian rhino vision 2020
1. The Indian Rhino vision aims to increase the number one horned rhinoceros to 3000 by
2020.
2. Kaziranga national park houses nearly 75% of world rhinoceros
3. WWF (WWF-India and WWF AREAS) and the International Rhino Foundation (IRF) are
the two major partnering of Indian Rhino vision 2020
Select the incorrect statements
a)
b)
c)
d)

2 only
2 and 3 only
1 and 3 only
All

Q.19) Solution (a)

Click here

www.iasbaba.com

Page 13

IASbaba Prelims 60 Days: [Day 9]

Q.20) Consider the following statements with respect to Ancient Nalanda University
1. The university taught/helped propagates Hinayana school of Buddhism.
2. It was started by a ruler known as Kumaragupta.
Select the correct answer with respect to Nalanda university
a)
b)
c)
d)

1 only
2 only
Both
None

Q.20) Solution (b)

It propagated Mahayana school of Buddhism

Q.21) Consider the following


1. Defence of the body
2. Metabolic activities
3. Damage repair
Which of the following are the primary functions of proteins?
a)
b)
c)
d)

1 and 2 only
2 and 3 only
1, 2 and 3 only
1 and 3 only

Q.21) Solution (c)

www.iasbaba.com

Page 14

IASbaba Prelims 60 Days: [Day 9]

Q.22) Recently it is found that sea ice cover surrounding Antarctica has been slightly
increasing in sharp contrast to drastic loss of ice occurring in the Arctic region. What can be
the possible reason for the same?
1. The topography of Antarctica and the depth of the ocean surrounding it
2. Topography of Antarctica and depth of ocean influence the winds and ocean currents to
drive the formation and evolution of Antarcticas sea ice cover and help sustain it.
Select the correct code
a) Only 1
b) Only 2
c) Both
d) None

Q.22) Solution (c)


The recent research revealed that as sea ice forms and builds up early in the sea ice growth
season, it gets pushed offshore and northward by winds, forming a protective shield of older,
thicker ice that circulates around the continent.
The persistent winds, which flow down slope off the continent and are shaped by Antarcticas
topography, pile ice up against the massive ice shield, enhancing its thickness.
This band of ice, which varies in width from roughly 100 to 1,000 km, encapsulates and protects
younger, thinner ice in the ice pack behind it from being reduced by winds and waves. The team
also used QuikScat radar data to classify the different types of Antarctic sea ice.
Older, thicker sea ice returns a stronger radar signal than younger, thinner ice does. They found
the sea ice within the protective shield was older and rougher (due to longer exposure to wind
and waves), and thicker (due to more snow accumulation).
As the sea ice cover expands and ice drifts away from the continent, areas of open water form
behind it on the sea surface, creating ice factories conducive to rapid sea ice growth,

www.iasbaba.com

Page 15

IASbaba Prelims 60 Days: [Day 9]

http://www.thehindu.com/sci-tech/energy-and-environment/why-sea-ice-cover-aroundantarctica-isrising/article8634112.ece?utm_source=RSS_Feed&utm_medium=RSS&utm_campaign=RSS_Syn
dication

Q.23) Which of the following are not affected by Solar Storm/flare?

1. Short wave radio communication


2. Lon range radio communication
3. Chromosphere
4. Ionosphere
5. Astronauts
Select the appropriate code
a) 1 and 2
b) 2 and 4
c) 1, 4 and 5
d) None

Q.23) Solution (d)


All are affected by Solar flare
The soft X-ray flux of X class flares increases the ionization of the upper atmosphere, which can
interfere with short-wave radio communication and can heat the outer atmosphere and thus
increase the drag on low orbiting satellites, leading to orbital decay.

www.iasbaba.com

Page 16

IASbaba Prelims 60 Days: [Day 9]

X-rays and UV radiation emitted by solar flares can affect Earth's ionosphere and disrupt longrange radio communications. Direct radio emission at decimetric wavelengths may disturb the
operation of radars and other devices that use those frequencies
Solar flares affect all layers of the solar atmosphere (photosphere, chromosphere, and corona)

The radiation risks posed by solar flares are a major concern in discussions of a manned mission
to Mars, the Moon, or other planets. Energetic protons can pass through the human body,
causing biochemical damage, presenting a hazard to astronauts during interplanetary travel.
Some kind of physical or magnetic shielding would be required to protect the astronauts. Most
proton storms take at least two hours from the time of visual detection to reach Earth's orbit. A
solar flare on January 20, 2005 released the highest concentration of protons ever directly
measured, giving astronauts as little as 15 minutes to reach shelter.

Q.24) Consider the following statements regarding Remittance


1. India is worlds largest remittance recipient in 2015
2. Indian remittance has declined as compared to last year.
3. While global remittance has decreased, remittance for developing countries increased in
2015
Select the correct code
a) Only 1
b) 1 and 3
c) 1 and 2
d) 1, 2 and 3

Q.24) Solution (d)

www.iasbaba.com

Page 17

IASbaba Prelims 60 Days: [Day 9]

As per recently report of World Bank, India remained the worlds largest remittance recipient in
2015. It was revealed by the World Banks annual report Migration and Development Brief. In
2015, India attracted about 69 billion US dollars in remittances, down from 70 billion in 2014
Key Highlights of Report Other large remittance recipients in 2015 were China (64 billion
dollars), Philippines (28 billion), Mexico (25 billion) and Nigeria (21 billion dollar). Global
scenario:
In 2015, global remittances which include those to high-income countries contracted by 1.7% to
581 billion US dollar compared to 592 billion in 2014.
Indian scenario: Remittances to India in 2015 decreased by 2.1 per cent to USD 68.9 billion. This
marks the first decline in remittances since 2009.
Developing countries: Officially recorded remittances to developing countries amounted to
431.6 billion dollars in 2015, an increase of 0.4 per cent over 430 billion dollars in 2014.

Q.25) Consider the following statements


1. Seawater is utilised to produce a solar fuel that is hydrogen peroxide (H2O2)
2. Chlorine in seawater is mainly responsible for enhancing the photo-catalytic activity
Select the correct code
a) Only 1
b) Only 2
c) Both
d) None

Q.25) Solution (c)


www.iasbaba.com

Page 18

IASbaba Prelims 60 Days: [Day 9]

http://www.thehindu.com/sci-tech/new-method-can-generate-power-fromseawater/article8632993.ece?utm_source=RSS_Feed&utm_medium=RSS&utm_campaign=RSS
_Syndication

www.iasbaba.com

Page 19

IASbaba Prelims 60 Days- [Day 10]


Q.1) Which of the following is correct about Mathew effect?
a) The Matthew effect (or accumulated advantage) is the phenomenon where "the rich get
richer and the poor get poorer
b) The Mathew effect means the idea of inflation increasing with employment rates
soaring
c) Mathew was a ancient historian who felt rich will help poor and vice versa hence an
equalizing effect
d) Mathew effect is a concept of psychology of people wishing for equality in extreme
poverty
Q.1) Solution (a)
The Principle of Cumulative Advantage states that once a social agent gains a small advantage
over other agents, that advantage will compound over time into an increasingly larger
advantage. The effect is well known and is embodied in "the rich get richer and the poor get
poorer". The principle is also known as the Matthew effect.
The term was coined by sociologist Robert Merton in a 1968 paper which described how the
more eminent scientists in a group tend get the most credit for the group's work, regardless of
who did the work.
The Principle of Cumulative Advantage lies at the heart of many large social problems. It's the
causal mechanism for income inequality growth, corruption growth, centralization of power
growth, and hardening of class stratification.

www.iasbaba.com

Page 1

IASbaba Prelims 60 Days- [Day 10]


Q.2) Which of the following institutions have their objectives stated to fight poverty in the
world?
1.
2.
3.
4.

International Monetary Fund


World Bank
World Trade Organization
World Health Organization

Select the correct answer:


a) 1 only
b) 1 and 2 only
c) 1, 2 and 3 only
d) 1, 2 and 4 only

Q.2) Solution (b)


Further reading: https://www.wto.org/english/thewto_e/whatis_e/what_we_do_e.htm
http://www.who.int/about/mission/en/

www.iasbaba.com

Page 2

IASbaba Prelims 60 Days- [Day 10]

Q.3) Which of the following are correct about Poverty?

1. Absolute poverty was defined as: a condition characterised by severe deprivation of


basic human needs, including food, safe drinking water, sanitation facilities, health,
shelter, education and information.
2. Relative poverty refers to a standard which is defined in terms of the society in which an
individual lives and which therefore differs between countries and over time.
3. Poverty gap is the mean shortfall of the total population from the poverty line (counting
the nonpoor as having zero shortfall), expressed as a percentage of the poverty line.
This measure reflects the depth of poverty as well as its incidence.
4. Poverty line is the level of income below which one cannot afford to purchase all the
resources one requires to live.

Select the correct answer:

a) 1 and 2 only
b) 1, 2 and 3 only
c) 1, 2 and 4 only
d) All the above

Q.3) Solution (d)


Povert is a condition where people's basic needs for food, clothing, and shelter are not being
met. Poverty is generally of two types:
(1) Absolute poverty is synonymous with destitution and occurs when people
cannot obtain adequate resources (measured in terms of calories or nutrition) to support a
www.iasbaba.com

Page 3

IASbaba Prelims 60 Days- [Day 10]


minimum level of physical health. Absolute poverty means about the same everywhere, and
can be eradicated as demonstrated by some countries.
(2) Relative poverty occurs when people do not enjoy a certain minimum level of living
standards as determined by a government (and enjoyed by the bulk of the population) that vary
from country to country, sometimes within the same country.

Q.4) Arrange following committees w.r.t. Poverty Alleviation in chronological order


1. Lakadwala Committee
2. Y K Alag Committee
3. C Rangarajan Committee
4. Suresh Tendulkar Committee
5. N C Saxena Committee

Select the correct answer:

a) 1 2 3 4 5
b) 2 1 4 5 3
c) 1 3 4 5 2
d) 2 5 4 3 1

Q.4) Solution (b)


Further reading: http://planningcommission.nic.in/reports/genrep/pov_rep0707.pdf

www.iasbaba.com

Page 4

IASbaba Prelims 60 Days- [Day 10]

Q.5) Consider the following

1. Dr. N.C. Saxena Committee was set up by the Ministry of Rural Development to advise it
on the suitable methodology for BPL Census and not for estimation of poverty.
2. The Planning Commission constituted an Expert Group under the Chairmanship of
Professor S.R. Hashim, with the objective of putting in place a uniform criterion to
identify the BPL households in urban areas so that objectivity and transparency is
ensured in delivery of benefits to the target groups.

Select the correct answer:

a) 1 only
b) 2 only
c) Both 1 and 2
d) None of the above

Q.5 Solution (c)


The Planning Commission constituted an Expert Group under the Chairmanship of Professor
S.R.Hashim to recommend the detailed methodology for identification of families living Below
Poverty Line in urban areas. The Hashim Committee submitted its final report on 24th
December, 2012 to the Planning Commission. In its report, the Hashim Committee
recommended three stage identification process to identify the families living Below Poverty
Line in urban areas which include automatic exclusion, automatic inclusion and scoring index of
the remaining urban families in this order.
www.iasbaba.com

Page 5

IASbaba Prelims 60 Days- [Day 10]


Dr. N.C. Saxena Committee was set up by the Ministry of Rural Development to advise it on the
suitable methodology for BPL Census and not for estimation of poverty. However, in the Report
submitted by the Expert Group on 21st August 2009 it is mentioned that the percentage of
people entitled to BPL status should be revised upwards to at least 50%. The committee has
suggested

proportionate

increase

in

the

state

level

poverty

estimates

also.

The recommendations of the Expert Group and other alternative methodologies are being
tested through a pilot socio-economic survey and a Participatory Rural Appraisal (PRA) exercise
in order to finalize the methodology for the final BPL Census.
Further reading: http://pib.nic.in/newsite/PrintRelease.aspx?relid=68126

Q.6) Which of the following are not correctly matched?

1. Lorenz Curve Poverty estimation


2. J Curve Taxation
3. Laffer Curve Devalutaion
4. Philips Curve Inflation and uemployment

Select the correct answer:

a) 1 only
b) 1 and 2 only
c) 1, 2 and 3 only
d) All of the above

Q.6) Solution (c)


In economics, the Lorenz curve is a graphical representation of the distribution of income or of
wealth. It was developed by Max O. Lorenz in 1905 for representing inequality of the wealth
distribution.
www.iasbaba.com

Page 6

IASbaba Prelims 60 Days- [Day 10]


The J-curve effect is seen in economics when a country's trade balance initially worsens
following a devaluation or depreciation of its currency.
The Phillips curve represents the relationship between the rate of inflation and the
unemployment rate.
In economics, the Laffer curve is a representation of the relationship between rates of taxation
and the resulting levels of government revenue.

Q.7) The Gini coefficient (also known as the Gini index or Giniratio) is a measure of statistical
dispersion intended to represent the income distribution of a nation's residents, and is the
most commonly used measure of inequality. Consider the following about Gini Coefficent.

1. A Gini coefficient of zero expresses perfect equality, where all values are the same (for
example, where everyone has the same income).
2. A Gini coefficient of 1 (or 100%) expresses maximal inequality among values (e.g., for a
large number of people, where only one person has all the income or consumption, and
all others have none, the Gini coefficient will be very nearly one).

Select the correct answer:

a) 1 only
b) 2 only
c) Both 1 and 2
d) None of the above

Q.7) Solution (c)


Further reading: http://data.worldbank.org/indicator/SI.POV.GINI

www.iasbaba.com

Page 7

IASbaba Prelims 60 Days- [Day 10]

Q.8) Which of the following is incorrect about the Digital Gender Atlas for Advancing Girls
Education in India?
a) In order to plan and execute educational interventions, the purpose of the Gender Atlas
is to help identify and ensure equitable education with a focus on vulnerable girls,
including girls with disabilities.

b) The Ministry of Women and Child Development presented the Digital Gender Atlas for
Advancing Girls Education in India
c) The Atlas provides comparative analysis of individual gender related indicators over
three years and that enables a visual assessment of the change and an understanding of
whether some intervention introduced in a geography at a particular point in time has
worked or not.
d) The main components of the Gender Atlas are : (i) Composite Gender Ranking (ii) Trend
Analysis of Gender Indicators (iii) Vulnerabilities based on educational indicators in
districts with substantial tribal, schedule caste, minority population

Q.8) Solution (b)

The Ministry of Human Resource Development, Department of School Education and Literacy,
presented the Digital Gender Atlas for Advancing Girls Education in India. The tool, which has
been developed with the support of UNICEF, will help identify low performing geographic
pockets for girls, particularly from marginalised groups such as scheduled castes, schedule
tribes and Muslim minorities, on specific gender related education indicators.
www.iasbaba.com

Page 8

IASbaba Prelims 60 Days- [Day 10]


The Atlas provides comparative analysis of individual gender related indicators over three years
and that enables a visual assessment of the change and an understanding of whether some
intervention introduced in a geography at a particular point in time has worked or not. It is
constructed on an open source platform with an inbuilt scope of updating data by authorized
persons to retain its dynamic character.
Further reading: http://pib.nic.in/newsite/PrintRelease.aspx?relid=116595

Q.9) Consider the following about Employment rates and surveys.

1. Labour Bureau has been entrusted the task of conducting Annual EmploymentUnemployment Survey in the country.
2. The labor force participation rate (LFPR) is the proportion of people eligible to
participate in the labor force who are actually participating in it by working or looking
for work. It is usually expressed as a percent of the total labor force-eligible population
in an economy.
3. Unemployment rate is defined most basically as the percentage of the total labor force
that is unemployed but actively seeking employment and willing to work.
Select the correct answer:

a) 1 and 3 only
b) 1 and 2 only
c) 2 and 3 only
d) All of the above

Q.9) Solution (d)


Further reading: http://labourbureau.nic.in/Report%20%20Vol%201%20final.pdf

www.iasbaba.com

Page 9

IASbaba Prelims 60 Days- [Day 10]

Q.10) Consider the following about Pradhan Mantri Kaushal Vikas Yojana (PMKVY).

1. Pradhan Mantri Kaushal Vikas Yojana (PMKVY) isthe flagship outcome-based skill
training scheme of the new Ministry of Skill Development & Entrepreneurship (MSDE).
2. The objective of this skill certification and reward scheme is to enable and mobilize a
large number of Indian youth to take up outcome based skill training and become
employable and earn their livelihood
3. Under PMKVY, trainees with prior experience or skills and competencies will not be
recognized and they will be given certification only post undergoing assessments.
4. The scheme will be implemented by the Labour Bureau of India.

Select the correct answer about PMKVY:

a) 1 and 2 only
b) 1 and 4 only
c) 2 and 3 only
d) None of the above

Q.10) Solution (a)

www.iasbaba.com

Page 10

IASbaba Prelims 60 Days- [Day 10]


Pradhan Mantri Kaushal Vikas Yojanais a unique initiative by the Government of India that aims
to offer 24 lakh Indian youth meaningful, industry relevant, skill based training. Under this
scheme, the trainees will be offered a financial reward and a government certification on
successful completion of training and assessment, which will help them in securing a job for a
better future.
The scheme will be implemented through the National Skill Development Corporation
(NSDC).

Further reading: http://www.skilldevelopment.gov.in/pmkvy.html


http://pmkvyofficial.org/Index.aspx
Q.11) Recently Ministry of Urban development released the second list of would be smart
cities. Consider the following statements with respect to development of green field smart
city.
1. The minimum area for the development of Greenfield smart city is 250 acres
2. GIFT in Gujarat is the best example for Green field development of a smart city

Select the correct option with respect to development of Green field smart city
a)
b)
c)
d)

1 only
2 only
Both
None

Q.11) Solution (c)

Greenfield development will introduce most of the Smart Solutions in a previously


vacant area (more than 250 acres) using innovative planning, plan financing and plan
implementation tools (e.g. land pooling/ land reconstitution) with provision for
affordable housing, especially for the poor. Greenfield developments are required
around cities in order to address the needs of the expanding population. One well
known example is the GIFT City in Gujarat. Unlike retrofitting and redevelopment,
greenfield developments could be located either within the limits of the ULB or within
the limits of the local Urban Development Authority (UDA).

www.iasbaba.com

Page 11

IASbaba Prelims 60 Days- [Day 10]

Q.12) Consider the following countries


1.
2.
3.
4.
5.

Italy
China
Russia
France
Japan

Which of the following countries does not belong to G7 countries?

a)
b)
c)
d)

1, 2 and 3 only
1, 2 and 4 only
2 and 3 only
All are members of G7 forum

Q.12) Solution (c)

China and Russia are not members of G7

Q.13) Union cabinet approved National capital goods policy recently. Consider the following
statements with respect to National Capital goods policy
1. The policy aims to create 21 million new jobs by 2025
2. The policy aims to increase exports from the current 27 percent to 40 percent of
production.
3. Ministry of commerce and Industry is the nodal agency for the formulation of NCGP
2016-05-26

Select the correct option

www.iasbaba.com

Page 12

IASbaba Prelims 60 Days- [Day 10]


a)
b)
c)
d)

1 and 2 only
2 and 3 only
1 and 3 only
All

Q.13) Solution (a)

It is spearheaded by ministry of heavy industries and public enterprises.


Click here

Q.14) Consider the following statements with respect to PAHAL scheme


1. The scheme was first launched in 2015
2. The scheme encourages the LPG customers to voluntarily give up subsidies
3. The scheme was launched under the aegis of Ministry of Petroleum and natural
resources

Select the correct answer


a)
b)
c)
d)

2 and 3 only
3 only
1 and 3 only
1, 2 and 3

Q.14) Solution (b)

The PAHAL (DBTL) scheme was earlier launched on 1st June 2013 and finally covered
291 districts. It required the consumer to mandatorily have an Aadhaar number for
availing LPG Subsidy. The government has comprehensively reviewed the scheme and
after examining the difficulties faced by the consumer substantively modified the
scheme prior to launch. The modified scheme has been re-launched in 54 districts on
15.11.2014 in the 1st Phase and to be launched in rest of the country on 1.1.2015

www.iasbaba.com

Page 13

IASbaba Prelims 60 Days- [Day 10]

Q.15) Recently Global statics about production of e-waste was released. Consider the
following statements with respect to e-waste
1. PCB (polychlorinated biphenyls), Beryllium, Barium are some of the commonly found Ewaste in India
2. India is the largest producer of E waste in the world

Select the correct option


a)
b)
c)
d)

1 only
2 only
Both
None

Q.15) Solution (a)

U.S., China, Japan and Germany, and India are top e-waste producing countries in the
world

Q.16) Consider the following


1. Itai Itai
2. Yokakaichi Asthama
3. Minimata

Select the above ailments with their causing pollutants


a)
b)
c)
d)

1 Mercury, 2 Cadmium, 3 Sulphur oxide


1 Cadmium, 2 Sulphur oxide, 3 Mercury
1 Sulphur oxide, 2 Cadmium, 3 Mercury
1 Sulphur oxide, 2 Mercury, 3 Cadmium

Q.16) Solution (b)


www.iasbaba.com

Page 14

IASbaba Prelims 60 Days- [Day 10]

Q.17) Consider the following


1. Transfat are saturated fatty acids
2. They are formed during the process of addition of hydrogen atoms to oil
3. Trans fat are naturally found in nature

Which of the following statements is/are false with respect to Trans fat?
a)
b)
c)
d)

1 and 2 only
1 and 3 only
3 only
None

Q.17) Solution (b)

Transfat

They are naturally found but in trace amount. So in general the statement They are naturally
found in nature may be taken as incorrect.
Q.18) Consider the following conventions and select the wrongly matched convention with its
intent
a) Stockholm convention deals with persistent organic pollutants
b) Basel convention deals with the control of hazardous wastes and their disposal
c) Montreal conventions deals with preventing the use of substances that deplete ozone
layer
d) Rotterdam convention are dedicated to the protection of ozone layer

Q.18) Solution (d)

www.iasbaba.com

Page 15

IASbaba Prelims 60 Days- [Day 10]

Rotterdam convention aims to promote shared responsibility and cooperative efforts


among parties in the international trade of certain hazardous chemicals in order to
protect human health and the environment from potential harm.

Q.19) Consider the following


1.
2.
3.
4.

India lies in close proximity with ring of fire


The Indian plate is moving northwards diverging with Eurasian plate
The Indian plate is moving northwards converging with Eurasian plate
Presence of active Volcanoes along Himalayan range

Which of the following statements does not explain the possible reason for presence of earth
quake in India?
a)
b)
c)
d)

1 and 2 only
2 and 3 only
1, 2 and 4 only
All

Q.19) Solution (c)

Indian plate is converging with Eurasian plate.

Q.20) Consider the following statements with respect to Roads in India


1. Bangalore is one such place in India where the National highway, North south corridor
and Golden Quadrilateral intersect
2. The intersection between North-South and East-West corridor is at Jhansi

Select the correct answer using the code given below


www.iasbaba.com

Page 16

IASbaba Prelims 60 Days- [Day 10]


a)
b)
c)
d)

1 only
2 only
Both 1 and 2
Neither of them

Q.20) Solution (c)

Both the statements are true

Q.21) Consider the following statements


1. The trends in enrolment reflect a decline in the percentage of enrolment in government
schools in rural areas
2. In the case of Scheduled Tribe (ST) students, parity between girls and boys has been
achieved across all levels of school and higher education.
Select the correct code
a)
b)
c)
d)

Only 1
Only 2
Both
None

Q.21) Solution (a)


In the case of Scheduled Tribe (ST) students, parity between girls and boys has not been
achieved across all levels of school and higher education.
Economic Survey, Social Infrastructure, Employment and Human Development

Q.22) Consider the following regarding Digital Gender Atlas for Advancing Girls Education in
India. It is a tool
1. To help identify low-performing geographic pockets for girls, particularly from
marginalized groups to provides comparative analysis of individual gender-related
indicators over the years.
www.iasbaba.com

Page 17

IASbaba Prelims 60 Days- [Day 10]


2. The tool has been developed in partnership with the United Nations Childrens Fund
(UNICEF)
Select the correct code
a)
b)
c)
d)

Only 1
Only 2
Both
None

Q.22) Solution (c)


Economic Survey, Social Infrastructure, Employment and Human Development

Q.23) Schemes that are aimed both at creating a conducive environment for industrial
development and doing business with ease and also expanding government support to
impart skill training for workers?
1.
2.
3.
4.
5.

Pandit Deendayal Upadhyay Shramev Jayate Karyakram


Shram Suvidha Portal
Random Inspection Scheme
Universal Account Number Scheme
Apprentice Protsahan Yojana

Select the correct code


a)
b)
c)
d)

1, 2 and 3
1, 2, 3, 4, 5
1, 2, 4, 5
1, 3, 4, 5

Q.23) Solution (b)


India Year Book- Labor, Skill Development and Employment

www.iasbaba.com

Page 18

IASbaba Prelims 60 Days- [Day 10]


Q.24) Consider the following
1.
2.
3.
4.
5.
6.

Malaria
Dengue
Chagas disease
Yellow fever
Japanese encephalitis
Zika

Which of the above are Vector Borne Diseases?


a)
b)
c)
d)

1, 2 and 4
1, 3, 4 and 5
1, 2, 3, 4 and 6
1, 2, 3, 4, 5 and 6

Q.24) Solution (d)


Economic Survey, Social Infrastructure, Employment and Human Development
http://www.who.int/mediacentre/factsheets/fs387/en/

Q.25) The Universal Health Coverage (UHC) index has been developed by
a)
b)
c)
d)

WHO
World Bank
United Nation
UNICEF

Q.25) Solution (b)


Economic Survey, Social Infrastructure, Employment and Human Development
The Universal Health Coverage (UHC) index has been developed by the World Bank to measure
the progress made in health sectors in select countries of the World. India ranks 143 among 190
countries in terms of per capita expenditure on health ($146 PPP in 2011). It has 157th position
according to per capita government spending on health which is just about $44 PPP. Indias
www.iasbaba.com

Page 19

IASbaba Prelims 60 Days- [Day 10]


performance on the indicator on treatment of diarrhoea needs improvement in terms of
enhancing the coverage. The impoverishment indicator reflects the financial risk protection
coverage, with a higher percentage reflecting better coverage.

www.iasbaba.com

Page 20

IASbaba Prelims 60 Days [Day 11]


Q.1) Consider the following countries
1.
2.
3.
4.

Congo
Gabon
Mali
Eritrea

Which of the African countries are located above equator?


a)
b)
c)
d)

1 and 2 only
2 and 3 only
3 and 4 only
1, 2 and 3 only

Q.1) Solution (c)


Refer Atlas

Q.2) Recently a group of researchers at Harvard University who are working on Alzheimer
disease came up with certain findings. Consider the following statements with respect to
Alzheimers disease

1. Alzheimers disease is the most common form of dementia.


2. Alzheimer is caused by Bacteria.
3. As of now there is no cure for Alzheimers
Select the correct option with respect to Alzheimers
a)
b)
c)
d)

1 and 2 only
2 and 3 only
1 and 3 only
All

Q.2) Solution (c)


The exact of cause of the disease is still unknown.

www.iasbaba.com

Page 1

IASbaba Prelims 60 Days [Day 11]


Q.3) Consider the following
1. The protocol of bio safety deals with Living modified organisms
2. The bio safety protocol is also known as Cartagena protocol
3. The Nagoya and Cartagena protocol deals with Genetic resources
Identify the wrong statement with respect to Bio safety
a)
b)
c)
d)

1 and 2 only
2 only
1 and 3 only
None of the above

Q.3) Solution (d)


All the statements are correct

Q.4) Mahila E Haat is an online platform for women for


a)
b)
c)
d)

Showcasing products made/manufactured/sold by women entrepreneurs.


Disburse loans for women belonging to BPL families
Provide helping hand in the form of financial assistance during Pregnancy period
E learning portal for rural women under Bharat Nirman programme

Q.4) Solution (a)


Click here
Q.5) Consider the following statements with respect to MUDRA bank
1.
2.
3.
4.

It is a subsidiary of SIDBI
It is a Non banking financial company
Loans under MUDRA are issued in three categories - Shishu, Kishore and Tarun
Only PSB(public sector banks) are entitled to disburse MUDRA loans

Select the correct option


a) 1 and 2 only
b) 2 and 3 only
c) 1, 2 and 3 only
www.iasbaba.com

Page 2

IASbaba Prelims 60 Days [Day 11]


d) 2, 3 and 4 only
Q.5) Solution (c)
Click here

Q.6) Consider the following statements with respect to Food Safety & Standards Authority
of India (FSSAI)
1. It is a non statutory body
2. It works under ministry of food processing
Select the correct option
a)
b)
c)
d)

1 only
2 only
Both
None

Q.6) Solution (d)

It is statutory body and works under ministry of health and family welfare

Q.7) Consider the following statements with respect to National Air Quality Index
1. It is an initiative under Swacch Bharat
2. The proposed air quality index includes 8 pollutants
3. CO2, CO, NH3 are some of the air pollutants included in the list
Select the correct option
a)
b)
c)
d)

1 and 3 only
1 and 2 only
2 and 3 only
All

www.iasbaba.com

Page 3

IASbaba Prelims 60 Days [Day 11]


Q.7) Solution (b)

Carbon dioxide is not included in the list of pollutants.

Click here

Q.8) Sahayog Kaijin is a joint coast guard exercise between India and
a)
b)
c)
d)

Oman
Japan
Bangladesh
Sri Lanka

Q.8) Solution (b)


Click here
Q.9) Consider the following
1. Mitra shakti India and Sri Lanka
2. Indradhanush India and UK
3. Varuna India and Russia
Which of the following military exercises are wrongly matched?
a)
b)
c)
d)

1 and 2 only
2 and 3 only
3 only
1 and 3 only

Q.9) Solution (c)


Varuna is a naval exercise between India and France

Q.10) Consider the following


1. Tsomoriri
www.iasbaba.com

Page 4

IASbaba Prelims 60 Days [Day 11]


2. Deepor Beel
3. Koleru Lake
Which of the above given lakes/Sanctuary is/are a designated Ramsar sites?
a)
b)
c)
d)

1 and 2 only
2 and 3 only
1 and 3 only
All

Q.10) Solution (d)


Click here

Q.11) Consider the following statements in regard to vaccines:


1) BCG, or bacille Calmette-Guerin provide immunization against diarrhea given to
child up to one to four month
2) DPT refers to a class of combination vaccines against three infectious diseases in
humans: diphtheria, pertussis (whooping cough), and tetanus given to child up to
one year
3) Oral Polio vaccine (OPV) is given to the child from 2 month to 6 year
Which of the above statements is/are incorrect?
a)
b)
c)
d)

2 and 3 only
1 only
2 only
1, 2 and 3

Q.11) Solution (b)

BCG, or bacille Calmette-Guerin, is a vaccine for tuberculosis (TB) disease. Many foreignborn persons have been BCG-vaccinated. BCG is used in many countries with a high
prevalence of TB to prevent childhood tuberculous meningitis and miliary disease.
DPT (also DTP and DTwP) refers to a class of combination vaccines against three
infectious diseases in humans: diphtheria, pertussis (whooping cough), and tetanus. The
vaccine components include diphtheria and tetanus toxoids and killed whole cells of the
organism that causes pertussis (wP).

www.iasbaba.com

Page 5

IASbaba Prelims 60 Days [Day 11]

There are two types of vaccine that protect against polio: inactivated polio vaccine (IPV)
and oral polio vaccine (OPV). IPV is given as an injection in the leg or arm, depending on
the patient's age. Polio vaccine may be given at the same time as other vaccines. Oral
Polio vaccine (OPV) is given to the child from 2 month to 6 year.

Q.12) Which among the following are features of Mammal?


1)
2)
3)
4)

Sweat glands and oil glands are found on skin


Tooth comes twice in these animals
There is no nucleus in its blood cells
All non-egg laying animals are mammals and hence man is also a mammal

Choose the correct code


a)
b)
c)
d)

1 and 4 only
2 and 4 only
1, 2 and 3 only
All the above

Q.12) Solution (c)


Mammal Characteristics
All mammals are warm blooded.
Most young are born alive.
They have hair or fur on their bodies.
Every mammal is a vertebrate.
All mammals have lungs to breathe air.
Its hearts are divided into four chambers.
Mammals feed milk to their babies.
There is no nucleus in its blood cells (except in camel and lama).
External ear is present in mammal.
Q.13) Which of the following statements is/are correct?
1) PSLV is designed mainly to deliver the earth observation or remote- sensing
satellites in LEO
2) GSLV is designed mainly to deliver the communication- satellites in GEO
3) An orbit is called sun-synchronous when the angle between the line joining the
centre of the Earth and the satellite and the Sun is constant throughout the orbit
4) Low Earth Orbit (LEO) is in the range of 600-900 Km altitude whereas Geosynchronous Earth orbit (GEO) is about 36000 Km altitude
www.iasbaba.com

Page 6

IASbaba Prelims 60 Days [Day 11]

Choose the correct code

a)
b)
c)
d)

1 and 2
3 and 4
1, 2 and 4
All of the above

Q.13) Solution (d)

Sun-Synchronous - circular polar orbits (The remote sensing satellites orbit the earth
from Pole-to- Pole)
GEO - zero deg inclination on equatorial plane (satellites in these orbits appear to
remain permanently fixed in the same position in the sky, as viewed from a particular
location on Earth, thus avoiding the need of a tracking ground antenna and hence are
useful for the communication applications)
Remaining are self-explanatory (as all the given options are correct)

Q.14) Match List I with List II and select the correct answer using the code given below the
Lists:
LIST I
LIST II
A. Dawn
1) Pluto
B. New Horizon
2) Ceres
C. Philae Lander
3) Binary Star
D. Cassini
4) Saturn
E. Nustar
5) comet 67P
A-B-C-D-E
a) 1-2-5-4-3
b) 2-1-5-3-4
c) 2-1-5-4-3
d) 1-4-5-2-3
Q.14) Solution (c)

Dawn is a space probe launched by NASA in September 2007 with the mission of
studying two of the three known protoplanets of the asteroid belt, Vesta and Ceres. It is
currently in orbit about its second target, the dwarf planet Ceres.

www.iasbaba.com

Page 7

IASbaba Prelims 60 Days [Day 11]

NASAs New Horizons mission is helping us understand worlds at the edge of our solar
system by making the first reconnaissance of the dwarf planet Pluto and by venturing
deeper into the distant, mysterious Kuiper Belt a relic of solar system formation.
Philae is a robotic European Space Agency lander that accompanied the Rosetta
spacecraft until it landed on comet 67P/ChuryumovGerasimenko, more than ten years
after departing Earth.
Philae's mission was to land successfully on the surface of a comet, attach itself, and
transmit data about the comet's composition.
Cassini is an unmanned spacecraft sent to the planet Saturn. It is a flagship-class NASA
ESAASI robotic spacecraft
NASAs Nuclear Spectroscopic Telescope Array, or NuSTAR, has successfully deployed its
lengthy mast, giving it the ability to see the highest energy X-rays in our universe, a step
closer in the hunt for black holes.
Dr. Bhaleraos team observed the binary star (binary star a pair of stars revolving
around each other) using a space x-ray telescope known as NuStar, a NASA space
mission. It is the first x-ray space telescope that can focus on very high energy x-rays.

Q.15) Consider the following in regard to INO:


1) Neutrinos are colloquially called ghost particles because they travel at almost the
speed of light, hardly interact with matter, and are very light.
2) Neutrinos come in three types, and spontaneously switch from one type to another
and the process is called Neutrino Oscillation
3) In the 1960s, a neutrino observatory located at the Kolar Gold Fields in Karnataka
became one of the worlds first experiments to observe neutrinos in the Earths
atmosphere
4) At present Dept. of Atomic Energy and Dept. of Science and Technology is funding
INO to come up near Bodi Hills, Theni district, TN
Which of the above statements is/are incorrect?
a)
b)
c)
d)

1 and 2
2 and 3
None of the above
All of the above

Q.15) Solution (c)

Self-explanatory (All the statements given explains about Neutrinos)


www.iasbaba.com

Page 8

IASbaba Prelims 60 Days [Day 11]

Q.16) When an incandescent electric bulb glows

a)
b)
c)
d)

the electric energy is completely converted into light energy


the electric energy is partly converted into light energy and partly into heat energy
the light energy is converted into electric energy
the electric energy is converted into magnetic energy

Q.16) Solution (b)

When an incandescent electric bulb glows the electric energy is partly converted into
light energy and partly into heat energy

Q.17) Which one among the following statements correctly defines the term Biofortification?
a) Enrichment of the nutrient quality of the soil using biological agents
b) Breeding crops to increase their nutritional values
c) Increase of organo-metallic compounds in the organism through energy flow in an
ecosystem
d) Adoption of genetic variable plants for breeding of high yielding varieties
Q.17) Solution (b)

Biofortification is the idea of breeding crops to increase their nutritional value. This can
be done either through conventional selective breeding, or through genetic engineering.
Biofortification differs from ordinary fortification because it focuses on making plant
foods more nutritious as the plants are growing, rather than having nutrients added to
the foods when they are being processed.

Q.18) 3G and 4G are wireless technologies, which are supposedly faster, more secure and
reliable. Present day 3G technology is capable of handling data around 2 Megabits per
second (1.8 - 2.5 GHz frequency band). What speed is expected from new 4G technology?
a)
b)
c)
d)

10-30 Megabits per second (1-5 GHz frequency band)


10-100 Megabits per second (2-10 GHz frequency band)
100 Megabits - 1 Gigabit per second (2-8 GHz frequency band)
More than 10 Gigabits per second (10-20 GHz frequency band)

www.iasbaba.com

Page 9

IASbaba Prelims 60 Days [Day 11]

Q.18) Solution (c)

3G should be capable of handling around 2 Megabits per second.


4G The speed and standards of this technology of wireless needs to be at least 100
Megabits per second and up to 1 Gigabit per second to pass as 4G.

Q.19) Which among the following is/are diseases caused by Protozoa?


1)
2)
3)
4)

Malaria
Diarrhea
Kala-azar
Sleeping sickness

Choose the appropriate code:


a)
b)
c)
d)

4 only
1 and 3 only
2 and 4 only
All of the above

Q.19) Solution (d)

All the given diseases are caused by Protozoa


Protozoan infections are parasitic diseases caused by organisms formerly classified in
the Kingdom Protozoa.
Protozoa are single-celled micro-organisms that get their food from the surrounding
environment or as scientists would say 'they are heterotrophs. They can be found in a
variety of habitats such as freshwater (including ponds & rivers), marine (the sea) and
even in the soil.

Q.20) Match List I with List II and select the correct answer using the code given below the
Lists:
List I
List II
A. Vitamin A
1.Non-clotting of blood
B. Vitamin B
2.Rickets
C. Vitamin C
3.Beriberi
D. Vitamin D
4.Colour Blindness
E. Vitamin K
5.Scurvy
www.iasbaba.com

Page 10

IASbaba Prelims 60 Days [Day 11]


a)
b)
c)
d)

A-B-C-D-E
4-3-5-2-1
4-1-5-2-3
4-1-5-3-2
2-1-5-4-3

Q.20) Solution (a)

www.iasbaba.com

Page 11

IASbaba 2016 Prelims 60 Days [Day 12]


Q.1) Consider the following statements:
1. Colonies of snowflake coral have been recently documented off the coast of
Vishakhapatnam.
2. These snowflake coral (Carijoa riisei) are native species of India.
3. It inhabits reefs and underwater structures such as shipwrecks and piers, attaching
itself to metal, concrete and even plastic.
Select the correct option:
a)
b)
c)
d)

Only 3
1 and 2
2 and 3
None of the above

Q.1) Solution: (a)


Friends of Marine Life (FML), a local NGO, has recorded the presence of several colonies of
the fast-growing alien species amid barnacle clusters on the rocky reef off the coast of
Kovalam in Thiruvananthapuram and Enayam, Kanyakumari.
A native of the tropical Western Atlantic and the Caribbean, C.riisei was first reported as an
invasive species from Hawaii in 1972. Since then, it has spread to Australia, Thailand,
Indonesia and the Philippines. It is considered an invasive species because of its capacity to
dominate space and crowd out other marine organisms. With its capacity to thickly settle
and occupy a variety of surfaces, C.riisei can destabilise the marine ecosystem
India: Gulf of Mannar, the Andaman and Nicobar Islands, Gulf of Kutch and Goa.

Q.2) Consider the following statements regarding the planets sixth mass biological
extinction event:
1. The number of rodents doubles and the abundance of the disease-causing
ectoparasites that they harbour decreases.
2. Megafauna face the highest rate of decline.
3. The number of invertebrate animals such as beetles, butterflies, spiders and worms
has increased by 20 per cent.
Select the incorrect option:
a) Only 3
b) Only 2

IASbaba 2016 Prelims 60 Days [Day 12]


c) Both 1 and 3
d) All of the above

Q.2) Solution (c)


An era of Anthropocene defaunation the current die-off largely associated with human
activity

Q.3) The Red list of threatened species is prepared by the:


a) Convention on International Trade in Endangered Species of Wild Fauna and Flora
(CITES)
b) International Union for Conservation of Nature (IUCN)
c) UN Environment Assembly (UNEA)
d) UN Environment Programme (UNEP)

Q.3) Solution: (b)


International Union for Conservation of Nature (IUCN)

Q.4) Consider the following statements:


1. Genetic diversity refers to the variation of genes within species.
2. Specie richness refers to the variety of species within a region.
Select the incorrect statement:
a)
b)
c)
d)

Only 1
Only 2
Both 1 and 2
Neither 1 nor 2

Q.4) Solution: (b)

Species diversity refers to the variety of species within a region.

IASbaba 2016 Prelims 60 Days [Day 12]

Specie richness (simplest measure of species diversity) refers to the number of


species per unit area.

Q.5) Which of the following has been set up under MAB program of UNESCO?
a)
b)
c)
d)

National Park
Sanctuary
Buffer Zone
Biosphere Reserves

Q.5) Solution: (d)


Source

Q.6) Where is Kedarnath Sanctuary located?


a)
b)
c)
d)

Bihar
Himachal Pradesh
Uttarakhand
Uttar Pradesh

Q.6) Solution: (c)


Uttarakhand (Musk Dur Project Himalayan musk deer)

Q.7) Identify the State:


1. Lai Haroba, Shim Lam and Thang-Ta are the trademark dances of this State.
2. The sacred groves are known as Umanglai.
3. Sangai is the State Animal and Nongin, its State Bird.
Select the correct option:
a) Mizoram
b) Manipur
c) Nagaland

IASbaba 2016 Prelims 60 Days [Day 12]


d) Assam

Q.7) Solution: (b)

Q.8) What do you mean by Kanams?


a)
b)
c)
d)

Backwaters of Kerala
Patch of biodiversity-rich small groves
Coral reef off the coast of Tamil Nadu
A small island near Lakshwadeep

Q.8) Solution: (b) Source

Q.9) Identify this animal with CITES listing


1. Its fur is famously known as Shahtoosh which is used to make luxury shawls.
2. It is listed as endangered.
Select the correct animal:
a)
b)
c)
d)

Lion-tailed Macquae
Slow Loris
Blue-eyed Black Lemur
Tibatian Antelope

Q.9) Solution: (d)

Q.10) BIOFIN is
a) A portable remote imaging spectrometer
b) Hybrid of debt & equity financing model for addressing the lack of finance in
leveraging Indias efforts to safeguard its environment
c) A pilot project to measure the hearing sensitivity of whales
d) A global partnership to help government plan and pay for its actions on biodiversity
conservation and sustainable use

IASbaba 2016 Prelims 60 Days [Day 12]


Q.10) Solution: (d)

Q.11) Consider the following statements regarding Peatlands


1. All wetlands include peat
2. Peatlands act as huge source of carbon
3. Peatlands regulate regional and local climate
Select the correct statement/s
a)
b)
c)
d)

Only 1
1 and 2
1, 2 and 3
2 and 3

Q.11) Solution (d)


Peatlands occur in a great variety of forms and are present in more than half of all Ramsar
sites throughout the world. Unwise use of this key wetland type may damage the features
that make peatlands so important for long-term carbon storage. Drainage of peatlands also
leads to loss of carbon and fertile soil, including severe soil subsidence and salt water
intrusion in lowland coastal areas.
Peatlands
Pristine peatlands are characterized by the presence of water and special vegetation. The
peat soil, often exceeding many meters in depth, consists of organic material and water and
is created by the accumulation of partially decomposed plant materials. The layers of peat
build up over sometimes thousands of years and preserve other materials including pollen
grains, human artefacts and ancient bodies, giving us an unrivalled window into the past.
Global carbon store and climate mitigation
It has been estimated that peatlands contain at least 550 Gt of carbon, which is almost
double the amount stored in the worlds forests. However, when peatlands are drained
huge amounts of carbon dioxide (CO2) are released making restoration of great importance
for climate change mitigation
Regulation functions of Peatlands:

regulation of global climate


regulation of regional and local climates
regulation of catchment hydrology

IASbaba 2016 Prelims 60 Days [Day 12]

regulation of catchment hydrochemistry


regulation of soil conditions

About 60% of the world's wetlands are peat.


http://www.peatsociety.org/peatlands-and-peat/functions-peatlands
http://www.ramsar.org/sites/default/files/documents/library/fs_8_peatlands_en_v5.pdf

Q.12) The definition of the word wetland, under the Convention, is any land area that is
saturated or flooded with water, either seasonally or permanently. Which of the given
options form part of an Inland wetland?
1.
2.
3.
4.
5.
6.
7.

Flood Plains
Swamps
Seagrass Meadows
Aquifers
Estuaries
Marshes
Coral Reefs

Select the correct option


a)
b)
c)
d)

1, 2, 3, 6 and 7
2, 3, 4, 5 and 6
1, 2, 4 and 6
1, 2 and 5

Q.12) Solution (c)

The definition of the word wetland, under the Convention, is any land area that is
saturated or flooded with water, either seasonally or permanently. Inland wetlands include
aquifers, lakes, rivers, streams, marshes, peatlands, ponds, flood plains and swamps.
Coastal wetlands include all coastlines, mangroves, saltmarshes, estuaries, lagoons,
seagrass meadows and coral reefs.

IASbaba 2016 Prelims 60 Days [Day 12]


Q.13) Consider the following regarding Conservation International
1. It is an international organization under United Nation Environment Program (UNEP)
that aims to help stabilize global climate, protect fresh water, and ensure human
well-being
2. Biodiversity Hotspots is an initiative of Conservation International
Select the incorrect statement/s
a)
b)
c)
d)

Only 1
Only 2
Both
None

Q.13) Solution (a)


Conservation International (CI)
An American non-profit environmental organization headquartered in Arlington, Virginia. Its goal is
to protect nature as a source of food, fresh water, livelihoods and a stable climate.
It employs scientists and policy experts to balance healthy ecosystems with sustainable human use.
Conservation International aims to help stabilize global climate, protect fresh water, and ensure
human well-being. To achieve their goals they work with indigenous peoples and non-governmental
organization. Conservation International's primary initiatives include climate, fresh water, food,
health, culture, and biodiversity.
Of all the significant initiatives Conservation International has achieved, its Biodiversity
Hotspots project is the most impressive. This project identifies and protects biological hotspots
places that exhibit the richest diversity and most threatened collections of plants and animals on our
planet.

Q.14) Identify the correct statement regarding Therukoothu


a)
b)
c)
d)

A Martial Art form of Tamil Nadu


A stage drama prevalent in Andhra Pradesh
A traditional folk art form of Tamil Nadu.
A form of regional folk dance of Karnataka

Q.14) Solution (c)

IASbaba 2016 Prelims 60 Days [Day 12]


http://www.thehindu.com/features/friday-review/theatre/from-the-village-ofkoothu/article6962630.ece
http://www.livemint.com/Leisure/Zg0RExEogup782Wn0V90zH/The-Mint-Planner-19February-2016.html

Q.15) Consider the statements w.r.t Sacred Groves


1. It is included in Wildlife Protection Act, 1972
2. Nakshatravana, is a sacred grove consisting of trees
3. The sacred groves are also found in Rajasthan
Select the correct statement/s
a)
b)
c)
d)

1 and 2
2 and 3
Only 1
1, 2 and 3

Q.15) Solution (b)

Historical references to sacred groves can be obtained from ancient classics as far back
as Kalidasa's Vikramuurvashiiya. There has been a growing interest in creating green patches
such as Nakshatravana.
Sacred groves did not enjoy protection via federal legislation in India. Some NGOs work with
local villagers to protect such groves. Traditionally, and in some cases even today, members
of the community take turns to protect the grove. However, the introduction of the
protected area category community reserves under the Wild Life (Protection) Amendment
Act, 2002 has introduced legislation for providing government protection to community
held lands, which could include sacred groves.

Q.16) Consider the following statements with respect to Swadesh Darshan scheme

1. The scheme aims for Integrated Development of Tourist Circuits around Specific
Themes.

IASbaba 2016 Prelims 60 Days [Day 12]


2. Under this scheme 12 pilgrimage sites that were part of HRIDAY will be merged with
Swadesh Darshan scheme

Select the correct option

a)
b)
c)
d)

1 only
2 only
Both
None

Q.16) Solution (a)

The second statement is the explanation of PRASAD scheme.


HRIDAY scheme works under Ministry of Urban development.
Click here

Q.17) Consider the following

1.
2.
3.
4.
5.
6.

Amritsar
Amravati
Dwarka
Gaya
Kedarnath
Kamakhaya

Which of the following pilgrimage sites are included under HRIDAY SCHEME?
a)
b)
c)
d)

1,2,3, and 5 only


2,3,4 and 5 only
1,2,3 and 4 only
All

Q.17) Solution (c)

IASbaba 2016 Prelims 60 Days [Day 12]

All the places are included in the HRIDAY scheme except Kamakhaya and Kedarnath

Q.18) Indian Air force successfully test fired BrahMos missile from Pokran test range.
Consider the following statements with respect to BrahMos missile

1. It is a long range supersonic cruise missile


2. It consists of ram jet engine
3. It is co developed by India and Russia

Select the correct option

a)
b)
c)
d)

2 and 3 only
3 only
1, 2 and 3 only
1 and 2 only

Q.18) Solution (a)

Click here
It is a short range supersonic cruise missile

Q.19) Consider the following statements with respect to Sansad Adarsh Gram Yojana

1. Strengthening of local democracy through strong and transparent Gram Panchayat is


an important objective of SAGY
2. As per the scheme each MP is required to develop three model villages by 2019

Select the incorrect statement

IASbaba 2016 Prelims 60 Days [Day 12]


a)
b)
c)
d)

1 only
2 only
Both
None

Q.19) Solution (d)

Click here

Q.20) Mawphlang sacred forest is found in which of the following north-eastern state of
India

a)
b)
c)
d)

Sikkim
Manipur
Meghalaya
Nagaland

Q.20) Solution (c)

Click here

Q.21) Consider the following

1. Australia Group
2. Wassaner Arrangement
3. Missile Control Technology Regime

IASbaba 2016 Prelims 60 Days [Day 12]


4. Nuclear Non Proliferation Treaty

India is a member of which of the above treaties?

a)
b)
c)
d)

1 and 3 only
1 and 2 only
3 only
None

Q.21) Solution (d)

The Australia Group is an informal group of countries (now joined by the European
Commission) established in 1985 (after the use of chemical weapons by Iraq in 1984)
to help member countries to identify those exports which need to be controlled so
as not to contribute to the spread of chemical and biological weapons.
The Missile Technology Control Regime (MTCR) is an informal and voluntary
partnership
among
34
countries
to
prevent
the
proliferation of missile and unmanned aerial vehicle technology capable of carrying a
500 kg payload for at least 300 km.

The Wassenaar Arrangement is a multilateral export control regime (MECR) with 41


participating states including many former COMECON (Warsaw Pact) countries. The
Wassenaar Arrangement was established to contribute to regional and international
security and stability by promoting transparency and greater responsibility in
transfers of conventional arms and dual-use goods and technologies, thus preventing
destabilizing accumulations.
India, Pakistan, Israel, South Sudan, North Korea are countries that have not signed
NPT

Q.22) Consider the following statements with respect to Indian nuclear test and its spill
over effects.

IASbaba 2016 Prelims 60 Days [Day 12]


1. Operation Shakti was the assigned code name of India's first successful nuclear
bomb test on 1974.
2. The NSG was founded in response to the Indian nuclear test in 1974
3. Nuclear Supplier Group is a multinational body concerned with reducing nuclear
proliferation by controlling the export and re-transfer of nuclear materials

Select the correct option

a)
b)
c)
d)

All
1 and 2 only
2 and 3 only
1 and only

Q.22) Solution (c)

Smiling Buddha was the assigned code name of India's first successful nuclear bomb
test on 1974.
The NSG was founded in response to the Indian nuclear test in May 1974 and first
met in November 1975. The test demonstrated that certain non-weapons specific
nuclear technology could be readily turned to weapons development. Nations
already signatories of the Nuclear Non-Proliferation Treaty (NPT) saw the need to
further limit the export of nuclear equipment, materials or technology.

Q.23) Recently conservation of Indigenous cow breeds were in news. Consider the
following statements with respect to cows.

1. Cows normally tend to produce two types of milk known as A1 and A2


2. A2 milk is considered to be richer and helps human beings to fight various diseases.
3. Indigenous cow breed like Gir, Sahiwal, Gersy normally produce A2 type of milk

Select the incorrect answer using the codes given below

IASbaba 2016 Prelims 60 Days [Day 12]

a)
b)
c)
d)

1 and 3 only
3 only
1 and 2 only
All

Q.23) Solution (b)

Gersy is not indigenous cow breed.

Q.24) Global Burden on Food borne diseases is a report released by which of the following
international agencies

a)
b)
c)
d)

FAO
WHO
UNDP
WTO

Q.24) Solution (b)

The report is released by WHO

Q.25) Consider the following FDI contributors of India for 2015-16

1.
2.
3.
4.
5.

Mauritius
Singapore
USA
Netherlands
Switzerland

Arrange the above countries in decreasing order of FDI in 2015-2016?

IASbaba 2016 Prelims 60 Days [Day 12]

a)
b)
c)
d)

12354
21435
21345
12435

Q.25) Solution (c)


Singapore at US$ 10.99 billion, followed by Mauritius (US$ 6.12 billion), USA (US$ 3.51
billion), Netherlands (US$ 2.15 billion)

Click
Click

IASBABA Prelims 60 Day plan Day 13

Q.1) Atlas mountain is present in which of the following African countries


a)
b)
c)
d)

Algeria and Morocco


Mali and Libya
Tunisia and Sudan
Algeria and Egypt

Q.1) Solution (a)

The Atlas Mountains extend some 2,500km across northwestern Africa, spanning
Morocco, Algeria and Tunisia, separating the Atlantic and Mediterranean coastline from
the Sahara Desert.

Q.2) Consider the following


1. The scheme aims to provide financial assistance to patients of BPL families.
2. Rashtriya Aarogya Nidhi is an initiative of Ministry of health and family welfare.

Select the correct answer with respect to Rashtriya Aarogya Nidhi ?


a)
b)
c)
d)

1 only
2 only
Both
None

Q.2) Solution (c)


Self explanatory

Q.3) Consider the following


1.
2.
3.
4.

Indus
Ravi
Jhelum
Chenab

www.iasbaba.com

IASBABA Prelims 60 Day plan Day 13

5. Sutlej
6. Beas

Arrange the following rivers tributaries of Indus from north to south


a)
b)
c)
d)

123456
124356
214365
134265

Q.3) Solution (d)


Refer Atlas

Q.4) Consider the following


1. Wild life sanctuaries
2. National parks
3. Bio sphere reserves

Arrange the above in decreasing order of their number


a)
b)
c)
d)

231
213
321
123

Q.4) Solution (d)

Wild life sanctuaries - 551


National parks - 96
Bio sphere reserves 14

Q.5) Which of the following statement/s is/are true with respect to CURIE
www.iasbaba.com

IASBABA Prelims 60 Day plan Day 13

1. It is known as Consolidation of University Research for Innovation and Excellence


2. It is launched by Dept of S & T
3. The programme is launched for Women only Universities

Select the incorrect statement with respect to CURIE


a)
b)
c)
d)

1 only
2 and 3 only
3 only
None of the above

Q.5) Solution (d)

CURIE (Consolidation of University Research for Innovation and Excellence) is a unique


programme of Department of Science and Technology for strengthening S&T
infrastructure and also to enhance research facilities in women only universities.

Q.6) Consider the following statements with respect to Indian Ocean Naval Symposium
(IONS)
1. IONS is a voluntary and co-operative initiative between 30 countries of the Indian Ocean
Region for maritime security and cooperation.
2. The first inaugural IONS Seminar was held by the Indian Navy in 2008
3. Pakistan, France reunion, Mauritius are some of the members of the IONS

Select the correct option


a)
b)
c)
d)

1, 2 and 3
2 and 3 only
1 and 3 only
1 and 2 only

www.iasbaba.com

IASBABA Prelims 60 Day plan Day 13

Q.6) Solution (a)

All the statements are correct

Q.7) Consider the following


1. Chanderi silk Madhya Pradesh
2. Dorukha shawls Tamil Nadu
3. Illkal saree Karnataka

Which of the following sarees/shawl is wrongly matched?


a)
b)
c)
d)

1 and 2 only
2 only
1 only
2 and 3 only

Q.7) Solution (b)

Dorukha shawl is from Kashmir

Q.8) Consider the following


1.
2.
3.
4.
5.

Nepal
Bhutan
Maldives
Malaysia
Bangladesh

Which of the above are BIMSTEC (Bay of Bengal initiative for multi sectoral technical and
economic cooperation) countries?
a) 3, 4 and 5 only
b) 1 and 2 only
www.iasbaba.com

IASBABA Prelims 60 Day plan Day 13

c) 1, 2 and 5 only
d) 2, 3, 4 and 5 only

Q.8) Solution (c)

Self explanatory

Q.9) Consider the following statement with respect to TAPI pipeline


1. The TAPI pipeline transport Crude oil
2. It is funded by ADB (Asian development bank)
3. The TAPI pipeline terminates at Indian town of Ludhiana

Select the correct statement


a)
b)
c)
d)

1 and 2 only
2 and 3 only
2 only
1, 2 and 3

Q.9) Solution (c)

It transport natural gas


TAPI pipeline ends in Fazilka Punjab

Q.10) Which of the following statements with respect to initiative of AMRIT is correct?
1. It is headed by Ministry of Health and family welfare.
2. AMRIT aims to provide drugs for cardiovascular and cancer diseases at highly affordable
prices.

Select the correct option from the codes given below:

www.iasbaba.com

IASBABA Prelims 60 Day plan Day 13

a)
b)
c)
d)

1 only
2 only
Both
None

Q.10) Solution (c)

Union Ministry of Health & Family Welfare has opened countrys first Affordable
Medicines and Reliable Implants for Treatment (AMRIT) outlet at AIIMS in New Delhi to
sell drugs at highly discounted rates.

Key facts

The pilot AMRIT outlet will provide drugs for cancer and cardiovascular diseases at
highly discounted rates. At this outlet the prices of 202 cancer and 186 cardio-vascular
drugs will reduced on an average by 60 to 90 per cent.

The pilot AMRIT outlet will be managed by the government-owned HLL Life care Ltd
(HLL),

Provide drugs at affordable prices only on the prescription of a doctor for AIIMS patients
and also other hospitals

Q.11) Which of the following statements is/are Constitutional Provision for Amendment of
the Indian Constitution?
1) Parliament has been bestowed with the constituent power, using which changes in the
Indian Constitution can be made
2) It is mentioned in Constitution that there will be no limitation on the constituent power
of the Parliament for amending it by adding, removing or improving the provisions made
in it
Choose the appropriate code:
a) 1 only
b) 2 only
www.iasbaba.com

IASBABA Prelims 60 Day plan Day 13

c) 1 and 2 only
d) None of the above
Q.11) Solution (c)
Explanation:
Article 368 in Part XX of the Constitution deals with the powers of Parliament to amend the
Constitution and its procedure.

It states that the Parliament may, in exercise of its constituent power, amend by way of
addition, variation or repeal any provision of the Constitution in accordance with the
procedure laid down for the purpose.
Source: Chapter 10 - Amendment of the Constitution, Laxmikanth
Q. 12) Which among the following takes place during election and impeachment of President
of India?
1) The elected members of the legislative assemblies and councils of the states and UTs of
Delhi and Puducherry participate in election but do not participate in impeachment
2) The nominated members of either House of Parliament do not participate in election
but participate in impeachment
Choose the appropriate code:
a) 1 only
b) 2 only
c) 1 and 2 only
d) None of the above

Q.12) Solution (b)


Explanation:
The elected members of the legislative assemblies of the states and UTs of Delhi and
Puducherry participate in election but do not participate in impeachment (no legislative
councils) Hence statement 1 is wrong
The nominated members of either House of Parliament do not participate in election but
participate in impeachment
Source: Chapter 17 - President, Laxmikanth
Q. 13) Consider the following with regard to Ordinance making power of President:
1) Ordinance making is the discretionary power of the President of India
www.iasbaba.com

IASBABA Prelims 60 Day plan Day 13

2) President can promulgate or withdraw an ordinance only on the advice of Cabinet


Ministers
Which of the statements given above is/are incorrect?
a) 1 only
b) 2 only
c) Both 1 and 2
d) Neither 1 nor 2
Q.13) Solution (c)
Explanation:
Article 123 of the Constitution empowers the President to promulgate ordinances during
the recess of Parliament. These ordinances have the same force and effect as an act of
Parliament, but are in the nature of temporary laws.

He can promulgate an ordinance only when both the Houses of Parliament are not in
session or when either of the two Houses of Parliament is not in session. An ordinance can
also be issued when only one House is in session because a law can be passed by both the
Houses and not by one House alone. An ordinance made when both the Houses are in
session is void. Thus, the power of the President to legislate by ordinance is not a parallel
power of legislation.
However, his power of ordinance-making is not a discretionary power, and he can
promulgate or withdraw an ordinance only on the advice of the council of ministers headed
by the prime minister. (not Cabinet Ministers)

Source: Chapter 17 - President, Laxmikanth


Q. 14) Which among the following are correct with regard to Council of Ministers and
Cabinet?
1) Council of Ministers does not meet, as a body, to transact government business. It has
no collective functions.
2) Cabinet meets, as a body, frequently and usually once in a week to deliberate and take
decisions regarding the transaction of government business. Thus, it has collective
functions.
3) Council of Ministers enforces the collective responsibility of the cabinet to the Lower
House of Parliament.
4) Cabinet is collectively responsible to the Lower House of the Parliament.
Choose the correct code from below options:
a) 1 and 2 only
www.iasbaba.com

IASBABA Prelims 60 Day plan Day 13

b) 2 and 4 only
c) 3 and 4 only
d) All of the above
Q.14) Solution (a)
Explanation:
Council of Ministers does not meet, as a body, to transact government business. It has no
collective functions.
Cabinet meets, as a body, frequently and usually once in a week to deliberate and take
decisions regarding the transaction of government business. Thus, it has collective
functions.
Cabinet enforces the collective responsibility of the council of ministers to the Lower House
of Parliament.
Council of Ministers is collectively responsible to the Lower House of the Parliament.
Chapter 20 Central Council of Ministers, Laxmikanth
Q. 15) Consider the following statements:
1) If the election of a person as President is declared void by the Supreme Court, acts done
by him before the date of such declaration of the Supreme Court are not invalidated and
continue to remain in force
2) Impeachment is a quasi-judicial procedure
3) During impeachment, the President cannot defend himself by taking service of the
Attorney General or the Solicitor General or the Additional Solicitors General of India
Which of the statements given above is/are incorrect?
a) 1 and 3 only
b) 1 only
c) 2 only
d) 3 only
Q.15) Solution (d)
Explanation:

If the election of a person as President is declared void by the Supreme Court, acts done by
him before the date of such declaration of the Supreme Court are not invalidated and
continue to remain in force
Impeachment is a quasi-judicial procedure

www.iasbaba.com

IASBABA Prelims 60 Day plan Day 13

During impeachment, the President can defend himself by taking service of the Attorney
General of India or any other lawyer he wishes for

Source: Chapter 17 - President, Laxmikanth


Q.16) Consider the following statements:
1) Central Legislation can over-ride the veto power of President in the case of suspensive
veto
2) State Legislature cannot over-ride the veto power of President in the case of suspensive
veto
Which of the statements given above is/are correct?
a) 1 only
b) 2 only
c) 1 and 2 only
d) None of the above
Q.16) Solution (c)
Explanation:
Suspensive Veto:

The President exercises this veto when he returns a bill for reconsideration of the
Parliament. However, if the bill is passed again by the Parliament with or without
amendments and again presented to the President, it is obligatory for the President to give
his assent to the bill. This means that the presidential veto is overridden by a re-passage of
the bill by the same ordinary majority.
When a bill is reserved by the governor for the consideration of the President, the President
may direct the governor to return the bill (if it is not a money bill) for the reconsideration of
the state legislature. If the bill is passed again by the state legislature with or without
amendments and presented again to the President for his assent, the President is not
bound to give his assent to the bill. This means that the state legislature cannot override the
veto power of the President.

Source: Chapter 17 - President, Laxmikanth


Q.17) Consider the case of constitutional amendment bill, where the consent of the state
legislatures is required. Which among the following statements is/are true?

www.iasbaba.com

IASBABA Prelims 60 Day plan Day 13

1) The Constitution does not prescribe the time frame within which the state legislatures
should ratify or reject an amendment submitted to them
2) The Constitution is silent on the issue whether the states can withdraw their approval
after according the same
Choose the appropriate code:
a) 1 only
b) 2 only
c) 1 and 2 only
d) None of the above

Q.17) Solution (c)


Explanation:
Self-explanatory (both statements are correct)
Source: Chapter 10 - Amendment of the Constitution, Laxmikanth
Q.18) Recently, there was news that Tamil Nadu government decided to remit the life
sentences of all the seven convicts in the Rajiv Gandhi assassination case. Which among the
following statements is/are true?
1) State has to consult the Centre before releasing prisoners prosecuted by the CBI or
under a Central law
2) Governor of a State has the power to suspend, remit or commute the sentence of any
person convicted of any offence against any law relating to a matter to which the
executive power of the State extends
Choose the appropriate code:
a) 1 only
b) 2 only
c) 1 and 2 only
d) None of the above
Q.18) Solution (c)
Explanation:

Tamil Nadu government decided to remit the life sentences of all the seven convicts in the
Rajiv Gandhi assassination case and release them as they had already served over 24 years
in prison.

www.iasbaba.com

IASBABA Prelims 60 Day plan Day 13

Under Section 435 of the Cr.PC, the State has to consult the Centre before releasing
prisoners prosecuted by the CBI or under a Central law.
In December last year, the Supreme Court had ruled that the State government had no
power to release the Rajiv Gandhi case convicts without the Centres concurrence.
Article 161: Governor of a State has the power to grant pardons, reprieves, respites or
remissions of punishment or to suspend, remit or commute the sentence of any person
convicted of any offence against any law relating to a matter to which the executive power

of the State extends. Please note that President can grant pardon to a person awarded
death sentence. But Governor of State does not enjoy this power.
Source: Chapter 17 - President, Laxmikanth
TN seeks Centre's views on freeing Rajiv case convicts - The Hindu
Q.19) With reference to Union Government, consider the following statements:
1) The Ministries Departments of the Government of India are created by the Prime
Minister on the advice of the Cabinet Secretary
2) Each of the Ministries is assigned to a Minister by the President of India on the advice of
the Prime Minister
Which of the statements given above is/are correct?
a) 1 only
b) 2 only
c) 1 and 2 only
d) None of the above
Q.19) Solution (b)
Explanation:

The Ministries/Departments of the Government of India are created by the President on the
advice of the Prime Minister under the Government of India (Allocation of Business) Rules,
1961
The business of the Government of India are transacted in the ministries/departments,
secretariats and offices (referred to as "Department") as per the distribution of subjects
specified in these Rules.
Each of the Ministry (ies) will be assigned to a Minister by the President on the advice of the
Prime Minister. Each department will be generally under the charge of a Secretary to assist
the Minister on policy matters and general administration.

Source: Chapter 20 Central Council of Ministers, Laxmikanth


www.iasbaba.com

IASBABA Prelims 60 Day plan Day 13

UPSC 2009 Question, Explanation link: http://www.eoi.gov.in/ashgabat/?0775?000#


Q.20) Consider the following statements in regard to constitutional amendment bill:
1) The constitutional amendment bill must be passed in each House by a majority of more
than 50 per cent of the total strength, which includes even those members who are
abstaining
2) In case of joint sitting, for the purpose of deliberation and passage of constitutional
amendment bill, the bill must be passed by a simple
majority Which of the statements given above is/are correct?
a) 1 only
b) 2 only
c) 1 and 2 only
d) None of the above
Q.20) Solution (d)
Explanation:

The constitutional amendment bill must be passed in each House by a special majority, that
is, a majority (that is, more than 50 per cent) of the total membership of the House and a
majority of two-thirds of the members of the House present and voting.

In 1 statement, it is explaining about Absolute Majority (hence statement 1 is wrong)

st

There is no provision of joint sitting, for the purpose of deliberation and passage of
constitutional amendment bill (so statement 2 is also wrong)

Source: Chapter 10 - Amendment of the Constitution, Laxmikanth

Q.21) For which of the following cases, constitutional amendment has been done?
1.
2.
3.
4.

Reduction in voting rights from 21 to 18


Introducing any language as official language
Substituting Odia for Oriya
Restrict the size of council of ministers to 15% of legislative members

Select the appropriate code


a) 1 and 3
www.iasbaba.com

IASBABA Prelims 60 Day plan Day 13

b) 2, 3 and 4
c) 1, 2 and 4
d) 1, 2, 3 and 4
Q.21) Solution (d)
st

Reduction in voting rights from 21 to 18- 61 Amendment


Introducing any language as official language- Amendment in schedule 8
Substituting Odia for Oriya- Amendment in Schedule
st

Restrict the size of council of ministers to 15% of legislative members- 91 Amendment

Q.22) Oath or affirmation which says Preserve, Protect and Defend Constitution of India is
taken by?
1. Governor
2. Prime Minister
3. President
4. Vice- President
5. Chief Minister
Select the correct code

a) 1, 2, 4 and 5
b) 2, 3, 4 and 5
c) 1 and 3
d) 3 and 4

Q.22) Solution (c)


Article 60 {Oath or affirmation by the President}
www.iasbaba.com

IASBABA Prelims 60 Day plan Day 13

Every President and every person acting as President or discharging the functions of the
President shall, before entering upon his office, make and subscribe in the presence of the Chief
Justice of India or, in his absence, the senior most Judge of the Supreme Court available, an
oath or affirmation in the following form, that is to say - "I, A.B., do swear in the name of God /
solemnly affirm that I will faithfully execute the office of President (or discharge the function
of the President) of India and will to the best of my ability preserve, protect and defend the
Constitution and the law and that I will devote myself to the service and well-being of the
people of India."

Article 159 {Oath or affirmation by the Governor}


Every Governor and every person discharging the functions of the Governor shall, before
entering upon his office, make and subscribe in the presence of the Chief Justice of the High
court exercising jurisdiction in relation to the State, or, in his absence, the senior most Judge of
that Court available, an oath or affirmation in the following form, that is to say - "I, A.B., do
Swear in the name of God / solemnly affirm that I will faithfully execute the office of
Governor (or discharge the functions of the Governor) of ...................... (name of the State)
and will to the best of my ability preserve, protect and defend the Constitution and the law
and that I will devote myself to the service and well-being of the people of ....................
(name of the State)."

Q.23) Consider the following regarding Attorney General of India


1. He belongs to the category of Government Servants
2. He is full time counsel of Government of India
3. He doesnt enjoy privileges and immunities that are available to a Member of
Parliament.
Select the incorrect code
a) Only 3
b) 1 and 2
c) 2 and 3
www.iasbaba.com

IASBABA Prelims 60 Day plan Day 13

d) 1, 2 and 3

Q.23) Solution (d)


In the performance of his official duties, the Attorney General has the right of audience in all
courts in the territory of India. Further, he has the right to speak and to take part in the
proceedings of both the Houses of Parliament or their joint sitting and any committee of the
Parliament of which he may be named a member, but without a right to vote. He enjoys all the
privileges and immunities that are available to a Member of Parliament.
However, the Attorney General is not a full-time counsel for the Government. He does not fall
in the category of government servants. Further, he is not debarred from private legal practice.
Source: Chapter on Attorney General of India, Laxmikanth

Q.24) The present relationship between the President and the council of ministers is
governed by the provisions of:
a) 42nd Amendment Act
b) 48th Amendment Act
c) 54th Amendment Act
d) 44th Amendment Act

Q.24) Solution (d)

Q.25) Consider the following statements:


1. An amendment to the Constitution of India can be initiated by an introduction of a bill in
the Lok Sabha only.
www.iasbaba.com

IASBABA Prelims 60 Day plan Day 13

2. If such an amendment seeks to make changes in the federal character of the


Constitution, the amendment also requires to be ratified by the legislature of all the
States of India.
Which of the statements given above is/are correct?
a) 1 only
b) 2 only
c) Both 1 and 2
d) Neither 1 nor 2

Q.25) Solution (d)

www.iasbaba.com

IASBABA 60 Day plan Day 14


Q.1) Consider the following
1. Measles Rubella
2. Japanese Encephalitis
3. Rotavirus
Which of the above diseases are covered under mission Indradhanush?
a)
b)
c)
d)

1, 2 and 3
2 and 3 only
1 only
None of the above

Q.1) Solution (c)

Q.2) Consider the following statements


1. National green tribunal is enacted under Article 21 of Indian constitution.
2. The tribunal is guided by principles of natural justice and not code of civil procedure,
1908
Select the correct option
a)
b)
c)
d)

1 only
2 only
Both
None

Q.2) Solution (c)

National Green Tribunal Act, 2010 (NGT) is an Act of the Parliament of India which
enables creation of a special tribunal to handle the expeditious disposal of the cases
pertaining to environmental issues. It was enacted under India's constitutional
provision of Article 21, which assures the citizens of India the right to a healthy
environment.

www.iasbaba.com

Page 1

IASBABA 60 Day plan Day 14

Q.3) Pradhan Mantri Krushi Sinchai Yojana, PMKSY aims to bring rationality and
investment in irrigation and bring water in every field. Consider the following
1. Accelerated Irrigation Benefit Programme (AIBP)
2. Integrated Watershed Management Programme (IWMP)
3. On Farm Water Management (OFWM)
Pradhan Mantri Krishi Sinchana yojana is formulated by amalgamating which of the above
schemes
a)
b)
c)
d)

1 and 2 only
2 and 3 only
1 and 3 only
All

Q.3) Solution (d)

Click here

Pradhan Mantri Krishi Sinchayee Yojana (PMKSY) has been formulated amalgamating
ongoing schemes viz. Accelerated Irrigation Benefit Programme (AIBP) of Ministry of
Water Resources, River Development & Ganga Rejuvenation; Integrated Watershed
Management Programme (IWMP) of Department of Land Resources; and On Farm
Water Management (OFWM) component of National Mission on Sustainable
Agriculture (NMSA) of Department of Agriculture and Cooperation.

Q.4) Consider the following


1. Pradhan Mantri Awas Yojana also known as Housing for all aims to provide quality
homes at affordable prices by 2022.
2. The scheme is eligible only for BPL families
Select the correct option with respect to PMAY (Pradhan Mantri Awas Yojana)
a)
b)
c)
d)

1 only
2 only
Both
None

www.iasbaba.com

Page 2

IASBABA 60 Day plan Day 14


Q.4) Solution (a)
The scheme is not only eligible for BPL families but also low income groups, economically
weaker section and women.

Q.5) Consider the following


1.
2.
3.
4.

Tamil Nadu
Assam
Kerala
Karnataka

Coffee is grown in which of the above states


a)
b)
c)
d)

1, 2 and 4 only
4 only
3 and 4 only
1, 3 and 4 only

Q.5) Solution (d)


Refer page number 37 in Oxford atlas

Q.6) Consider the following


1. Horn of Africa
2. Gulf of Mannar
3. Sri Lanka
Which of the above is/are biodiversity hotspot?
a)
b)
c)
d)

1 and 2 only
2 only
1 and 3 only
2 and 3 only

www.iasbaba.com

Page 3

IASBABA 60 Day plan Day 14


Q.6) Solution (c)
List of Biodiversity Hotspots

North and Central America: California Floristic Province, Madrean pine-oak


woodlands, Mesoamerica
The Caribbean: Caribbean Islands
South America: Atlantic Forest, Cerrado, Chilean Winter Rainfall-Valdivian Forests,
Tumbes-Choc-Magdalena, Tropical Andes
Europe: Mediterranean Basin
Africa: Cape Floristic Region, Coastal Forests of Eastern Africa, Eastern Afromontane,
Guinean Forests of West Africa; Horn of Africa; Madagascar and the Indian Ocean
Islands; Maputaland-Pondoland-Albany; Succulent Karoo
Central Asia: Mountains of Central Asia;

South Asia: Eastern Himalaya, Nepal; Indo-Burma, India and Myanmar; Western
Ghats, India; Sri Lanka

South East Asia and Asia-Pacific: East Melanesian Islands; New Caledonia; New
Zealand; Philippines; Polynesia-Micronesia; Southwest Australia; Sundaland;
Wallacea;
East Asia: Japan; Mountains of Southwest China
West Asia: Caucasus; Irano-Anatolian

Q.7) Consider the following statements with respect to defects of eye and there correction
1. Myopia is a condition of long sightedness
2. People with myopia use convex lens
Select the correct option from code given below
a)
b)
c)
d)

1 only
2 only
Both
None

Q.7) Solution (d)

People affected by Myopia does not able to see far off objects

www.iasbaba.com

Page 4

IASBABA 60 Day plan Day 14

In a myopic eye, the image of a distant object is formed in front of the retina and not
at the retina itself. This defect may arise due to

(i) Excessive curvature of the eye lens, or (ii) elongation of the eyeball. This defect
can be corrected by using a concave lens of suitable power.

Source Ncert Chap 10 class 10

th

Q.8) Domestic electric wiring is basically a


a)
b)
c)
d)

Series connection
Parallel connection
Combination of series and parallel
Series connection with each room and parallel connection elsewhere.

Q.8) Solution (b)

Parallel connection because electricity can be supplied equally to all the instruments
in a house and also when one circuit is broken electric current still flows in other
circuits.

Q.9) Global energy architecture performance index report (GEAPI) is released by which of
the following international institutions.
a)
b)
c)
d)

World economic forum


United Nations development programme
United Nations environment programme
International renewable energy agency

Q.9) Solution (a)

World economic forum releases Global energy architecture performance index


report (GEAPI).

It explored the energy architecture of 126 countries based on their ability to provide
energy access across three dimensions of the "energy triangle" affordability,
environmental sustainability, security and access.
India has been ranked at 90 out of 126 countries.
Click here to know more about the report.

Q.10) Consider the following


www.iasbaba.com

Page 5

IASBABA 60 Day plan Day 14


1. Project Mausam is transnational initiative meant to revive its ancient maritime
routes and cultural linkages between countries of the Indian Ocean world
2. It was launched at 38th World Heritage Session at Doha in 2012
Select the correct option
a)
b)
c)
d)

1 only
2 only
Both
None

Q.10) Solution (a)

Project Mausam is transnational initiative meant to revive its ancient maritime


routes and cultural linkages between countries of the Indian Ocean world
It was launched at 38th World Heritage Session at Doha in 2014

Q.11) The final interpreter of the provisions of the Constitution of India within the House
is
a) Attorney General
b) President
c) Speaker
d) Supreme Court
Q.11) Solution (c)
Explanation:

Speaker is the final interpreter of the provisions of (a) the Constitution of India, (b) the
Rules of Procedure and Conduct of Business of Lok Sabha, and (c) the parliamentary
precedents, within the House.

Q.12) Consider the following statements in regard to representation of UTs in the second
chamber:
1) The Constitution has empowered the Parliament to prescribe the manner of
choosing the representatives of the UTs to the second chamber
2) The Parliament has enacted the Union Territories (Indirect Election to the Second
Chamber) Act, 1965, by which the representatives of the UTs are chosen
3) The representatives of each UT in the second chamber are indirectly elected by
members of an electoral college specially constituted for the purpose

www.iasbaba.com

Page 6

IASBABA 60 Day plan Day 14


4) This election is held in accordance with the system of proportional representation by
means of the single transferable vote
Which of the statements given above is/are correct?
a)
b)
c)
d)

2 and 3 only
3 and 4 only
1, 3 and 4 only
All of the above

Q.12) Solution (b)


Explanation:
Representation of Union Territories in Rajya Sabha

The Constitution has empowered the Parliament to prescribe the manner of choosing
the representatives of the UTs in the Lok Sabha (not second chamber i.e., Rajya Sabha).
Accordingly, the Parliament has enacted the Union Territories (Direct Election to the
House of the People) Act, 1965, by which the representatives of the UTs are chosen.
The representatives of each union territory in the Rajya Sabha are indirectly elected by
members of an electoral college specially constituted for the purpose. This election is
also held in accordance with the system of proportional representation by means of the
single transferable vote.
Out of the seven union territories, only two (Delhi and Puducherry) have representation
in Rajya Sabha. The populations of other five union territories are too small to have any
representative in the Rajya Sabha.

Q.13) Consider the following statements:


1) Under the Rules of Rajya Sabha, the Chairman nominates from amongst the
members a panel of vice-chairpersons
2) Any of them can preside over the House when the office of the Speaker or the
Deputy Speaker is vacant
Which of the statements given above is/are incorrect?
a)
b)
c)
d)

1 only
2 only
Both 1 and 2
Neither 1 nor 2

www.iasbaba.com

Page 7

IASBABA 60 Day plan Day 14


Q.13) Solution (b)
Explanation:

Under the Rules of Rajya Sabha, the Chairman nominates from amongst the members a
panel of vice-chairpersons.
Any one of them can preside over the House in the absence of the Chairman or the
Deputy Chairman.
It must be emphasized here that a member of the panel of vice-chairpersons cannot
preside over the House, when the office of the Chairman or the Deputy Chairman is
vacant.
During such time, the Chairmans duties are to be performed by such member of the
House as the president may appoint for the purpose. The elections are held, as soon as
possible, to fill the vacant posts.

Q.14) Which among the given statements is/are true in regard to the Speaker of Lok
Sabha?
1) He is the guardian of powers and privileges of the members, the House as a whole
and its committees
2) President enables the House to elect the new Speaker
3) Parliamentary Affairs Committee, which looks after the progress of government
business in the Parliament, is chaired by the Speaker
Select the appropriate code
a)
b)
c)
d)

1 only
1 and 2 only
2 only
All of the above

Q.14) Solution (a)


Explanation:

Speaker is the guardian of powers and privileges of the members, the House as a whole
and its committees.

www.iasbaba.com

Page 8

IASBABA 60 Day plan Day 14

The main duty of Speaker Pro Tem is to administer oath to the new members. He also
enables the House to elect the new Speaker.
Parliamentary Affairs Committee, which looks after the progress of government business
in the Parliament, is chaired by the Home Minister (not Speaker).

Q.15) From the following devices of Parliamentary Proceedings, select only the Indian
innovated Parliamentary procedures:
1)
2)
3)
4)

Zero hour
Short notice question
Calling attention motion
No-day- yet-named motion

Choose appropriate option:


a)
b)
c)
d)

1 and 3
2 and 3
1, 2 and 4
2, 3 and 4

Q.15) Solution (a)


Explanation:

Question Hour: The first hour of every parliamentary sitting is slotted for this. During
this time, the members ask questions and the ministers usually give answers. The
questions are of three kinds, namely, starred, unstarred and short notice.
A short notice question is one that is asked by giving a notice of less than ten days. It is
answered orally.
Unlike the question hour, the zero hour is not mentioned in the Rules of Procedure.
Thus it is an informal device available to the members of the Parliament to raise matters
without any prior notice.
The zero hour starts immediately after the question hour and lasts until the agenda for
the day (ie, regular business of the House) is taken up. In other words, the time gap
between the question hour and the agenda is known as zero hour. It is an Indian
innovation in the field of parliamentary procedures and has been in existence since
1962.
Calling Attention Motion: It is introduced in the Parliament by a member to call the
attention of a minister to a matter of urgent public importance, and to seek an
authoritative statement from him on that matter. Like the zero hour, it is also an Indian

www.iasbaba.com

Page 9

IASBABA 60 Day plan Day 14


innovation in the parliamentary procedure and has been in existence since 1954.
However, unlike the zero hour, it is mentioned in the Rules of Procedure.
Q.16) Consider the following statements in regard to Money Bill and Financial Bill:
1) Money bills and financial bills can be introduced only on the recommendation of the
President
2) Financial Bills are governed by the same legislative procedure which is applicable to
an ordinary bill
3) The only special feature of Financial Bills is that it cannot be passed by either House
of Parliament unless the President has recommended to that House the
consideration of the bill
Which of the statements given above is/are correct?
a) 1 and 3 only
b) 1 and 2 only
c) 2 only
d) 3 only
Q.16) Solution (c)
Explanation:

Money Bill and Financial Bill (I) can be introduced only on the recommendation of the
President, but recommendation of the President is not necessary for the introduction of
Financial bill (II)
Financial Bills (both I and II) are governed by the same legislative procedure which is
applicable to an ordinary bill
The only special feature of Financial Bill (II) is that it cannot be passed by either House of
Parliament unless the President has recommended to that House the consideration of
the bill (Financial Bill (I) has no such special feature)

Q.17) Which among the following statements is/are not true in regard to Speaker and
Deputy Speaker of Lok Sabha?
1) The institutions of Speaker and Deputy Speaker originated in India under the
provisions of the Government of India Act of 1935
2) President administers the oath of the Speaker and Speaker administers the oath of
Deputy Speaker and remaining members of the LS
3) Speaker acts as strictly a non-party man in India, he resigns from the membership of
his party on his election to the exalted office

www.iasbaba.com

Page 10

IASBABA 60 Day plan Day 14


Select the appropriate code
a)
b)
c)
d)

1 and 2 only
2 and 3 only
2 only
All of the above

Q.17) Solution (d)


Explanation:

The institutions of Speaker and Deputy Speaker originated in India in 1921 under the
provisions of the Government of India Act of 1919 (MontagueChelmsford Reforms). At
that time, the Speaker and the Deputy Speaker were called the President and Deputy
President respectively and the same nomenclature continued till 1947.
The Speaker and the Deputy Speaker, while assuming their offices, do not make and
subscribe any separate oath or affirmation.
In Britain, the Speaker is strictly a non-party man. There is a convention that the Speaker
has to resign from his party and remain politically neutral. This healthy convention is not
fully established in India where the Speaker does not resign from the membership of his
party on his election to the exalted office.

Q.18) Which among the following is/are correct in regard to Motions associated with
Parliamentary functions?
1) No-Confidence Motion and Censure motion can be moved against the entire council
of ministers
2) Calling Attention Motion is introduced in the Parliament to draw attention of the
House to a definite matter of urgent public importance
3) All resolutions come in the category of substantive motion
Select the appropriate code
a)
b)
c)
d)

1 and 3 only
2 and 3 only
2 only
All of the above

Q.18) Solution (a)


Explanation:
www.iasbaba.com

Page 11

IASBABA 60 Day plan Day 14

Censure motion can be moved against an individual minister or a group of ministers or


the entire council of ministers. No-Confidence Motion can be moved against the entire
council of ministers only.
It is Adjournment Motion (not Calling Attention Motion), which is introduced in the
Parliament to draw attention of the House to a definite matter of urgent public
importance.
Calling Attention Motion is introduced in the Parliament by a member to call the
attention of a minister (not the House) to a matter of urgent public importance, and to
seek an authoritative statement from him on that matter.
All resolutions come in the category of substantive motions, that is to say, every
resolution is a particular type of motion.

Q.19) Match List I with List II and select the correct answer using the code given below the
Lists:
List I
List II
(Parliamentary Committees)
(Consists of)
A. Public Accounts Committee
1) 22 members (15 from LS and 7 from RS)
B. Estimates Committee
2) 30 members (All from LS only)
C. Standing committee
3) 31 members (21 from LS and 10 from RS)
D. Committee on Empowerment
4) 30 members (20 from LS and 10 from RS)
of Women

a)
b)
c)
d)

A-B-C-D
1-2-4-3
2-1-4-3
2-1-3-4
1-2-3-4

Q.19) Solution (d)


Explanation:
Self Explanatory
Note: This questions intention is to make you aware of the Committees of the
Parliament and its composition (as UPSC has started asking questions on these recently)

www.iasbaba.com

Page 12

IASBABA 60 Day plan Day 14


Q.20) Consider the following statements with regard to Parliamentary Privileges:
1) Without these privileges, the Parliament can neither maintain their authority, dignity
and honor nor can protect their members from any obstruction in the discharge of
their parliamentary responsibilities
2) The Constitution has extended Parliamentary privileges to the Parliament, including
the attorney general of India
Which of the statements given above is/are correct?
a)
b)
c)
d)

1 only
2 only
Both 1 and 2
Neither 1 nor 2

Q.20) Solution (d)


Explanation:

Parliamentary privileges are special rights, immunities and exemptions enjoyed by the
two Houses of Parliament, their committees and their members. They are necessary in
order to secure the independence and effectiveness of their actions.
Without these privileges, the Houses (not the Parliament as it includes President of India
and both the houses) can neither maintain their authority, dignity and honor nor can
protect their members from any obstruction in the discharge of their parliamentary
responsibilities.
The Constitution has also extended the parliamentary privileges to those persons who
are entitled to speak and take part in the proceedings of a House of Parliament or any of
its committees. These include the attorney general of India and Union ministers.
It must be clarified here that the parliamentary privileges do not extend to the president
who is also an integral part of the Parliament.
Note: As Parliament consists of President, both the statements are incorrect.

Note: Source for all these questions is Laxmikanth, Parliament Chapter

Q.21) Consider the following regarding Public Accounts Committee

www.iasbaba.com

Page 13

IASBABA 60 Day plan Day 14


1. The chairman of the committee must belong to the ruling party
2. The function of the committee is to examine the annual audit reports of the
Comptroller and Auditor General of India (CAG), which are laid before the Parliament
by the Speaker
3. It is a non-advisory and non-executive body
Select the correct code
a) 1 and 2
b) Only 2
c) Only 3
d) None

Q.21) Solution (d)


Until 1966-67, the chairman of the committee belonged to the ruling party. However, since
1967 a convention has developed whereby the chairman of the committee is selected
invariably from the Opposition. The function of the committee is to examine the annual
audit reports of the Comptroller and Auditor General of India (CAG), which are laid before
the Parliament by the President. The CAG submits three audit reports to the President,
namely, audit report on appropriation accounts, audit report on finance accounts and audit
report on public undertakings.
However, the effectiveness of the role of the committee is limited by the following:
(a) It is not concerned with the questions of policy in broader sense.
(b) It conducts a post-mortem examination of accounts (showing the expenditure already
incurred).
(c) It cannot intervene in the matters of day-to-day administration.
(d) Its recommendations are advisory and not binding on the ministries.
(e) It is not vested with the power of disallowance of expenditures by the departments.
(f) It is not an executive body and hence, cannot issue an order. Only the Parliament can
take a final decision on its findings
Q.22) Consider the following w.r.t Ethics Committee of Lok Sabha and Rajya Sabha
1. Ethics Committee has an Ad Hoc status in Lok Sabha whereas a Permanent Standing
Committee Status in Rajya Sabha
2. Both the houses explicitly provide for a Register of Members Interest, where MPs
have to declare their interest in certain categories

www.iasbaba.com

Page 14

IASBABA 60 Day plan Day 14


3. Lok Sabhas Ethics Committee acts both on complaints as well as takes up issues suo
motu, Rajya Sabhas committee acts only on complaints made either by any member
of the public or any other member of the House
Select the incorrect statement/s
a) 1 and 2 Only
b) 1, 2 and 3
c) 2 and 3
d) Neither of them

Q.22) Solution (b)


Lok Sabha has a new permanent Standing Committee on Ethics which came into force on
12th August, 2015. Until now, the ethics committee in Lok Sabha has been an ad hoc one.
Rajya Sabha has explicitly provided for a Register of Members Interest, where MPs have to
declare their interest in 5 categories: remunerative directorship, remunerated activity,
majority shareholding, paid consultancy and professional engagement. In addition to that,
members are required to declare any financial interest on an issue that is being debated in
the House or under consideration by any other standing committee and hence refrain from
taking part to avoid conflict of interest.
Lok Sabha does not maintain such a registry of members interests and apart from disclosing
their assets and liabilities, MPs are not obliged to declare other financial interests that might
be in direct or indirect conflict with their role as public servants.
Another significant point of difference between the two Houses is that while Rajya Sabhas
Ethics Committee acts both on complaints as well as takes up issues suo motu, Lok Sabhas
committee acts only on complaints made either by any member of the public or any other
member of the House.
Read more- click
Q.23) Consider the following
1. Temperature
2. Pressure
3. Medium
What are the factors that affect the speed of sound from above?
a) Only 3
b) 1 and 3
www.iasbaba.com

Page 15

IASBABA 60 Day plan Day 14


c) 2 and 3
d) 1, 2 and 3

Q.23) Solution (d)


Here are the factors that affect the speed of sound:
Medium
Medium has a huge effect of the speed of sound. When most people discuss the speed of
sound they are talking about the propagation of sound waves through the medium of Air.
For anyone who has gone underwater and listen to people talking above it is likely that one
would notice the muted an odd way that voices sound underwater. This is because the
medium of water greatly bends, distorts and changes the speed of sound wave.
There is a whole aspect of science that measure and defines the effect of different mediums
(gaseous and liquid) on the speed of sound. This is called Fluid Dynamics. Underwater
communication is possible if you understand how this wave propagation as well as another
important factor (pressure).
Because of elasticity of materials sound will, as a rule of thumb, generally travel faster in
solids than in liquids and faster in liquids than in gases.
Temperature
Temperature has a large effect on the speed of sound. Not as much as the Medium does,
but far more than anything else. Temperature affects the speed of sound because
temperature can affect the elastic qualities of different mediums. At the very basics lower
temperatures will decrease the speed of sound while higher temperatures will increase the
speed of sound, all other factors being equal.
Pressure
Pressure is the final factor that has a significant impact on the speed of sound. The effect of
pressure on the speed of sound is due to the materials inertial properties. In short, the more
pressure that is applied to the material or medium the denser it becomes and the greater
the inertia becomes. This makes any interactions between particles slower. Therefore the
speed of sound throughout the medium is slowed due to the greater pressure.

Q.24) Consider the following regarding Aerogels


1. They are liquid having low density
2. They have high thermal conductivity
3. Metal Oxide aerogels are the most common type of aerogels
Select the incorrect code
www.iasbaba.com

Page 16

IASBABA 60 Day plan Day 14


a) 1 and 2
b) 2 and 3
c) Only 3
d) 1, 2 and 3

Q.24) Solution (d)


Aerogels are solid having low density and low thermal conductivity. Silica aerogel is the
most common type of aerogel, and the most extensively studied and used.
http://www.thehindu.com/todays-paper/tp-national/paper-waste-turned-into-nontoxicaerogel/article8207322.ece
Q.25) Consider the following w.r.t Red Panda
1. It is an arboreal animal
2. It is categorized as Threatened under IUCN Red List
3. They eat mostly bamboo, and may eat small mammals, birds and eggs etc
Select the appropriate code
a) Only 1
b) 1 and 2
c) 1, 2 and 3
d) 1 and 3

Q.25) Solution (d)


Arboreal means spending most time on trees.
They are endangered not threatened. Red pandas are excellent climbers, and forage largely
in trees. They eat mostly bamboo, and may eat small mammals, birds, eggs, flowers, and
berries. In captivity, they were observed to eat birds, flowers, maple and mulberry leaves,
and bark and fruits of maple, beech, and mulberry
Click
Click

www.iasbaba.com

Page 17

IASBABA 60 Day plan Day 14

www.iasbaba.com

Page 18

IASbaba 60 Day Plan- Prelims Test 2016 GEOGRAPHY & CURRENT AFFAIRS [DAY 15]

Q.1) Which among the following are benefits of Atmosphere?


1)
2)
3)
4)
5)

It protects from harmful radiation


It contains living gases
Acts as Green house, allows long-wave radiation and traps short-wave radiation
Acts as medium for fast air transport
Storehouse for water vapor and leads to precipitation

Choose the appropriate code:


a)
b)
c)
d)

2, 3 and 5 only
1, 2, 3 and 5 only
1, 2, 4 and 5 only
All of the above

Q.1) Solution (c)

Only 3rd statement is wrong, because atmosphere acts as Green house by allowing shortwave radiation (from Sun) and trapping long-wave terrestrial radiation (from Earths
surface)

Q.2) Arrange the following rays according to their wavelengths (from short-wavelength to
long)
1)
2)
3)
4)
5)

X-rays
-rays
UV-rays
Visible rays
Infrared rays

Choose the appropriate code:


a) 2-1-3-5-4
b) 1-2-3-5-4
c) 1-2-3-4-5
www.iasbaba.com

Page 1

IASbaba 60 Day Plan- Prelims Test 2016 GEOGRAPHY & CURRENT AFFAIRS [DAY 15]

d) 2-1-3-4-5
Q.2) Solution (d)
The types of electromagnetic radiation are listed below in fig.

(See: http://www.darvill.clara.net/emag/images/scale.jpg)
Q.3) Consider the following statements in regard to Westerlies:
1) Westerlies blow from the Sub-tropical high Pressure Belt to the Sub-polar low Pressure
Belt in the temperature latitudes between 30 N and 60 S latitudes
2) They are more constant and stronger in the Southern Hemisphere because there are no
large landmasses to interrupt them
3) It got the name Westerlies because they blow out of the west
Which of the statements given above is/are incorrect?
a)
b)
c)
d)

1, 2 and 3 only
1 and 2 only
1 only
None

Q.3) Solution (c)

www.iasbaba.com

Page 2

IASbaba 60 Day Plan- Prelims Test 2016 GEOGRAPHY & CURRENT AFFAIRS [DAY 15]

Westerlies blow from the Sub-tropical high Pressure Belt to the Sub-polar low Pressure Belt
in the temperature latitudes between 30 and 60, on either side of the equator
They are more constant and stronger in the Southern Hemisphere because there are no
large landmasses to interrupt them
In places they become so strong, these winds are known as Roaring Forties or the Brave
West winds and the Furious Fifties
The belts of the Westerlies move north and south following the Suns movement. These are
known as Westerlies because they blow out of the west.

Q.4) Which of the following are non-metallic minerals?


1)
2)
3)
4)

Gypsum
Quartz
Lead
Mica

Choose the appropriate code:


a)
b)
c)
d)

3 and 4 only
1, 2 and 4 only
1, 3 and 4 only
All of the above

Q.4) Solution (b)

Lead is metallic mineral, where others are non-metallic

Q.5) Which among the following statements is/are true in regard to El-Nino?
1) El Nino is a narrow warm current which occasionally appears off the coast of Peru in
December
2) El Nino has the potential of increasing the surface water temperatures of the sea by
10C
3) El Nino refers to a cyclical warming of the Central and Eastern Equatorial Pacific that
frequently corresponds to a drought in India
www.iasbaba.com

Page 3

IASbaba 60 Day Plan- Prelims Test 2016 GEOGRAPHY & CURRENT AFFAIRS [DAY 15]

Choose the appropriate code:


a)
b)
c)
d)

2 and 3 only
1 and 2 only
1 and 3 only
All of the above

Q.5) Solution (d)

Self-explanatory All the statements are correct

Q.6) Arrange the following Iron ores based upon their quality or pure iron content:
1)
2)
3)
4)

Limonite
Siderite
Magnetite
Haematite

Choose the appropriate code:


a)
b)
c)
d)

1-2-3-4
3-4-1-2
3-4-2-1
4-3-2-1

Q.6) Solution (b)

Magnetite: This is the best quality of iron ore and contains 72 per cent pure iron. It
possesses magnetic property and hence is called magnetite. It is found in Andhra Pradesh,
Jharkhand, Goa, Karnataka etc
Haematite: It contains 60 per cent to 70 per cent pure iron and is found in Andhra Pradesh,
Jharkhand, Orissa, Chhattisgarh, Goa etc

www.iasbaba.com

Page 4

IASbaba 60 Day Plan- Prelims Test 2016 GEOGRAPHY & CURRENT AFFAIRS [DAY 15]

Limonite: It contains 40 per cent to 60 per cent pure iron. It is of yellow or light brown
colour
Siderite: It contains many impurities and has just 40-50 per cent pure iron. However, due to
presence of lime, it is self fluxing

Q.7) Consider the following statements in regard to Depressions:


1) Depressions are low pressure systems that form in the mid-latitude region (30-60 N/S)
when warm air meets cold air
2) When this occurs the warm air is forced above the colder air. It then cools and clouds
form which produce rain
3) Winds in depressions move in an clockwise direction in the northern hemisphere and
anti-clockwise in the southern hemisphere
4) Depressions are characterized by fronts
Which of the statements given above is/are correct?
a)
b)
c)
d)

2, 3 and 4 only
2 and 4 only
1, 2 and 4 only
All of the above

Q.7) Solution (c)

Depressions are low pressure systems that form in the mid-latitude region (30-60 N/S)
when warm air meets cold air. When this occurs the warm air is forced above the colder air.
It then cools and clouds form which produce rain.
Winds in depressions move in an anti-clockwise direction in the northern hemisphere and
clockwise in the southern hemisphere (hence 3rd statement is wrong)

Q.8) Humidity of the air


a)
b)
c)
d)

Increases with the increase in atmospheric temperature


Decreases with the increase in atmospheric temperature
Is not affected by the change in atmospheric temperature
Does not show any consistent behaviour with the change in atmospheric temperature

www.iasbaba.com

Page 5

IASbaba 60 Day Plan- Prelims Test 2016 GEOGRAPHY & CURRENT AFFAIRS [DAY 15]

Q.8) Solution (a)


Self Explantory - Humidity of the air increases with the increase in atmospheric temperature

Q.9) Consider the following statements with regard to cyclones:


1) The centre of Tropical cyclone is characterized by extremely low pressure. There is no
temperature variation in their different parts.
2) Normally Tropical cyclones move from east to west under the influence of trade winds
and they mainly occur in summer.
3) Temperate Cyclones always travel from west to east and are produced only in winter.
4) Extratropical cyclones have cold air at their core, and derive their energy from the
release of potential energy when cold and warm air masses interact.
Which of the statements given above is/are correct?
a)
b)
c)
d)

1, 2 and 4 only
1 and 4 only
2 and 4 only
1, 2 and 3 only

Q.9) Solution (a)

The centre of Tropical cyclone is characterized by extremely low pressure. There is no


temperature variation in their different parts.
Normally Tropical cyclones move from east to west under the influence of trade winds and
they mainly occur in summer.
Temperate Cyclones always travel from west to east and more cyclones are produced in
winter than in summer. (not only in winter)
Extratropical cyclones have cold air at their core, and derive their energy from the release of
potential energy when cold and warm air masses interact. These storms always have one or
more fronts connected to them, and can occur over land or ocean.

Q.10) Which among the following statements is/are not true with regard to monsoons?
www.iasbaba.com

Page 6

IASbaba 60 Day Plan- Prelims Test 2016 GEOGRAPHY & CURRENT AFFAIRS [DAY 15]

1) Southwest monsoon brings rain during summer whereas Northeast monsoon brings rain
during winter
2) Direction of Northeast monsoon is land to sea so it doesnt contains moisture and brings
dryness and coldness after blowing through Bay of Bengal and brings rainfall only in
Tamil Nadu
3) During Southwest monsoon, Indian Subcontinent has high pressure and the direction of
air movement is from Australia to Indian subcontinent
Choose the appropriate code:
a)
b)
c)
d)

2 and 3 only
1 and 2 only
2 only
3 only

Q.10) Solution (d)

During Southwest monsoon, Indian Subcontinent has low pressure and the direction of air
movement is from Australia(high) to Indian subcontinent(low)

www.iasbaba.com

Page 7

IASbaba 60 Day Plan- Prelims Test 2016 GEOGRAPHY & CURRENT AFFAIRS [DAY 15]

Q.11) Consider the following minerals


1.
2.
3.
4.
5.
6.

Limestone
Natural Gas
Gypsum
Uranium
Iron ore
Thorium

The minerals included in the wholesale price index are?


a)
b)
c)
d)

1, 2, 3 and 5
2, 3, 4, 5 and 6
1, 3 and 5
1, 3, 4, 5 and 6

Q.11) Solution (c)


The minerals included in the wholesale price index are bauxite, chromite, iron ore, copper ore,
zinc concentrate, manganese ore, barytes, dolomite, fireclay, gypsum, kaolin, limestone,
magnesite, phosphorite, graphite, steatite and sillamanite.
Source- Ministry of Mines- Annual Report
http://www.mines.nic.in/writereaddata/UploadFile/Mines_AR_2015-16_English.pdf

Q.12) The number of mines which reported mineral production in India has decreased over
the period of time. Consider the following
1.
2.
3.
4.
5.

Madhya Pradesh
Gujrat
Jharkhand
Tamil Nadu
Andra Pradesh

Arrange in decreasing order of number of mines present in the state?


www.iasbaba.com

Page 8

IASbaba 60 Day Plan- Prelims Test 2016 GEOGRAPHY & CURRENT AFFAIRS [DAY 15]

a)
b)
c)
d)

1-2-3-4-5
2-1-3-5-4
4-3-1-2-5
1-3-2-4-5

Q.12) Solution (d)


Indian mining industry is characterized by a large number of small operational mines. The
number of mines which reported mineral production (excluding minor minerals, petroleum
(crude), natural gas and atomic minerals) in India was 1,878 in 2015-16 as against 3,524 in the
previous year. Out of 1878 reporting mines, 245 were located in Madhya Pradesh followed by
Jharkhand (208), Gujarat (192), Tamil Nadu (185), Andhra Pradesh (169), Odisha (158),
Chhattisgarh (154), Maharashtra (121), Karnataka (115), West Bengal (101), Goa (75) and
Rajasthan (66).

Q.13) The cases of HIV infected blood transfusion have seen a rise. Consider the following
1. It is mandatory to screen the blood donated for traces of HIV, Hepatitis C, malaria and
syphilis before transfusion
2. NACO is an autonomous body under Department of Health and Family Welfare,
Government of India which is primarily responsible for controlling spread of HIV
infection
3. India has the highest number of population living with HIV in the world as per UN report
Choose the correct code
a)
b)
c)
d)

Only 2
2 and 3
1 and 2
1, 2 and 3

Q.13) Solution (c)


www.iasbaba.com

Page 9

IASbaba 60 Day Plan- Prelims Test 2016 GEOGRAPHY & CURRENT AFFAIRS [DAY 15]

India has the third largest number of people living with HIV in world (2.1 million). Top two are
South Africa and Nigeria. [UNAIDS Gap report]. Around 36% of Indian adults with the virus have
access to antiretroviral treatment, the report said. An annual report by Indias National AIDS
Control Organization noted that the transgender population was emerging as a risk group with
high vulnerability and high levels of HIV.

Q.14) Operation Hill Vijay is being undertaken to declare Parasnath jungles Naxal-free.
Consider the following regarding famous Parasnath Hill
1. Parasnath hill (Shikharji) is located towards the eastern end of the Chota Nagpur Plataea
2. It is a famous Jain pilgrimage site
3. Bhagwan Parshwanath was the last Tirthankara
Select the correct code
a)
b)
c)
d)

Only 1
1 and 2
2 and 3
1 and 3

Q.14) Solution (b)


He was not the last Tirthankara.
Learn about Operation Vijay Hill
http://www.thehindu.com/news/national/jharkhand-dgp-supervises-operation-hillvijay/article8673129.ece

Q.15) Consider the following regarding Global Slavery Index 2016


1. India has the distinction of having the highest number of people in the world trapped in
modern slavery
2. 2016 Global Slavery Index is released by Australia based human right group, Walk Free
www.iasbaba.com

Page 10

IASbaba 60 Day Plan- Prelims Test 2016 GEOGRAPHY & CURRENT AFFAIRS [DAY 15]

Select the correct code


a) Only 1
b) Only 2
c) Both
d) None

Q.15) Solution (c)


http://www.business-standard.com/article/current-affairs/india-tops-global-slavery-indexwith-18-35-million-people-enslaved-116053100436_1.html

Q.16) Salma Dam is in news. Salma Dam is in which of the following countries
a)
b)
c)
d)

Afghanistan
Iran
Sri Lanka
Bhutan

Q.16) Solution (a)


Salma dam is present in Afghanistan. PM Modi will inaugurate the dam as it is constructed by
India. The dam is expected to provide irrigation facilities for nearly 40 thousand hectars of land.

Q.17) Consider the following


1.
2.
3.
4.

Rhizobium
Clostridium
Azotobacter
Pseduomonas

www.iasbaba.com

Page 11

IASbaba 60 Day Plan- Prelims Test 2016 GEOGRAPHY & CURRENT AFFAIRS [DAY 15]

Which of the above is/are free living nitrogen bacteria present in the soil?
a)
b)
c)
d)

1 and 2 only
2 and 3 only
1 and 3 only
1,2 and 3

Q.17) Solution (b)

Rhizobium is a present in the legumes of a plant.


Pseduomonas is denitrifying bacteria

Q.18) Consider the following


1.
2.
3.
4.

A group of volcanic islands


Group of coral islands
A group of islands that lie in the centre of the oceans
A group of islands that lie in close proximity

Which of the above statements does not belong to the category of Archipelago?
a)
b)
c)
d)

1, 2 and 3 only
2 and 3 only
1,2 and 4 only
All

Q.18) Solution (a)

A group of islands that lie in close proximity is known as archipelago

Q.19) Consider the following


1.
2.
3.
4.

Bhangar
Terai
Bhabar
Khadar

www.iasbaba.com

Page 12

IASbaba 60 Day Plan- Prelims Test 2016 GEOGRAPHY & CURRENT AFFAIRS [DAY 15]

As one travels from the Gangetic plain towards Shiwaliks in the north, the correct sequence of
regions he/she will cross is?
a)
b)
c)
d)

1234
1243
4321
4123

Q.19) Solution (d)


Source Ncert 11th Physical environment
Similar explanation can also be found in NCERT also
The Bhabar belt is adjacent to the foothills of the Himalayas and consists of boulders and
pebbles which have been carried down by the river streams. As the porosity of this belt is very
high, the streams flow underground. The bhabar is generally narrow about 715 km wide.
The Terai belt lies next to the Bhabar region and is composed of newer alluvium. The
underground streams reappear in this region. The region is excessively moist and thickly
forested. It also receives heavy rainfall throughout the year and is populated with a variety of
wildlife.
The Bangar belt consists of older alluvium and forms the alluvial terrace of the flood plains.
In the Gangetic plains, it has a low upland covered by laterite deposits.
The Khadir belt lies in lowland areas after the Bangar belt. It is made up of fresh newer
alluvium which is deposited by the rivers flowing down the plain.

Q.20) Consider the following statements with respect to GSLV Mark 3


1. It features an Indian cryogenic technology
2. It is used to launch only remote sensing satellite in geostationary orbit
3. It is currently used to launch satellites weighing more than 3 tons

www.iasbaba.com

Page 13

IASbaba 60 Day Plan- Prelims Test 2016 GEOGRAPHY & CURRENT AFFAIRS [DAY 15]

Select the incorrect option


a)
b)
c)
d)

1 and 3 only
2 and 3 only
1 and 2 only
1,2 and 3

Q.20) Solution (b)


It is used to launch communication satellite and it is currently under development

Q.21) Consider the statements


1. Moons gravity at Earth is much stronger than Suns
2. Lunar tidal force is stronger than solar tidal force
Select the correct code
a) Only 1
b) Only 2
c) Both
d) None

Q.21) Solution (b)


Tides are caused by tidal forces, and the answer to your question lies in the definition of a tidal
force. A tidal force is related to gravity, but it isn't the same thing. It's really the difference
between the strength of gravity at two locations.
The gravitational attraction between two objects (say the Earth and the Moon) decreases with
distance. This means that the Moon's gravity pulls most strongly on the side of the Earth closest
to the Moon and least strongly on the side of the Earth farthest from the Moon. Tidal forces on
www.iasbaba.com

Page 14

IASbaba 60 Day Plan- Prelims Test 2016 GEOGRAPHY & CURRENT AFFAIRS [DAY 15]

the side of Earth closest to the Moon pull material (mostly water) toward the Moon. Tidal
forces on the other side of Earth actually pull material away from the Moon. The resulting
deformation of Earth looks the same when the moon is at opposite sides of its orbit, like full
moon and new moon or first quarter and third quarter. That's why tides around the equator are
higher during both a new moon and a full moon.
The Sun also affects the Earth's tides. However, tidal forces due to the Sun are about half as
strong as those due to the Moon. This seems strange, because the Sun's gravity at Earth is
much stronger than the Moon's. But remember that tides concern the difference between
gravity's pull at opposite sides of the Earth. The radius of the Earth is a very small fraction of the
distance between the Sun and the Earth, about 0.005%. As a result, the difference between the
Sun's gravitational pull on either end of the Earth is small. In contrast, the radius of the Earth is
about 1.7% of the distance between the Earth and the Moon. So even though the Moon's
gravity isn't as strong as the Sun's, lunar tidal forces are stronger than solar tidal forces, so lunar
tides are stronger than solar tides.

Q.22) Consider the following statements with respect to National Pension Scheme
1. National pension scheme was launched in the year 2015
2. NRIs may subscribe to the NPS as an investment option under FEMA 1999
3. NPS has been provided for all citizens of the country including the unorganized sector
workers on voluntary basis.
Select the correct option with respect to NPS
a)
b)
c)
d)

1 and 2 only
2 and 3 only
1 and 3 only
All

Q.22) Solution (b)

The National Pension System (NPS) was launched on 1 January, 2004 with the objective
of providing retirement income to all the citizens.
RBI took the decision in consultation with the Union Government to enable National
Pension System (NPS) as an investment option for NRIs under Foreign Exchange
Management Act (FEMA), 1999.

www.iasbaba.com

Page 15

IASbaba 60 Day Plan- Prelims Test 2016 GEOGRAPHY & CURRENT AFFAIRS [DAY 15]

NRIs may subscribe to the NPS governed and administered by the Pension Fund
Regulatory and Development Authority (PFRDA)
NPS aims to institute pension reforms and to inculcate the habit of saving for retirement
amongst the citizens.
Initially, NPS was introduced for the new government recruits (except armed forces).
With effect from 1 May, 2009, NPS has been provided for all citizens of the country
including the unorganized sector workers on voluntary basis.

Q.23) The Government of India scheme Vikalp refers to


a) It aims to provide confirmed accommodation to waitlisted passengers in alternate
trains.
b) It aims to ensure 100 percent attendance in tribal areas.
c) The scheme aims to provide free electricity in rural areas
d) The scheme aims to ensure safety of women passengers in public transport through the
use of ICT

Q.23) Solution (a)


Indian Railways has launched Alternate Train Accommodation Scheme (ATAS) called as Vikalp
to provide confirmed accommodation to waitlisted passengers in alternate trains.

Q.24) Consider the following statements with respect to Olive Ridely turtles
1. They are also known as Pacific Ridely sea turtles.
2. Olive Ridley sea turtle is placed under Schedule - I of Indian Wildlife (Protection) Act,
1972 (amended 1991).
3. It is endangered as per IUCN red list
Select the correct option
www.iasbaba.com

Page 16

IASbaba 60 Day Plan- Prelims Test 2016 GEOGRAPHY & CURRENT AFFAIRS [DAY 15]

a)
b)
c)
d)

1 and 2 only
2 and 3 only
1 and 3 only
All

Q.24) Solution (a)

Also known as the Pacific Ridley Sea Turtle, it is a medium-sized species of sea turtle
found in warm and tropical waters, primarily in the Pacific and Indian Oceans.
Gahirmatha Beach in Kendrapara district of Odisha, a part of the Bhitarkanika Wildlife
Sanctuary
Bhitarkanika Mangroves were designated a Ramsar Wetland of International
Importance in 2002
Olive Ridley sea turtle has found place in Schedule - I of Indian Wildlife (Protection) Act,
1972 (amended 1991).
All the species of sea turtles in the coastal water of Orissa are listed as "vulnerable" as
per IUCN Red Data Book.
The Sea Turtles are protected under the 'Migratory Species Convention' and CITES
(Convention of International Trade on Wildlife Flora and Fauna). India is a signatory
nation to all these conventions

Q.25) Which of the following statements regarding H1N1 is/are correct?


1. It is transmitted by an Influenza A virus subtype H1N1
2. The H stands for hemagglutinin. And N for neuraminidase- both proteins on the outer
layer of the virus
3. Swine influenza is a respiratory disease that occurs in pigs that is caused by the
Influenza A virus.
4. It is a Non-contagious disease
Select the correct answer using the code given below:
a)
b)
c)
d)

1 ,2 and 3 only
2 and 4 only
1, 2 and 4 only
All the above

www.iasbaba.com

Page 17

IASbaba 60 Day Plan- Prelims Test 2016 GEOGRAPHY & CURRENT AFFAIRS [DAY 15]

Q.25) Solution (a)

It is transmitted by an Influenza A virus subtype H1N1.The H stands for hemagglutinin.


And N for neuraminidase- both proteins on the outer layer of the virus
It is a pandemic outbreak Means Disease outbreak occurs over a wide geographic area
and affects a very high proportion of the population.
Swine influenza is a respiratory disease that occurs in pigs that is caused by the
Influenza A virus.
Its a highly contagious disease that can easily spread from person to person.

www.iasbaba.com

Page 18

IASbaba 60 Day Plan- Prelims Test 2016 GEOGRAPHY & CURRENT AFFAIRS [DAY 16]

Q.1) A great variety of soils is found over the globe. Which among the following are causes for
this variation?
1) Nature of rocks
2) Physical characteristics of land
3) Climate
4) Vegetation cover
Choose the appropriate code:
a) 1 and 3 only
b) 1, 2 and 3 only
c) 1, 3 and 4 only
d) All of the above
Q.1) Solution (d)
Explanation:
Self-explanatory, all the given factors are responsible for the variations of soils
Source: Chapter 7 Soils, Principles of Geography (Old NCERT)

Q.2) Which among the following statements is/are true in regard to soils?
1) Soil is not only a mixture of solid and liquid substances but also gaseous substances
2) Soil is a non-renewable resource
3) The rate of removal of fine particles from the surface (due to running water and wind) is
the same as the rate of addition of particles to the soil layer.
4) Soils with high lime content are called acid and those with low lime content are called
alkaline
Choose the appropriate code:
a)
b)
c)
d)

3 and 4 only
1, 2 and 3 only
1, 3 and 4 only
All of the above

Q.2) Solution (b)


www.iasbaba.com

Page 1

IASbaba 60 Day Plan- Prelims Test 2016 GEOGRAPHY & CURRENT AFFAIRS [DAY 16]

Explanation:
Only 4th Statement is wrong - Soils with low lime content are called acid and those with high
lime content are called alkaline
Source: Chapter 7 Soils, Principles of Geography (Old NCERT)

Q.3) Consider the following statements:


1) Laterite soils are formed due to the heat combined with the rate of alternation between
wet and dry intervals of monsoon climate
2) Hot summer and low rainfall develops black soil, irrespective of the parent rock
3) In tropical/equatorial region since biodiversity/vegetation is thick, humus content is also
very high
Which of the statements given above is/are correct?
a)
b)
c)
d)

2 and 3 only
1 and 2 only
1 and 3 only
All of the above

Q.3) Solution (b)


Explanation:
Climate of an area is such a major factor that over a long period, it tends to reduce
differences caused by the parent materials. That is why two different parent material may
develop the same soil in one type of climatic regime.
Likewise, the same parent material may produce two different types of soil in two types of
climates.
Statement 1 is correct self explanatory; Laterite has been derived from the Latin word
Later which means brick. The laterite soils develop in areas with high temperature and
high rainfall.
Statement 2 is also correct and above explanation is the reason why black soil are found in
some districts of Tamil Nadu, irrespective of the parent rock
Statement 3 is wrong. Tropical/equatorial region consists of humid climate, therefore
bacterial activity is intense. Consumption of humus is so much that soils are left poor in it.
www.iasbaba.com

Page 2

IASbaba 60 Day Plan- Prelims Test 2016 GEOGRAPHY & CURRENT AFFAIRS [DAY 16]

In cold climate (temperate region), bacterial activity is limited and the soils are rich in
humus.

Source: Chapter 6 Soils, India Physical Environment and Chapter 7 Soils, Principles of
Geography (Old NCERT)

Q.4) Which among the following statements is/are not true?


1) Pedocals are soils that are found in humid climates extending from high-latitude
coniferous forests, mid latitude deciduous forest lands to low-climate tropical forests
and grasslands
2) Pedalfers are soils that are found in arid, semi-arid and the sub-humid zones in the
world
3) Pedocals are very rich in calcium and mineral salts
Choose the appropriate code:
a)
b)
c)
d)

1 and 2 only
2 and 3 only
3 only
None of the above

Q.4) Solution (a)


Explanation:
Pedalfers are soils that are found in humid climates extending from high-latitude coniferous
forests, mid latitude deciduous forest lands to low-climate tropical forests and grasslands
Pedocals are soils that are found in arid, semi-arid and the sub-humid zones in the world
Pedocals are very rich in calcium and mineral salts
Source: Chapter 7 Soils, Principles of Geography (Old NCERT)

Q.5) Consider the following statements:


1) These soils are called black earths
www.iasbaba.com

Page 3

IASbaba 60 Day Plan- Prelims Test 2016 GEOGRAPHY & CURRENT AFFAIRS [DAY 16]

2) They are rich in humus, rich in calcium, unleached and have a crumby or nut-like
structure
3) These are fertile soils requiring little fertilizing even when used year after year.
4) The Steppes of Ukraine, the central part of USA, central Africa, South America and
Australia have these type of soils

Select the appropriate soil which the statements are referring to:
a)
b)
c)
d)

Chernozems
Prairie soils
Grey brown Podsolic soil
Red and Yellow Pedalfers

Q.5) Solution (a)


Explanation:
Self-explanatory
Source: Chapter 7 Soils, Principles of Geography (Old NCERT)

Q.6) Which of the following are true about Desert soils?


1) They are grey in colour in temperate region and red in hot deserts of the tropics
2) The soils are unleached and alkaline
3) These soils are very much lacking in humus because of little vegetation found
Choose the appropriate code:
a) 2 and 3 only
b) 1 and 2 only
c) 1 and 3 only
www.iasbaba.com

Page 4

IASbaba 60 Day Plan- Prelims Test 2016 GEOGRAPHY & CURRENT AFFAIRS [DAY 16]

d) All of the above


Q.6) Solution (d)
Explanation:
Desert soils are grey in colour in temperate region and red in hot deserts of the tropics
The soils are unleached and alkaline but very much lacking in humus because of little
vegetation found
Source: Chapter 7 Soils, Principles of Geography (Old NCERT)

Q.7) Consider the following statements with regard to Usar and Reh:
1) Usar is a term collectively applied to all kinds of saline and alkaline soils in the plains of
north India, particularly in Uttar Pradesh
2) Reh is a white, grayish or ash-coloured salt that are found in low-lying plain areas in dry
periods
Which of the statements given above is/are incorrect?
a)
b)
c)
d)

1 only
2 only
Both 1 and 2
Neither 1 nor 2

Q.7) Solution (d)


Explanation:
Both the statements are correct and self explanatory
Source: Chapter 6 Soils, India Physical Environment and Chapter 7 Soils, Principles of
Geography (Old NCERT)

Q.8) Consider the following statements:

www.iasbaba.com

Page 5

IASbaba 60 Day Plan- Prelims Test 2016 GEOGRAPHY & CURRENT AFFAIRS [DAY 16]

1) Alluvial soils are rich in potash and phosphorous


2) The colour of the alluvial soils varies from the light grey to ash grey
3) Sikkim has only Mountain soil and no presence of Alluvial/Terai soil
Which of the statements given above is/are correct?
a)
b)
c)
d)

2 only
1 and 2 only
1 and 3 only
All of the above

Q.8) Solution (a)


Explanation:
Alluvial soils are rich in potash but poor in phosphorous
The colour of the alluvial soils varies from the light grey to ash grey
Sikkim has both Mountain soil and Terai soil (which is alluvial)
Source: Chapter 6 Soils, India Physical Environment and Orient BlackSwan Atlas (Thematic
map Soils)

Q.9) Which among the following statements is/are correct with regard to Zonal, Intrazonal
and Azonal Soils?
1) Soil occurring over wide areas on well-drained land which have been there long enough
for the climate and organisms to have expressed their full influence is called Zonal Soils
2) Soils affected by some local conditions not involving climate or vegetation, such as poor
drainage or those where parent material exerts a strong influence is Intrazonal soils
3) Immature soils without well developed soil characteristics are called Azonal Soils
Choose the appropriate code:
a)
b)
c)
d)

2 and 3 only
1 and 2 only
1 and 3 only
All of the above

www.iasbaba.com

Page 6

IASbaba 60 Day Plan- Prelims Test 2016 GEOGRAPHY & CURRENT AFFAIRS [DAY 16]

Q.9) Solution (d)


Explanation:
Zonal Soils:
The zonal soils are formed on normal sites from ordinary siliceous rocks and show clearly
the impress of climate and vegetation. In short, these are formed under conditions of good
soil drainage through the prolonged action of climate and vegetation, e.g. chestnut soils.
This kind of soils have following types 1) Tundra Soils
2) Podzols
3) Brown Forest Soils
4) Laterite Soils/Latosols/Ferralsols
5) Chernozem/Prairie/Steppe
6) Grumusol/Reddish Brown Soils
7) Desert (Seirozems and Red Desert) Soils
Intrazonal Soils:
The intrazonal soils include the soils from less common parent materials and those
influenced by high ground water or under conditions of very poor drainage (such as in bogs,
flood-plain meadows, or in the playa lake basins of the deserts) or upon limestone.
Depending on the role played by water, presence of calcium in the parent material and the
location, intra-zonal soils may be 1) Hydromorphic
2) Calcimorphic
3) Halomorphic
Azonal Soils:
The azonal soils are youthful, owing to recent renewal by sedimentation or erosion. This
have no well-developed profile characteristics. These soils are common where the parent
material is being continuously eroded and deposited, e.g. alluvial Soils (newer or younger
Khadar and older Bhangar soils) or lithosols (those at high altitudes on resistant parent
material). These soils have poorly developed horizons due to three reasons:
1) Lack of Time For instance, in new flood plains alluvium is being continually eroded and
deposited.
2) Parent Material Azonal soils like regosols result from loose sand and loess.
3) Geomorphology Lithosols result on steep slopes where soil is eroded as soon as it is
deposited.

Q.10) Consider the following characteristics with regard to Red soils:


www.iasbaba.com

Page 7

IASbaba 60 Day Plan- Prelims Test 2016 GEOGRAPHY & CURRENT AFFAIRS [DAY 16]

1) Red soils are the largest soil group in India


2) Most of the red soils have come into existence due to weathering of ancient crystalline
and metamorphic rocks
3) They are usually poor growing soils, low in nutrients and humus and difficult to cultivate
because of its low water holding capacity.
4) Red and Yellow soil is abundantly found in southern belt i.e., Tamil Nadu and Karnataka
Choose the appropriate code:
a)
b)
c)
d)

3 and 4 only
1, 2 and 3 only
1, 3 and 4 only
All of the above

Q.10) Solution (b)


Explanation:
Red soils are the largest soil group in India. Most of the red soils have come into existence
due to weathering of ancient crystalline and metamorphic rocks
They are usually poor growing soils, low in nutrients and humus and difficult to cultivate
because of its low water holding capacity.
Red and Yellow soil is not found in southern belt (i.e., Tamil Nadu and Karnataka) Refer
Orient BlackSwan Atlas (Thematic map Soils)
Due to presence of haematite and limonite its colour ranges from red to yellow, usually
found on Jharkhand, Chattisgarh and Orissa
Source: Chapter 7 Soils, D. R. Khullar and Refer Orient BlackSwan Atlas (Thematic map Soils)

Q.11) There is a lot of fuss created about UK exiting the EU (European Union). Consider the
following statements with respect to European Union
1. EU is economic political union comprising of 28 countries.
2. The headquarters of EU is in Brussels, Belgium
3. Among the EU cities Paris is the largest city

Select the correct answer using the codes given below


a) 1 and 2 only
www.iasbaba.com

Page 8

IASbaba 60 Day Plan- Prelims Test 2016 GEOGRAPHY & CURRENT AFFAIRS [DAY 16]

b) 2 and 3 only
c) 1 and 3 only
d) All

Q.11) Solution (a)


Among the EU London is the largest city

Q.12) Consider the following statement with respect to TRAI (Telecom Regulatory Authority
of India)
1. TRAI is an apex body to regulate telecom services, including fixation/revision of tariffs
for telecom services which were earlier vested in the State government
2. Recently TRAI was accorded the status of Non banking financial company

Select the correct option


a)
b)
c)
d)

1 only
2 only
Both
None

Q.12) Solution (d)

The Telecom Regulatory Authority of India (TRAI) was, thus, established with effect from
20th February 1997 by an Act of Parliament, called the Telecom Regulatory Authority of
India Act, 1997, to regulate telecom services, including fixation/revision of tariffs for
telecom services which were earlier vested in the Central Government.
It is not a non banking financial company

www.iasbaba.com

Page 9

IASbaba 60 Day Plan- Prelims Test 2016 GEOGRAPHY & CURRENT AFFAIRS [DAY 16]

Q.13) Virtual reality is making headlines nowadays. Consider the following statement with
respect to the term Virtual reality
1. Virtual reality is the creation of a virtual environment presented to our senses in such a
way that we experience it as if we were really there.
2. Medicine is one of the biggest beneficiaries with the development of virtual reality.
3. It is also called floating multimedia

Select the correct option


a)
b)
c)
d)

1 and 2 only
2 and 3 only
Only 1
All

Q.13) Solution (a)

Click here to know more about virtual reality


This question can be asked in Mains from S&T part of GS 3, from prelims perspective
know the basics.

Q.14) The kind of unemployment when the economy experiences many ups and downs is
known as
a)
b)
c)
d)

Frictional unemployment
Structural unemployment
Cyclical unemployment
Seasonal unemployment

Q.14) Solution (c)

Click here

Cyclical Unemployment
www.iasbaba.com

Page 10

IASbaba 60 Day Plan- Prelims Test 2016 GEOGRAPHY & CURRENT AFFAIRS [DAY 16]

Over time, the economy experiences many ups and downs. That's what we call cyclical
unemployment because it goes in cycles. Cyclical unemployment occurs because of
these cycles. When the economy enters a recession, many of the jobs lost are
considered cyclical unemployment.
For example, during the Great Depression, the unemployment rate surged as high as
25%. That means one out of four people were willing and able to work, but could not
find work! Most of this unemployment was considered cyclical unemployment.
Eventually, unemployment came down again. As you can see, at least part of
unemployment can be explained by looking at the cycles, or the ups and downs of the
economy.

Q.15) Consider the following statements with respect to SAARC countries


1. SAARC is an economic and political organization of eight countries in Southern Asia that
includes Bangladesh, Bhutan, India, Mauritius, Nepal, Pakistan and Sri Lanka
2. The headquarters of SAARC is located in Kathmandu, Nepal

Select the correct option


a)
b)
c)
d)

1 only
2 only
Both
None

Q.15) Solution (b)

Mauritius is not a member of SAARC

Q.16) Consider the following statements with respect to Intended Nationally Determined
Contributions
1. Intended Nationally Determined Contributions (INDCs) is a term used under the United
Nation Environment Programme (UNEP) for reductions in greenhouse gas emissions
www.iasbaba.com

Page 11

IASbaba 60 Day Plan- Prelims Test 2016 GEOGRAPHY & CURRENT AFFAIRS [DAY 16]

2. As part of INDC India aims to reduce the Emissions Intensity of its GDP by 33 to 35 Per
Cent by 2030 from 2005 Level

Select the correct option


a)
b)
c)
d)

1 only
2 only
Both
None

Q.16) Solution (b)

Intended Nationally Determined Contributions (INDCs) is a term used under the United
Nations Framework Convention on Climate Change (UNFCCC) for reductions
in greenhouse gas emissions
Click here

Q.17) Consider the following statements with respect to Renewable energy and India
1. India has the target to produce 1,00,000 MW of solar power by 2022.
2. India allows 49 % FDI in renewable energy sector to promote domestic industries as part
of Make in India campaign

Select the correct option


a)
b)
c)
d)

1 only
2 only
Both
None

Q.17) Solution (a)


India allows 100 % FDI in renewable energy sector
www.iasbaba.com

Page 12

IASbaba 60 Day Plan- Prelims Test 2016 GEOGRAPHY & CURRENT AFFAIRS [DAY 16]

Q.18) Consider the following


1.
2.
3.
4.

Prithvi
Agni
BrahMos
Akash

Arrange the above missiles in decreasing order of their range


a)
b)
c)
d)

1234
2134
2143
1243

Q.18) Solution (b)

Agni missile has the highest range of up to 8000 kms


Prithvi missile has the range up to 600 kms
BrahMos missile has the range up to 290 kms
Akash missile has the range up to 30 kms

Q.19) Consider the following statements with respect to Insider trading


a) Transactions of shares/security by someone having access to confidential information of
a listed company.
b) The trading of shares in between the promoters of a listed company to stop a possible
takeover.
c) The trading of virtual commodities on floor of commodity stock exchanges on the last
working days of the weeks.
d) None of the above

Q.19) Solution (a)

www.iasbaba.com

Page 13

IASbaba 60 Day Plan- Prelims Test 2016 GEOGRAPHY & CURRENT AFFAIRS [DAY 16]

Insider trading is the buying or selling of a security by someone who has access to material, non
public information about the security.

Q.20) Bollgard technology is associated with which of the following crop?


a)
b)
c)
d)

Wheat
Rice
Cotton
Sun flower

Q.20) Solution (c)


It is associated with cotton
Click here

Q.21) Consider the following statement/s


a) Stapled Visa doesnt leave permanent trace on ones passport
b) China had issued stapled visa to residents of Indian J&K and Arunanchal Pradesh
c) Both correct
d) Both incorrect

Q.21) Solution (c)


Click

Q.22) Consider the following statements regarding a particular soil


1. Found in the summits of Western Ghats, Eastern Ghats, Vindhyas, Sathpuras and Malwa
plateau.
www.iasbaba.com

Page 14

IASbaba 60 Day Plan- Prelims Test 2016 GEOGRAPHY & CURRENT AFFAIRS [DAY 16]

2. Subjected to intense leaching and thus not fit for agriculture unless manures and
fertilizers are used.
3. It is the end product of weathering process and thus it is indefinitely durable.
Which is the soil that is spoken about in the above statements?
a) Forest soil
b) Mountain Soil
c) Laterite soil
d) Red soil

Q.22) Solution (c)


Although first two statements ie 1 and 2 do comply with mountain soil and forest soil, the third
statement is unique for laterite soil.

Q.23) Consider the statements regarding red and yellow soils.


1. Soil looks yellowish when iron diffused in the soil gets hydrated.
2. Red and yellow soils are mainly found in the deltaic regions especially in the Eastern
Indian coast.
3. Less fertile coarser soils are found in the uplands while fertile fine soils occur in the low
lying regions.
Choose the correct code
a) 1 only
b) 1 and 2 only
c) 1 and 3 only
d) 2 and 3 only
www.iasbaba.com

Page 15

IASbaba 60 Day Plan- Prelims Test 2016 GEOGRAPHY & CURRENT AFFAIRS [DAY 16]

Q.23) Solution (c)


This type of soils get red colour when iron is diffused in the crystalline and metamorphic rocks.
The same soil turns yellow when the iron diffused gets turned into hydrated form. These soils
occur generally in the drier regions of Eastern and Southern India and not in the deltaic regions.

Q.24) Black soils are known for its fertility with little or no evidence of exhaustion. This is
because
1. Black soils with equal proportion of clay, gravel and coarse sand has high moisture
retention capacity
2. Cracks developed during dry season allow oxygenation of the soils.
3. Black soil has low level of kankar thus supports wide variety of crops.
Choose the incorrect reason from the codes below
a) 1 Only
b) 2 and 3 only
c) 1 and 3 only
d) All the above

Q.24) Solution (c)


Black soil is mainly composed of clay, thus it has high moisture retention capacity . Black soils
are in fact rich in kankar nodules which add to the fertility of the soil.

www.iasbaba.com

Page 16

IASbaba 60 Day Plan- Prelims Test 2016 GEOGRAPHY & CURRENT AFFAIRS [DAY 16]

Q.25) Consider the statements the effects of Oil Spill


1. It reduces Photosynthetic activities in water bodies.
2. It results into killing of birds and mammals that die of hypothermia
3. Food web of water bodies gets even more complex
Select the correct code:
a) 1 Only
b) 1 and 2 Only
c) 1 and 3 Only
d) All of the above

Q.25) Solution (b)


Oil spill can prove fatal for plant, animal and human life. The substance is so toxic that it can
cause massive loss of species (food web will get simplify) that live in the sea. Oil spill penetrates
into the plumage and fur of birds, breaks down the insulating capabilities of feather which
makes them heavier, disallow them to fly and kill them via poisoning or hypothermia.
http://www.conserve-energy-future.com/effects-of-oil-spills.php

http://www.thehindu.com/news/international/us-judge-orders-20-billion-settlement-for-bpoil-spill/article8436793.ece
http://www.thehindu.com/news/cities/mumbai/news/oil-spill-throws-traffic-off-gear-oneway/article8263754.ece

www.iasbaba.com

Page 17

IASbaba 60 Day Plan- Prelims Test 2016 SCIENCE & TECHNOLOGY & CURRENT AFFAIRS [DAY 17]

Q.1) DNA Origami involves making shapes out of DNA in


a)
b)
c)
d)

1 Dimension
2 Dimension
1 and 2 Dimensions
2 and 3 Dimensions

Q.1) Solution (d)


http://www.thehindu.com/opinion/op-ed/what-is-dna-origami/article8660871.ece

Q.2) Consider the following


1.
2.
3.
4.

Change in state
Change in temperature
Change in colour
Evolution of a gas

Which of the above observations leads to a chemical reaction?


a)
b)
c)
d)

1, 3 and 4
2, 3 and 4
1, 2 and 3
1, 2, 3 and 4

Q.2) Solution (d)


Refer to NCERT, Chemical Reactions and Equations

Q.3) Consider the following


1.
2.
3.
4.
5.
6.

Fuelling of Space Shuttle


Photosynthesis
Corrosion of metals
Rancidity
Enzymes
Combustion

www.iasbaba.com

Page 1

IASbaba 60 Day Plan- Prelims Test 2016 SCIENCE & TECHNOLOGY & CURRENT AFFAIRS [DAY 17]

Which of the above involves Redox reaction?


a)
b)
c)
d)

1, 2 and 5
2, 3, 5 and 6
1, 2, 4 and 5
1, 2, 3, 4, 5 and 6

Q.3) Solution (a)


Combustion, Rancidity and Corrosion: Oxidation Reaction.
The reduction and oxidation reaction or redox reaction is a chemical reaction during which
the oxidation state of two or more of the reactants change. Oxidation reaction is not only
about just combining with oxygen. Most redox reactions do not even involve oxygen at all.
Oxidation in the context of a redox reaction refers to the oxidation state of an atom being
increased.
Conversely reduction refers to oxidation state of an atom being reduced.
Many chemical reactions do not involve any change in oxidation state.
For example when we react aqueous solutions of copper (II) sulphate and sodium hydroxide
to form copper (II) hydroxide which precipitates and a solution of sodium sulphate forms
and there are no changes in oxidation state.
Oxidation is the process of addition of oxygen or any electronegative radical or removal of
hydrogen or electropositive radical. Its a process in which an atom or a group of atoms
taking part in a chemical reaction loses one or more electrons.
The species which undergo the loss of electron during the reaction is called as reducing
agent or reductant.
An oxidation reaction is always couple with reduction reaction which refers as addition of
electron to the reaction species (Oxidant or oxidising agent).
These coupled reactions are known as redox reaction.
We can observe many oxidation reactions in our daily life like corrosion of metal, rancidity
and combustion.
Combustion: it is the most common example of oxidation reaction.
Combustion or burning of any material involves oxidation reaction coupled with reduction.
The complete combustion of substance generally released carbon dioxide and water.
www.iasbaba.com

Page 2

IASbaba 60 Day Plan- Prelims Test 2016 SCIENCE & TECHNOLOGY & CURRENT AFFAIRS [DAY 17]

For example the burning of wood released a large amount of energy with carbon dioxide
and water vapour.
That energy uses for heating home, drive automobile, operate industrial processes and
much other purpose.
Corrosion: You must have seen the rusting of your car and the burning of magnesium metal
in oxygen to form magnesium oxide in your daily life are also an oxidation reaction.
Similarly rusted iron sheets or green surface of copper utensils and tableware or tarnish
surface of aluminium surfaces are also due to oxidation of metal surface.
Most of metal surfaces oxidised due to atmospheric oxygen and forms metal oxides on the
surface of metal.
For example; corrosion of iron forms iron oxide which is also called as rust.
4Fe +3O2 ==> 2Fe2O3
Rancidity: Oxidation reactions are also responsible for the spoiling of food.
Oxidation-reduction reactions also fuel the most advanced form of transportation known
today, the space shuttle. The actual orbiter vehicle is relatively small compared to its
external power apparatus, which consists of two solid rocket boosters on either side, along
with an external fuel tank.
Inside the solid rocket boosters are ammonium perchlorate (NH4ClO4) and powdered
aluminium, which undergo an oxidation-reduction reaction that gives the shuttle enormous
amounts of extra thrust. As for the larger single external fuel tank, this contains the gases
that power the rocket: hydrogen and oxygen.
Because these two are extremely explosive, they must be kept in separate compartments.
When they react, they form water, of course, but in doing so, they also release vast
quantities of energy. The chemical equation for this is: 2H 2 + O 2 2H 2 O + energy.
On January 28, 1986, something went terribly wrong with this arrangement on the space
shuttle Challenger. Cold weather had fatigued the O-rings that sealed the hydrogen and
oxygen compartments, and the gases fed straight into the flames behind the shuttle itself.
This produced a powerful and uncontrolled oxidation-reduction reaction, an explosion that
took the lives of all seven astronauts aboard the shuttle.
Enzymes in the human body regulate oxidation-reduction reactions. These complex
proteins, of which several hundred are known, act as catalysts, speeding up chemical
processes in the body. Oxidation-reduction reactions also take place in the metabolism of
food for energy, with substances in the food broken down into components the body can
use.
www.iasbaba.com

Page 3

IASbaba 60 Day Plan- Prelims Test 2016 SCIENCE & TECHNOLOGY & CURRENT AFFAIRS [DAY 17]

Photosynthesis involves the reduction of carbon dioxide into sugars and the oxidation of
water into molecular oxygen.

Q.4) Consider the following statements


1.
2.
3.
4.

All non-metals are either solid or gases


All metals are solid at room temperature
All non-metals are non-lustrous
All non-metals are bad conductors of electricity

Select the incorrect code


a)
b)
c)
d)

1, 3 and 4
1, 2 and 3
2 and 4 Only
1, 2, 3 and 4

Q.4) Solution (d)


Bromine is a non-metal which is liquid
Mercury is a metal but exists as liquid at room temperature.
Iodine is a non-metal but lustrous
Graphite is a non-metal (allotrope of carbon) but good conductor of electricity

Q.5) Consider the following


1.
2.
3.
4.
5.
6.

Sodium
Gold
Magnesium
Copper
Calcium
Platinum

Which of the above are found in nature as Free Elements?


a)
b)
c)
d)

1, 2 and 6
2, 3 and 5
2, 4 and 6
1, 2, 4, 5 and 6

www.iasbaba.com

Page 4

IASbaba 60 Day Plan- Prelims Test 2016 SCIENCE & TECHNOLOGY & CURRENT AFFAIRS [DAY 17]

Q.5) Solution (c)


Some metals are found in Free State and some in the form of their compounds. Reactive
elements like Sodium, Potassium, Magnesium and Calcium are not found in nature as free
elements.

Q.6) Choose the incorrect statement


a) When a solid melts its temperature changes
b) Water vapour at 100 degree Celsius have more energy than normal water at the
same temperature
c) Change in pressure can change the state of matter
d) Both (a) and (b)

Q.6) Solution (a)


When a solid melts, its temperature doesnt change. The heat energy is used by the particles
to overcome the force of attraction.
http://www.dummies.com/how-to/content/why-temperature-remains-constant-during-aphase-ch.html

Q.7). Recently NASAs new horizon had sent close-up images of Pluto. Consider following
statements about New Horizons
1. New Horizons is an interplanetary space probe that was launched as a part of NASA's
New Frontiers program
2. A space probe is a robotic spacecraft that leaves Earth orbit and explores space.
3. Space environment around Pluto and its moons has a lot of dust.
Select the correct statements
a)
b)
c)
d)

1 only
1 and 2 only
All of the above
None of the above

www.iasbaba.com

Page 5

IASbaba 60 Day Plan- Prelims Test 2016 SCIENCE & TECHNOLOGY & CURRENT AFFAIRS [DAY 17]

Q.7) Solution (b)


Space environment around Pluto and its moons is almost empty, containing only about six
dust particles per cubic mile, according to data collected by a student-built instrument riding
on NASAs New Horizons spacecraft.
Further reading - http://www.thehindu.com/sci-tech/science/space-around-pluto-nearlydust-freenasa/article8370579.ece
Further reading - https://en.wikipedia.org/wiki/New_Horizons
Space probe may approach the Moon; enter interplanetary space; flyby, orbit, or land on
other planetary bodies; or approach interstellar space.

Q.8) Consider the following statements about jet lag


1. It is a physiological condition which results from alterations to the body's circadian
rhythms resulting from rapid long-distance trans-meridian travel on high-speed
aircraft.
2. It is also known as desynchronosis or circadian dysrhythmia
Select the correct statements
a)
b)
c)
d)

1 only
2 only
Both 1 and 2
Neither 1 nor 2

Q.8) Solution (c)


Mental health implication - Jet lag may affect the mental health of vulnerable individuals.
When travelling across time zones, there is a "phase-shift of body temperature, rapid-eyemovement sleep, melatonin production, and other circadian rhythms
More reading - https://en.wikipedia.org/wiki/Jet_lag
http://www.thehindu.com/sci-tech/health/fresh-method-developed-to-deal-withjetlag/article8668714.ece

www.iasbaba.com

Page 6

IASbaba 60 Day Plan- Prelims Test 2016 SCIENCE & TECHNOLOGY & CURRENT AFFAIRS [DAY 17]

Q.9) Mentha also known as mint is a very beneficial crop for farmers in India. Consider the
statements regarding crop Mentha
1. India is a net importer of mentha
2. It is included in Minimum Support Price (MSP)
3. Menthol is obtained from plant mentha
Select the correct statement/s
a)
b)
c)
d)

Only 1
1 and 2
1, 2 and 3
Only 3

Q.9) Solution (d)


Just as rice, wheat and cotton are among the 25 crops that qualify for a governmentmandated minimum support price, the CSIR is framing a proposal to push for mentha (the
plant that gives menthol), lavender, lemongrass, artemisia annua (the source of the
antimalarial compound artemisinin), geranium, lavender and rose oil to be included in such
a category.
India was an importer of mentha 40 years ago. Today it is the worlds leading producer. This
is because mentha varieties were specifically bred to exude greater quantities of menthol
mint oil. In 2014, mentha acreage spanned 3,00,000 hectares.
http://www.thehindu.com/todays-paper/tp-national/csir-wants-farmers-to-consideralternatives/article8669484.ece

Q.10) Consider the following statements regarding e-waste


1. Pyro metallurgy and hydro metallurgy are the processes used to extract gold from ewaste
2. Pyro metallurgy is a process in which leaching chemicals are used.
3. Hydro metallurgy is a process to extract gold using high temperatures.
Select the incorrect statements
a)
b)
c)
d)

Only 1
Only 1 and 2
Only 2 and 3
All of the above

www.iasbaba.com

Page 7

IASbaba 60 Day Plan- Prelims Test 2016 SCIENCE & TECHNOLOGY & CURRENT AFFAIRS [DAY 17]

Q.10) Solution (c)


Pyro metallurgy is a process to extract gold using high temperature and hydro metallurgy is
a process to in which leaching chemicals are used.

Q.11) Consider the following statements:


1. Orbital speed of a satellite is independent of its mass
2. Orbital speed of a satellite depends upon the radius of orbit (height of satellite from
the surface of earth)
3. Period of revolution of a satellite is independent of its mass
4. Escape velocity is independent of the mass, shape and size of the body and its
direction of projection
Which of the statements given above is/are correct?
a)
b)
c)
d)

3 and 4 only
1, 2 and 3 only
1, 3 and 4 only
All of the above

Q.11) Solution (d)

Orbital speed of a satellite is independent of its mass. Hence satellites of different


masses revolving in the orbit of same radius have same orbital speed.
Orbital speed of a satellite depends upon the radius of orbit (height of satellite from the
surface of earth). Greater the radius of orbit, lesser will be the orbital speed.
The orbital speed of a satellite revolving near the surface of earth is 7.9 km/sec
Time taken by a satellite to complete one revolution in its orbit is called its period of
revolution.
Period of revolution of a satellite depends upon the height of satellite from the surface
of earth. Greater the height more will be the period of revolution.
However, period of revolution of a satellite is independent of its mass.
The period of revolution of satellite revolving near the surface of earth is 1 hour 24
minute (84 minute)

www.iasbaba.com

Page 8

IASbaba 60 Day Plan- Prelims Test 2016 SCIENCE & TECHNOLOGY & CURRENT AFFAIRS [DAY 17]

Q.12) Consider the following statements and select the incorrect statement/s from the
codes given below:
1. The addition of detergent or soap decreases the viscosity of water and thus
increases the cleaning ability
2. If a clean and dry needle is kept slowly on the surface of water, it floats due to
surface tension of water
Choose the appropriate code:
a) 1 only
b) 2 only
c) Both 1 and 2
d) Neither 1 nor 2

Q.12) Solution (a)

Both the given statements are examples of surface tension property of water
If a clean and dry needle is kept slowly on the surface of water, it floats due to surface
tension of water
The addition of detergent or soap decreases the surface tension of water and thus
increases the cleaning ability
Surface Tension is the property of a liquid by virtue of which it has the tendency to have
the area of its free surface minimum as if it were under tension like a stretched elastic
membrane
A liquid drop attains spherical shape due to surface tension as for given volume, sphere
has minimum surface area

Q.13) Identify the incorrect pair/s from the following:


1.
2.
3.
4.

Biggest Solar Observatory : : Kodaikanal


National Remote Sensing Centre : : Mahendragiri
INSAT Master Control Facility : : Hassan
Indian Deep Space Network (IDSN) : : Byalalu

Choose the correct code:


a)
b)
c)
d)

3 only
3 and 4 only
1 and 2 only
2 only

www.iasbaba.com

Page 9

IASbaba 60 Day Plan- Prelims Test 2016 SCIENCE & TECHNOLOGY & CURRENT AFFAIRS [DAY 17]

Q.13) Solution (c)


Explanation:
Biggest Solar Observatory is in Udaipur, Rajasthan on an island in the Fateh Sagar Lake.
The sky conditions at Udaipur are quite favourable for solar observations. Since the
observatory is situated amidst a large mass of water, air turbulence which occurs due to
ground heating by sun's rays is decreased. This improves the image quality and accuracy
(average between 1-2 arc seconds).
National Remote Sensing Centre is in Hyderabad not Mahendragiri
Refer the below fig. for other space related centres

www.iasbaba.com

Page 10

IASbaba 60 Day Plan- Prelims Test 2016 SCIENCE & TECHNOLOGY & CURRENT AFFAIRS [DAY 17]

Q.14) Which among the following waves are examples of Electromagnetic waves?
1) X-rays
2) -rays
3) -rays
4) -rays
5) Ultrasonic wave
Choose the appropriate code:
a)
b)
c)
d)

1, 2, 3 and 4 only
1 and 2 only
1 and 5 only
1, 2 and 5 only

Q.14) Solution (b)

Electromagnetic waves or non-mechanical waves are those waves which do not require
medium for their propagation i.e. which can propagate even through the vacuum are
called non-mechanical wave.
In other words, electromagnetic waves are waves which can travel through the vacuum
of outer space. Mechanical waves, unlike electromagnetic waves, require the presence
of a material medium in order to transport their energy from one location to another.
Following waves are not electro-magnetic 1) Cathode rays 2) -rays 3) -rays 4) Sound
wave 5) Ultrasonic wave 6) Canal rays
Some important Electromagnetic waves 1) -rays 2) X-rays 3) UV rays 4) Visible
radiation 5) Infrared rays 6) Radio waves

Q.15) Consider the following statements in regard to Indias 3-stage Nuclear Power
Programme:
1) The first stage Pressurised Heavy Water Reactors (PHWRs) use natural uranium as
fuel and heavy water as moderator and coolant
2) The second stage Fast Breeder Reactors (FBRs) use plutonium (which is produced by
irradiation of uranium-238) and liquid sodium as coolant
3) The third stage is based on the thorium-uranium-233 cycle. Uranium-233 is obtained
by irradiation of thorium.
Which of the statements given above is/are correct?
a) 2 and 3 only
b) 1 and 2 only
www.iasbaba.com

Page 11

IASbaba 60 Day Plan- Prelims Test 2016 SCIENCE & TECHNOLOGY & CURRENT AFFAIRS [DAY 17]

c) 1 and 3 only
d) All of the above
Q.15) Solution (d)

All the statements are correct and self-explanatory

Q.16) Match List I with List II and select the correct answer using the code given below the
Lists:
LIST I
LIST II
A. Anupam-Ameya
1. BARC
B. PARAM Yuva
2. ISRO
C. SAGA
3. C-DAC
D. Vikram-100
4. Physical Research Laboratory

a)
b)
c)
d)

A-B-C-D
1-2-3-4
1-3-2-4
4-2-3-1
4-3-2-1

Q.16) Solution (b)

VIKRAM-100 is the 13th fastest supercomputer in India and is more powerful than 200
desktop computers
VIKRAM -100 was inaugurated on 26 June 2015, by Prof. U. R. Rao at the Physical
Research Laboratory
Vikram-100 is a High Performance Computing (HPC) Cluster (named after eminent
scientist Dr Vikram Sarabhai)
Physical Research Laboratory (PRL), is an organisation that was founded by him at
Ahmedabad in 1947

Q.17) Which of the following are illustrations of total internal reflection?


1) Shining of air bubble in water
2) Sparkling of diamond
3) Increase in duration of suns visibility
4) Mirage and looming
5) Ophthalmoscope

www.iasbaba.com

Page 12

IASbaba 60 Day Plan- Prelims Test 2016 SCIENCE & TECHNOLOGY & CURRENT AFFAIRS [DAY 17]

Choose the appropriate code:


a) 1, 2, 3 and 4 only
b) 1 , 2 and 5 only
c) 1, 2, 3 and 5 only
d) All of the above

Q.17) Solution (d)


Total Internal Reflection:
If light is propagating from denser medium towards the rarer medium and angle of
incidence is more than critical angle, then the light incident on the boundary is reflected
back in the denser medium, obeying the laws of reflection. This phenomenon is called
total internal reflection as total light energy is reflected, no part is absorbed or
transmitted.
For total internal reflection,
1) Light must be propagating from denser to rarer medium
2) Angle of incidence must exceed the critical angle
Q.18) Which among the following statements is/are true in regard to DNA and RNA?
1) Sugar is deoxyribose type in DNA, whereas sugar is just ribose type in RNA
2) DNA is double stranded structure, whereas RNA is single stranded structure
3) DNA is found in both nucleus and cytoplasm, RNA is found only in nucleus
Which of the statements given above is/are correct?
a)
b)
c)
d)

2 and 3 only
1 and 2 only
1 and 3 only
All of the above

Q.18) Solution (b)

DNA is found mainly in nucleus, whereas RNA is found in both nucleus and cytoplasm

www.iasbaba.com

Page 13

IASbaba 60 Day Plan- Prelims Test 2016 SCIENCE & TECHNOLOGY & CURRENT AFFAIRS [DAY 17]

Q.19) Which among the following statements is/are true?


1) Apart from blood groups O, A, B and AB, there is a special type of blood group (hh)
2) A type blood can accept blood from A type or O type and not from AB or B
type donors
3) The main reason behind the difference in blood of human is the glyco protein which
is found in RBC called antigen
Which of the statements given above is/are correct?
a)
b)
c)
d)

2 and 3 only
1 and 2 only
1 and 3 only
All of the above

Q.19) Solution (d)

Apart from blood groups O, A, B and AB, there is a special type of blood group (hh)
(hh)- a rare special blood group first discovered in Bombay in 1952, and hence
christened as Bombay Blood.
People who carry this rare blood type, about 1 in 10, 000 Indians, can accept blood only
from another Bombay Blood type individual, and not from anyone who is O, A, B or AB
type.

Q.20) Consider the following statements with regard to Atomic Energy Regulatory Board
(AERB)
1) AERB is engaged in the development of nuclear power technology, applications of
radiation technologies in the fields of agriculture, medicine, industry, and basic
research.
2) The safety standards formulated by AERB are at par with those recommended by the
international organisations such as the International Atomic Energy Agency (IAEA)
and the International Commission on Radiological Protection (ICRP).
Choose the appropriate code from below:
a)
b)
c)
d)

Only 1 is correct
Only 2 is correct
Both 1 and 2 are correct
Both 1 and 2 are incorrect

www.iasbaba.com

Page 14

IASbaba 60 Day Plan- Prelims Test 2016 SCIENCE & TECHNOLOGY & CURRENT AFFAIRS [DAY 17]

Q.20) Solution (b)

Department of Atomic Energy (not AERB), established in 1954 is engaged in the


development of nuclear power technology, applications of radiation technologies in the
fields of agriculture, medicine, industry, and basic research.
Atomic Energy Regulatory Board (AERB) is an independent body, the Atomic Energy
Regulatory Board (AERB) monitors safety.
The safety standards formulated by AERB are at par with those recommended by the
international organisations such as the International Atomic Energy Agency (IAEA) and
the International Commission on Radiological Protection (ICRP).

Q.21) Razmnama is a Persian translation of which of the following Indian works


a)
b)
c)
d)

Ramayana
Mahabharata
Bhagavathgeetha
Atarvanaveda

Q.21) solution (b)


PM modi mentioned this in a speech at a cultural exchange function organised during his
visit to Iran

Q.22) Leelavathi of Bhaskara is an ancient treatise related to which of the following


subject
a)
b)
c)
d)

Mathematics
Medicine
Surgery
Poetry

Q.22) solution (a)


PM modi mentioned this in a speech at a cultural exchange function organised during his
visit to Iran

www.iasbaba.com

Page 15

IASbaba 60 Day Plan- Prelims Test 2016 SCIENCE & TECHNOLOGY & CURRENT AFFAIRS [DAY 17]

Q.23) Consider the following


1. Decrease in temperature.
2. Exposure to low tides
3. Run off and pollution
Which of the above is/are major cause of coral bleaching?
a)
b)
c)
d)

1 and 2 only
2 and 3 only
1 and 3 only
All

Q.23) solution (d)

Q.24) Consider the following


1. Olive Ridley turtles
2. Hawks bill turtles
3. Leather back turtle
Which of the above group of turtles come for mass nesting to India?
a)
b)
c)
d)

1 and 2 only
2 and 3 only
1 and 3 only
All

Q.4) solution (d)


Hawksbill turtle come for mass nesting in Lakshadweep and Andaman Islands, leather back
turtles in Andaman and Olive Ridley turtles in Orissa

Q.25) Pradhan Mantri Fasal bhima yojana aims to unburden the farmers who are exposed
to the vulnerabilities of natural calamities. Consider the following statements
1. The scheme aims to provide 2% annual insurance premium for Rabi crops
2. For horticulture crops the rate of premium is 10 %
Select the correct answer
www.iasbaba.com

Page 16

IASbaba 60 Day Plan- Prelims Test 2016 SCIENCE & TECHNOLOGY & CURRENT AFFAIRS [DAY 17]

a)
b)
c)
d)

1 only
2 only
Both
None

Q.25) solution (d)


For Rabi crop 1.5 percent
For horticulture crop 5 percent

Q.26) Consider the following rivers


1. Mandovi
2. Zuari
3. Barak
Which of the above river does not flow in the state of Goa?
a)
b)
c)
d)

2 and 3 only
1 and 3 only
3 only
1 and 2 only

Q.26) solution (c)


Mandovi and zuari flow in the state of Goa

Q.27) Operation Red lotus is a group of Maratha Mughal alliance led by which of the
following freedom fighter
a)
b)
c)
d)

Balaji baji rao


Shah Alam
Tantya tope
Balaji Vishwanath

Q.27) solution (c)

www.iasbaba.com

Page 17

IASbaba 60 Day Plan- Prelims Test 2016 SCIENCE & TECHNOLOGY & CURRENT AFFAIRS [DAY 17]

Operation Red Lotus, 1857 A Maratha-Mughal alliance against the British led by Tatya
Tope

Q.28) Consider the following


1. World bank New York
2. WTO Washington
3. UNESCO Paris
Which of the above international organisation and there headquarters is wrongly matched?
a)
b)
c)
d)

1 and 3 only
1 and 2 only
2 and 3 only
None of the above

Q.28) solution (b)

World bank headquarters is in Washington and WTO headquarters is in Geneva Switzerland

Q.29) Amir Khusro, a scholar poet and musicologist of rare talents was present in the
court of which medieval Indian ruler
a)
b)
c)
d)

Allauddin khilji
Akbar
Iltimush
Balban

Q.29) solution (a)

Amir Khusro, a scholar poet and musicologist of rare talent in the court of Allauddin Khilji
(13th Century, A.D.) is credited with the introduction of entirely new forms and styles in
Hindustani music which are still in practice today.

www.iasbaba.com

Page 18

IASbaba 60 Day Plan- Prelims Test 2016 SCIENCE & TECHNOLOGY & CURRENT AFFAIRS [DAY 17]

Q.30) Consider the following statement with respect to CERN


1. CERN is the worlds largest body of experts in nuclear and particle physics, where
scientists and engineers are probing the fundamental structure of universe by using
the most sophisticated scientific instruments and advanced systems.
2. CERN is the birthplace of www (World Wide Web) which is regularly used to
browse the internet.
Select the correct option with respect to CERN
a)
b)
c)
d)

1 only
2 only
Both
None

Q.30) solution (c)

CERN is the worlds largest body of experts in nuclear and particle physics, where
scientists and engineers are probing the fundamental structure of universe by using
the most sophisticated scientific instruments and advanced systems. CERN was
established in 1954 and is based in Geneva.
Significantly, CERN is the birthplace of www (World Wide Web) which is regularly
used to browse the internet. Similarly, the touch screen widely used in smart phones
was also first developed by CERN.

www.iasbaba.com

Page 19

IASbaba 60 Day Plan- Prelims Test 2016 ENVIRONMENT & CURRENT AFFAIRS [DAY 18]

Q.1) Which of the following is/are legally binding?


1. Nagoya Protocol
2. Convention on Biological Diversity
3. Paris Agreement on Climate Change
Select the correct code
a)
b)
c)
d)

Only 2
1 and 2
1, 2 and 3
1 and 3

Q.1) Solution (c)


http://cdkn.org/2015/12/feature-is-the-paris-agreement-legally-binding/?loclang=en_gb

Q.2) Which among the following can effectively be examples of Secondary Succession?
1.
2.
3.
4.

Formation of a new Island after volcanic eruption


The renewal of a forest after a fire
A flooded land
The renewal of a crop after harvesting

Select the correct option


a)
b)
c)
d)

1, 2 and 3
2, 3 and 4
1, 3 and 4
All

Q.2) Solution (b)

Examples of secondary succession include:

The renewal of a forest after a fire: The fire itself destroys a majority of different
types of trees and plant life. Because seeds and roots and other plant and tree parts
remain in and on the soil, gradually the plants and trees begin to grow again and
eventually return to the state of the original ecosystem.

www.iasbaba.com

Page 1

IASbaba 60 Day Plan- Prelims Test 2016 ENVIRONMENT & CURRENT AFFAIRS [DAY 18]

The renewal of a crop after harvesting: A crop is completed harvested when it


becomes ripe. Without new seeds being planted, the crop can regenerate the
following year due to the plants and seeds that remained after harvesting.
A forest renews after logging: A large amount of trees were chopped down by
loggers in order to create building materials. Over time, trees grow in and the area
returns to its previous state.
Renewal after disease: A plant population can be very negatively affected by a
variety of infectious plant diseases. If the entire population dies, but the soil and
roots remain, it is possible for secondary succession to occur and for the population
of those plants to return.
A flood can ruin farmlands. However, because the soil remains after the waters
recede, over the course of many years a natural secondary succession can occur and
the vegetation that had previously grown there can grow again.
Plants can be very susceptible to attack from pests, particularly if there is an
overpopulation of those pests. When this occurs, the plant population in one area
can be completely destroyed. However, when the pest overpopulation is resolved,
the plants are able to live again and thrive in the soil in which they previously had
lived.

Q. 3) Consider the following with regard to National Green Tribunal (NGT)


1. NGT is bound by procedure laid down under the Code of Civil Procedure, 1908
2. He tribunal has Original as well as Appellate jurisdiction
3. Tribunal is competent to hear cases for several acts such as Forest Right Act,
Biological Diversity Act, Environment (Protection) Act, Water & Air (Prevention &
control of Pollution) Acts.
Select the incorrect code
a)
b)
c)
d)

1 and 2
Only 2
1 and 3
1, 2 and 3

Q.3) Solution (c)


The Tribunal's dedicated jurisdiction in environmental matters shall provide speedy
environmental justice and help reduce the burden of litigation in the higher courts. The
Tribunal shall not be bound by the procedure laid down under the Code of Civil Procedure,
1908, but shall be guided by principles of natural justice. The Tribunal is mandated to make

www.iasbaba.com

Page 2

IASbaba 60 Day Plan- Prelims Test 2016 ENVIRONMENT & CURRENT AFFAIRS [DAY 18]

and endeavour for disposal of applications or appeals finally within 6 months of filing of the
same.
The Tribunal has Original Jurisdiction on matters of substantial question relating to
environment (i.e. a community at large is affected, damage to public health at broader
level) & damage to environment due to specific activity (such as pollution).
Also Tribunal is competent to hear cases for several acts such as Forest (Conservation) Act,
Biological Diversity Act, Environment (Protection) Act, Water & Air (Prevention & control of
Pollution) Acts etc. and also have appellate jurisdiction related to above acts after
establishment of Tribunal within a period of 30 days of award or order received by
aggrieved party.

Q.4) Consider the following statements about the Green India Mission (GIM). Its mission
is/are
1. To increase forest/tree cover to the extent of 5 million hectares
2. To enhance ecosystem services like timber and non-timber produce
3. To enhance provisioning services like carbon sequestration, hydrological services and
biodiversity
Identify the incorrect statement/s
a)
b)
c)
d)

1 and 2
2 and 3
Only 1
1 and 3

Q.4) Solution (b)


National Mission for a Green India (GIM)
To increase forest/tree cover to the extent of 5 million hectares (mha) and improve quality
of forest/tree cover on another 5 mha of forest/non-forest lands;
To improve/enhance eco-system services like carbon sequestration and storage (in forests
and other ecosystems), hydrological services and biodiversity; along with provisioning
services like fuel, fodder, and timber and non-timber forest produces (NTFPs); and to
increase forest based livelihood income of about 3 million households.

www.iasbaba.com

Page 3

IASbaba 60 Day Plan- Prelims Test 2016 ENVIRONMENT & CURRENT AFFAIRS [DAY 18]

Q.5) Loss of biodiversity in a region may lead to


1. Decline in plant production
2. Increased resistance to environmental perturbations such as drought
3. Decreased variability in certain ecosystem processes such as plant productivity,
water use, and pest and disease cycles
Select the correct statement/s
a)
b)
c)
d)

1 and 2
Only 3
Only 1
Only 2

Q.5) Solution (c)


Lowered resistance to environmental perturbations such as drought and increased
variability in certain ecosystem processes such as plant productivity, water use, and pest
and disease cycles

Q.6) Consider the following assumptions regarding Pugmark method


1. The entire potential tiger habitat in India had to be effectively covered during the
pugmark census.
2. All the four paw prints of every individual tiger in the surveyed area had to be
detected during the censuses.
3. The same hind pugmark of each one of these individual tigers must be lifted from
suitable and comparable substrates or from standardized soil track-plots.
4. The shape of each pugmark lifted had to be recorded without distortion by the
thousands of census personnel involved in the operation.
5. Supervisory officials were expected to be subsequently able to segregate the
pugmarks of each individual tiger correctly, based on footprint shape, track
measurements, and prior local knowledge.
Select the appropriate code
a)
b)
c)
d)

Failure of assumptions 1 and 2 would lead to under-counts


Failure of assumptions 3 and 4 would lead to over-counts
Failure of assumption 5 could lead to either undercounts or over-counts.
a, b and c holds true

www.iasbaba.com

Page 4

IASbaba 60 Day Plan- Prelims Test 2016 ENVIRONMENT & CURRENT AFFAIRS [DAY 18]

Q.6) Solution (d)


Learn about Pugmark technique.
http://www.conservationindia.org/articles/why-the-pugmark-census-used-to-monitor-tigerpopulations-failed

Q.7) Consider the following about South Asia Wildlife Enforcement Network
1. India is a formal member of SAWEN
2. SAWEN, South Asia Wildlife Enforcement Network is an initiative to control the
transboundary Wildlife Crime
Select the correct statement/s
a)
b)
c)
d)

Only 1
Only 2
Both
None

Q.7) Solution (c)


http://pib.nic.in/newsite/PrintRelease.aspx?relid=138834

Q.8) Consider the following regarding Bharat Stage (BS) Emission Norms in India
1. Currently, India is following BS IV Norms
2. India has planned to implement BS V norms from 2019
3. BS VI, particulate matter emission for diesel cars and nitrogen oxide levels are
expected to be substantially higher than in BS IV.
Select the incorrect code
a)
b)
c)
d)

1 and 2
Only 2
2 and 3
Only 3

Q.8) Solution (c)

www.iasbaba.com

Page 5

IASbaba 60 Day Plan- Prelims Test 2016 ENVIRONMENT & CURRENT AFFAIRS [DAY 18]

India has planned to switch directly from BS IV to BS VI, skipping BS V


http://www.thehindubusinessline.com/opinion/all-you-wanted-to-know-about-bs-viemission-norms/article8120879.ece

Q.9) Consider the following countries


1.
2.
3.
4.
5.

Myanmar
Vietnam
China
Philippines
Malaysia

Mekong river flows through which of the above given countries?


a)
b)
c)
d)

1,2,3 &4
1,2,3 and 5 only
1,2 and 3 only
2&4 only

Q.9) Solution (c)


Refer atlas, it does not flow in Philippines and Malaysia

Q.10) Consider the following


1. Rashtriya Swasthya Bima Yojana (RSBY) insures people working in organised sector
only
2. As per this scheme organised sector worker and his family (unit of five) will be
covered with total sum of Rs. 30,000/- per family per annum
Select the correct option
a)
b)
c)
d)

1 only
2 only
Both
None

www.iasbaba.com

Page 6

IASbaba 60 Day Plan- Prelims Test 2016 ENVIRONMENT & CURRENT AFFAIRS [DAY 18]

Q.10) solution (d)

Rashtriya Swasthya Bima Yojana (RSBY) insures people working in unorganised


sector only
As per this scheme unorganised sector worker and his family (unit of five) will be
covered with total sum of Rs. 30,000/- per family per annum

Q.11) Consider the following


1. Sikkim is the first organic state in India
2. Indias highest peak Kanchenjunga is located in the state of Sikkim
3. River Teesta flows through Sikkim
Select the correct option with respect to the state of Sikkim
a)
b)
c)
d)

1, 2 & 3
1 & 2 only
2 & 3 only
1 & 3 only

Q.11) solution (a)

Sikkim is the first organic state in India


Indias highest peak Kanchenjunga is located in the state of Sikkim
River Teesta flows through Sikkim

Q.12) Consider the following statements regarding Zika Virus


1. It is named after the Zika forest in Ethiopia
2. The mosquitoes that spread dengue and chikungunya viruses are the same affecting
Zika virus
3. It can be spread through sexual contact
Select the correct code
a)
b)
c)
d)

1 and 2
2 and 3
Only 2
1, 2 and 3

Q.12) Solution (b)

www.iasbaba.com

Page 7

IASbaba 60 Day Plan- Prelims Test 2016 ENVIRONMENT & CURRENT AFFAIRS [DAY 18]

Q.13) Consider the following statements with respect to Spaceward bound programme
1. Spaceward bound programme is a project by NASA
2. The objective is to train the next generation of space explorers.
3. India has joined hands with NASA in this experiment to conduct microbiological
expeditions in places like Ladakh that has extreme climatic conditions
Select the correct option
a)
b)
c)
d)

1 & 2 only
2 & 3 only
1 & 3 only
All

Q.13) Solution (d)

An expedition to Ladakh to study the similarities of certain parts of the regions


topography and microbial life to Martian surroundings.
India is part of Spaceward Bound programme for the first time.
Ladakh offers a high UV exposed, dry ecosystem with Mars analogue topological
features.
The Space ward Bound is a NASA project.
It educates future space explorers and funds expeditions to places with extreme
climate conditions.
The objective is to train the next generation of space explorers.

Q.14) Corruption perception index is a report released by which of the following


international organisations
a)
b)
c)
d)

World bank
Transparency international
United Nations development programme
Amnesty international

Q.14) solution (b)


It is released by Transparency International

www.iasbaba.com

Page 8

IASbaba 60 Day Plan- Prelims Test 2016 ENVIRONMENT & CURRENT AFFAIRS [DAY 18]

Q.15) Nai Manzil is a government of India scheme related to development of which of the
following
a)
b)
c)
d)

Skill development
Recapitalisation of banks
Health
Tourism

Q.15) solution (a)

Under the scheme girls from minority communities will be imparted three month
skill development training in seven identified sectors relevant to the region

Q.16) Mobile academy, IT initiative of government of India is related to which of the


following option
a) Its a mobile-based application aimed at providing training services to 9 million
ASHAs.
b) It will be an IT-enabled tool to help tobacco users to quit tobacco.
c) It is an audio-based mobile service that delivers weekly audio messages to families
about pregnancy, child birth and child care.
d) It is a Mobile based application for filing quick IT returns
Q.16) Solution (a)
Its a mobile-based application aimed at providing training services to 9 million ASHAs. This
will aid in enhancing their inter-personal skills. Once registered, ASHAs can access the 240minute course via their mobile phones.

Q.17) The Red Data List published by the International Union for Conservation of Nature
and Natural Resources (IUCN) contain lists of
1. Information on plants, fungi and animals
2. Information on plants and animals only
3. It divides species into Six categories
Select the correct answer using the codes given below
a)
b)
c)
d)

1 and 3
1, 2 and 3
Only 3
Only 1

www.iasbaba.com

Page 9

IASbaba 60 Day Plan- Prelims Test 2016 ENVIRONMENT & CURRENT AFFAIRS [DAY 18]

Q.17) Solution (d)


http://www.iucnredlist.org/about/introduction

Q.18) Sacred groves are, tracts of forest were set aside, and all the trees and wildlife
within were venerated and given total protection. Sacred groves are found in following
places
1.
2.
3.
4.

Melghat Maharashtra
Aravalli Rajasthan
Western ghats - Karnataka and Maharashtra
Sarguja, Chanda and Bastar Madhya Pradesh

Select the correct answers


a)
b)
c)
d)

1, 3 and 4 only
2, 3 and 4 only
None of the above
All of the above

Q.18) Solution (b)


India has also a history of religious and cultural traditions that emphasised protection of
nature. In many cultures, tracts of forest were set aside, and all the trees and wildlife within
were venerated and given total protection. Such sacred groves are found in Khasi and Jaintia
Hills in Meghalaya, Aravalli Hills of Rajasthan, Western Ghat regions of Karnataka and
Maharashtra and the Sarguja, Chanda and Bastar areas of Madhya Pradesh. In Meghalaya,
the sacred groves are the last refuges for a large number of rare and threatened plants.

Q.19) Which of the floral groups are not found in India?


1.
2.
3.
4.
5.
6.
7.

Algae
Fungi
Bacteria
Lichens
Bryophytes
Pteridophytes
Gymnosperms

www.iasbaba.com

Page 10

IASbaba 60 Day Plan- Prelims Test 2016 ENVIRONMENT & CURRENT AFFAIRS [DAY 18]

8. Angiosperms
Select the correct answers
a)
b)
c)
d)

6 only
6, 7 and 8 only
5, 6, and 8
All of the above are found in India

Q.19) Solution (d)


All the floral groups found in India. Simple examples can be read from class 9 ncert

Q.20) What is common to the techniques (i) in vitro fertilisation, (ii) Cryo preservation and
(iii) tissue culture?
a)
b)
c)
d)

All are in situ conservation methods.


All are ex situ conservation methods.
All require ultra-modern equipment and large space.
All are methods of conservation of extinct organisms.

Q.20) Solution (b)

Q.21) Consider the following statements about Biodiversity


1. In biosphere diversity exists only at species level, genes level and ecological level.
2. It is the term popularized by socio biologist Robert May.
3. The Western Ghats have a greater amphibian species diversity than the Eastern
Ghats
Select the incorrect answer:
a)
b)
c)
d)

1 only
Only 1 and 2
Only 1, 2 and 3
All of the above

www.iasbaba.com

Page 11

IASbaba 60 Day Plan- Prelims Test 2016 ENVIRONMENT & CURRENT AFFAIRS [DAY 18]

Q.21) Solution (b)


In our biosphere immense diversity (or heterogeneity) exists not only at the species level
but at all levels of biological organisation ranging from macromolecules within cells to
biomes. Biodiversity is the term popularised by the socio biologist Edward Wilson to
describe the combined diversity at all the levels of biological organisation.
The most important of them are
Genetic diversity: A single species might show high diversity at the genetic level over its
distributional range. The genetic variation shown by the medicinal plant Rauwolfia vomitoria
growing in different Himalayan ranges might be in terms of the potency and concentration
of the active chemical (reserpine) that the plant produces. India has more than 50,000
genetically different strains of rice, and 1,000 varieties of mango.
Species diversity: The diversity at the species level. For example, the Western Ghats have
greater amphibian species diversity than the Eastern Ghats.
Ecological diversity: At the ecosystem level, India, for instance, with its deserts, rain forests,
mangroves, coral reefs, wetlands, estuaries, and alpine meadows has greater ecosystem
diversity than a Scandinavian country like Norway.
Source 12th Biology NCERT chapter biodiversity and conservation P.no. 258

Q.22) Consider following statements about IUCN RED LIST


1. It uses a set of criteria which are not relevant to all species and all regions of the
world.
2. The IUCN Red List is recognized as the most authoritative guide to the status of
biological diversity.
Select the correct answers
a)
b)
c)
d)

Only 1
Only 2
Both
None

Q.22) Solution (b)


http://www.iucn.org/resources/conservation-tools/iucn-red-list-threatened-species

www.iasbaba.com

Page 12

IASbaba 60 Day Plan- Prelims Test 2016 ENVIRONMENT & CURRENT AFFAIRS [DAY 18]

Q.23) Consider the following statements


1. Agasthyamala Biosphere Reserve has been included in UNESCOs list of World
Biosphere Reserve Network.
2. Agasthyamala Biosphere Reserve rests in between Tamil Nadu and Karnataka
3. Kannikarans are the indigenous tribes residing in this area
Select the incorrect answer
a)
b)
c)
d)

2 only
1 and 3 only
2 and 3 only
None of the above

Q.23) Solution (a)


http://www.thebetterindia.com/49714/agasthyamala-unesco-biosphere-reserve-network/

Q.24) The ecological footprint is a measure of human demand on the Earth's ecosystems.
It measures1. How much of the biological capacity of the planet is demanded by a given human
activity or population
2. Ecological Footprints is calculated for overall activity of a nation or population and
activities such as industrialization etc.
3. The disadvantage of Ecological Footprint is that it cannot be calculated for individual
people or area.
Select the incorrect statement/s
a)
b)
c)
d)

1 and 2
Only 2
1 and 3
Only 3

Q.24) Solution (d)


The Ecological Footprint is a resource accounting tool that measures how much biologically
productive land and sea is used by a given population or activity, and compares this to how
much land and sea is available. Productive land and sea areas support human demands for
food, fibre, timber, energy, and space for infrastructure. These areas also absorb the waste
www.iasbaba.com

Page 13

IASbaba 60 Day Plan- Prelims Test 2016 ENVIRONMENT & CURRENT AFFAIRS [DAY 18]

products from the human economy. The Ecological Footprint measures the sum of these
areas, wherever they physically occur on the planet. The Ecological Footprint is used widely
as a management and communication tool by governments, businesses, educational
institutions, and non-governmental organizations.

Ecological Footprint accounts answer a specific research question: how much of the biological
capacity of the planet is demanded by a given human activity or population? To answer this
question, the Ecological Footprint measures the amount of biologically productive land and water
area an individual, a city, a country, a region, or all of humanity uses to produce the resources it
consumes and to absorb the waste it generates with todays technology and resource management
practices. This demand on the biosphere can be compared to biocapacity, a measure of the

amount of biologically productive land and water available for human use. Biologically
productive land includes areas such as cropland, forest, and fishing grounds, and excludes
deserts, glaciers, and the open ocean
Ecological Footprints can be calculated for individual people, groups of people (such as a
nation), and activities (such as manufacturing a product).
The Ecological Footprint of a person is calculated by considering all of the biological
materials consumed, and all of the biological wastes generated, by that person in a given
year. These materials and wastes each demand ecologically productive areas, such as
cropland to grow potatoes, or forest to sequester fossil carbon dioxide emissions. All of
these materials and wastes are then individually translated into an equivalent number of
global hectares.

Q.25) Consider the statements regarding Coral bleaching


1. It is the loss of zooxanthellae through either expulsion or loss of algal pigmentation.
2. It happens due to increase in ocean temperature only
3. All the corals undergoing bleaching are dead corals
Select the correct code
a)
b)
c)
d)

1 and 3
Only 2
Only 1
2 and 3

Q.25) Solution (c)

www.iasbaba.com

Page 14

IASbaba 60 Day Plan- Prelims Test 2016 ENVIRONMENT & CURRENT AFFAIRS [DAY 18]

First statement is true. Coral bleaching not only happens because of warm temperature but
can also happen due to Cold temperature. Not all bleaching events are due to warm water.
In January 2010, cold water temperatures in the Florida Keys caused a coral bleaching event
that resulted in some coral death. Water temperatures dropped 12.06 degrees
Fahrenheit lower than the typical temperatures observed at this time of year. Researchers
will evaluate if this cold-stress event will make corals more susceptible to disease in the
same way that warmer waters impact corals.
When a coral bleaches, it is not dead. Corals can survive a bleaching event, but they are
under more stress and are subject to mortality.

http://www.theaustralian.com.au/news/nation/great-barrier-reef-scientists-exaggeratedcoral-bleaching/news-story/99810c83f5a420727b12ab255256774b

www.iasbaba.com

Page 15

IASbaba 60 Day Plan- Prelims Test 2016 MODERN INDIA & CURRENT AFFAIRS [DAY 19]

Q.1) Congress Session of Calcutta in the year 1906 after Bengal Partition was significant in
many ways. Which of the following statements are correct about this Congress Session?
1.
2.
3.
4.

It was presided by Dr. Rashbihari Ghosh.


Congress adopted Swaraj as the goal of Indian people.
A resolution on partition of Bengal was passed.
The Congress got split into moderates and extremists.

Select the correct code from the following:


a)
b)
c)
d)

1,2 and 3
2 and 3
1,3 and 4
All of the above

Q.1) Solution (b)


The session was presided by Dada Bhai Naoroji. The congress split into two parts in the next
session i.e. Surat session of 1907.

Q.2) Home Rule league movement of 1916 played an important role in creating political
awareness in masses and putting up a united front against the British rule. Consider the
following statements regarding the Indian Home Rule League:
1. It was designed on the lines of Irish Home Rule league.
2. It was started first by Annie Besant and later by Bal Gangadhar Tilak.
3. The aim of the league was to attain independence from the British rule and have self
government.
4. The two leagues cooperated with each other as well as with Congress and Muslim
League.
5. Tilaks movement was concentrated in Maharashtra while Besants movement in rest of
the country.
6. Montagues August declaration to gradually develop self governing institutions in India
led to the end of Home rule Movement.
Which of the above statements are correct?
a) 1,2,3,4 and 5
www.iasbaba.com
Page 1

IASbaba 60 Day Plan- Prelims Test 2016 MODERN INDIA & CURRENT AFFAIRS [DAY 19]

b) 1,4,5 and 6
c) 1,3,4,5 and 6
d) 1,2,3,5 and 6

Q.2) Solution (b)


It was started first by B.G. Tilak in April 1916 in Poone and later by Annie Besant in September
1916 in Madras.
The aim of the movement was to get self government for India within the British Empire.
(Source: History Class XII TN Board)

Q.3) On what charges were Bhagat Singh, Sukhdev and Rajguru sentenced to death?
1.
2.
3.
4.

For throwing bomb in the assembly


For killing of Asst. Superintendent of Police J.P. Saunders
For Kakori Conspiracy case
For the Hunger strike unto death in Lahore Central Jail

Which of the above reasons are correct?


a)
b)
c)
d)

1 and 2
2 only
2 and 3
1,2 and 4

Q.3) Solution (b)


The charge on which the three revolutionaries were sentenced to death was killing of Saunders.

Q.4) In which of the following movements did Mahatma Gandhi undertook fast unto death
for the first time to get his demands fulfilled?
a)
b)
c)
d)

Champaran Satyagraha
Kheda Satyagraha
Ahmadabad mill strike
Non-cooperation movement

www.iasbaba.com
Page 2

IASbaba 60 Day Plan- Prelims Test 2016 MODERN INDIA & CURRENT AFFAIRS [DAY 19]

Q.4) Solution (c)


Mahatma Gandhi undertook a fast unto death for the cause of Ahmadabad mill workers. Finally
the mill owners conceded to the just demands of the workers.

Q.5) Consider the following statements regarding Khudai Khidmatgar movement:


1. It was a non-violent freedom struggle against the British by pathans of North-West
Frontier province.
2. It was led by Maulana Abul Kalam Azad.
3. The movement is also known as the Red Shirt movement as the volunteers used to
wear red shirts.
Which of the above statements are correct?
a)
b)
c)
d)

All of the above


1 and 2
2 and 3
1 and 3

Q.5) Solution (d)


The movement was lead by Khan Abdul Ghaffar Khan also known as the Frontier Gandhi.

Q.6) Who amongst the following personalities have won Bharat Ratna award despite being a
non-Indian citizen?
1.
2.
3.
4.

Mother Teresa
Khan Abdul Ghaffar Khan
Nelson Mandela
Granville Austin

Select the correct code from the following:


a) 1,2 and 3
b) 2 and 3
www.iasbaba.com
Page 3

IASbaba 60 Day Plan- Prelims Test 2016 MODERN INDIA & CURRENT AFFAIRS [DAY 19]

c) 1,3 and 4
d) 3 only
Q.6) Solution (b)
Khan Abdul Ghaffar khan was the first non Indian to get a Bharat Ratna in 1987. Nelson
Mandela won the award in 1990.

Q.7) After the August offer of 1940, the radicals and leftists wanted to start a mass civil
disobedience movement. But Mahatma Gandhi insisted on individual Satyagraha. Which of
the following statements are correct about individual satyagraha of 1940?
1. It was not to seek independence but to affirm the right of speech.
2. It was started by Acharya Vinoba Bhave first and then by Jawaharlal Nehru.
3. Since it was not a mass movement, it attracted little enthusiasm and later in December,
Gandhiji suspended it.
Select the correct code from the following:
a)
b)
c)
d)

1 and 2
2 and 3
1 and 3
All of the above

Q.7) Solution (d)


The Congress was in a confused state again after the August Offer. The radicals and leftists
wanted to launch a mass Civil Disobedience Movement, but here Gandhi insisted on Individual
Satyagraha. The Individual Satyagraha was not to seek independence but to affirm the right of
speech. The other reason of this Satyagraha was that a mass movement may turn violent and
he would not like to see the Great Britain embarrassed by such a situation. This view was
conveyed to Lord Linlithgow by Gandhi when he met him on September 27, 1940.
The non-violence was set as the centerpiece of Individual Satyagraha. This was done by
carefully selecting the Satyagrahis. The first Satyagrahi selected was Acharya Vinoba Bhave,
who was sent to Jail when he spoke against the war. Second Satyagrahi was Jawahar Lal Nehru.
Third was Brahma Datt, one of the inmates of the Gandhis Ashram. They all were sent to jails
for violating the Defense of India Act. This was followed by a lot of other people. But since it
was not a mass movement, it attracted little enthusiasm and in December 1940, Gandhi
www.iasbaba.com
Page 4

IASbaba 60 Day Plan- Prelims Test 2016 MODERN INDIA & CURRENT AFFAIRS [DAY 19]

suspended the movement. The campaign started again in January 1941, this time, thousands of
people joined and around 20 thousand people were arrested.

Q.8) Consider the following statements:


1. Leaders and influential personalities returned their honorary posts and titles.
2. Students came out of Government educational institutions and national schools as Kashi
Vidhyapeet, Bihar Vidyapeet and Jamia Millia Islamia were set up.
3. People switched to swadeshi and took up charkha.
4. Movement abruptly called off by a violent incident in Gorakhpur District.
Which of the following mass movements is being discussed in the above statements?
a)
b)
c)
d)

Swadeshi Movement 1905-06


Non-cooperation Movement 1921-22
Civil Disobedience movement 1929-30
Quit India Movement 1942

Q.8) Solution (b)


Self explanatory

Q.9) The British government adopted the strategy of talking to different political parties by
convening the Round Table Conferences. Which of the following statements are incorrect
about these conferences?
1. Congress boycotted the first round table conference and civil disobedience movement
was going on in India.
2. After Gandhi-Irwin pact the civil disobedience movement was called off and Ghandhiji
attended the second round table conference.
3. Civil disobedience movement was relaunched after second round table conference.
4. Communal award was given in third round table conference.
Select the code from the following:
a) 4 only
b) 3 and 4
www.iasbaba.com
Page 5

IASbaba 60 Day Plan- Prelims Test 2016 MODERN INDIA & CURRENT AFFAIRS [DAY 19]

c) 1 and 3
d) 1,2 and 3
Q.9) Solution (a)
The Communal Award was basically a proposal on minority representation.
Thus, this award accorded separate electorates for Muslims, Europeans, Sikhs, Indian
Christians, Anglo Indians, Depressed Classes, and even Marathas. (Some seats in Bombay were
given to Marathas).
The depressed classes were given seats which had to be filled by election from the special
constituencies in which only they could vote. However, they were eligible to vote in the general
constituencies as well. The labor, Commerce and Industry, Mining and Planting, Landholders
were also given special electorates. Sikhs were 13.2% of the population in Punjab. Here they
were given 32 seats out of the total 175 seats.

Q.10) Which of the following slogans are correctly matched with the leaders who gave them?

1.
2.
3.
4.

Slogan
Do or Die
You give me blood, Ill give you freedom
Dilli Chalo
Swaraj is my birth right

Leader
Mahatma Gandhi
Subhash Chandra Bose
Pt. Jawaharlal Nehru
Bal Gangadhar Tilak

Select the correct code from the following:


a) All of the above
b) 1,2 and 3
c) 2,3 and 4
d) 1,2 and 4
Q.10) Solution (d)
Dilli Chalo was a war cry given by Subhash Chandra Bose to Indian National Army.

www.iasbaba.com
Page 6

IASbaba 60 Day Plan- Prelims Test 2016 MODERN INDIA & CURRENT AFFAIRS [DAY 19]

Q.11) The civil disobedience movement was started by Salt Satyagraha- by breaking the salt
tax law. No one imagined that such a small thing would stir the emotions of the masses in
India. Which of the following statements are correct about the salt satyagraha?
1. The salt march was started by Gandhi ji from his Sabarmati ashram to a coastal village of
Dandi.
2. The march was started with disciplined and trained volunteers of Sabarmati Ashram.
3. The path of the march was kept secret from the authorities so that they do not cause a
hindrance.
4. The Salt Satyagraha in Madras was led by Sarojini Naidu.
Select the code from the following:
a)
b)
c)
d)

1 and 2
3 and 4
1,2 and 4
All of the above

Q.11) Solution (a)


The path was declared in detail with halts and timings of the meetings. Mahatma Gandhi
wanted this march to become popular and get as much coverage as possible. Prior declaration
also made it possible for the people from all over India to join the march.
In Madras province, the salt Satyagraha was led by C. Rajagopalachari.
Q.12) The Government of India Act of 1935 was passed on the basis of the report of the
Simon Commission, the outcome of the Round Table Conferences and the White Paper issued
by the British Government in 1933. This was one of the most comprehensive Acts that shaped
the later Constitution of India. Which of the following was not a provision of this Act?
a) It introduced diarchy at the provinces and provincial subjects were divided into
reserved subjects and transferred subjects.
b) Division of power into three lists: Federal, provincial and concurrent.
c) Provincial Legislatures of Bengal, Madras, Bombay, United Provinces, Bihar and Assam
were made bicameral.
d) Extension of the principle of Separate Electorates to Sikhs, Europeans, Indian Christians
and Anglo Indians.
www.iasbaba.com
Page 7

IASbaba 60 Day Plan- Prelims Test 2016 MODERN INDIA & CURRENT AFFAIRS [DAY 19]

Q.12) Solution (a)


Abolition of Dyarchy and the introduction of Provincial Autonomy in the provinces. The
Governor was made the head of the Provincial Executive but he was expected to run the
administration on the advice of the Council of Ministers. Thus provincial government was
entursted to the elected Ministers. They were responsible to the popularly elected Legislative
Assemblies.
Introduction of Dyarchy at the Centre. The Governor-General and his councillors administered
the Reserved subjects. The Council of Ministers were responsible for the Transferred
subjects.

Q.13) Consider the following statements regarding Rowlatt Act


1. As per this act, any person could be arrested on the basis of suspicion
2. Arrested person could file an appeal or petition against his arrest
3. It was called the Black Act and was widely opposed
Select the correct statements
a)
b)
c)
d)

Only 1
1 and 3
2 and 3
All

Q.13) Solution (b)


In 1917, a committee was set up under the president ship of Sir Sydney Rowlatt to look into the
militant Nationalist activities. On the basis of its report the Rowlatt Act was passed in March
1919 by the Central Legislative Council. As per this Act, any person could be arrested on the
basis of suspicion. No appeal or petition could be filed against such arrests. This Act was called
the Black Act and it was widely opposed. An all-India hartal was organized on 6 April 1919.
Meetings were held all over the country. Mahatma Gandhi was arrested near Delhi. Two
prominent leaders of Punjab, Dr Satya Pal and Dr. Saifuddin Kitchlew, were arrested in
Amritsar.
www.iasbaba.com
Page 8

IASbaba 60 Day Plan- Prelims Test 2016 MODERN INDIA & CURRENT AFFAIRS [DAY 19]

Q.14) Consider the statements regarding Nehru Report of 1928


1. List of Fourteen Points as Muslim League demands were put forward against Nehru
Report
2. It advocated provincial autonomy and bicameral legislature
Select the incorrect code
a)
b)
c)
d)

Only 1
Only 2
Both
None

Q.14) Solution (d)


The Report published by this Committee came to be known as the Nehru Report. The Report
favoured:

Dominion Status as the next immediate step.


Full responsible government at the centre.
Autonomy to the provinces.
Clear cut division of power between the centre and the provinces.
A bicameral legislature at the centre

However, the leader of the Muslim League, Mohammad Ali Jinnah regarded it as detrimental to
the interests of the Muslims. Jinnah convened an All India Conference of the Muslims where he
drew up a list of Fourteen Points as Muslim League demand.

Q.15) Consider the following statements and identify the personality associated with it
1. He started the Indian National Union (1926) and the All India Nationalist Muslim Party
(1929)
2. Autobiographies like Tazhiran, Ghubir-i-khatir, Kahani and India wins Freedom
belongs to him
3. He started Al-Hilal, the Urdu Newspaper
www.iasbaba.com
Page 9

IASbaba 60 Day Plan- Prelims Test 2016 MODERN INDIA & CURRENT AFFAIRS [DAY 19]

Correct code
a)
b)
c)
d)

Khan Abdul Ghaffar Khan


M.A. Jinnah
Maulana Abul Kalam Azad
Liyaqat Ali

Q.15) Solution (c)


Read this article- Click

Q.16) Which of the following is/are not a lake?


1.
2.
3.
4.
5.

Hunza
Shyok
Otteri
Zaskar
Chandratal

Select the correct code


a)
b)
c)
d)

Only 2 and 4
1, 2 and 4
Only 1 and 2
1, 2, 3 and 4

Q.16) Solution (b)


Tsomorir- J&K
Chandratal- Himachal Pradesh
Hunza-Shyok-Zaskar rivers of J&K
Click here

www.iasbaba.com
Page 10

IASbaba 60 Day Plan- Prelims Test 2016 MODERN INDIA & CURRENT AFFAIRS [DAY 19]

Q.17) Consider the following


1.
2.
3.
4.

Hague Convention- To amend Warsaw Convention


Hague Code of Conduct- Ballistic Missile Proliferation
Hague Agreement- International Registration of Industrial designs
Hague Protocol- Civil Aspects of International Child Abduction

Choose the incorrect match


a)
b)
c)
d)

1, 3 and 4
3 and 4
1 and 4
None

Q.17) Solution (c)


Hague Convention- Civil Aspects of International Child Abduction
Hague Code of Conduct- Ballistic Missile Proliferation
Hague Agreement- International Registration of Industrial designs
Hague Protocol- To amend Warsaw Convention

Q.18) Select the correct statement


a)
b)
c)
d)

River Rhone flows through Germany


Germany borders with Slovenia, France and Switzerland
Black Forest is a large forested mountain in southwestern Germany
(b) and (c) are correct

Q.18) Solution (c)

Q.19) Identify the place referring to following statements


1. It is one of the World Heritage Sites
2. It is also known as Pearl of Desert and Tadmur
3. It is an oasis
www.iasbaba.com
Page 11

IASbaba 60 Day Plan- Prelims Test 2016 MODERN INDIA & CURRENT AFFAIRS [DAY 19]

Select the correct code


a)
b)
c)
d)

Palmyra
Cultural Landscape of Maymand
Ephesus
Susa

Q.19) Solution (a)

Q.20) Consider the following statements


1. A winter festival is in the named after this
2. It lies on the border of Arunanchal Pradesh and Myanmar
3. It lies on the crest of Pitkai Hills
Select the correct code
a)
b)
c)
d)

Rohtang Pass
Sela Pass
Pangsau Pass
Bum La Pass

Q.20) Solution (c)

http://theculturetrip.com/asia/india/articles/pangsau-pass-winter-festival-transcendingboundaries/

Q.21) Consider the following International organizations


1.
2.
3.
4.

The European Organization for Nuclear Research


The International Labour Organization (ILO)
World Health Organization
World Trade Organization

www.iasbaba.com
Page 12

IASbaba 60 Day Plan- Prelims Test 2016 MODERN INDIA & CURRENT AFFAIRS [DAY 19]

Which of the following International Organization is located in Switzerland?


a)
b)
c)
d)

1 and 2 only
2, 3 and 4 only
1, 2, 3, and 4
1 and 4 only

Q.21) Solution (c)


Click here

Q.22) Consider the following


1. The International Space Station (ISS) is a habitable artificial satellite, located in Medium
Earth orbit.
2. International Space Station can often be seen with naked eye
3. The ISS serves as a microgravity and space environment research laboratory in which
crew members conduct experiments in physics, astronomy, meteorology but not in the
field of in biology and human biology
Select the incorrect option
a)
b)
c)
d)

1 and 3 only
2 and 3 only
1 and 2 only
All

Q.22) Solution (a)

It is located in Low Earth Orbit. Approx 400 kms


Click here

Q.23) Consider the following statements


1. The -cells secretes a hormone called Insulin
www.iasbaba.com
Page 13

IASbaba 60 Day Plan- Prelims Test 2016 MODERN INDIA & CURRENT AFFAIRS [DAY 19]

2. The -cells secretes a hormone called glucagon


3. Prolonged hyperglycemia leads to a complex disorder called diabetes mellitus
Select the incorrect statement with respect to Diabetes
a)
b)
c)
d)

1 and 2 only
2 and 3 only
1 and 3 only
1, 2 and 3

Q.23) Solution (a)

The two main types of cells in the Islet of Langerhans are called -cells and -cells.
The -cells secretes a hormone called glucagon, while the -cells secretes insulin.
Prolonged hyperglycemia leads to a complex disorder called diabetes mellitus which is
associated with loss of glucose through urine and formation of harmful compounds
known as ketone bodies. Diabetic patients are successfully treated with insulin therapy.

Q.24) Consider the following Glaciers


1. Pindari Glacier
2. Gangotri Glacier
3. Panchchuli Glacier

Which of the above glacier is present in the state of Himachal Pradesh?


a)
b)
c)
d)

1 and 3 only
1 and 2 only
2 and 3 only
None

Q.24) Solution (d)


The entire above glaciers is present in the state of Uttarakhand

www.iasbaba.com
Page 14

IASbaba 60 Day Plan- Prelims Test 2016 MODERN INDIA & CURRENT AFFAIRS [DAY 19]

Q.25) Consider the following


1.
2.
3.
4.
5.

Interest payment
Grants to States
Defence
Loans to public enterprises
Loans to States

Which of the following fall under Plan Expenditure?


a)
b)
c)
d)

1, 2 and 3 only
2, 3 and 4 only
1,3,4 and 5 only
None of the above

Q.25) Solution (d)


Defence, Loans to public enterprises, Loans to states fall under non plan capital expenditure.
Whereas Interest payment, grant to states comes under non plan revenue expenditure.

www.iasbaba.com
Page 15

IASbaba 60 Day Plan- Prelims Test 2016 MODERN INDIA & CURRENT AFFAIRS [DAY 20]

Q.1) Which of the following statements are the correct reasons of Vellore Mutiny?
1.
2.
3.
4.

There was a racial prejudice and Indian sepoys were considered inferior.
Wearing ear rings and caste marks were prohibited.
The sepoys were asked to shave the chin and trim the moustache.
It was a popular belief that all the soldiers would be slowly converted to Christianity.

Select the code from the following:


a)
b)
c)
d)

1 and 4
1,3 and 4
1,2 and 3
All of the above

Q.1) Solution (d)


Several causes are attributed to the Vellore Mutiny. Indian sepoys had to experience numerous
difficulties when they went to serve in the Companys army. The sepoys were forced to serve
under the Company since their earlier patrons (the native chieftains) were all disappearing from
the scene.
The strict discipline, practice, new weapons, new methods and uniforms were all new to the
sepoys. Anything new appears to be difficult and wrong for a man who is well-settled in the old
way of life for a long-time. Sir John Cradock, the commander-in-chief, with the -approval of
Lord-William Bentinck, the Governor of Madras, introduced a new form of turban, resembling a
European hat. Wearing ear rings and caste marks were also prohibited.
The sepoys were asked to shave the chin and to trim the moustache. The sepoys felt that these
were designed to insult them and their religious and social traditions. There was also a popular
belief that this was the beginning of a process by which all of them would be converted to
Christianity. The English treated the Indian sepoys as their inferior. There was the racial
prejudice.

www.iasbaba.com

Page 1

IASbaba 60 Day Plan- Prelims Test 2016 MODERN INDIA & CURRENT AFFAIRS [DAY 20]

Q.2) During 17th and 18th Century, the East India company had a monopoly in the eastern
trade. Bribes were often given to monarchs to have their political backing. Which of the
following statements are correct about the relations between east India company and the
crown of England?
1. In 17th century company gave huge loans to the crown.
2. Charles II granted series of charters, empowering company to build forts, raise army and
make peace and war with the eastern powers.
3. Company was allowed to administer justice to all Englishmen and others living in English
settlements.
Select the code from the following:
a)
b)
c)
d)

1 and 2
2 and 3
1 and 3
All of the above

Q.2) Solution (d)


Self explanatory.
Source: Old Modern India NCERT

Q.3) Battle of Plassey between The Nawab of Bengal Siraj ud Daulah and English East India
company made company a major contender of political power in India. Consider the
following statements:
1. Siraj ud Daulah was defeated treacherously and there was hardly any fight in Battle of
Plassey.
2. British put Mir Qasim as a puppet nawab in place of Siraj ud Daulah.
3. Mir Qasim was an able administrator who tried to maintain public discipline and tried to
free himself from the influence of the British.
Which of the above statements are correct?
a) 1 and 2
b) 2 and 3
c) 1 and 3
www.iasbaba.com

Page 2

IASbaba 60 Day Plan- Prelims Test 2016 MODERN INDIA & CURRENT AFFAIRS [DAY 20]

d) All of the above

Q.3) Solution (c)


British put Mir Jafar as a puppet nawab in place of Siraj ud Daulah.

Q.4) Fourth Anglo-Mysore war was a decisive war between British and Mysore. Tipu sultan
was killed defending the city. Which of the following statements are correct about fourth
Anglo-Mysore war?
1. Mysore was attacked from all four sides by British, Marathas and the Nizam of
Hyderabad.
2. Tipus troops were outnumbered 4:1.
3. British annexed almost all of Mysore. The core area was restored to the eldest son of
Tipu whose ancestors ruled till 1947.
Select the correct code from the following:
a)
b)
c)
d)

1 and 2
2 and 3
1 and 3
All of the above

Q.4) Solution (a)


The Fourth Anglo-Mysore War (1799) saw the death of Tipu Sultan and further reductions in
Mysorean territory. Mysore's alliance with the French was seen as a threat to the East India
Company and Mysore was attacked from all four sides. Tipu's troops were outnumbered 4:1 in
this war. Mysore had 35,000 soldiers, whereas the British commanded 60,000 troops. The
Nizam of Hyderabad and the Marathas launched an invasion from the north. The British won a
decisive victory at the Siege of Seringapatam (1799). Tipu was killed during the defence of the
city. Much of the remaining Mysorean territory was annexed by the British, the Nizam and the
Marathas. The remaining core, around Mysore and Srirangapattana, was restored to the Indian
prince belonging to the Wodeyar dynasty, whose forefathers had been the actual rulers before
Hyder Ali became the de facto ruler. The Wodeyars ruled the remnant state of Mysore until
1947, when it joined the Union of India.
www.iasbaba.com

Page 3

IASbaba 60 Day Plan- Prelims Test 2016 MODERN INDIA & CURRENT AFFAIRS [DAY 20]

Q.5) Bahramji Malabari was one of the most prominent Parsi social reformers of India. Which
of the following statements are correct about him?
1. He criticized the prohibition of widow remarriage by Hindus and placed the blame
squarely with that religion's "priestly class" and the "social monopolists" (i.e. the
Brahmin caste) for their "vulgar prejudices.
2. Malabari published a set of Notes on Infant Marriage and Enforced Widowhood, that he
sent to 4,000 leading Englishmen and Hindus.
3. He was instrumental in passing the Age of Consent Act in 1894.
4. Malabari published a book in England An Appeal from the daughters of India.
Select the correct code from the following:
a)
b)
c)
d)

1,2 and 3
2,3 and 4
1,3 and 4
All of the above

Q.5) Solution (d)


Behramji Merwanji Malabari was an Indian poet, publicist, author, and social reformer best
known for his ardent advocacy for the protection of the rights of women.
He wrote extensively about raising the age of girls for marriage which gained sympathy of the
people of India. The pundits and shastris of Benares and Mathura listening to his lectures on
social reform started accepting the fairness of his arguments. Malabari published a book in
England An Appeal from the daughters of India.
He chose journalism as his career and devoted his life to fighting the evils of child marriage and
enforced widowhood among the Hindus. He was a fearless journalist. He travelled extensively
throughout India and made three trips to England. He was vociferous in his struggle to get
infant marriage and correlated evils abolished. Seeking womens emancipation he advocated,
If new India is to be blessed with a generation of free and enlightened sons, a nation to
manage its own affairs the Hindus of today might to see in their midst a race of free,
enlightened mothers. Lokmanya Bal Gangadhar Tilak advised Malabari to look after the
interests of women in his own community but Malabari relentlessly persisted in his efforts and
succeeded.

www.iasbaba.com

Page 4

IASbaba 60 Day Plan- Prelims Test 2016 MODERN INDIA & CURRENT AFFAIRS [DAY 20]

He became famous for raising the issue of freedom of Hindu widows to remarry, which was
debated throughout India. His successful campaign from 1884-1891 resulted in passing the Age
of Consent Act in 1894. His stand was based on the principles of justice and humanity. His
invaluable Notes on Infant Marriage and Enforced Widowhood published in 1894 was the
vade mecum of social reformers.

Q.6) The doctrine of lapse was an annexation policy purportedly devised by Lord Dalhousie,
who was the Governor General for the East India Company in India between 1848 and 1856.
Which of the following statements are correct about the policy of Doctrine of Lapse?
1. This doctrine was based on the idea that in case a ruler of dependent state died
childless, the right of ruling over the State reverted or lapsed to the sovereign.
2. The state was not to pass to an adopted heir unless the adoption was approved by the
British authorities.
3. Satara was the first state to be annexed under this policy.
4. Awadh was the last state to be annexed under Doctrine of lapse which triggered the
great revolt of 1857.
Select the code from the following:
a)
b)
c)
d)

1,2 and 3
2,3 and 4
1,3 and 4
All of the above

Q.6) Solution (a)


Awadh was not annexed under Doctrine of Lapse. It was annexed by British citing the reason
misadministration. This was one major reason which triggered the revolt of 1857.

www.iasbaba.com

Page 5

IASbaba 60 Day Plan- Prelims Test 2016 MODERN INDIA & CURRENT AFFAIRS [DAY 20]

Q.7) Which of the following Acts ended the monopoly of trade of East India company in
India?
a)
b)
c)
d)

Charter Act of 1793


Charter Act of 1813
Charter Act of 1833
Government of India Act 1858

Q.7) Solution (b)


The earlier charter act of 1793 had given the East India Company a monopoly to trade with East
for a period of 20 years. However, the rise of Napoleon Bonaparte had brought hard days to the
businessmen of England.
Charter act of 1813 ended the monopoly of the East India Company in India, however the
companys monopoly in trade with china and trade in tea with India was kept intact. Thus, trade
with India for all commodities except Tea was thrown open to all British subjects. This lasted till
1833 when the next charter abolished the trade of the company.

Q.8) Consider the following statements regarding Permanent Settlement method of land
revenue:
1. It was introduced in Bengal and Bihar by Warren Hastings.
2. The zamindars were made owners of the land and ownership was made hereditary and
transferrable.
3. Cultivators were reduced to low status of mere tenants.
4. Zamindars were to give 50% of the rental they derived to the state, keeping 50% to
themselves.
Which of the above statements are correct?
a)
b)
c)
d)

2 and 3
1,2 and 3
2,3 and 4
All of the above

www.iasbaba.com

Page 6

IASbaba 60 Day Plan- Prelims Test 2016 MODERN INDIA & CURRENT AFFAIRS [DAY 20]

Q.8) Solution (a)


Zamindars were to give 10/11 of the rental they derived from the peasantry to the state,
keeping 1/11 for them.

Q.9) The revolt of 1857 came as a bitter shock to the British empire. Since it started as an
army revolt, key changes were made in the Army to avoid such a situation again. Which of
the following statements are correct regarding the changes made in the Indian Army?
1. The proportion of Europeans to Indians in the army was increased.
2. European troops were kept in key geographical and military positions.
3. Policy of divide and rule was implemented by making battalions on the basis of caste
and religion.
4. The older policy of excluding Indians from officer corps was abandoned and loyal Indians
were given higher posts.
Select the code from the following:
a)
b)
c)
d)

2,3 and 4
1,2 and 3
1,3 and 4
All of the above

Q.9) Solution (b)


The older policy of excluding Indians from officer corps was strictly continued and not Indian
could rise over the position of subedar.

Q.10) Consider the following statements regarding the Khaksar movement:


1. It was started by a charismatic leader, Allama Mashriqi in Lahore.
2. Its aim was revival of orthodox Islam in India and establish a Muslim government.
3. Allama encouraged members of the movement to serve the people regardless of their
caste or religion; and Khaksars were expected to convince others to join the movement
through "love and affection.
4. Mahatma Gandhi Called Allama Mashriqi to join Quit India movement in 1942.

www.iasbaba.com

Page 7

IASbaba 60 Day Plan- Prelims Test 2016 MODERN INDIA & CURRENT AFFAIRS [DAY 20]

Select the correct statement from above:


a)
b)
c)
d)

1,2 and 3
1,3 and 4
2,3 and 4
1 and 2

Q.10) Solution (b)


The Khaksar movement was a social movement based in Lahore, Punjab, British India,
established by Allama Mashriqi in 1931, with the aim of freeing India from the rule of the British
Empire and establish a Hindu-Muslim government in India.
Mashriqi had said in 1931 that the Khaksar movement had three distinct objectives; "to
emphasize the idea of superiority of God, unity of the nation and service to mankind".
Allama Mashriqi was called to join the Quit India Movement. Mashriqi was apprehensive of its
outcome and did not agree with the Congress Working Committee's resolution.

Q.11) Which of the following political entities supported the Quit India Movement?
1.
2.
3.
4.
5.

Muslim League
Rashtriya Swayamsevak Sangh
Hindu Mahasabha
Communist Party of India
Princely States

Select the code from the following:


a)
b)
c)
d)

1 and 4
2,3 and 5
2 and 3
None of the above

Q.11) Solution (d)


The Muslim League opposed the Quit India Movement as it was of the view that if the British
left India in its current state, Muslims as a minority would be oppressed by the Hindu majority.
www.iasbaba.com

Page 8

IASbaba 60 Day Plan- Prelims Test 2016 MODERN INDIA & CURRENT AFFAIRS [DAY 20]

Hindu nationalist parties like the Hindu Mahasabha openly opposed the call for the Quit India
Movement and boycotted it officially. Vinayak Damodar Savarkar, the president of the Hindu
Mahasabha at that time, even went to the extent of writing a letter titled "Stick to your Posts",
in which he instructed Hindu Sabhaites who happened to be "members of municipalities, local
bodies, legislatures or those serving in the army...to stick to their posts" across the country, and
not to join the Quit India Movement at any cost.
the RSS, under M.S. Golwalkar refused to join in the Quit India Movement as well. However,
such a non-committal attitude during the Indian freedom movement also led to the Sangh
being viewed with distrust and anger, both by the general Indian public, as well as certain
members of the organization itself.
The Communist Party of India was banned at that time by the British government. In order to
get the ban lifted, as well as to assist the Soviet Union in its war against Nazi Germany, it
supported the British war effort, despite support for Quit India by many industrial workers. In
response the British lifted the ban on the party.

Q.12) Consider the following statements about Tattvabodhini Sabha:


1. It was founded by Rabindranath Tagore.
2. The objective of Tattvabodhini Sabha was to encourage religious inquiries and
disseminate the essence of Upanishads.
Which of the above statements are correct?
a)
b)
c)
d)

1 only
2 only
Both 1 and 2
Neither 1 nor 2

Q.12) Solution (b)


It was founded by Debendranath Tagore.

www.iasbaba.com

Page 9

IASbaba 60 Day Plan- Prelims Test 2016 MODERN INDIA & CURRENT AFFAIRS [DAY 20]

Q.13) Consider the following matches:

1.
2.
3.
4.

Author

Literature

Premchand
New lamps for Old
Bhagat Singh
Shyama Prasad Mukherji

Bharat Durdasha
Aurobindo Ghosh
Why I am an Atheist
Young India

Select the correct code from the following:


a) 3 only
b) 1,3 and 4
c) 2 and 3
d) All of the above
Q.13) Solution (c)
Bharat Durdasha- Bhartendu Harishchandra
Young India- Lala Lajpat Rai
Q.14) Consider the following statements regarding Baba Ramchandra:
1. He left for Fiji as an indentured labourer and changed his name to Ramchandra Rao to
conceal his identity as Brahmin.
2. He returned to India, became a Sanyasi and started working for the upliftment of
peasantry.
3. In 1920, he organized an all farmers association in Oudh, called Oudh Kisan Sabha.
Which of the above statements are correct?
a)
b)
c)
d)

1 and 2
2 and 3
1 and 3
All of the above

Q.14) Solution (d)

www.iasbaba.com

Page 10

IASbaba 60 Day Plan- Prelims Test 2016 MODERN INDIA & CURRENT AFFAIRS [DAY 20]

Baba Ramchandra was a sanyasi who had earlier been to Fiji as an indentured labourer His
battlefield was Awadh where he led the peasants against the exploitation of zamindars and
British.
He moved around the region with a copy of the Ramayana under his arm, blending readings
from this popular Hindu epic with denunciations of both the British Raj and the landlords, and
appealed to the peasants to act together against their exploiters. Although he began by seeking
to harmonise tenant-landlord relations, Ram Chandra soon considered this to be a wasted
effort and began to mobilise the peasants. He encouraged peasants to pay only the required
rent and refrain from customary donations. In 1919 he led the first peasant protest against the
landlords and by 1920 had organised all the farmers associations in Oudh, forming the Oudh
Kisan Sabha (Oudh Farmers Association). He was arrested on a number of occasions for
organising public protests.

Q.15) The second half of the 19th century saw a remarkable growth in the vernacular press.
They played a critical role in creating socio-political awareness in the masses. They also
became a vehicle in exposing the exploitative policies of the British government. Alarmed, by
its growth, the government passed a Vernacular Press Act. Which of the following statements
are correct about this act?
1. It was passed by Lord Ripon in 1878.
2. The magistrates of the districts were empowered, to call upon a printer and publisher of
any kind to enter into a Bond, undertaking not to publish anything which might rouse
feelings of disaffection against the government.
3. The magistrate was authorized to deposit a security, which could be confiscated if the
printer violated the Bond.
Select the code from the following:
a)
b)
c)
d)

1 and 2
2 and 3
1 and 3
All of the above

Q.15) Solution (b)


Vernacular Press Act was passed by Lord Lytton and it was later repealed by Lord Ripon.

www.iasbaba.com

Page 11

IASbaba 60 Day Plan- Prelims Test 2016 MODERN INDIA & CURRENT AFFAIRS [DAY 20]

Q.16) Consider the following statements with Rare earth metals


1. As per IUPAC they are 17 rare earth metals
2. Among the rare earth metals there are 15 rare earth metals belonging to lanthanoid
group and other two being scandium and yttrium
3. China has the largest reserves of rare earth metals
Select the correct option
a)
b)
c)
d)

1 &2 only
2&3 only
1&3 only
All

Q.16) Solution (d)


Rare earth metals:
As per IUPAC total 17 elements are considered as rare earth metal:the 15 from lanthanoid
group and two others:Scandium and Yttrium.
China produces 97% of world production
India has 2nd largest reserve of REM.
Mostly found in placer deposits
High end use, especially in electronics, tablet, wind and solar energy
Few examples: lanthanum, dyprosium and yttrium

Use:
lanthanum: oil refineries
dyprosium :used in hybrid vehicles, wind turbines and stealth helicopters
yttrium : Used in military jet engines

www.iasbaba.com

Page 12

IASbaba 60 Day Plan- Prelims Test 2016 MODERN INDIA & CURRENT AFFAIRS [DAY 20]

Q.17) Consider the fund SMILE as mentioned in the budget 2016-17

a) It is debt fund by SIDBI that will be used to provide soft term loans for MSME sectors
to meet debt to equity norms
b) It is fund that will be used to provide rehabilitation care for women affected by
violence
c) It is fund used to develop scientific temper among students in school
d) None of these
Q.17) Solution (a)

Budget 2016-17 announced two fund namely Indian aspiration fund and SMILE fund
India Aspiration Fund (IAF) to be set up as a fund of funds under the Small Industries
Development Bank of India (SIDBI) in order to boost the startup ecosystem in the country. The
fund will have an initial corpus of Rs.2,000 crore. Life Insurance Corp. of India (LIC) will be a
partner and co-investor in the fund.
Another Fund
The second fund is a debt fund called SIDBI Make in India Loan for Enterprises (SMILE), which
was announced in the Union budget in February. The fund will provide soft term loans and
loans in the nature of quasi-equity to MSMEs to meet debt-to-equity norms and pursue growth
opportunities in existing MSMEs.

Q.18) Consider the following statements with respect to Kanheri caves

1. It is located in close proximity to Sanjay Gandhi National park in Madhya Pradesh


2. Kanheri caves are constructed with the influence of Buddhism
Select the correct answer using the codes given below
a)
b)
c)
d)

1 only
2 only
Both
None

Q.18) Solution (b)


www.iasbaba.com

Page 13

IASbaba 60 Day Plan- Prelims Test 2016 MODERN INDIA & CURRENT AFFAIRS [DAY 20]

Kanheri National park is present in Maharashtra

Q.19) Consider the following

1.
2.
3.
4.
5.

George Yule
Sarojini Naidu
Annie besant
Chadrashekar Azad
Badruddin Tyabji

Select the above personalities who presided over the sessions of Indian National congress
a)
b)
c)
d)

1 and 4 only
1,2,3 and 5 only
2,3,4 and 5 only
All presided over INC sessions

Q.19) Solution (b)

Chandra Shekar Azad did not preside over INC sessions

Q.20) Silambham is a Martial art patronised by which of the following ancient Indian

kingdoms
a)
b)
c)
d)

Cholas
Cheras
Pandyas
Pallavas

Q.20) Solution (a)

A martial art called Silambam was patronised by the Chola rulers. Ancient and medieval Tamil
texts mention different forms of martial traditions but the ultimate expression of the loyalty of
the warrior to his commander was a form of martial suicide called Navakandam

www.iasbaba.com

Page 14

SUR IAS academy Gujarat

https://telegram.me/suriasHJA

IASbaba 60 Day Plan- Prelims Test 2016 ECONOMICS & CURRENT AFFAIRS [DAY 21]

Q.1) RBI merged two funds to create a new Financial Inclusion Fund (FIF) to support
developmental and promotional activities for expanding reach of banking services.
Consider the following w.r.t FIF
1. The new FIF will be administered by NABARD.
2. FIF is created by merging National Innovation Fund and Financial Inclusion
Technology Fund

a)
b)
c)
d)

Only 1
Only 2
Both
None

Q.1) Solution (d)

The Financial Inclusion Fund (FIF) and Financial Inclusion Technology Fund (FITF)
was constituted in the year 2007-08 for a period of five years with a corpus of Rs. 500
crore each to be contributed by Government of India (GOI), RBI and NABARD in the
ratio of 40:40:20.
GOI has merged the FIF and FITF to form a single Financial Inclusion Fund.
The Reserve Bank of India has finalised the new scope of activities and guidelines for
utilisation of the new FIF in consultation with GOI.
The new FIF will be administered by the reconstituted Advisory Board constituted by
GOI and will be maintained by NABARD.
The objectives of the FIF shall be to support developmental and promotional activities including creating of
FI infrastructure across the country, capacity building of stakeholders, creation of awareness to address
demand side issues, enhanced investment in Green Information and Communication Technology (ICT) solution,
research and transfer of technology, increased technological absorption capacity of financial service
providers/users with a view to securing greater financial inclusion. The fund shall not be utilized for normal
business/banking activities.

Select the correct statement/s

https://rbi.org.in/Scripts/NotificationUser.aspx?Id=10074&Mode=0

Q.2) Consider the following regarding River Information Services (RIS)


1. They are software based system, combination of modern tracking equipment
designed to optimize traffic and transport processes in inland navigation.
2. They would facilitate environment protection and better safety provisions for ship to
Information Services (RIS) are combination of modern tracking equipment related hardware and
ship collisions. River
software designed to optimize traffic and transport processes in inland navigation. The system enhances
Select the correct code
a)
b)
c)
d)

Only 1
Only 2
Both
None

swift electronic data transfer between mobile vessels and shore (Base stations) through advance and
real-time exchange of information. RIS aims to streamline the exchange of information between
waterway operators and users. This would facilitate:Enhancement of inland navigation safety in ports and rivers.
Better use of the inland waterways
Environmental protection
control stations will monitor the vessels plying in this river stretch via Automatic Identification System
(AIS) and will communicate with Vessels via VHF.
NW 1 per chhe , phase 2 farraka to patana and phase 3 patana to varanasi

Q.2) Solution (b)


http://pib.nic.in/newsite/PrintRelease.aspx?relid=134228

www.iasbaba.com

Page 1

IASbaba 60 Day Plan- Prelims Test 2016 ECONOMICS & CURRENT AFFAIRS [DAY 21]

Q.3) Consider the following statements about inflation.


1. Inflation means a persistent rise the price of goods and services.
2. Inflation increases the purchasing power of money.
3. Inflation hurts those with fixed and rising incomes equally.
Which of the above are correct?
a) 1 only
b) 2 only
c) 2 and 3 only
d) All of the above

Q.3) Solution (a)


Inflation is the long term rise in the prices of goods and services caused by the devaluation
of currency. Inflationary problems arise when we experience unexpected inflation which is
not adequately matched by a rise in peoples incomes.
If incomes do not increase along with the prices of goods, everyones purchasing power has
been effectively reduced, which can in turn lead to a slowing or stagnant economy.
Moreover, excessive inflation can also wreak havoc on retirement savings as it reduces the
purchasing power of the money that savers and investors have squirreled away.

Q.4) Which of the following are probable effects of inflation?


1. It reduces savings
2. Leads to depreciation of currency
3. Make imports cheap
4. Increases borrowing costs for businesses
Select the correct answer:
a) 1 only
b) 1 and 2 only
c) 1, 2 and 3 only
d) 1, 2 and 4 only

www.iasbaba.com

Page 2

IASbaba 60 Day Plan- Prelims Test 2016 ECONOMICS & CURRENT AFFAIRS [DAY 21]

Q.4) Solution (d)

The major effects of inflation are as follows: 1. Effects on Redistribution of Income and
Wealth 2. Effects on Production 3. Other Effects!
Inflation affects different people differently. This is because of the fall in the value of
money. When price rises or the value of money falls, some groups of the society gain, some
lose and some stand in-between. Broadly speaking, there are two economic groups in every
society, the fixed income group and the flexible income group.
People belonging to the first group lose and those belonging to the second group gain. The
reason is that the price movements in the case of different goods, services, assets, etc. are
not uniform. When there is inflation, most prices are rising, but the rates of increase of
individual prices differ much. Prices of some goods and services rise faster, of others slowly
and of still others remain unchanged. We discuss below the effects of inflation on
redistribution of income and wealth, production, and on the society as a whole.

Q.5) The GDP price deflator is an economic metric that accounts for inflation by converting
output measured at current prices into constant-dollar GDP. Consider the following.
1. It is the ratio of the value of goods and services an economy produces in a particular
year at current prices to that at prices prevailing during any other reference (base)
year.
2. The deflator covers the entire range of goods and services produced in the economy
as against the limited commodity baskets for the wholesale or consumer price
indices it is seen as a more comprehensive measure of inflation.
Select the correct answer:
a) 1 only
b) 2 only
c) Both 1 and 2
d) None of the above

www.iasbaba.com

Page 3

IASbaba 60 Day Plan- Prelims Test 2016 ECONOMICS & CURRENT AFFAIRS [DAY 21]

Q.5) Solution (c)

The GDP deflator is a measure of price inflation. It is calculated by dividing Nominal GDP by
Real GDP and then multiplying by 100. (Based on the formula). Nominal GDP is the market
value of goods and services produced in an economy, unadjusted for inflation. Real GDP is
nominal GDP, adjusted for inflation to reflect changes in real output. Trends in the GDP
deflator are similar to changes in the Consumer Price Index, which is a different way of
measuring inflation.
Further reading: http://indianexpress.com/article/explained/why-deflator-is-best-indicatorof-price-rise/

Q.6) Which of the following are correct about Producer Price Index?
a) The Producer Price Index (PPI) program measures the change in prices received by
the seller.
b) The PPI provides a broader coverage in terms of products and industries and it also
includes services, something which is not a part of the WPI.
c) PPI measures the price pressure due to increase in cost of finished goods.
d) All of the above.

Q.6) Solution (b)


The Producer Price index (PPI) is a family of indexes that measures the average change
in selling prices received by domestic producers of goods and services over time. PPIs
measure price change from the perspective of the seller and differ from other indexes,
such as the Consumer Price Index, that measure price change from the purchaser's
perspective. The PPI looks at three areas of production: industry-based, commoditybased and commodity-based final demand intermediate demand.
Further reading: http://indianexpress.com/article/business/economy/new-ppi-index-likelyto-replace-wpi-by-fy16/

www.iasbaba.com

Page 4

IASbaba 60 Day Plan- Prelims Test 2016 ECONOMICS & CURRENT AFFAIRS [DAY 21]

Q.7) Consider the following.


1. Demand Pull inflation involves inflation decreasing as real GDP rises and
unemployment decreases.
2. It can be a result of easy money policy and also referred to as growth inflation.
Select the correct answer:
a) 1 only
b) 2 only
c) Both 1 and 2
d) None of the above

Q.7) Solution (b)


Inflation means a sustained increase in the general price level. However, this increase in the
cost of living can be caused by different factors. The main two types of inflation are
Demand pull inflation this occurs when the economy grows quickly and starts to
overheat Aggregate demand (AD) will be increasing faster than aggregate supply (LRAS).
Cost push inflation this occurs when there is a rise in the price of raw materials, higher
taxes, e.t.c
1. Demand Pull Inflation - This occurs when AD increases at a faster rate than AS. Demand
pull inflation will typically occur when the economy is growing faster than the long run trend
rate of growth. If demand exceeds supply, firms will respond by pushing up prices.

Q.8) Which of the following is incorrect?


a) Supply shock inflation is caused by reduced supplies due to increased prices of
inputs.
b) A type of persistent inflation that occurs by deficiencies in the condition of the
economy is structural inflation.
c)

Speculation and cartelization can cause inflation and hence vitiate the market
dynamics.

d) None of the above.

www.iasbaba.com

Page 5

IASbaba 60 Day Plan- Prelims Test 2016 ECONOMICS & CURRENT AFFAIRS [DAY 21]

Q.8) Solution (d)


This occurs when there is an increase in the cost of production for firms causing aggregate
supply to shift to the left. Cost push inflation could be caused by rising energy and
commodity prices.
Speculation involves trading a financial instrument involving high risk, in expectation of
significant returns. Speculation involves trading a financial instrument involving high risk, in
expectation of significant returns. The motive is to take maximum advantage from
fluctuations in the market.
A cartel is a grouping of producers that work together to protect their interests. Cartels are
created when a few large producers decide to co-operate with respect to aspects of their
market. Once formed, cartels can fix prices for members, so that competition on price is
avoided. In this case cartels are also called price rings. They can also restrict output released
onto the market, such as with OPEC and oil production quotas, and set rules governing
other aspects of the behaviour of members. Setting rules is especially important in
oligopolistic markets, as predicted in game theory. A significant attraction of cartels to
producers is that they set rules that members follow, thus reducing risks that would exist
without the cartel.

Q.9) Consider the relation of inflation with BOP.


1. Inflation discourages exports as domestic sales are attractive
2. Inflation may erode external competitiveness of domestic products if it leads to
higher production costs.
3. Inflation leads to currency appreciation and hence increases imports.
4. High inflation thus boosts trade and hence solve BOP crisis as such.
Select the correct answer:
a) 1 only
b) 1 and 2 only
c) 1 and 3 only
d) 3 and 4 only

www.iasbaba.com

Page 6

IASbaba 60 Day Plan- Prelims Test 2016 ECONOMICS & CURRENT AFFAIRS [DAY 21]

Q.9) Solution (b)


Inflation not only creates problems within the economy, but also in the sphere of external
trade of a country, that is, countrys trade balances with the rest of the World. Countrys
trade relations with the other countries involve exports and imports of goods and services
and how much a country will export and import depends, amongst other thing, on the
domestic price level and variation in it, that is, the rate of inflation.
Countrys transactions with the other countries, which are recorded in balance of payments
(BOPs), get adversely affected if the domestic price rise is high. High rate of inflation in the
domestic market makes domestic goods unattractive to the foreigners and therefore,
reduces demand for exports. Moreover, because of high domestic prices, residents prefer to
buy foreign goods which implies increase in imports. The result of falling exports and
increasing imports, on account of high domestic inflation, is the adverse disequilibrium in
the BOPs which, if not kept within limits, can assume serious proportion and spell a BOPs
crisis.
The BOPs crisis, which India experienced in 1991, was of a similar nature. Policy mistakes in
the form of high and unsustainable fiscal deficit financed through creation of new money
led to unprecedented growth in money supply. The resulting inflation entailed high growth
in imports than exports and finally, led to a very serious BOPs crisis involving steep decline
in foreign exchange reserves and possibility of default on external payment front.

Q.10) Which are possible consequences of inflation?


1. Repo rates may go up to squeeze out money.
2. More OMOs may be conducted by RBI by selling G-Secs.
3. Governments fiscal deficit may go up in need to subsidise and make services
affordable.
Select the correct answer:
a) 2 only
b) 1 and 2 only
c) 2 and 3 only
d) 1 and 3 only

www.iasbaba.com

Page 7

IASbaba 60 Day Plan- Prelims Test 2016 ECONOMICS & CURRENT AFFAIRS [DAY 21]

Q.10) Solution (d)


The steps generally taken by the RBI to tackle inflation include a rise in repo rates (the rates
at which banks borrow from the RBI), a rise in Cash Reserve Ratio and a reduction in rate of
interest on cash deposited by banks with RBI. The signals are intended to spur banks to raise
lending rates and to reduce the amount of credit disbursed. The RBI's measures are
expected to suck out a substantial sum from the banks. In effect, while the economy is
booming and the credit needs grow, the central bank is tightening the availability of credit.
The RBI also buys dollars from banks and exporters, partly to prevent the dollars from
flooding the market and depressing the dollar indirectly raising the rupee. In other words,
the central bank's interactions have a desirable objective to keep the rupee devalued
which will make India's exports more competitive, but they increase liquidity.

Q.11) Which of the following is correct about Inflation tax?


a) Inflation tax is a term which refers to the financial loss of value suffered by holders
of cash and as well those on fixed income, due to the effects of inflation;
b) Inflation tax is the tax people pay in developed countries to help governments fight
inflation.
c) Inflation tax is imposed on the super-rich to subsidize the poor against the risks of
inflation.
d) Inflation tax is imposed on those who strike and cause harm to public property in
times of high inflation.
Q.11) Solution (a)
Inflation tax is not an actual legal tax paid to a government; instead "inflation tax" refers to
the penalty for holding cash at a time of high inflation. When the government prints more
money or reduces interest rates, it floods the market with cash, which raises inflation in the
long run. If an investor is holding securities, real estate or other assets, the effect of inflation
may be negligible. If a person is holding cash, though, this cash is worth less after inflation
has risen. The degree of decrease in the value of cash is termed the inflation tax for the way
it punishes people who hold assets in cash, which tend to be lower- and middle-class wage
earners.

www.iasbaba.com

Page 8

IASbaba 60 Day Plan- Prelims Test 2016 ECONOMICS & CURRENT AFFAIRS [DAY 21]

Q.12) Which of the following is incorrect about Wholesale Price Index?


a) Wholesale Price Index (WPI) represents the price of goods at a wholesale stage i.e.
goods that are sold in bulk and traded between organizations instead of consumers.
b) The WPI has 3 categories namely Primary article group, fuel and power category and
the manufactured products category.
c) Primary articles have the highest weightage whereas the manufactured products
have the lowest weightage.
d) Indian WPI is published by the Economic Advisor, Ministry of Commerce and
Industry.

Q.12) Solution (c)


WPI represents the price of goods at a wholesale stage i.e. goods that are sold in bulk and
traded between organizations instead of consumers.
Definition: Wholesale Price Index (WPI) represents the price of goods at a wholesale stage
i.e. goods that are sold in bulk and traded between organizations instead of consumers. WPI
is used as a measure of inflation in some economies.
Description: WPI is used as an important measure of inflation in India. Fiscal and monetary
policy changes are greatly influenced by changes in WPI. In the United States, Producer Price
Index (PPI) is used to measure inflation.
WPI is an easy and convenient method to calculate inflation. Inflation rate is the difference
between WPI calculated at the beginning and the end of a year. The percentage increase in
WPI over a year gives the rate of inflation for that year.

Q.13) Call and Term money markets are the sub-markets of Indian money market. Which
of the following statements are correct about these short term markets?
1. Call money refers to borrowing and lending of funds for one day.
2. Term money refers to borrowing and lending of funds for a period of more than 14
days.
3. Interest rates in these markets are fixed by banks.
4. Since banks work as both lenders and borrowers in these markets, they are also
known as Inter-Bank market.
Select the code from the following:
a) 1,2 and 3
www.iasbaba.com

Page 9

IASbaba 60 Day Plan- Prelims Test 2016 ECONOMICS & CURRENT AFFAIRS [DAY 21]

b) 2,3 and 4
c) 1,2 and 4
d) All of the above
Q.13) Solution (c)
Call Money, Notice Money and Term Money markets are sub-markets of the Indian Money
Market. These refer to the markets for very short term funds.

Call Money refers to the borrowing or lending of funds for 1 day.


Notice Money refers to the borrowing and lending of funds for 2-14 days.
Term money refers to borrowing and lending of funds for a period of more than 14
days.

Interest rates in these markets are market determined i.e. by the demand and supply of
short term funds. In India, 80% demand comes from the public sector banks and rest 20%
comes from foreign and private sector banks.
Since banks work as both lenders and borrowers in these markets, they are also known as
Inter-Bank market. The short term fund market in India is located only in big commercial
centres such as Mumbai, Delhi, Chennai and Kolkata. The intervention of RBI is prominent in
the short term funds money market in India. Call Money / Notice Money market is most
liquid money market and is indicator of the day to day interest rates. If the call money rates
fall, this means there is a rise in the liquidity and vice versa.

Q.14) Consider the following statements:


1.
2.
3.
4.

Import of gold is inflationary


Providing subsidies through Direct Benefit Transfer causes demand pull inflation
International increase in crude prices will cause cost push inflation in Indian market
Inflow of FOREX causes inflation

Which of the above statements are correct?


a)
b)
c)
d)

1 and 4
1,2 and 3
2,3 and 4
All of the above

Q.14) Solution (d)

www.iasbaba.com

Page 10

IASbaba 60 Day Plan- Prelims Test 2016 ECONOMICS & CURRENT AFFAIRS [DAY 21]

With import of gold and inflow of FOREX, RBI has to print equivalent amount of currency.
This increases the liquidity and hence inflation increases.
International increase in crude oil causes increase in cost of production. This causes cost
push inflation.
DBT increases the money in consumers hand, causing demand pull inflation.

Q.15) Which of the following statements correctly explains the term Stagflation?
a)
b)
c)
d)

Persistent high inflation combined with high unemployment


Reduction of the general level of prices in the economy
Decrease in the rate of inflation
Constant prices of commodities for a long time in an economy

Q.15) Solution (a)


Stagflation is persistent high inflation combined with high unemployment and stagnant
demand in a country's economy.

Q.16) Which of the following economic entities is are benefitted with inflation?
1. Debtors
2. Exporters
3. Savings bank account holders
4. Creditors
Select the code from the following:
a)
b)
c)
d)

1 and 2
1 only
2,3 and 4
None of the above

16) Solution (b)


Debtors will be benefitted. As the market value of money is reduced, and the amount paid
by a debtor is fixed, he will be paying less value in reality.

www.iasbaba.com

Page 11

IASbaba 60 Day Plan- Prelims Test 2016 ECONOMICS & CURRENT AFFAIRS [DAY 21]

Q.17) Consider the following statements regarding sugarcane and sugar pricing policy.
1. The concept of Statutory Minimum Price (SMP) of sugarcane was replaced with the
Fair and Remunerative Price (FRP)' of sugarcane.
2. Sugar is an essential commodity under the Essential Commodities Act, 1955.
3. The Central Government has been following a policy of partial control and dual
pricing for sugar.
Which of the above statements are correct?
a)
b)
c)
d)

1 and 2
2 and 3
1 and 3
All of the above

Q.17) Solution (d)


Sugar is an essential commodity under the Essential Commodities Act, 1955. The Central
Government has been following a policy of partial control and dual pricing for sugar. Under
this policy, a certain percentage of sugar produced by sugar factories (raised to 20% for
2009-10 sugar season and revised to 10% for 2010-11 season and 2011-12 sugar season) is
requisitioned by the Government as compulsory levy at a price fixed by the Central
Government in every sugar season. Levy sugar is distributed under the Public Distribution
System (PDS) at a uniform retail issue price throughout the country. The non-levy (free sale)
sugar is allowed to be sold as per the quantity released by the Central Government under
the regulated release mechanism.

Q.18) Consider the following statements with respect to Missile technology control regime
(MTCR)
1. For non-signatories of missile technology control regime the maximum permissible
payload of a missile can be up to 1 ton
2. The range of missiles for non-signatories of MTCR should not exceed more than 300
Kms
Select the correct answer
a)
b)
c)
d)

1 only
2 only
Both
None

www.iasbaba.com

Page 12

IASbaba 60 Day Plan- Prelims Test 2016 ECONOMICS & CURRENT AFFAIRS [DAY 21]

Q.18) Solution (b)


Maximum payload for non-signatory is only 500 kg

Q.19) Consider the following statements with respect to RBI


1. RBI or Reserve bank of India was established under the government of India act 1935
2. A palm tree and a Bengal tiger form the logo of RBI
Select the correct option
a)
b)
c)
d)

1 only
2 only
Both
None

Q.19) Solution (c)


Click here

Q.20) Consider the following


1.
2.
3.
4.

Tiger
One horned rhinoceros
Elephant
Peacock

Which of the following animals are found in the ten rupees Indian currency note?
a)
b)
c)
d)

1,2&4 only
2,3&4 only
1,3&4 only
1,2&3 only

Q.20) Solution (d)


Peacock is not found

www.iasbaba.com

Page 13

IASbaba 60 Day Plan- Prelims Test 2016 ECONOMICS & CURRENT AFFAIRS [DAY 21]

Q.21) The recent New Delhi Declaration is related to which of the following themes
a)
b)
c)
d)

Satellite coordination to monitor human induced greenhouse gases


To curb violence against women in tribal areas
To enhance scientific temper among school going students
To aim 100 % adult literacy by 2020

Q.21) Solution (a)

For the first time, under the impetus of the Indian Space Research Organisation (ISRO) and the French space agency,
Centre National dEtudes Spatiales (CNES), space agencies of over 60 countries have agreed to engage their satellites for
coordination in their methods and data to monitor human-induced greenhouse gas emissions.
The Conference of Parties (COP21) climate conference held in Paris in last December acted as a wake-up call in this
context

Click here

Q.22) Consider the following


a) Chimmonoy Wild life sanctuary
b) Parambikulam wild life sanctuary
c) Eravikilam national park
Which of the above wild life sanctuary/national park is not located in Kerala?
a)
b)
c)
d)

1&2 only
3 only
2&3 only
All are present in Kerala

Q.22) Solution (d)


Refer atlas

Q.23) Consider the following statements.


1. An extremely low aggregate demand.
2. Comparatively lower inflation.
3. Production houses go for forced labour cut in order to reduce production cost.
Which of the following stage of Business cycle best represents the above characteristics?
a)
b)
c)
d)

Recession
Recovery
Depression
Boom

www.iasbaba.com

Page 14

IASbaba 60 Day Plan- Prelims Test 2016 ECONOMICS & CURRENT AFFAIRS [DAY 21]

Q.23) Solution (c)


The major traits of depression could be as given below:
(a) An extremely low aggregate demand in the economy causes activities to decelerate;
(b) The inflation being comparatively lower;
(c) The employment avenues start shrinking forcing unemployment rate to grow fast;
(d) To keep the business going, production houses go for forced labour-cuts or
retrenchment (to cut down the production cost and be competitive in the market) etc.
Recession Vs Depression:
Recession is somewhat similar to the phase of depression we may call it a mild form of
depression fatal for economies as this may lead to depression if not handled with care and
in time.
When a downturn in the GDP lasts for at least six months of time, it is termed as recession.
When the GDP drops by more than 10 percent, it results in depression.

Q.24) Consider the following statements with regards to inflation indexed bonds (IIBs).
1. IIBs will provide inflation protection to principal only.
2. Existing tax provisions will be applicable on interest payment and capital gains on
IIBs.
3. Foreign institutional investors (FIIs) are not allowed to invest in IIBs.
Which of the above statements are NOT correct?
a)
b)
c)
d)

2 and 3 only
1 and 2 only
1 and 3 only
All of the above

Q.24) Solution (c)


Inflation Indexed Bonds (IIBs) were issued in the name of Capital Indexed Bonds (CIBs)
during 1997. The CIBs issued in 1997 provided inflation protection only to principal and not
to interest payment. IIBs will provide inflation protection to both principal and interest
payments.
Tax provisions will be applicable on interest payment and capital gains on IIBs. There will be
no special tax treatment for these bonds.

www.iasbaba.com

Page 15

IASbaba 60 Day Plan- Prelims Test 2016 ECONOMICS & CURRENT AFFAIRS [DAY 21]

IIBs would be Government securities (G-Sec) and the different classes of investors eligible to
invest in G-Secs would also be eligible to invest in IIBs. FIIs would be eligible to invest in the
IIBs but subject to the overall cap for their investment in G-Secs (currently USD 25 billion).
For more details:
https://rbi.org.in/scripts/FAQView.aspx?Id=91

Q.25) GPS-Aided Geo Augmented Navigation (GAGAN) system will provide augmentation
service over which of the following areas?
1.
2.
3.
4.
5.
6.

Bay of Bengal
Middle East
South East Asia
Africa
Russia
Scandinavia

Choose the correct code


a)
b)
c)
d)

1,2,3 and 4
2,4,5 and 6
3,4,5 and 6
1 and 3 only

Q.25) Solution (a)


GAGAN will provide augmentation service for the GPS over the country, the Bay of Bengal,
South East Asia and Middle East and up to Africa
Read More Click Here

Q.26) Consider the following statements with respect to Kaladan Multi-modal Transit
Transport Project
1. It is a project between India, Bangladesh and Myanmar
2. The project consists of only naval routes
3. It will reduce the need to transport goods through the Chickens neck
Which of the following is incorrect?
a) Only 1
www.iasbaba.com

Page 16

IASbaba 60 Day Plan- Prelims Test 2016 ECONOMICS & CURRENT AFFAIRS [DAY 21]

b) Only 2
c) 1 and 2
d) 2 and 3

Q.26) Solution (c)

Kaladan Multi-modal Transit Transport Project is between India and Myanmar


The project consists of road and naval routes
It will reduce the need to transport good through the narrow Siliguri corridor, also
known as Chicken's Neck

Siliguri Corridor Click Here


Kaladan Multi-modal Transit Transport Project Click Here
The Siliguri Corridor, or Chicken's Neck, is a narrow stretch of land, located in the Indian state of West Bengal, that connects India's northeastern
states to the rest of India, with the countries of Nepal and Bangladesh lying on either side of the corridor. The kingdom of Bhutan lies on the northern
side of the corridor.
The city of Siliguri, in the state of West Bengal, is the major settlement in this area and the central node that connects Bhutan, Nepal, Sikkim, Darjeeling
hills, Northeast India and the rest of India.

The Kaladan Multi-Modal Transit Transport Project is a project that will connect the eastern Indian seaport of Kolkata with Sittwe seaport in
Myanmar by sea; it will then link Sittwe seaport to Paletwa in Myanmar via Kaladan river boat route and then from Paletwa on to Mizoram by road
transport
The project has several sections/legs:[3]

539 km shipping route from seaport of Kolkata in India to Sittwe seaport in Myanmar via Bay of Bengal
158 km river boat route from Sittwe seaport to Paletwa jetty via Kaladan river in Myanmar
110 km road route from Paletwa jetty to Indo-Myanmar border in Myanmar
100 km route from Indo-Myanmar border to Lawngtlai in Mizoram in India by road on National Highway 54 (India) (NH-54), which then
continues further to Dabaka in Assam via 850 km long NH-54 which in turn is part of the larger East-West Corridor connecting North East India
with the rest of India.

SUR IAS academy Gujarat


https://telegram.me/suriasHJA

www.iasbaba.com

Page 17

SUR IAS ACADEMY GUJARAT - https://telegram.me/suriasHJA


IASbaba 60 Day Plan- Prelims Test 2016 ECONOMICS & CURRENT AFFAIRS [DAY 22]

Q. 1) Which of the following are correct w.r.t. Asset Reconstruction Companies?


1. It is set up to reconstruct or re-package assets to make them more saleable. The assets
in question here are loans from banks, card companies, financial institutions etc.
2. The word "asset reconstruction" in India was used in Narsimham I report where it
was envisaged for the setting up of a central Asset Reconstruction Fund with money
contributed by the Central Government.
3. ARCs focus on NPAs and allow the banking system to act as "clean bank".

Select the correct answer


a) 1 only
b) 2 only
c) 2 and 3 only
d) All of the above

Q.1) Solution (d)


ARC is in news for multiple reasons especially with NPAs being all time high. Hence we expect a
question. Here is the basic understanding about ARCs.
An Asset Reconstruction Company (ARC) is a company that is set up to do exactly what the
name suggests reconstruct or re-package assets to make them more saleable. The assets in
question here are loans from banks, card companies, financial institutions etc.
Why do we need ARCs? Bad loans are essentially of two types those that are a consequence
of routine banking operations and those that are a reflection of a greater systematic rot, as in
the Indian context where the bulk of non-performing assets (NPAs) are due to government
interference/loan waivers/difficulties in recovering dues etc. There are essentially two
approaches to tackling NPAs one, leave the banks to manage their own bad. Two, do the
same thing on a concerted, central level, through a centralized agency or agencies. ARCs are
centralised agencies for resolving bad loans created out of a systematic crisis. Non-performing

www.iasbaba.com

Page 1

IASbaba 60 Day Plan- Prelims Test 2016 ECONOMICS & CURRENT AFFAIRS [DAY 22]

assets (NPAs) can be assigned to ARCs by banks at a discounted price, enabling a one-time
clearing of the balance sheet of banks of sticky loans.
At the same time, the ARC can float bonds and recover dues from the borrowers directly. ARCs
can have several alternate structures. They can either be publicly or privately owned or a
combination of both, and can be either separately capitalised units or wholly-owned
subsidiaries.

Q.2) Which of the following are correct about Monetary Policy?


1. The strategy to influence money supply and interest rates to affect output and inflation.
2. Balancing savings and investments is a major objective along with generating
employment.
3. Exchange rate stabilization is achieved through monetary policy.
4. Monetary policy can be expansionary or contractionary depending on the status of the
macroeconomic parameters.
Select the correct answer
a) 1 and 3 only
b) 1 and 4 only
c) 3 and 4 only
d) All of the above

Q.2) Solution (d)


Monetary policy consists of the actions of a central bank, currency board or other regulatory
committee that determine the size and rate of growth of the money supply, which in turn
affects interest rates. Monetary policy is maintained through actions such as modifying the
interest rate, buying or selling government bonds, and changing the amount of money banks
are required to keep in the vault (bank reserves).

Further

reading:

http://www.yourarticlelibrary.com/policies/monetary-policy-meaning-

objectives-and-instruments-of-monetary-policy/11134/
www.iasbaba.com

Page 2

IASbaba 60 Day Plan- Prelims Test 2016 ECONOMICS & CURRENT AFFAIRS [DAY 22]

Q.3) Consider the following statements about Cash Reserve Ratio:


1. It is the ratio of deposits which banks have to keep with RBI as savings account.
2. Banks get a fixed interest on CRR from RBI.
3. With increase in CRR the lending capacity of banks will increase.
Which of the above statements are correct?
a)
b)
c)
d)

1 and 2
2 and 3
1 and 3
None of the above

Q.3) Solution (d)


It is the ratio of Deposits which banks have to keep with RBI. Under CRR a certain percentage of
the total bank deposits has to be kept in the current account with RBI. Banks dont earn
anything on that.
Banks will not have access to this amount. They cannot use this money for any of their
economic or commercial activities. Banks cant lend this portion of money to corporate or
individual borrowers. With increase in CRR the lending capacity of banks will decrease as they
will have less money to lend.

Q.4) Which of the following is incorrect about Bank rate?


a) It is rate at which RBI lends long term to commercial banks.
b) It requires collateral security like in case of repo-reverse repo rate.
c) It is a penal rate and is aligned with MSF in 2011.
d) Lower bank rates can help to expand the economy, when unemployment is high, by
lowering the cost of funds for borrowers.

Q.4) Solution (b)


Bank rate being long term lending rate doesnt require collaterals like the G-secs in case of Repo
or MSF.
www.iasbaba.com

Page 3

IASbaba 60 Day Plan- Prelims Test 2016 ECONOMICS & CURRENT AFFAIRS [DAY 22]

Bank rate is the rate charged by the central bank for lending funds to commercial banks. Bank
rates influence lending rates of commercial banks. Higher bank rate will translate to higher
lending rates by the banks. In order to curb liquidity, the central bank can resort to raising the
bank rate and vice versa
Further reading: https://www.rbi.org.in/scripts/FAQView.aspx?Id=51

Q.5) Consider the following about ready forward contracts.


1. Its a transaction where 2 parties agree to sell and repurchase the same security.
2. Repo rate is the rate at which the central bank of a country (Reserve Bank of India in
case of India) lends money to commercial banks in the event of any shortfall of funds.
3. The difference between Repo and Reverse repo rate is always 100 basis points with the
latter being lower.
4. The difference between Repo and MSF is always 100 basis points with the latter being
higher.
Select the correct answer:
a) 1 and 3 only
b) 1 and 2 only
c) 2 and 3 only
d) 1, 3 and 4 only

Q.5) Solution (b)


Repurchase Options or in short Repo, is a money market instrument, which enables
collateralised short term borrowing and lending through sale/purchase operations in debt
instruments.
"repo" means an instrument for borrowing funds by selling securities with an agreement to
repurchase the securities on a mutually agreed future date at an agreed price which includes
interest for the funds borrowed; "reverse repo" means an instrument for lending

www.iasbaba.com

Page 4

IASbaba 60 Day Plan- Prelims Test 2016 ECONOMICS & CURRENT AFFAIRS [DAY 22]

funds by purchasing securities with an agreement to resell the securities on a mutually agreed
future date at an agreed price which includes interest for the funds lent."
This is the general definition of Repo and Reverse Repo in India. The securities transacted here
can be either government securities or corporate securities or any other securities which the
Central bank permits for transaction. Non-sovereign securities are used in many global markets
for repo operations. Unlike them, Indian repo market predominantly uses sovereign securities,
though repo is allowed on corporate bonds and debentures
The current repo, reverse repo and MSF are 6.5%, 6% and 7% respectively. Hence it is no
bound rule that there has to be a 100 basis points difference between them.

Q.6) Marginal standing facility (MSF) is a window for banks to borrow from the Reserve Bank
of India in an emergency situation when inter-bank liquidity dries up completely. Which of
the following is incorrect about MSF?
a) MSF is a penal rate as the repo limit is exhausted and also SLR limit is breached at times.
b) MSF operations also become necessary as the repo operations are limited to a specific
period of the day.
c) Only scheduled commercial banks can use this route with government securities
including SLR as collateral.
d) If banks do not have excess SLR it cannot borrow under MSF.

Q.6) Solution (d)


Marginal Standing Facility (MSF) is a new scheme announced by the Reserve Bank of India (RBI)
in its Monetary Policy (2011-12) and refers to the penal rate at which banks can borrow money
from the central bank over and above what is available to them through the LAF window.
MSF, being a penal rate, is always fixed above the repo rate. The MSF would be the last resort
for banks once they exhaust all borrowing options including the liquidity adjustment facility by
pledging government securities, where the rates are lower in comparison with the MSF. The
MSF would be a penal rate for banks and the banks can borrow funds by pledging government
securities within the limits of the statutory liquidity ratio. The scheme has been introduced by
www.iasbaba.com

Page 5

IASbaba 60 Day Plan- Prelims Test 2016 ECONOMICS & CURRENT AFFAIRS [DAY 22]

RBI with the main aim of reducing volatility in the overnight lending rates in the inter-bank
market and to enable smooth monetary transmission in the financial system.
To balance the liquidity, RBI uses the sole independent "policy rate" which is the repo rate (in
the LAF window) and the MSF rate automatically gets adjusted to a fixed per cent above the
repo rate (MSF was originally intended to be 1% above the repo rate). MSF is at present aligned
with the Bank rate.
Even if the Banks do not have excess SLR they still can opt for MSF and it is allowed within
certain limits. The other statements are true.

Q.7) Consider the following about SLR. Which of the following are correct?
1. The ratio of liquid assets to net demand and time liabilities (NDTL) that they should keep
in designated liquid assets is called statutory liquidity ratio (SLR).
2. To control expansion of bank credit and ensure solvency of commercial banks is major
objective.
3. Banks hold public sector bonds, current account balances with other banks and gold as
SLR.
4. Banks can also hold multilateral bonds and foreign exchange as part of SLR

Select the correct answer:


a) 1 only
b) 1 and 2 only
c) 1, 2 and 3 only
d) All of the above.

Q.7) Solution (c)


The Statutory Liquidity Ratio (SLR) is a prudential measure under which (as per the Banking
Regulations Act 1949) all Scheduled Commercial Banks in India must maintain an amount in one

www.iasbaba.com

Page 6

IASbaba 60 Day Plan- Prelims Test 2016 ECONOMICS & CURRENT AFFAIRS [DAY 22]

of the following forms as a percentage of their total Demand and Time Liabilities (DTL) / Net
DTL (NDTL);
[i] Cash.
[ii] Gold; or
[iii] Investments in un-encumbered Instruments that include;
(a) Treasury-Bills of the Government of India.
(b) Dated securities including those issued by the Government of India from time to time
under the market borrowings programme and the Market Stabilization Scheme (MSS).
(c) State Development Loans (SDLs) issued by State Governments under their market
borrowings programme.
(d) Other instruments as notified by the RBI.
Traditionally the amount to be held thus was stipulated to be no lower than 25 percent and not
exceeding 40 percent of the banks total DTL. However, effective from January, 2007 the floor
of 25 percent on the SLR was removed following an amendment of the Banking Regulation Act,
1949.
Further reading: http://www.arthapedia.in/index.php?title=Statutory_Liquidity_Ratio

Q.8) Consider the following about SLR and CRR.


1. SLR restricts the banks leverage in pumping more money into the economy while CRR is
the portion of deposits that banks have to maintain within central bank.
2. CRR is allowed only in Cash and Government securities while SLR is allowed in multiple
other designated forms.
Select the correct answer:
a) 1 only
b) 2 only
c) Both 1 and 2
d) None of the above

Q.8) Solution (a)

www.iasbaba.com

Page 7

IASbaba 60 Day Plan- Prelims Test 2016 ECONOMICS & CURRENT AFFAIRS [DAY 22]

The question tries to catch the aspirant on reading and attention to detail. CRR is allowed only
in Cash is a known fact.
Cash Reserve Ratio (CRR): Each bank has to keep a certain percentage of its total deposits with
RBI as cash reserves.
Statutory Liquidity Ratio (SLR): Amount of liquid assets such as precious metals(Gold) or other
approved securities, that a financial institution must maintain as reserves other than the cash.
Formula: SLR rate = (liquid assets / (demand + time liabilities)) 100%
Time liabilities are liabilities which the banks are liable to pay after a certain period of time. E.g.
A 1 year fixed deposit. Demand liabilities are liabilities which the banks are liable to pay on
being demanded by the customer. E.g. A savings account
CRR limits the ability of the banks to pump more money into the economy. SLR is used to limit
the expansion of bank credit, for ensuring the solvency of banks (even if all the loans by the
bank go bad, the bank can still retrieve a part of it by selling the gold/govt securities.
Further reading: http://www.livemint.com/Money/VLvbh3hAk9sSOCvYdqd7SI/DYK-Differencebetween-CRR-and-SLR.html

Q.9) Which of the following is incorrect about qualitative and quantitative methods used by
RBI to control credit supply?
a) Qualitative method controls the manner of channelizing cash and credit in the economy.
b) Qualitative method restricts credit for certain section and expands for others depending
on the erstwhile situation
c) Marginal requirement is increased for those business activities where the flow of credit
is to be restricted and hence a quantitative method.
d) Under rationing of credit RBI fixes ceiling of credit setting a limit to the loans and
advances that can be made to particular sector is a qualitative method.

Q.9) Solution (c)


Some of the methods employed by the RBI to control credit creation are: I. Quantitative
Method II. Qualitative Method.
www.iasbaba.com

Page 8

IASbaba 60 Day Plan- Prelims Test 2016 ECONOMICS & CURRENT AFFAIRS [DAY 22]

The various methods employed by the RBI to control credit creation power of the commercial
banks can be classified in two groups, viz., quantitative controls and qualitative controls.
Quantitative controls are designed to regulate the volume of credit created by the banking
system qualitative measures or selective methods are designed to regulate the flow of credit in
specific uses.
Quantitative or traditional methods of credit control include banks rate policy, open market
operations and variable reserve ratio. Qualitative or selective methods of credit control include
regulation of margin requirement, credit rationing, regulation of consumer credit and direct
action.
Marginal requirement by the name sounds quantitative but is a qualitative method. So be
careful.
Further reading: http://www.yourarticlelibrary.com/banking/important-methods-adapted-byrbi-to-control-credit-creation/23490/

Q.10) Consider the following about Market Stabilization scheme.


1. MSS (Market Stabilisation Scheme) securities are issued with the objective of providing
the RBI with a stock of securities with which it can intervene in the market for managing
liquidity.
2. These securities are issued not to meet the government's expenditure.
3. The amounts raised under the MSS will be held in a separate identifiable cash account
titled the Market Stabilization Scheme Account (MSS Account) to be maintained and
operated by the RBI.
Select the correct answer
a) 1 only
b) 2 only
c) 1 and 3 only
d) All of the above

Q.10) Solution (d)


www.iasbaba.com

Page 9

IASbaba 60 Day Plan- Prelims Test 2016 ECONOMICS & CURRENT AFFAIRS [DAY 22]

This scheme came into existence following a MoU between the Reserve Bank of India (RBI) and
the Government of India (GoI) with the primary aim of aiding the sterilization operations of the
RBI. Historically, the RBI had been sterilizing the effects of significant capital inflows on
domestic liquidity by offloading parts of the stock of Government Securities held by it. It is
pertinent to recall, in this context, that the assets side of the RBIs Balance Sheet (July 1 to June
30) includes Foreign Exchange Reserves and Government Securities while liabilities are
primarily in the form of High Powered Money (consisting of Currency with the public and
Reserves held in the RBI by the Banking System). Thus, any rise in Foreign Exchange Reserves
resulting from the intervention of the RBI in the Foreign Exchange Markets (with the intention,
say, to maintain the exchange rate on the face of huge capital inflows) entails a corresponding
rise in High Powered Money. The Money Supply in the economy is linked to High Powered
Money via the money multiplier. Therefore, on the face of large capital inflows, to keep the
liabilities side constant so as to not raise the Supply of Money, corresponding reduction in the
stock of Government Securities by the RBI is necessary.

Q.11) Consider the following about fixed and floating interest rates.
1. A floating interest rate is an interest rate that is allowed to move up and down with the
rest of the market or along with an index but linked to an underlying benchmark rate.
2. A fixed interest rate is an interest rate on a liability, such as a loan or mortgage, that
remains fixed either for the entire term of the loan or for part of this term.
Select the correct answer:
a) 1 only
b) 2 only
c) Both 1 and 2
d) None of the above

Q.11) Solution (c)

www.iasbaba.com

Page 10

IASbaba 60 Day Plan- Prelims Test 2016 ECONOMICS & CURRENT AFFAIRS [DAY 22]

A floating interest rate, also known as a variable or adjustable rate, refers to any type
of debt instrument, such as a loan, bond, mortgage, or credit, that does not have a fixed
rate of interest over the life of the instrument.
Floating interest rates typically change based on a reference rate (a benchmark of any financial
factor, such as the Consumer Price Index). One of the most common reference rates to use as
the basis for applying floating interest rates is the London Inter-bank Offered Rate,
or LIBOR (the rates at which large banks lend to each other)
A fixed interest rate is an interest rate on a liability, such as a loan or mortgage, that remains
fixed either for the entire term of the loan or for part of this term. A fixed interest rate may be
attractive to a borrower who feels that the interest rate might rise over the term of the loan,
which would increase his or her interest expense. A fixed interest rate, therefore, avoids
the interest rate risk that comes with a floating or variable interest rate, wherein the interest
rate payable on a debt obligation depends on a benchmark interest rate or index.
The UPSC trend of asking questions is coming down to fundamentals of the current and
trending topics. This is relevant w.r.t Chinas devaluation and after effects.

Q.12) The Reserve Bank of India (RBI) constituted and Expert Committee to Revise and
Strengthen the Monetary Policy Framework under the Chairmanship of Dr. Urjit R.Patel.
Consider the following.
1. It has suggested that the apex bank should adopt the new CPI (consumer price index) as
the measure of the nominal anchor for policy communication.
2. The committee asked the Central Government to ensure that the fiscal deficit as a ratio
to GDP (gross domestic product) is brought down to 3.0 per cent by 2016-17.
3. The Patel panel felt that the monetary policy decision-making should be vested with a
monetary policy committee (MPC) which has participation of both government and RBI.
4. The FSLRC under Justice (retd) B N Srikrishna had also suggested for the formation of an
MPC.
Select the correct answer:
www.iasbaba.com

Page 11

IASbaba 60 Day Plan- Prelims Test 2016 ECONOMICS & CURRENT AFFAIRS [DAY 22]

a) 1 and 2 only
b) 1 and 3 only
c) 1, 2 and 3 only
d) 1, 2 and 4 only

Q.12) Solution (d)


Urjit patel Committee is avery important committee of RBI. It suggested only RBI members in
the MPC with no government nominees. This was point of conflict even.
Further reading:

http://www.thehindu.com/business/Economy/urjit-panel-suggests-4-cpi-

inflation-target/article5602626.ece
https://rbi.org.in/Scripts/PublicationReportDetails.aspx?ID=743

Q.13) What is common for all the following?


1. S S Tarapore Committee of 2006
2. Percy Mistry Committee on 2007
3. Jahangir Aziz working group of 2008
4. Dr. Raghuram Rajan Committee of 2009
5. Justice B N Srikrishnas FSLRC of 2013

Select the correct answer.

a) Capital and Current account convertibility.


b) Poverty alleviation
c) Public Debt management Agency
d) Categorisation of states for discretionary grants.

Q.13) Solution (c)


There were multiple committees that suggested PDMA. Especially with the agency formation
becoming point of tussle between Ministry of Finance and RBI, we expect a question this time.
www.iasbaba.com

Page 12

IASbaba 60 Day Plan- Prelims Test 2016 ECONOMICS & CURRENT AFFAIRS [DAY 22]

These committees have been reffered in government documents repeatedly especially post
2015 budget.
Further reading:
http://www.arthapedia.in/index.php?title=Public_Debt_Management_Agency_(PDMA)

Q.14) Which of the following is incorrect about Public Debt Management Agency the
government intends to set up?
a) Public Debt Management Agency (PDMA) is a specialized independent agency that
manages the internal and external liabilities of the Central Government in a holistic
manner and advises on such matters in return for a fee.
b) PDMA is considered to be set up with the objective of "maximising the cost of raising
and servicing public debt over the long-term within an acceptable level of risk at all
times, under the general superintendence of the central government".
c) An autonomous PDMA can be the catalyst for wider institutional reform, including
building a government securities market, and bring in transparency about public debt.
d) Genesis of the thinking on an independent debt management office is traced back to
the Committee on Capital Account Convertibility (1997) and the Review Group of
Standing Committee on International Financial Standards & Codes (2004).

Q. 14) Solution (b)


Public Debt Management Agency (PDMA) is a specialized independent agency that manages the
internal and external liabilities of the Central Government in a holistic manner and advises on
such matters in return for a fee. In other words, PDMA is the Investment Banker or Merchant
Banker to the Government. PDMA manages the issue, reissue and trading of Government
securities, manages and advises the Central Government on its contingent liabilities and
undertakes cash management for the central government including issuing and redeeming of
short term securities and advising on its cash management.

www.iasbaba.com

Page 13

IASbaba 60 Day Plan- Prelims Test 2016 ECONOMICS & CURRENT AFFAIRS [DAY 22]

PDMA is considered to be set up with the objective of "minimising the cost of raising and
servicing public debt over the long-term within an acceptable level of risk at all times, under the
general superintendence of the central government". This will guide all of its key functions,
which include managing the public debt, cash and contingent liabilities of Central Government,
and related activities.
Further reading:
http://www.arthapedia.in/index.php?title=Public_Debt_Management_Agency_(PDMA)

Q.15) Which of the following is not correctly matched?


a) Lakadwala Poverty estimation
b) Justice R V Eshwar Taxation
c) Kirit Parikh Disinvestment
d) Urjit patel Monetary Policy

Q.15) Solution (c)


The Kirit Parikh panel was set up by the Petroleum and Natural Gas Ministry to suggest a
methodology for pricing of diesel and cooking fuel.
The panel has said that the Government should take steps to pass on the impact of rise in price
of diesel to consumers, and move rapidly towards making the price of diesel marketdetermined. The expert group recommends that in view of high under-recovery of Rs. 10.51
per litre on diesel, HSD prices be raised by Rs. 5 per litre with immediate effect. The balance
under-recovery should be made up through a subsidy of Rs. 6 per litre to public sector oil
marketing companies (OMCs). The subsidy on diesel should be capped at Rs. 6 per litre litre.
This will imply freeing of price of diesel beyond this cap, the panel has said.

Q.16) Consider the following about Phillips Curve.


1. The Phillips curve represents the relationship between the rate of inflation and
the unemployment rate.
www.iasbaba.com

Page 14

IASbaba 60 Day Plan- Prelims Test 2016 ECONOMICS & CURRENT AFFAIRS [DAY 22]

2. Phillips found a consistent inverse relationship: when unemployment was high, wages
increased slowly; when unemployment was low, wages rose rapidly.
3. It showed the rate of wage inflation that would result if a particular level of
unemployment persisted for some time.
4. Price stability has a trade-off against unemployment and some level of inflation could be
considered desirable in order to minimize unemployment, is the core argument.
Select the correct answer:
a) 1 and 2 only
b) 1 and 4 only
c) 2, 3 and 4 only
d) 1, 2, 3 and 4 only

Q.16) Solution (d)


The Phillips curve is an economic concept developed by A. W. Phillips stating
that inflation and unemployment have a stable and inverse relationship. According to the
Phillips curve, the lower an economy's rate of unemployment, the more rapidly wages paid to
labor increase in that economy.
Further reading: http://www.econlib.org/library/Enc/PhillipsCurve.html

Q.17) Consider the following.


1. Deflation is used to describe instances when the inflation rate has reduced marginally
over the short term
2. Disinflation is a general decline in prices, often caused by a reduction in the supply of
money or credit.
Select the correct answer:
a) 1 only
b) 2 only
c) Both 1 and 2
d) None of the above
www.iasbaba.com

Page 15

IASbaba 60 Day Plan- Prelims Test 2016 ECONOMICS & CURRENT AFFAIRS [DAY 22]

Q.17) Solution (d)


Deflation and disinflation are used with reference to change in general price levels in an
economy. While these two terms sound similar, they have very different meanings. As we
know, inflation refers to the rise in price levels in an economy, and deflation is the opposite, a
fall in price levels. Disinflation, on the other hand, refers to a slower rate of inflation.
Further

reading:

http://www.livemint.com/Money/rmR2KGnzn18ltvP6zBgpsO/Dejargoned-

deflation-and-disinflation.html

Q.18) If deflation is general decline in prices caused by a reduction in the supply of money
then which of the following is not the possible consequence of the same?
a) Demand from businesses and consumers to buy products falls
b) Employment growth rate stabilizes and is upward in trend
c) Debt servicing becomes more expensive
d) Governments can resort to tax cuts to boost demand from consumers and businesses

Q.18) Solution (b)


Deflation is when prices actually fall. Such a situation does seem favorable but in reality its not
a position that any economy would like to be in. Deflation usually moves hand in hand with
economic slowdown, lower productivity and loss of jobs. Just like how inflation decreases the
value of money, deflation increases its value. This incentivizes people to save money to buy
later when goods are cheaper, which, in turn, leads to further slowing of economic growth.
During deflation, value of money increases and goods are cheaper but you may end up earning
less thanks to slowing growth.
During the Great Depression, there was severe deflation; it had moved into double digits. Japan
has been struggling with deflation for the past two decades. Only recently did its Prime

www.iasbaba.com

Page 16

IASbaba 60 Day Plan- Prelims Test 2016 ECONOMICS & CURRENT AFFAIRS [DAY 22]

Minister, Shinzo Abe, declare that the period of deflation has come to an end and that
economic recovery is on its way.
A positive impact of deflation is increased export competitiveness as most other economies are
in an inflationary trend. Japan has benefited from competitive exports in the past decade or so.

Q.19) Consider the following about inflation.


1. When the government doesnt attempt to curb price rise allowing the free market
mechanism to function it is said to be open inflation.
2. When international bodies interrupts a price rise in a country through price control
measures and subsidies it is said to be internationally controlled inflation.
Select the correct answer:
a) 1 only
b) 2 only
c) Both 1 and 2
d) None of the above

Q.19) Solution (a)


Inflation is often open and suppressed. Inflation is open when markets for goods or factors of
production are allowed to function freely, setting prices of goods and factors without normal
interference by the authorities. Thus open inflation is the result of the uninterrupted
operation of the market mechanism.
There are no checks or controls on the distribution of commodities by the government.
Increase in demand and shortage of supplies persists which tend to lead to open inflation.
Unchecked open inflation ultimately leads to hyper-inflation.
Suppressed Inflation: On the contrary when the government imposes physical and monetary
controls to check open inflation, it is known as repressed or suppressed inflation. The market
mechanism is not allowed to function normally by the use of licensing, price controls and
rationing in order to suppress extensive rise in prices.
www.iasbaba.com

Page 17

IASbaba 60 Day Plan- Prelims Test 2016 ECONOMICS & CURRENT AFFAIRS [DAY 22]

Q.20) Suppressed inflation adversely affects the economy. What are the probable
consequences?
1. When the distribution of commodities is controlled, the prices of uncontrolled
commodities rise very high.
2. Suppressed inflation reduces the incentive to work because people do not get the
commodities which they want to have.
3. Suppressed inflation leads to black marketing, corruption, hoarding and profiteering. It
invites extralegal powers of control.
Select the correct answer:
a) 1 and 2 only
b) 1 and 3 only
c) 2 and 3 only
d) 1, 2 and 3 only

Q.20) Solution (d)


Suppressed inflation results when efforts are made to increase domestic production and reduce
import demand by tariffs, import restrictions, limits on foreign loans, voluntary import
agreements, etc. So long as such controls exist, the present demand is postponed and there is
diversion of demand from controlled to uncontrolled commodities. But as soon as these
controls are removed, there is open inflation. Its Effects:
(1) When the distribution of commodities is controlled, the prices of uncontrolled commodities
rise very high.
(2) Suppressed inflation reduces the incentive to work because people do not get the
commodities which they want to have.
(3) Controlled distribution of goods also leads to misallocation of resources. This results in the
diversion of productive resources from essential to non-essential industries.

www.iasbaba.com

Page 18

IASbaba 60 Day Plan- Prelims Test 2016 ECONOMICS & CURRENT AFFAIRS [DAY 22]

(4) Frictions increase in the labour market when high inflation is associated with higher
unemployment.
(5) Suppressed inflation leads to black marketing, corruption, hoarding and profiteering. It
invites extralegal powers of control.

Q.21) The Nairobi Package was adopted at the WTO's Tenth Ministerial Conference, held in
Nairobi, Kenya. It contains a series of Ministerial Decisions on
1.
2.
3.
4.
5.

Special Safeguard Mechanism for Developed Countries


Issues of Least Developed Countries
Export Competition
Export of Cotton, Pulses and Rice and its by products
Public Stockholdings

Select the correct code


a)
b)
c)
d)

1, 2, 3 and 5
2, 3, 4 and 5
1, 2, 3 and 4
2, 3 and 5

Q.21) Solution (d)


The Nairobi Package was adopted at the WTO's Tenth Ministerial Conference, held in Nairobi,
Kenya, from 15 to 19 December 2015. It contains a series of six Ministerial Decisions on
agriculture, cotton and issues related to least-developed countries (LDCs). A Ministerial
Declaration outlining the Package and the future work of the WTO was adopted at the end of
the five-day Conference.
https://www.wto.org/english/thewto_e/minist_e/mc10_e/nairobipackage_e.htm
http://www.firstpost.com/business/nothing-at-nairobi-wto-ministerial-leaves-india-anddeveloping-countries-in-the-lurch-2553428.html
http://www.thehindu.com/business/Industry/wto-nairobi-meeting-government-to-respond-towtos-nairobi-package-in-parliament/article8011127.ece
www.iasbaba.com

Page 19

IASbaba 60 Day Plan- Prelims Test 2016 ECONOMICS & CURRENT AFFAIRS [DAY 22]

Q.22) Consider the following statements regarding Channapatana handcraft


1.
2.
3.
4.

Channapatana toys are special type of wooden toys


They are protected by GI tag
Channapatana handcraft can be traced back to the reign of Tipu Sultan
It flourished with the assistance of Persian Art form

Select the correct code


a)
b)
c)
d)

1, 2 , 3 and 4
Only 1
1, 3 and 4
1, 2 and 3

Q.22) Solution (a)


http://www.thehindu.com/news/cities/bangalore/bid-to-restore-channapatna-craft-to-itsoriginal-glory/article7941995.ece

Q.23) In a major boost to the sustainable development of the Eastern Ghats, with special
focus on its fragile environment, the United Nations University has sanctioned one more
Regional Centre of Expertise (RCE) to India. What are the RCEs in India?
1.
2.
3.
4.
5.
6.

RCE-Srinagar, working on western Himalayas


RCE-Guwahati on Eastern Himalayas
RCE-Gujarat on wetland ecosystems
RCE-TERI (Mumbai) on Youth empowerment and energy
RCE-Kodagu on traditional knowledge and tribal communities of Western Ghats.
RCE-Tirupati will work on Eastern Ghats, coastal communities, marine ecosystem,
biodiversity and sustainable development.

Select the correct code


a) 1, 2, 3, 4 and 6
www.iasbaba.com

Page 20

IASbaba 60 Day Plan- Prelims Test 2016 ECONOMICS & CURRENT AFFAIRS [DAY 22]

b) 1, 2, 5 and 6
c) 2, 4 and 6
d) 1, 3, 4 and 5

Q.23) Solution (b)


RCE- Chandigarh on Wetland ecosystems
RCE- TERI (Goa)on Youth empowerment and energy
http://www.thehindu.com/news/national/andhra-pradesh/rcetirupati-to-focus-on-easternghats/article7983642.ece

Q.24) Consider the following with respect to Ganges River Dolphin


1. Ganges river dolphin is the national aquatic animal of India
2. Ganges river dolphin is Vulnerable as per IUCN red list
3. Vikramshila Gangetic dolphin sanctuary is the only protected area for the endangered
Gangetic dolphins in Asia

Select the correct answer using the codes given below


a)
b)
c)
d)

1 and 2 only
2 and 3 only
1 and 3 only
All

Q.24) Solution (c)

It is endangered as per IUCN red list


Vikramshila Gangetic dolphin sanctuary is the only protected area for the endangered
Gangetic dolphins in Asia
It is located in Bhagalpur District of Bihar, India. Only a few hundred dolphins remain in
India, of which half are found here. WWF-India and Aaranyak a NGO has been working
closely with various government departments to protect these blind river Dolphins of
India.

www.iasbaba.com

Page 21

IASbaba 60 Day Plan- Prelims Test 2016 ECONOMICS & CURRENT AFFAIRS [DAY 22]

Q.25) Consider the following lakes


1.
2.
3.
4.
5.

Lake Erie
Huron
Michigen
Ontario
Superior

Arrange the above lakes from West to East


a)
b)
c)
d)

53124
53134
53214
53241

Q.25) Solution (c)

Q.26) Consider the following countries


1. Liechtenstein
2. Norway
www.iasbaba.com

Page 22

IASbaba 60 Day Plan- Prelims Test 2016 ECONOMICS & CURRENT AFFAIRS [DAY 22]

3. Switzerland

Which of the above is a Non EU country?


a)
b)
c)
d)

1 and 2 only
2 and 3 only
3 only
All

Q.26) Solution (d)


European countries that are not part of the European Union include Norway,
Iceland, Liechtenstein, Albania, Switzerland, Turkey, Russia, Macedonia andMontenegro.
Of
these, two countries, Russia and Turkey, straddle Europe and Asia.

Q.27) Consider the following statement with respect to SHILP SAMPADA


1. The objective of this scheme is to upgrade the technical and entrepreneurial skill of
Backward Classes by way of providing training and financial assistance
2. The maximum loan limit under this scheme is Rs.1 Lakh.
3. The scheme is spearheaded by Ministry of Social justice and empowerment

Select the correct answer


a)
b)
c)
d)

1 and 2 only
2 and 3 only
1 and 3 only
All

Q.27) Solution (c)


Self explanatory

www.iasbaba.com

Page 23

IASbaba 60 Day Plan- Prelims Test 2016 ECONOMICS & CURRENT AFFAIRS [DAY 22]

Q.28) Which of the following properties of light is observed in the phenomena of Rainbow
Formation?
1.
2.
3.
4.

Dispersion
Refraction
Total internal reflection
Reflection

Select the correct answer using the options given below


a)
b)
c)
d)

1,2,3 only
2,3,5 only
3 only
All of these

Q.28) Solution (d)

A rainbow is a natural spectrum appearing in the sky after a rain shower


It is caused by dispersion of sunlight by tiny water droplets, present in the atmosphere.
A rainbow is always formed in a direction opposite to that of the Sun.
The water droplets act like small prisms. They refract and disperse the incident sunlight,
then reflect it internally, and finally refract it again when it comes out of the raindrop
Due to the dispersion of light and internal reflection; different colours reach the
observers eye.
Source Ncert 10th std in 10 chap

Q.29) According to RBI guidelines, Basel III norms are to be implemented by all banks till
2019. Consider the following statements:
1. Capital to Risk weighted Assets Ratio (CRAR) is the percentage of banks risk weighted
credit exposures.
2. RBI has fixed CRAR as 9%.
3. Capital Adequacy Ratio (CAR) is expressed as CRAR.
4. Higher the CAR, higher is the risk.
Which of the above statements are correct?
www.iasbaba.com

Page 24

IASbaba 60 Day Plan- Prelims Test 2016 ECONOMICS & CURRENT AFFAIRS [DAY 22]

a)
b)
c)
d)

1,2 and 3
2,3 and 4
1,3 and 4
All of the above

Q.29) Solution (a)


The capital adequacy ratio (CAR) is a measure of a bank's capital. It is expressed as a percentage
of a bank's risk weighted credit exposures. Also known as capital-to-risk weighted assets ratio
(CRAR), it is used to protect depositors and promote the stability and efficiency of financial
systems around the world.
In simple terms Capital Adequacy Ratio is Capital/risk. Lower the CAR, higher is the risk.

Q.30) Consider the following statements:


1. When Repo Rate increases, borrowing from RBI becomes more expensive.
2. Increase in Reverse Repo decreases the liquidity in the market.
3. Reverse Repo is fixed .5 percent points more than Repo.
Which of the above statements are correct?
a)
b)
c)
d)

1 and 2
2 and 3
1 and 3
All of the above

Q.30) Solution (a)


Reverse Repo is fixed .5 percentage points less than Repo.

www.iasbaba.com

Page 25

IASbaba 60 Day Plan- Prelims Test 2016 SCIENCE & TECH & CURRENT AFFAIRS [DAY 23]

Q.1) A fuel cell is a device that generates electricity by a chemical reaction. Consider the
following
1.
2.
3.
4.

Molten carbonate fuel cells


Phosphoric Acid fuel cells
Hydrogen fuel cells
Solid oxide fuel cells

Which of the above is/are types of fuel cells?


a)
b)
c)
d)

Only 3
2 and 3
2, 3 and 4
1, 2, 3 and 4

Q.1) Solution (d)


Similar to batteries and all other electrochemical cells, fuel cells have two electrodes, and an
electrolyte sandwiched in between. Fuel cells are distinguished by the type of electrolyte
they use. The more common fuel cells on the market are:

PEM Polymer Exchange Membrane also known as Polymer Electrolyte


Membrane Fuel Cell (PEMFC)
Solid Oxide Fuel cell (SOFC),
Molten Carbonate Fuel cell (MCFC),
Alkaline Fuel Cell (MFC),
Phosphoric Acid fuel cell (PAFC).
Direct Methanol fuel cell (DMFC) is the only type of fuel cell that is distinguished
by its fuel (Methanol) and not its electrolyte.

http://www.thehindubusinessline.com/specials/clean-tech/change-is-in-theair/article7912910.ece
http://www.thehindu.com/features/metroplus/Motoring/honda-begins-sale-of-zeroemissions-car-the-clarity-fuel-cell/article8336475.ece
Read this to know the working of fuel cells
http://www.fuelcellenergy.com/why-fuelcell-energy/how-do-fuel-cells-work/

www.iasbaba.com

Page 1

IASbaba 60 Day Plan- Prelims Test 2016 SCIENCE & TECH & CURRENT AFFAIRS [DAY 23]

Q.2) Consider the following w.r.t Asian and African Elephants?


1. Both the males and females have tusks in Asian and African elephants
2. African elephants have keener sense of smell as compared to Asian elephants
Select the correct code
a)
b)
c)
d)

Only 1
Only 2
Both
None

Q.2) Solution (d)


Why is there an increase in tuskless males among Asian elephants? Why do only males have
tusks in Asia, while in Africa, both the sexes have tusks? Why does the elephant in the subcontinent have a keener sense of smell sharper than that of a dog than its African
counterparts?
The answers to these questions are there in the genome of the Asian elephant, which has
been sequenced perhaps for the first time in India
Read morehttp://www.thehindu.com/news/national/karnataka/genome-of-the-asianelephant-sequenced-for-the-first-time-in-india/article8006026.ece

Q.3) Which of the following is the correct definition of Transcriptome?


a) It includes only those RNA molecules found in a specified cell population, and usually
includes the amount or concentration of each RNA molecule in addition to the
molecular identities.
b) It is the set of all messenger RNA molecules in one cell or a population of cells.
c) It is the set of all transfer RNA molecule in two or more cells
d) None of the above is correct

Q.3) Solution (b)


Reference taken from- http://www.thehindu.com/news/national/karnataka/genome-ofthe-asian-elephant-sequenced-for-the-first-time-in-india/article8006026.ece

www.iasbaba.com

Page 2

IASbaba 60 Day Plan- Prelims Test 2016 SCIENCE & TECH & CURRENT AFFAIRS [DAY 23]

Q.4) Digital convergence refers to the convergence of four industries into one
conglomerate. Identify those four industries?
1.
2.
3.
4.
5.
6.

Media
Consumer Electronics
Information Technology
Telecommunication
Entertainment
Cognitive Science

Select the appropriate code


a)
b)
c)
d)

1, 3, 4 and 5
2, 3, 4 and 5
2, 3, 4 and 6
3, 4, 5 and 6

Q.4) Solution (b)


Digital convergence refers to the convergence of four industries into one conglomerate,
ITTCE (Information Technologies, Telecommunication, Consumer Electronics, and
Entertainment).

Q.5) Consider the following about totipotent, pluripotent and multipotent cells:
1. The correct order from most differentiated to least differentiated- is
pluripotent>multipotent>totipotent
2. Embryonic stem cells are considered pluripotent cells
3. Multipotent cells are more limited than pluripotent cells
Select the correct code
a)
b)
c)
d)

1 and 2
2 and 3
1, 2 and 3
1 and 3

Q.5) Solution (b)

www.iasbaba.com

Page 3

IASbaba 60 Day Plan- Prelims Test 2016 SCIENCE & TECH & CURRENT AFFAIRS [DAY 23]

These terms refer to the differentiating potential of the particular cell in its development
stage. In general, going from least differentiated to most differentiated, you could arrange
the terms as totipotent=>pluripotent=>multipotent. Now this means that a totipotent cell is
not differentiated at all, and can create any cell type in the body and certain placental
tissues. Only embryonic cells in very early stages of development (the first few cell divisions
from a zygote or fertilized ovum) are totipotent. A pluripotent stem cell can create cells
from any of the three germ layers in the body, thus the least differentiated pluripotent cells
can become any cell in the body. Embryonic stem cells are one source of pluripotent cells.
Totipotent cells can form all the cell types in a body, plus the extraembryonic, or placental,
cells. Embryonic cells within the first couple of cell divisions after fertilization are the only
cells that are totipotent. Pluripotent cells can give rise to all of the cell types that make up
the body; embryonic stem cells are considered pluripotent. Multipotent cells can develop
into more than one cell type, but are more limited than pluripotent cells; adult stem cells
and cord blood stem cells are considered multipotent
Reference of pluripotent cells taken from this news
http://www.thehindu.com/sci-tech/science/crisprcas-method-of-gene-editing-causescrisper-debates/article8203123.ece
Also learn about stem cells and induced pluripotent stem cells

Q.6) Since the advent of the antibiotic superdrug in the 1940s, a new villain, the
superbug has emerged and has been making national and international headlines over
the past two decades. Consider the following
1.
2.
3.
4.

Methicillin-resistant Staphylococcus aureus (MRSA)


Drug-resistant tuberculosis (MDR- and XDR-TB)
Drug-resistant Enterococcus
Drug-resistant Streptococcus pneumoniae

Which of the above falls under the category of Superbug?


a)
b)
c)
d)

1, 3 and 4
2, 3 and 4
1, 2 and 3
1, 2, 3 and 4

www.iasbaba.com

Page 4

IASbaba 60 Day Plan- Prelims Test 2016 SCIENCE & TECH & CURRENT AFFAIRS [DAY 23]

Q.6) Solution (d)


Reference taken from here- http://www.thehindu.com/opinion/op-ed/the-battle-againstsuperbugs/article8660875.ece?utm_source=RSS_Feed&utm_medium=RSS&utm_campaign=
RSS_Syndication

Q.7) Consider the following regarding Uranium production in the world


1. India is among top 5 uranium producing countries in the world
2. Kazakhstan produces the largest share of uranium from mines followed by Australia
and Canada
3. Worlds maximum uranium production comes from underground mining
Select the incorrect code
a)
b)
c)
d)

Only 1
1 and 3
1, 2 and 3
None

Q.7) Solution (c)


India is not even in top 10. Kazakhstan- Canada-Australia
Over two-thirds of the world's production of uranium from mines is from Kazakhstan,
Canada and Australia. An increasing amount of uranium, now 48%, is produced by in situ
leaching.
Reference taken from here- http://www.thehindu.com/data/nuclear-energy-a-globalfactsheet/article8701264.ece

Q.8) Why does the Sun appear white in colour at noon?


a)
b)
c)
d)

A little of the blue colour is scattered.


Blue colour is scattered the most
Red colour is scattered the most
All the colours are scattered away

www.iasbaba.com

Page 5

IASbaba 60 Day Plan- Prelims Test 2016 SCIENCE & TECH & CURRENT AFFAIRS [DAY 23]

Q.8) Solution (a)


At noon, the sun is overhead and the light coming from the sun travels a relatively shorter
distance through the atmosphere to reach the earth. In this case the blue light is not
sacttered much. As the light coming from the overhead sun contains almost all its
component colours in the right proportion, the sun appears white to us at noon.
NCERT- NCERT- class 10th

Q.9) Consider the following regarding The Natural Resources Data Management System
(NRDMS) Programme
1. It was launched with the objectives of developing and demonstrating scientific
methods and techniques for national level planning
2. Ministry of Environment and Forest is the nodal agency to govern its function and
proper execution The Natural Resources Data Management System (NRDMS) Programme was launched in 1982
objectives of developing and demonstrating scientific methods and techniques for local level planning

Select the correct code


a)
b)
c)
d)

Only 1
Only 2
Both
None

Q.9) Solution (d)

http://pib.nic.in/newsite/PrintRelease.aspx?relid=142346

Q.10) Consider the following regarding sunrise and sunset


1. The time difference between actual sunrise and sunset is 4 minutes
2. This is because of the phenomenon of Total Internal Reflection
3. The apparent flattening of the Suns disc at sunrise and sunset is due to atmospheric
refraction.
Select the incorrect code
a) Only 1
b) Only 3
c) 1 and 2
www.iasbaba.com

Page 6

IASbaba 60 Day Plan- Prelims Test 2016 SCIENCE & TECH & CURRENT AFFAIRS [DAY 23]

d) 1, 2 and 3

Q.10) Solution (c)


Time difference is 2 minutes and it is due to atmospheric refraction
A ray of light bends when it travels from one medium to another. This bending of light is
called refraction of light. If the light ray travels from a rare medium to a denser medium, it
bends towards the imaginary normal and if it travels from a dense medium to a rarer
medium, it bends away from the imaginary normal. Depending on the density of the
different medium, the speed of the travelling light ray keeps varying, and this causes it to
slow down or speed up, therefore bending in the process.
So how this refraction of light is connected to our advanced sunrise and delayed sunset?
Imagine the journey of light rays from the sun. Their initial journey is through vacuum and
then through the atmosphere of the earth and then it is finally seen by us. In this case,
vacuum will be a rare medium and the earths atmosphere with all its temperature changes,
winds, different gases, will be a denser medium in comparison.
During sunrise, when the sun is just below the horizon, our atmosphere causes the light
rays to bend and we see the sun early. Similarly, at sunset, the apparent position of the sun
is visible to us and not the actual position due to the same bending of light rays effect.
To sum up, due to refraction we see the sun rise about two minutes before its actually
there and during sunset we see it for around two minutes more, even though it has already
moved from that position.

Q.11) Consider the following statements


1.
2.
3.
4.

It is not a member of United Nations but a member of IMF and World Bank
It declared its independence from Serbia
It is a landlocked country
It is one of the Muslim majority territories on European mainland

Identify the country?


a)
b)
c)
d)

Albania
Kosovo
Bosnia
Slovenia

www.iasbaba.com

Page 7

IASbaba 60 Day Plan- Prelims Test 2016 SCIENCE & TECH & CURRENT AFFAIRS [DAY 23]

Q.11) Solution (b)


Statement 1 As of 23 June 2015, 108 UN member countries recognise the Republic of
Kosovo, while 85 member countries do not recognize them.] It is not itself a member of the
UN, however it is a member of the International Monetary Fund (IMF), World Bank.
Statement 2 It declared its independence from Serbia in 2008
Statement 3 Kosovo is landlocked in the central Balkan Peninsula. It is bordered by the
Republic of Macedonia and Albania to the south, Montenegro to the west, and the
uncontested territory of Serbia to the north and east
Statement 4 Click Here
Refer this
http://www.thehindu.com/todays-paper/tp-international/unesco-rejects-kosovos-bid-formembership/article7863025.ece

Religion in Kosovo
Religion

Population %

Islam

1,663,202 95.60

Christians 64,275
Catholic 38,438
Orthodox 25,837

3.69
2.20
1.48

Other

1,188

0.06

None

1,842

0.10

Not stated 10,023

0.55%

Q.12) Acid attacks are one of the most gruesome kind of acts causing not only physical
harm to the victim but psychological and emotional harm as well. Which of the following
steps should be taken to reduce the effect of acid on the skin?
1. Burnt area should be washed with clean water or slightly saline water.
2. Burnt are should be washed with milk. Milk is basic and neutralizes acid.
3. Burnt area should be washed with a strong base to quickly neutralize it so that it
does not react further.

www.iasbaba.com

Page 8

IASbaba 60 Day Plan- Prelims Test 2016 SCIENCE & TECH & CURRENT AFFAIRS [DAY 23]

Select the code from the following:


a)
b)
c)
d)

1 only
1 and 2
1 and 3
All of the above

Q.12) Solution (a)


Milk is slightly acidic and also the reaction of milk and acid is exothermic which can cause
further damage. It can also contaminate the burnt area.
Strong base is itself a corrosive agent. And the reaction with acid is highly exothermic.

Q.13) Mitochondria is an organelle found in most of the cells. It is known as the


powerhouse of the cell. Which of the following statements are correct related to
mitochondria?
1. The biochemical processes of respiration and energy production occurs here.
2. Mitochondrial disease can pass to a progeny only through mothers in human beings.
3. Three parent baby is a technique used to prevent the transfer of mitochondrial
defect to the babies.
Select the correct code from the following:
a)
b)
c)
d)

1 and 2
2 and 3
1 and 3
All of the above

Q.13) Solution (d)


In a male germ cell (sperm), the tail part contains the mitochondria. As tale does not take
part in the process of fertilization, it is not passed to a progeny from fathers.
Mitochondria is present in the cytoplasm of the mothers cells. In three parent baby, the
nucleus from mothers egg cell is extracted and planted in the cytoplasm of another
womens egg cell that does not have a mitochondrial defect. This new cell is fertilized by

www.iasbaba.com

Page 9

IASbaba 60 Day Plan- Prelims Test 2016 SCIENCE & TECH & CURRENT AFFAIRS [DAY 23]

fathers sperm. Since the genetic characteristics lie in the nucleus, the baby gets his genetic
traits from two parents only.

Q.14) Which of the following forces are not the example of contact forces?
1.
2.
3.
4.
5.

Friction force
Electrostatic force
Magnetic force
Gravitational force
Air resistance force

Select the code from the following:


a)
b)
c)
d)

1 only
1, 3, 4 and 5
2, 3 and 4
2, 4 and 5

Q.14) Solution (c)


Friction force is an example of a contact force. Rest of the three forces acts from a distance
and dont require body to body contact.
Contact Forces

Action-at-a-Distance Forces

Frictional Force

Gravitational Force

Tension Force

Electrical Force

Normal Force

Magnetic Force

Air Resistance Force


Applied Force
Spring Force

Q.15) Consider the following statements about Jal-Tarang:


1. It is an Indian percussion instrument.
2. It consists of ceramic or metal bowls tuned by putting different amount of water in
them.
3. It is played by hitting the bowls with sticks.
www.iasbaba.com

Page 10

IASbaba 60 Day Plan- Prelims Test 2016 SCIENCE & TECH & CURRENT AFFAIRS [DAY 23]

Which of the above statements are correct?


a)
b)
c)
d)

2 and 3
All of the above
1 and 3
None of the above

Q.15) Solution (b)


Jal Tarang is an Indian melodic percussion instrument. It consists of a set of ceramic or metal
bowls tuned with water. The bowls are played by striking the edge with beaters, one in each
hand.
https://www.youtube.com/watch?v=BhNcjLy5CeA

Q.16) The 2014 Nobel Prize for physics has been awarded to a trio of scientists in Japan
and the US for the invention of blue light emitting diodes (LEDs). Why the invention of
blue LED was so special?
1. It was the first LED to be invented.
2. Together with Green and red LED, white light could be produced.
3. Blue LED is the most difficult to make as it uses, Gallium nitride and not Gallium
Phosphide.
Select the correct code from the following:
a)
b)
c)
d)

1 and 2
2 and 3
1 and 3
All of the above

Q.16) Solution (b)


LEDs are basically semiconductors that have been built so they emit light when they're
activated. Different chemicals give different LEDs their colors. Engineers made the first LEDs
in the 1950s and 60s. Early iterations included laser-emitting devices that worked only when
bathed in liquid nitrogen. At the time, scientists developed LEDs that emitted everything

www.iasbaba.com

Page 11

IASbaba 60 Day Plan- Prelims Test 2016 SCIENCE & TECH & CURRENT AFFAIRS [DAY 23]

from infrared light to green light but they couldn't quite get to blue. That required
chemicals, including carefully-created crystals that they weren't yet able to make in the lab.
A white LED light is easy to make from a blue one. Engineers use a blue LED to excite some
kind of fluorescent chemical in the bulb. That converts the blue light to white light.
Two of the prize winners, Isamu Akasaki and Hiroshi Amano, worked together on producing
high-quality gallium nitride, a chemical that appears in many of the layers in a blue LED. The
previous red and green LEDs used gallium phosphide, which was easier to produce. Akasaki
and Amano discovered how to add chemicals to gallium nitride semiconductors in such a
way that they would emit light efficiently. The pair built structures with layers of gallium
nitride alloys.
The third prize-winner, Shuji Nakamura, also worked on making high-quality gallium nitride.
He figured out why gallium nitride semiconductors treated with certain chemicals glow. He
built his own gallium nitride alloy-based structures.

Q.17) Pole star is amongst the most useful stars for humans because of its fixed location in
the sky. Consider the following statements with respect to pole star:
1. Pole star is helpful in finding the north direction in night and helped in navigation in
ancient times.
2. Pole star can be pointed with the help of Ursa Major constellation.
3. Pole star is not visible in the Southern Hemisphere.
Which of the above statements are correct?
a)
b)
c)
d)

1 and 2
2 and 3
1 and 3
All of the above

Q.17) Solution (d)


Self explanatory.

www.iasbaba.com

Page 12

IASbaba 60 Day Plan- Prelims Test 2016 SCIENCE & TECH & CURRENT AFFAIRS [DAY 23]

A reference from current affairs


http://timesofindia.indiatimes.com/city/goa/A-mid-summer-tryst-with-thestars/articleshow/52368133.cms

Q.18) In cricket, the bowlers are now mastering the art of reverse swinging the ball. This
helps them to take the pitch out of the equation, and move the ball in the air. Which of
the following statements are correct about reverse swing?
1. For reverse swing to happen the ball needs to be rough on one side and shiny on the
other.
2. The ball moves towards the shining side.
3. Due to friction, the air pressure on the rough side is less and shiny side is more.
Select the code from the following:
a)
b)
c)
d)

1 and 2
2 and 3
1 and 3
All of the above

Q.18) Solution (a)


The reverse swinging phenomenon can be explained with the help of Bernoullis Principle.
The bowlers deliberately make one side a little rough and polish the other side. This makes
the two sides contrasting. Now when the ball is released, the rough side encounters high
aerodynamic friction, which increases the air pressure on the rough side. On the other hand,
the smooth side reduces the aerodynamic drag, hence less air pressure.
What Bernoulli's principle states is this: (in very very simple words)
The pressure difference will create a net force which has a direction. Remember Pressure is
simply force per area. This force created will make the ball move sideways. Also the ball will
move towards the shiny side, which has less air pressure.

www.iasbaba.com

Page 13

IASbaba 60 Day Plan- Prelims Test 2016 SCIENCE & TECH & CURRENT AFFAIRS [DAY 23]

Q.19) Consider the following situation:


If an object is released from a moving train, the object will go:
a)
b)
c)
d)

In the opposite direction of the moving train.


In the same direction of the moving train.
It will fall straight downwards.
None of the above

Q.19) Solution (b)


The object will be in inertia of motion. As it was moving with train, it had a component of
velocity in the direction of motion. If it is released, that component will still be there and it
will fall in the direction of motion of the train.

Q.20) Steam burns are more severe than the burns caused by boiling water because:
a)
b)
c)
d)

The temperature of steam is higher than the boiling water.


Steam contains latent heat.
Steam hits the body with a pressure while water does not.
The given statement is false.

Q.20) Solution (b)


Steam burns are more severe as steam contains latent heat.

Q.21) Which of the following statements about Amber are correct?


1.
2.
3.
4.

It is a fossilized tree resin.


It is helpful in paleontology as it contains preserved specimens of plants and insects.
It is highly sought after because of its vibrant colour and is used in jewelry.
Amber is used as an ingredient in perfumes and healing agent in folk medicines.

www.iasbaba.com

Page 14

IASbaba 60 Day Plan- Prelims Test 2016 SCIENCE & TECH & CURRENT AFFAIRS [DAY 23]

Select the code from the following:


a)
b)
c)
d)

1 and 2
1 and 3
2,3 and 4
All of the above

Q.21) Solution (d)


Self explanatory. Term is taken from NCERT.
You can also refer to these links
http://timesofindia.indiatimes.com/environment/30-million-year-old-fossil-flowers-foundpreserved-in-amber/articleshow/51026034.cms
http://www.thehindu.com/opinion/op-ed/huge-amber-deposit-discovery-in-westernindia/article850900.ece

Q.22) Consider the following with respect to Nilgai


1. Nilgai is endemic to Indian subcontinent
2. It is the largest antelope in Asia
3. It is vulnerable as per IUCN red list

Select the correct answer using the codes given below


a)
b)
c)
d)

1 and 3 only
1 and 2 only
2 and 3 only
All

Q.22) Solution (b)


It is least concerned
References

www.iasbaba.com

Page 15

IASbaba 60 Day Plan- Prelims Test 2016 SCIENCE & TECH & CURRENT AFFAIRS [DAY 23]

http://www.thehindu.com/news/national/other-states/revered-nilgai-turns-farmersenemy/article8711052.ece
http://www.thehindu.com/news/cities/Delhi/nilgai-blues-hit-capitalsheart/article8652515.ece

Q.23) Consider the following countries


1. Honduras
2. Belize
3. El Salvador

Select the countries not bordering Mexico


a)
b)
c)
d)

1 and 3 only
1 and 2 only
2 and 3 only
All border Mexico

Q.23) Solution (a)


Guatemala and Belize are the two countries in the south bordering Mexico
Mexico is in news for backing India for NSG membership. Also Mexican President took a
drive with our PM for dinner
Learn briefly about Mexico in particular. You never know UPSC :P

Q.24) Consider the following statements with respect to SURYAMITRA Mobile App
1. It is developed by National Institute of Solar Energy (NISE)
2. SURYAMITRA is a skill development program designed to create skilled manpower in
installation, commissioning, and Operation & Maintenance of solar power plants and
equipment.
Select the correct option
a)
b)
c)
d)

1 only
2 only
Both
None

www.iasbaba.com

Page 16

IASbaba 60 Day Plan- Prelims Test 2016 SCIENCE & TECH & CURRENT AFFAIRS [DAY 23]

Q.24) Solution (c)


Click here

Q.25) Hand in Hand is a joint military exercise conducted between which of the
following two countries
a)
b)
c)
d)

India and Sri Lanka


Sri Lanka and China
China and India
China and Bangladesh

Q.25) Solution (c)


Click here

Q.26) Consider the following statements regarding Gold monetisation Scheme


1. Its objective is to mobilize unutilized gold from individuals, households and
institutions and make them available to gold-base industries
2. The tenure of gold deposits is likely to be for a minimum of two years.
3. The minimum quantity of deposits is pegged at 10 gram to encourage even small
deposits

Choose the incorrect answer using the code given below


a)
b)
c)
d)

1 and 3 only
2 and 3 only
1 and 2 only
All the above

Q.26) Solution (b)

Gold Monetisation Scheme, which would replace both the present Gold Deposit and
Gold Metal Loan Schemes. The new scheme would allow the depositors of gold to
earn interest in their metal accounts and the jewellers to obtain loans in their metal
account. Banks/ other dealers would also be able to monetise this gold.

www.iasbaba.com

Page 17

IASbaba 60 Day Plan- Prelims Test 2016 SCIENCE & TECH & CURRENT AFFAIRS [DAY 23]

The tenure of gold deposits is likely to be for a minimum of one year. The minimum
quantity of deposits is pegged at 30 gram to encourage even small deposits. The gold
can be in any form, bullion or jewellery.

http://www.thehindu.com/business/Industry/government-announces-third-round-of-goldbond-scheme/article8314695.ece

www.iasbaba.com

Page 18

IASbaba 60 Day Plan- Prelims Test 2016 ENVIRONMENT & CURRENT AFFAIRS [DAY 24]

Q.1) Which of the following are the features of newly notified E-Waste Management Rules,
2016 by the Ministry of Environment, Forest and Climate Change?
1. These rules for the first time will bring the producers under Extended Producer
Responsibility (EPR).
2. For the first time, Compact Fluorescent Lamp (CFL) and other mercury containing lamp
brought under the purview of rules.
3. Provision for Pan India EPR Authorization by CPCB has been introduced replacing the
state wise EPR authorization.
4. Micro and Small industry sector as defined in Micro Small and Medium Developmental
Act, 2006 are completely exempted with EPR Responsibility.
Choose the appropriate code:
a) 1 and 4 only
b) 1, 3 and 4 only
c) 1, 2 and 3 only
d) All of the above
Q.1) Solution (d)
Explanation:
All the given statements are correct and self-explanatory
India has emerged as the worlds fifth largest electronic waste (e-waste) producer.
Exemption continues for micro enterprises; however small enterprises, which have been
referred as one of the major source of generation of e-waste, have been included in the
rules for responsibility as manufacturer, without burdening them with EPR responsibility as
applicable to Producers.
Source: http://pib.nic.in/newsite/PrintRelease.aspx?relid=138319
Q.2) Select the incorrect pair from the below given important environment related days and
its 2016 theme:
a) World Oceans Day (8th June every year) : : Healthy oceans, healthy planet
b) World Environment Day (5th June every year) : : Go Wild for Life -- Zero tolerance for
the illegal trade in wildlife
c) Earth Day (22nd April every year) : : Wetlands for our Future Sustainable Livelihoods
d) None of the above

www.iasbaba.com

Page 1

IASbaba 60 Day Plan- Prelims Test 2016 ENVIRONMENT & CURRENT AFFAIRS [DAY 24]

Q.2) Solution (c)


Explanation:
Wetlands for our Future Sustainable Livelihoods is the 2016 Theme of World Wetland
Day (which is celebrated on 2nd February every year)
Every year Earth Day is being observed on the 22nd April to build support for environmental
protection around the world. This year is 46th anniversary of Earth Day.
Earth Days 2016 Theme: Trees for the Earth!.
The theme signifies that trees are crucial for combating climate change as they absorb
harmful levels of Carbon dioxide (CO2) from the atmosphere. This years Earth Day
coincides with the signing of the historic Paris Agreement on Climate Change.
The World Oceans Day is being observed globally on 8th June every year to raise global
awareness of the current challenges faced by the international community in connection
with the oceans.
2016 Theme: Healthy oceans, healthy planet. The theme highlights main urgent need to
curb the plastic pollution.
Do you know?
Every year May 22 is being observed as International Biodiversity Day (IDB) across the world to
promote conservation and sustainable use of biodiversity.
2016 Theme: Mainstreaming Biodiversity; Sustaining People and their Livelihoods. The theme
highlights the role of biodiversity in underpinning development. The United Nations (UN) also
has highlighted the importance of biodiversity as an important cross-cutting issue in in the
Sustainable Development Goals (SDGs).
The day commemorates the adoption of Agreed Text of the Convention of Biological Diversity
(CBD) on 22 May 1992 by the Nairobi Final Act.
Source: Various newspaper articles and environment related websites

www.iasbaba.com

Page 2

IASbaba 60 Day Plan- Prelims Test 2016 ENVIRONMENT & CURRENT AFFAIRS [DAY 24]

Q.3) Recently, the Union Government has launched the Centre for Environmental Health in
New Delhi. Consider the following statements in regard to it:
1. The centre will assess the impact of environment-related problems like climate change,
air pollution, pesticide use and sanitation on health.
2. It is a joint initiative of the Bombay Natural History Society (BNHS) and Public Health
Foundation of India (PHFI).
3. The initiative was launched by Union Minister for Health and Family Welfare.
Which of the statements given above is/are correct?
a) 2 and 3 only
b) 1 and 2 only
c) 1 and 3 only
d) All of the above
Q.3) Solution (c)
Explanation:
It is a joint initiative of the Tata Institute of Social Science (TISS) and Public Health
Foundation of India (PHFI).
Centre for Environmental Health will assess the impact of environment-related problems
like climate change, air pollution, pesticide use and sanitation on health.
The centre will conduct research across wide range of environmental health issues,
including water, chemical exposure and hygiene.
It will also establish a policy engagement platform with regular meetings with the civil
society, government, academia and private sector to develop strategies for better
implementation of the eco-friendly policies.
Source:
http://www.uniindia.com/nadda-launches-centre-for-environmentalhealth/india/news/488858.html

www.iasbaba.com

Page 3

IASbaba 60 Day Plan- Prelims Test 2016 ENVIRONMENT & CURRENT AFFAIRS [DAY 24]

Q.4) Which among the following statements are correct in regard to recently adopted historic
Paris Climate Agreement at the 21st Conference of the Parties of the UNFCCC?
1. The agreement facilitates enforcement of global GHGs reduction measures, adaptation
and finance in the post-2020 i.e. in post Kyoto Protocol scenario.
2. All member countries agreed to work to limit global temperature rise to well below 2
degrees Celsius, and to strive for 1.5 degrees Celsius.
3. All member countries agreed to take into account the principle of equity (climate
justice) and common but differentiated responsibilities (CBDR) and respective
capabilities.
4. Developed countries will provide 100 billion dollars annually to the developing
counterparts beginning in 2020 and it would increase with time.
Choose the appropriate code:
a) 1 and 4 only
b) 1, 3 and 4 only
c) 1 and 3 only
d) All of the above

Q.4) Solution (d)


Explanation:
Self explanatory All the given statements are correct

Q.5) If a private company wants to set up an industrial or infrastructure projects in a forest


land, then:
1. The Forest (Conservation) Act, 1980 (FCA) governs diversion or use of forest land for
non-forest purposes such as industrial or infrastructure projects.
2. A company diverting forest land must provide alternative land for taking up
compensatory afforestation.
3. For the afforestation purpose, the company should pay for planting new trees in the
alternative land provided to the state.

www.iasbaba.com

Page 4

IASbaba 60 Day Plan- Prelims Test 2016 ENVIRONMENT & CURRENT AFFAIRS [DAY 24]

4. The loss of forest ecosystem must also be compensated by paying for net present value
of forest (NPV).
Which of the statements given above is/are incorrect?
a) None of the above
b) 1 and 4 only
c) 4 only
d) 1 only
Q.5) Solution (a)
Explanation:
Self explanatory All the given statements are correct
Q.6) Ministry of Environment, Forest and Climate Change recently released a new Four-color
Classification Scheme for industries based on their pollution potential. Select the correctly
matched new categorization system:
1.
2.
3.
4.

Red category: Pollution Index (PI) score of 60 and above.


Orange category: PI score of 41 to 59.
Yellow category: PI score of 21 to 40.
Green category: PI score below and upto 20.

Choose the appropriate code


a) 1 and 2 only
b) 1, 3 and 4 only
c) 1 and 4 only
d) 1, 2 and 4 only
Q.6) Solution (a)
Explanation:
Green category: PI score of 21 to 40.
White category: PI score below and upto 20.
There is no Yellow category.

www.iasbaba.com

Page 5

IASbaba 60 Day Plan- Prelims Test 2016 ENVIRONMENT & CURRENT AFFAIRS [DAY 24]

Q.7) Recently, the Union cabinet gave its formal approval for adopting the Statute of the
South Asia Wildlife Enforcement Network (SAWEN). Select the incorrect statement from the
following in regard to SAWEN:
a) SAWEN is regional inter-governmental wildlife law enforcement support body launched
in January, 2011 in Paro, Bhutan.
b) SAWEN aims to strengthen, promote and co-ordinate regional co-operation for curbing
illegal wildlife trade that threatens the wild flora and fauna of South Asia.
c) Except Pakistan, all other South Asian countries will be the part of SAWEN.
d) None of the above
Q.7) Solution (c)
Explanation:
All 8 South Asian countries will be the part of SAWEN (i.e. including Pakistan)

Q.8) Consider the following statements regarding Agenda 21


1. It is a non-binding and voluntary implementation plan of United Nations with regard to
sustainable development
2. United Nation Convention to Combat Desertification (UNCCD), a legally binding
international agreement stemmed from Agenda 21
3. Agenda 21 also mentions about protection of atmosphere
Select the appropriate code
a) 1 and 2 Only
b) 2 and 3 Only
c) 1 and 3 Only
d) 1, 2 and 3

Q.8) Solution (d)


www.iasbaba.com

Page 6

IASbaba 60 Day Plan- Prelims Test 2016 ENVIRONMENT & CURRENT AFFAIRS [DAY 24]

All the statements are correct.

Q.9) Consider the following International Conventions


1. Aarhus Convention
2. Geneva Convention
3. Stockholm Convention
4. Rotterdam Convention
5. Basel Convention
6. Bonn Convention
Which of the above are related to Hazardous Substances?
a) 1, 2, 3, 5 and 6
b) 2, 3, 4 and 5
c) 1, 2, 3, 4, 5 and 6
d) 1, 2, 3, 4 and 5

Q.9) Solution (b)


Aarhus Convention- The UNECE Convention on Access to Information, Public Participation in
Decision-making and Access to Justice in Environmental Matters, usually known as the Aarhus
Convention
Bonn Convention- The Convention on the Conservation of Migratory Species of Wild Animals -more commonly abbreviated to just the Convention on Migratory Species (CMS) or the Bonn
Convention -- aims to conserve terrestrial, marine and avian migratory species throughout their
range.

www.iasbaba.com

Page 7

IASbaba 60 Day Plan- Prelims Test 2016 ENVIRONMENT & CURRENT AFFAIRS [DAY 24]

Q.10) Amur falcons has one of the longest and most fascinating migratory paths in the avian
kingdom. Consider the following statements regarding Amur falcons:
1.
2.
3.
4.

It is one of the largest raptor species.


It migrates from Mongolia to South Africa via India.
It is amongst the critically endangered species and very few left in wild.
It considered a delicacy in Nagaland and widely hunted during their short stay in India.

Select the correct statements from above:


a)
b)
c)
d)

1 and 3
2 and 4
3 only
All of the above

Q.10) Solution (b)


It is a small raptor, of the size of a pigeon. It covers one of the largest migratory distances from
Mongolia and Russia to South Africa. It is not a critically endangered species and is found in
large numbers. However, due to indiscriminate killing, their numbers are rapidly coming down.
It is considered as a delicacy in Nagaland and widely hunted and sold in the local market.

Q.11) Endosulfan has been used as a pesticide in agriculture for a long time. India has agreed
to phase out completely the use and manufacture of endosulfan by 2017. Which of the
following statements are correct about endosulfan?
1.
2.
3.
4.
5.

It is not pest specific and it can negatively impact the populations of beneficial insects.
Endosulfan is acutely neurotoxic to both insects and mammals.
Endosulfan has high potential to bio-accumulate and bio-magnify.
Endosulfan is banned across the world under Stockholm convention.
Endosulfan is the cause of death of Indian vultures.

Select the code from the following:


a) 1, 2 and 3
b) 1, 3 and 4
c) 2, 3, 4 and 5
www.iasbaba.com

Page 8

IASbaba 60 Day Plan- Prelims Test 2016 ENVIRONMENT & CURRENT AFFAIRS [DAY 24]

d) All of the above


Q.11) Solution (a)
Endosulfan ban is being discussed under Stockholm convention for Persistent Organic
Pollutants but nothing is yet decided.
Cause of death of Indian vultures is the use of diclofinac.

Q.12) Government of India has being trying to protect the critically endangered Great Indian
Bustard. Which of the following statement are correct about Great Indian Bustard?
1. To protect the bird and create awareness, this has been made the state bird of Haryana.
2. It is a flight less bird.
3. It is found in arid and semi-arid grassland.
Select the code from the following:
a)
b)
c)
d)

All of the above


1 and 2
2 and 3
3 only

Q.12) Solution (d)


Great Indian Bustard is one of the heaviest flying birds. It has been declared as the state bird of
Rajasthan.

www.iasbaba.com

Page 9

IASbaba 60 Day Plan- Prelims Test 2016 ENVIRONMENT & CURRENT AFFAIRS [DAY 24]

Q.13) Which of the following gas emissions are checked when pollution check of vehicles are
done in India?
1.
2.
3.
4.

Carbon mono Oxide


Hydrocarbons
Carbon di-oxide
Sulphur Oxides

Select the correct code from the following:


a)
b)
c)
d)

1 and 2
3 and 4
1,2 and 4
All of the above

Q.13) Solution (a)


Only the amount of Carbon mono Oxide and Hydrocarbons is checked in exhaust from the
vehicle in pollution check.

Q.14) India is amongst the first and foremost countries to have a forest policy. Although the
first policy of 1894 was designed on how to exploit the forest resources, constant revisions
were made in the policy after independence of India. The current policy was made in 1988,
known as The National Forest Policy 1988. Which of the following statements are incorrect
about this policy?
1. Maintenance of environmental stability through the preservation and restoration of
ecological balance.
2. Conservation of forests as a national heritage with vast varieties of flora and fauna.
3. Control of soil erosion and denudation in catchment areas of rivers, lakes and reservoirs.
4. Check on the extension of sand-dunes in desert areas.
5. To meet the needs of fuel-wood, fodder and minor forest products for the rural and
tribal people.
6. Encouragement of efficient utilization of forest produce and optimum substitution of
wood.
www.iasbaba.com

Page 10

IASbaba 60 Day Plan- Prelims Test 2016 ENVIRONMENT & CURRENT AFFAIRS [DAY 24]

Select the code from the following:


a)
b)
c)
d)

1,2 and 3
4,5 and 6
5 and 6
None of the above

Q.14) Solution (d)


Since all the statements are part of the policy, none of the above statements are incorrect.
The National Forest Policy 1988:
The main emphasis of the National Forest Policy 1988 is on the protection, conservation,
regeneration and development of forests. The main points of the National Forest Policy 1988
are:
(a) Maintenance of environmental stability through the preservation and restoration of
ecological balance.
(b) Conservation of forests as a national heritage with vast varieties of flora and fauna.
(c) Control of soil erosion and denudation in catchment areas of rivers, lakes and reservoirs.
(d) Check on the extension of sand-dunes in desert areas of Rajasthan and along sea-coasts.
(e) Substantial increase in forest cover through massive afforestation and social forestry
programmes.
(f) To meet the needs of fuel-wood, fodder and minor forest products for the rural and tribal
people.
(g) Augment the productivity of the forests to meet national needs.
(h) Encouragement of efficient utilisation of forest produce and optimum substitution of wood.
(i) Steps to create massive movement of people with the involvement of women folk to achieve
these
objectives
and
to
minimise
pressure
on
existing
forests.

(j) Involvement of people in forest management under joint forest management.


www.iasbaba.com

Page 11

IASbaba 60 Day Plan- Prelims Test 2016 ENVIRONMENT & CURRENT AFFAIRS [DAY 24]

Q.15) Since the solid waste generation is rising in urban areas of India, many waste to energy
plants are being started. They serve the dual purpose of waste disposal as well as generation
of electricity. Which of the following statements are correct about waste to energy
generation?
1. The cost of waste to energy is somewhat higher than other renewable sources.
2. The chimneys of incinerators include acid gases, nitrogen oxide, heavy metals,
particulates, and dioxin, which are carcinogenic.
3. The efficiency of waste to energy plants is very low.
Select the correct code from the following:
a)
b)
c)
d)

1 and 2
2 and 3
1 and 3
All of the above

Q.15) Solution (d)


While the Indian Governments own figures would suggest that the cost of waste to energy is
somewhat higher than other renewable sources, it is still an attractive option, as it serves a dual
role of waste disposal and energy production.
Smoke and ash emitted by the chimneys of incinerators include acid gases, nitrogen oxide,
heavy metals, particulates, and dioxin, which are carcinogenic. While incineration pollution
control technology is evolving to reduce these pollutants, it has been found that even with
controls in place, some remaining dioxin still enters the atmosphere.

Q.16) Consider the following statements regarding Minamata Convention:


1. It is an international treaty designed to protect human health and the environment from
anthropogenic emissions and releases of mercury.
2. It is named after Japanese city Minamata which went through devastating incident of
mercury poisoning.
3. India has ratified the Minamata convention.
www.iasbaba.com

Page 12

IASbaba 60 Day Plan- Prelims Test 2016 ENVIRONMENT & CURRENT AFFAIRS [DAY 24]

4. The developed countries have promised to help financially, technically, and


technologically to developing countries.
Which of the above statements are correct?
a)
b)
c)
d)

1, 2 and 3
2, 3 and 4
1, 2 and 4
All of the above

Q.16) Solution (c)


India is a signatory to Minamata convention but it has still not ratified it.

Q.17) Consider the following:


International Conventions
1.
2.
3.
4.

Convention of Nuclear Safety


United Nations Convention on the Law of the Sea
Convention on Persistent Organic Pollutants
Convention on the Control of Transboundary
Movements of Hazardous Wastes and their Disposal

Location
Vienna
Montego Bay
Stockholm
Basel

Which of the above are correctly matched?


a)
b)
c)
d)

1,2 and 3
2,3 and 4
1,3 and 4
All of the above

Q.17) Solution (d)


Self explanatory.

www.iasbaba.com

Page 13

IASbaba 60 Day Plan- Prelims Test 2016 ENVIRONMENT & CURRENT AFFAIRS [DAY 24]

Q.18) Consider the following statements:


1. Mixing fly ash in soil can help farmers increase production of crops and vegetables
2. Fly ash can be mixed with cement thus reducing the cost of construction.
3. Fly ash can cover the surface of plant leaves and thus helps in increasing nutrient
capacity.
4. Fly ash bricks are light weight and offer high strength and durability.
5. Use of fly-ash instead of lime in agriculture can reduce net CO2 emission and also
reduce global warming.
Which of the above statements are correct w.r.t Fly Ash?
a)
b)
c)
d)

1, 2 , 3 and 4
2, 3, 4 and 5
1, 2, 4 and 5
All of the above

Q.18) Solution (c)


Fly ash in atmosphere acts as a pollutant. It can cover the leaf surface by making a thin layer
which reduces photosynthesis and productivity of plants. But its use in soils shows different
result. Fly ash is a resourceful material and can be effectively utilized as soil modifier in large
quantity and micro fertilizer in converting wasteland (barren land, rocky nature, sandy and
water logged soil, highly alkali and acidic soil etc.) into agriculturally productive land. "Best
thing about fly ash is, that it retains water in ground and helps bacterial actions to take place to
cultivate good quality of crops or vegetables.
Refer to these
http://scialert.net/fulltext/?doi=ajar.2010.1.14
http://timesofindia.indiatimes.com/city/bhopal/Fertilizer-costly-fly-ash-can-workmagic/articleshow/33835877.cms

www.iasbaba.com

Page 14

IASbaba 60 Day Plan- Prelims Test 2016 ENVIRONMENT & CURRENT AFFAIRS [DAY 24]

Q.19) Which of the following is not concerned with the Wildlife Protection Act?
1.
2.
3.
4.
5.
6.

National Parks
Wildlife Sanctuaries
Conservation Reserves
Biosphere Reserves
Community Reserves
Tiger Reserves

Select the correct code


a)
b)
c)
d)

3,4 and 5
Only 4
4 and 5
None of the above

Q.19) Solution (b)


National Parks, Wildlife Sanctuaries, Conservation Reserves, Community Reserves and Tiger
Reserves are established as per provisions of Wildlife Protection Act, there is no law as such
under which Biosphere Reserves are established.

Q.20) Consider the following statements with respect to India Biodiversity Award
1. It is a joint initiative by Ministry of Environment, Forest and Climate Change, National
Biodiversity Authority and United Nations Environment Programme
2. The categories for the award are aligned with Biological Diversity Act, 2002
3. Satpura Tiger Reserve won the India Biodiversity Award this year
Which of the following statements is/are incorrect?
a)
b)
c)
d)

Only 1
Only 3
1 and 3
All of the above

www.iasbaba.com

Page 15

IASbaba 60 Day Plan- Prelims Test 2016 ENVIRONMENT & CURRENT AFFAIRS [DAY 24]

Q.20) Solution (c)


It is joint initiative of the Union Ministry of Environment, Forest and Climate Change (MoEFCC),
National Biodiversity Authority (NBA) and United Nations Development programme (UNDP).
Click here
Pakke Tiger Reserve in East Kameng district of Arunachal Pradesh has received the 'India
Biodiversity Award 2016' Click Here

Q.21) Recently Himachal Pradesh fisheries department has started an artificial propagation
programme for rehabilitation and conservation of Golden Mahseer fish. Consider the
following statements with respect to Golden Mahseer Fish.
1.
2.
3.
4.

It is the longest-living freshwater fish


It is native to mountain and sub-mountain regions
It inhabit only rivers
It is an omnivore

Select the correct code


a)
b)
c)
d)

1,2 and 3
1,2 and 4
1 and 2
All of the above

Q.21) Solution (b)

Golden Mahseer Fish is the longest-living freshwater fish and is native to mountain and
sub-mountain regions.
It inhabit both rivers and lakes and is an omnivore

Read More Click Here and Click Here

www.iasbaba.com

Page 16

IASbaba 60 Day Plan- Prelims Test 2016 ENVIRONMENT & CURRENT AFFAIRS [DAY 24]

Q.22) Consider the following


1.
2.
3.
4.

Argentina
Brazil
Uruguay
Paraguay

Which of the above countries does not belong to MERCOSUR Trade Bloc?
a)
b)
c)
d)

All belong to MERCOSUR


1 and 2 only
2 and 3 only
3 and 4 only

Q.22) Solution (a)


Click here

Q.23) Consider the following statements with respect to Free Trade agreement
1.
2.
3.
4.

Goods
Services
IPR
Investment

Which of the above is included in FTA (Free trade agreement) between two countries?
a)
b)
c)
d)

1 and 2 only
2 and 4 only
1 and 3 only
All

Q.23) Solution (d)


FTAs are arrangements between two or more countries or trading blocs that primarily agree to
reduce or eliminate customs tariff and non tariff barriers on substantial trade between them.
www.iasbaba.com

Page 17

IASbaba 60 Day Plan- Prelims Test 2016 ENVIRONMENT & CURRENT AFFAIRS [DAY 24]

FTAs normally cover trade in goods (such as agricultural or industrial products) or trade in
services (such as banking, construction, trading etc.).
FTAs can also cover other areas such as intellectual property rights (IPRs), investment,
government procurement and competition policy, etc.

Q.24) Consider the following


1.
2.
3.
4.

Norway
Switzerland
Sweden
Iceland

Which of the above countries is a member of The European Free Trade association?
a) 1 and 2 only
b) 1,2 and 4 only
c) 2 and 4 only
d) 2 and 3 only
Q.24) Solution (b)

EFTA was in news yesterday as India is pushing to work out the modalities concerning to
finalising free trade Agreement with EFTA
To know more about EFTA click here

Q.25) Consider the following research institutes


1. National institute of Ocean technology is located in the state of Goa
2. National institute of Oceanography is located in Chennai
Select the correct statement with respect to location and research institute is concerned
a)
b)
c)
d)

1 only
2 only
Both
None

www.iasbaba.com

Page 18

IASbaba 60 Day Plan- Prelims Test 2016 ENVIRONMENT & CURRENT AFFAIRS [DAY 24]

Q.25) Solution (d)


Location and research institute is interchanged

Q.26) Consider the following statements with respect to SAFTA


1. SAFTA or South Asian free trade agreement come under the ambit of ASEAN
(Association of South East Asian nations)
2. Pakistan is not a member of SAFTA
Select the correct statements
a)
b)
c)
d)

1 only
2 only
Both
None

Q.26) Solution (d)

It comes under the ambit of SAARC


Pakistan is a member of SAFTA
It was signed during 2004 SAARC summit

www.iasbaba.com

Page 19

IASbaba 60 Day Plan- Prelims Test 2016 POLITY & CURRENT AFFAIRS [DAY 25]

Q.1) Consider the following statements in regard to the President of India:


1. President is the highest decision-making authority in our politico-administrative system.
2. President can require the Prime Minister to submit, for consideration of the council of
ministers, any matter on which a decision has been taken by a minister but, which has
not been considered by the council.
3. President can make regulations for the peace, progress and good government of the
Andaman and Nicobar Islands, Lakshadweep, Dadra and Nagar Haveli and Daman and
Diu.
4. In the case of Puducherry and Delhi, the President can legislate by making regulations
but only when the assembly is suspended or dissolved.
Which of the statements given above are correct?
a)
b)
c)
d)

1, 3 and 4
2 and 3
1, 2 and 3
All of the above

Q.1) Solution (b)


Explanation:
Cabinet is the highest decision-making authority in our politico-administrative system.
Only in the case of Puducherry (not Delhi), the President can legislate by making regulations
but only when the assembly is suspended or dissolved.

Q.2) Consider the following statements in regard to the removal of Vice President of India:
1. The Vice-President may be removed on any ground considered sufficient by the two
Houses of the Parliament
2. In both the cases of removal of the President and the Vice-President, the power of
removal is lodged in the two Houses of Parliament
3. A Vice President may be removed from his office by a resolution of the council of States
passed by an absolute majority and agreed to by the House of the People
4. A Vice President may be removed from his office by a resolution of the council of States
passed by a special majority and agreed to by the House of the People
www.iasbaba.com

Page 1

IASbaba 60 Day Plan- Prelims Test 2016 POLITY & CURRENT AFFAIRS [DAY 25]

Which of the statements given above is/are incorrect?


a)
b)
c)
d)

2 and 4 only
3 and 4 only
3 only
4 only

Q.2) Solution (b)


Explanation:
The Vice-President may be removed on any ground considered sufficient by the two Houses
of the Parliament (as no ground has been mentioned in the Constitution for his removal).
In both the cases of removal of the President and the Vice-President, the power of removal
is lodged in the two Houses of Parliament, and coordinated action is called for on their part
to effect the removal of either dignitary.
Article 67 (b) states that a Vice President may be removed from his office by a resolution
of the council of States passed by a majority of all the then members of the council and
agreed to by the House of the People
In other words, Vice President can be removed by a resolution of the Rajya Sabha passed by
an effective majority (more than 50% of effective membership(total membership - Vacant
seats)) and agreed to by a simple majority (50% of present and voting members) of the Lok
Sabha.
Q.3) Which among the following qualifications for a person to be chosen as a member of the
Parliament is laid down by the Constitution of India?
1. He must be not less than 30 years of age in the case of the Rajya Sabha and not less than
25 years of age in the case of the Lok Sabha.
2. He must be registered as an elector for a parliamentary constituency. This is same in the
case of both, the Rajya Sabha and the Lok Sabha.
3. He must make and subscribe to an oath or affirmation before the person authorised by
the election commission for this purpose, according to the form prescribed in the Fourth
Schedule.
Choose the appropriate code:
www.iasbaba.com

Page 2

IASbaba 60 Day Plan- Prelims Test 2016 POLITY & CURRENT AFFAIRS [DAY 25]

a)
b)
c)
d)

1 only
2 only
1 and 2 only
All of the above

Q.3) Solution (a)


Explanation:
He must be not less than 30 years of age in the case of the Rajya Sabha and not less than 25
years of age in the case of the Lok Sabha.
He must be registered as an elector for a parliamentary constituency. This is same in the
case of both, the Rajya Sabha and the Lok Sabha. (This statement is incorrect because the
Constitution does not lay down this qualification, it is provided in the Representation of
People Act (1951))
He must make and subscribe to an oath or affirmation before the person authorised by the
election commission for this purpose, according to the form prescribed in the Third
Schedule (not fourth).

Q.4) Consider the following statements and select the correct statement/s from the codes
given below:
1. Presiding officer decides on questions as to disqualifications of members of the
Parliament, in consultation with the Election Commission
2. Finance Minister causes to be laid before the Parliament the annual financial statement
3. No demand for a grant can be made except on Presidents recommendation
Choose the appropriate code:
a)
b)
c)
d)

1 and 3 only
2 and 3 only
3 only
All of the above

Q.4) Solution (c)


www.iasbaba.com

Page 3

IASbaba 60 Day Plan- Prelims Test 2016 POLITY & CURRENT AFFAIRS [DAY 25]

Explanation:
President decides on questions as to disqualifications of members of the Parliament, in
consultation with the Election Commission
President causes to be laid before the Parliament the annual financial statement
No demand for a grant can be made except on Presidents recommendation

Q.5) Which among the following statements is not true in regard to Presidents powers?
1. Presidents power to give or not to give assent to the bills passed by the Parliament is a
discretionary power, except in case of money bills and constitutional amendment bills.
2. The object of conferring this discretionary power on the President is to prevent hasty
and ill-considered legislation by the Parliament and to prevent a legislation which may
be unconstitutional.
3. It is not obligatory for the President to give his assent even if the bill is again passed by
the state legislature and sent again to him for his consideration.
Choose the appropriate code:
a) 1 only
b) 1 and 2 only
c) 2 only
d) 1 and 3 only
Q.5) Solution (b)
Explanation:
President has no such discretionary powers (hence, statement 1 and 2 is wrong)
President can act on his discretion (that is, without the advice of the ministers) under the
following situations:
1) Appointment of Prime Minister when no party has a clear majority in the Lok Sabha or
when the Prime Minister in office dies suddenly and there is no obvious successor.
2) Dismissal of the council of ministers when it cannot prove the confidence of the Lok
Sabha.
www.iasbaba.com

Page 4

IASbaba 60 Day Plan- Prelims Test 2016 POLITY & CURRENT AFFAIRS [DAY 25]

3) Dissolution of the Lok Sabha if the council of ministers has lost its majority.
With respect to the government bills when the cabinet resigns (after the passage of the bills
but before the assent by the President) and the new cabinet can advise the President not to
give his assent to such bills.

Q.6) Whenever a bill seeking to replace an ordinance is introduced in the House


1. A statement explaining the circumstances that had necessitated immediate legislation
by ordinance should also be placed before the House.
2. A statement explaining why the concerned House is seeking to replace an ordinance
should be placed before the President.
3. The ordinance may cease to operate even earlier than the prescribed expiry of six
weeks.
Which of the statements given above is/are correct?
a) 1 only
b) 2 only
c) 3 only
d) Both 2 and 3
Q.6) Solution (a)
Explanation:
Only 1st statement is correct - Self-explanatory

Q.7) Consider the following differences with regard to the President and the Governor of
India:
1. Governor has both constitutional and situational discretion power
2. President has no constitutional discretion, he has only some situational discretion
3. The pardoning power of the President is independent of the judiciary and is an
executive power
4. Both the Governor and the President have concurrent power in respect of suspension,
remission and commutation of death sentence
Which of the statements given above is/are correct?
www.iasbaba.com

Page 5

IASbaba 60 Day Plan- Prelims Test 2016 POLITY & CURRENT AFFAIRS [DAY 25]

a)
b)
c)
d)

1, 2 and 4 only
3 only
3 and 4
All of the above

Q.7) Solution (d)


Explanation:
All the statements given are correct Self explanatory
Governor has both constitutional and situational discretion power. For instance:
(Examples of constitutional discretion)
Reservation of a bill for the consideration of the President
Recommendation for the imposition of the Presidents Rule in the state
(Examples of situational discretion)
Appointment of chief minister when no party has a clear-cut majority in the state legislative
assembly or when the chief minister in office dies suddenly and there is no obvious
successor
Dismissal of the council of ministers when it cannot prove the confidence of the state
legislative assembly
President has no constitutional discretion, he has some situational discretion. In other words,
the President can act on his discretion (that is, without the advice of the ministers) under the
following situations:
Appointment of Prime Minister when no party has a clear majority in the Lok Sabha or when
the Prime Minister in office dies suddenly and there is no obvious successor.
Dismissal of the council of ministers when it cannot prove the confidence of the Lok Sabha.
Dissolution of the Lok Sabha if the council of ministers has lost its majority.

Q.8) Consider the following statements in regard to the Prime Minister of India:
1. The Prime Minister holds office for a term of five years from the date on which he
enters upon his office.
2. However, if he loses the confidence of the Lok Sabha, he must resign or the President
can dismiss him.
www.iasbaba.com

Page 6

IASbaba 60 Day Plan- Prelims Test 2016 POLITY & CURRENT AFFAIRS [DAY 25]

3. Prime Minister gets the salary and allowances that are payable to a member of
Parliament, only his sumptuary allowance varies
Which of the statements given above is/are correct?
a) 1 and 2 only
b) 2 only
c) 2 and 3 only
d) All of the above
Q.8) Solution (c)
Explanation:
The term of the Prime Minister is not fixed and he holds office during the pleasure of the
President.

Q.9) Consider the following statements:


1. In India the President is only a nominal executive and the real powers are vested in the
prime minister.
2. The executive power of the Union shall be vested in Prime Minister and shall be
exercised by him either directly or through officers subordinates to him.
3. Council of ministers headed by the Prime Minister advices the President with regard to
summoning and proroguing of the sessions of the Parliament.
4. Orders and other instruments made and executed in the name of the President shall be
authenticated in such manner as may be specified in rules made by the Parliament.
Which of the statements given above is/are incorrect?
a)
b)
c)
d)

2 only
1 and 2 only
2 and 4 only
All of the above

Q.9) Solution (d)


www.iasbaba.com

Page 7

IASbaba 60 Day Plan- Prelims Test 2016 POLITY & CURRENT AFFAIRS [DAY 25]

Explanation:
President is only a nominal executive and the real powers are vested in the council of
ministers headed by the prime minister.
The executive power of the Union shall be vested in President and shall be exercised by him
either directly or through officers subordinates to him in accordance with this Constitution
(Article 53).
Prime Minister advises the President with regard to summoning and proroguing of the
sessions of the Parliament.
Orders and other instruments made and executed in the name of the President shall be
authenticated in such manner as may be specified in rules to be made by the President.
Q.10) Consider the following in regard to President of India:
1. A person to be eligible for election as President should have completed 35 years of age,
i.e, should be qualified for election as a member of the second chamber.
2. The salary and allowances of the president are charged from Consolidated Fund of India.
3. President may be elected for any number of terms

Which of the statements given above is/are correct?


a) 1 and 2 only
b) 2 and 3 only
c) 1 and 3 only
d) All of the above
Q.10) Solution (b)
A person to be eligible for election as President should have completed 35 years of age, i.e,
should be qualified for election as a member of the Lok Sabha (not Rajya Sabha).

Q.11) Which among the following is/are correct statements in regard to Council of ministers
and Cabinet?
1. Cabinet functions are determined by the Council of ministers
2. Council of ministers supervises the implementation of its decisions by the cabinet
www.iasbaba.com

Page 8

IASbaba 60 Day Plan- Prelims Test 2016 POLITY & CURRENT AFFAIRS [DAY 25]

Choose the appropriate code:


a) 1 only
b) 2 only
c) Both 1 and 2
d) Neither 1 nor 2
Q.11) Solution (d)
Council of ministers functions are determined by the cabinet
Council of ministers implements the decisions taken by the cabinet. Cabinet supervises
the implementation of its decisions by the council of ministers
Q.12) Consider the following statements in regard to Council of Ministers:
1. When the Lok Sabha passes a no-confidence motion against the council of ministers, all
the ministers have to resign including those ministers who are from the Rajya Sabha.
2. The council of ministers can advise the president to dissolve the Lok Sabha on the
ground that the House does not represent the views of the electorate faithfully and call
for fresh elections.
3. It is the duty of every minister to stand by cabinet decisions and support them both
within and outside the Parliament. If any minister disagrees with a cabinet decision and
is not prepared to defend it, he must resign.
4. President can remove a minister even at a time when the council of ministers enjoys the
confidence of the Lok Sabha.
Which of the statements given above is/are correct?
a)
b)
c)
d)

3 and 4 only
2, 3 and 4 only
1, 2 and 3 only
All of the above

Q.12) Solution (d)


Explanation:
All the given statements are correct Self-explanatory
Q.13) Who among the following hold office during the pleasure of the President?
www.iasbaba.com

Page 9

IASbaba 60 Day Plan- Prelims Test 2016 POLITY & CURRENT AFFAIRS [DAY 25]

1)
2)
3)
4)

Attorney General
Prime Minister
Governor
Council of Ministers

Choose the appropriate code:


a) 1 only
b) 1, 2 and 3 only
c) 1 and 3 only
d) All of the above
Q.13) Solution (d)
Q.14) Which provision is regarded as the foundation of the Parliamentary system of
government?
a) Interdependence between Legislature and Executive organs
b) The council of ministers shall be collectively responsible to the Lok Sabha
c) The executive power of the Union shall be vested in President and shall be exercised by
him either directly or through officers subordinate to him
d) Lok Sabha is elected directly by the people

Q.14) Solution (b)

Q.15) Consider the below statements in regard to the President of India:


1. President is the supreme executive authority of the Central government and chief
coordinator of Central administration.
2. He is the supreme commander of the defence forces of India.
3. In that capacity, he appoints the chiefs of the Army, the Navy and the Air Force and can
declare war or conclude peace, subject to the approval of the Parliament.
Which of the statements given above is/are correct?

www.iasbaba.com

Page 10

IASbaba 60 Day Plan- Prelims Test 2016 POLITY & CURRENT AFFAIRS [DAY 25]

a)
b)
c)
d)

1 and 3 only
2 and 3 only
1 and 2 only
None

Q.15) Solution (b)


Explanation:
Statement 1 is not correct - Cabinet is the supreme executive authority of the Central
government and chief coordinator of Central administration.

Q.16) Consider the following:


Term
1.
2.
3.
4.

Pardon
Reprieve
Respite
Commutation

Meaning
Completely relief from any punishment
Temporary suspension of any punishment
Awarding less sentence
Changing one punishment to another

Which of the above terms are correctly matched?


a)
b)
c)
d)

1 and 3
2 and 4
1, 2 and 4
All of the above

Q.16) Solution (d)


A President is empowered with the power to pardon under Article 72 of the Indian
Constitution. Article 72 says that the President shall have the power to grant pardons,
reprieves, respites or remissions of punishment or to suspend, remit or commute the sentence
of any person convicted of any offence. The meaning of these terms is as follows:
Pardon: Complete pardon
Reprieve: Temporary suspension of sentence
Respite: awarding less sentence
www.iasbaba.com

Page 11

IASbaba 60 Day Plan- Prelims Test 2016 POLITY & CURRENT AFFAIRS [DAY 25]

Remission: Reducing amount of sentence


Commutation: Changing one punishment to another.

Q.17) Consider the following statements regarding impeachment of the President of India:
1. The President of India can be impeached from the office for violation of Constitution,
insolvency and using public office for private gains.
2. A motion of impeachment can be initiated only in Loksabha.
3. The proposal to prefer such charge is contained in a resolution which has been moved
after at least fourteen days' notice in writing, signed by not less than one-third of the
total number of members of the House, has been given of their intention to move the
resolution.
4. If the resolution is passed by a special majority (two thirds of present and voting), the
President stands impeached from his office.
Which of the above statements is/are incorrect regarding the impeachment of the President
of India?
a)
b)
c)
d)

1 and 2
4 only
None of the above
All of the above

Q.17) Solution (d)


All of the above statements are incorrect regarding the impeachment of the President of India.
Procedure for impeachment of the President
(1) When a President is to be impeached for violation of the Constitution, the charge shall be
preferred by either House of Parliament.
(2) No such charge shall be preferred unless(a) The proposal to prefer such charge is contained in a resolution which has been
moved after at least fourteen days' notice in writing, signed by not less than one-fourth
www.iasbaba.com

Page 12

IASbaba 60 Day Plan- Prelims Test 2016 POLITY & CURRENT AFFAIRS [DAY 25]

of the total number of members of the House, has been given of their intention to move
the resolution, and
(b) Such resolution has been passed by a majority of not less than two-thirds of the total
membership of the House.
(3) When a charge has been so preferred by either House of Parliament, the other House shall
investigate the charge or cause the charge to be investigated and the President shall have the
right to appear and to be represented at such investigation.
(4) If as a result of the investigation a resolution is passed by a majority of not less than twothirds of the total membership of the House by which the charge was investigated or caused to
be investigated, declaring that the charge preferred against the President has been sustained,
such resolution shall have the effect of removing the President from his office as from the date
on which the resolution is so passed.

Q.18) The Ordinance mechanism has been devised to enable the executive to deal with a
situation that may suddenly and immediately arise when the Parliament is not in session
(either one or both houses). Which of the following statements are correct regarding
Ordinance making in India?
1. Article 123 of Indian Constitution empowers the president to promulgate Ordinances.
2. Maximum life of an ordinance can be 6 months and 6 weeks.
3. An Ordinance can only be promulgated on the subjects on which Parliament is
competent to make laws.
Select the code from the following:
a)
b)
c)
d)

1 and 2
2 and 3
1 and 3
All of the above

Q.18) Solution (d)


www.iasbaba.com

Page 13

IASbaba 60 Day Plan- Prelims Test 2016 POLITY & CURRENT AFFAIRS [DAY 25]

Article 123 of the Indian constitution empowers the President to promulgate ordinances.
An Ordinance promulgated under this article shall have the same force and effect as an Act of
Parliament, but every such Ordinance
Shall be laid before both Houses of Parliament and shall cease to operate at the expiration of
six weeks from the reassembly of Parliament, or, if before the expiration of that period
resolution disapproving it are passed by both Houses, upon the passing of the second of those
resolutions. As maximum gap between two session of a house can be 6 months therefore
maximum life of a ordinance can be 6 months and 6 weeks; and May be withdrawn at any time
by the President.

Q.19) Article 74(1) requires the President to have a Council of Ministers with the Prime
Minister at the head to aid and advice him in the exercise of his power. Which of the
following statements regarding Council of Ministers are correct?
1. Ministers are appointed by the Prime Minister.
2. India has a three-tier ministry consisting of cabinet ministers, ministers of state and the
deputy ministers.
3. The term Cabinet is not present in the original Constitution.
Select the code from the following:
a)
b)
c)
d)

1 and 2
2 and 3
1 and 3
All of the above

Q.19) Solution (b)


Ministers are appointed by the President on the advice of the Prime Minister.
www.iasbaba.com

Page 14

IASbaba 60 Day Plan- Prelims Test 2016 POLITY & CURRENT AFFAIRS [DAY 25]

Q.20) Consider the following statements with respect to Swarna Bharat Yojana
1. The scheme is aimed to decrease gold imports by mobilising gold in the domestic
market
2. Gold monetisation, Indian gold coins, Gold bond scheme are some of the ancillary
schemes related to Swarna Bharat Yojana
Select the correct answer using the codes given below
a)
b)
c)
d)

1 only
2 only
Both
None

Q.20) Solution (c)


Both the statements are true
http://timesofindia.indiatimes.com/india/Now-Swarna-Bharat-campaign-by-centralgovernment/articleshow/49660918.cms

Q.21) Swasth Dharaa Khet Haraa is a slogan of which of the following scheme by Government
of India
a)
b)
c)
d)

Pradhan Mantri Krishi sinchana Yojana


Pradhan Mantri Fasal Bima Yojana
Rashtriya Krishi Vikas Yojana
Soil health card scheme

Q.21) Solution (d)


Soil health card scheme has the slogan of Swasth Dharaa Khet Haraa

Q.22) Consider the following statements with respect to World Wild Life Fund
www.iasbaba.com

Page 15

IASbaba 60 Day Plan- Prelims Test 2016 POLITY & CURRENT AFFAIRS [DAY 25]

1. The fund is created under the aegis of United Nations environment programme
2. Panda is the official logo of world wildlife fund
Select the correct option
a)
b)
c)
d)

1 only
2 only
Both
None

Q.22) Solution (b)


WWF is not created under UNEP,
World Wildlife Fund (WWF) and its Aims
When Sir Julian Huxley, the renowned British biologist, went to Africa for a research trip. What
he did sea there, shook him completely. He was dismayed by the unclenched destruction of
nature and the hunting of endangered species.
When Sir Huxley returned in 1960, he shared his concern, that many species would become
extinct it the condition persists in Africa, with fellow scientist and other prominent
personalities. Finally they agreed to begin an international organization with the scientific,
technical and financial resources to conduct and fund conservation efforts across the globe.

In 1961, World Wildlife Fund (WWF) was created. Sir Huxley and Sir Peter Scott, the famous
ornithologist who created the Panda logo, were among the founder members of World Wildlife
Fund.
The aims of the World Wildlife Fund are to halt the destruction of the earths natural
environment and to prepare a future in which all human beings can live in harmony with
nature. World Wildlife Fund does this by protecting all forms of life on earth, and making sure
that the earths natural resources like water and forests are not consumed faster than they can
be renewed. World Wildlife Fund encourages the people to curtail the amount of waste they
normally produce, and to cut down on pollution.

www.iasbaba.com

Page 16

IASbaba 60 Day Plan- Prelims Test 2016 POLITY & CURRENT AFFAIRS [DAY 25]

http://www.thehindu.com/sci-tech/energy-and-environment/global-tiger-population-up-by-22per-cent/article8461131.ece

Q.23) Shyam Benegal Committee is related to which of the following subjects


a)
b)
c)
d)

Tribal alienation
Public distribution system
Film certification
Regulation of MSME

Q.23) Solution (c)


Shyam Benegal committee on film certification
Shyam Benegal committee was set up by the Union Government in January 2016 to lay down
norms for film certification to give give sufficient and adequate space for artistic and creative
expression. The committee was asked to take note of best practices in various parts of the
world.
The committee was also asked to lay down procedures and guidelines for the benefit of the
Central Board of Film Certification (CBFC) to follow and examine staffing patterns with a view to
recommending a framework that would provide efficient and transparent user friendly services.
Recommendations
1. CBFC as a Certification Body
The CBFC should only be a film certification body. Its scope should be restricted to categorizing
the suitability of the film to audience groups on the basis of age and maturity.
2. Refusing Certification
The CBFC should refuse certification only when a film contains anything that contravenes the
provisions of Section 5B (1) of the Cinematograph Act, 1952 or when its content crosses the
ceiling laid down in the highest category of certification.
3. Categorisation of Films

www.iasbaba.com

Page 17

IASbaba 60 Day Plan- Prelims Test 2016 POLITY & CURRENT AFFAIRS [DAY 25]

The committee recommended that the categorisation of films should be more specific. The UA
Category can be broken up into further sub-categories UA12+ and UA15+.
The A category should also be sub-divided into A and AC (Adult with Caution) categories.
http://www.thehindu.com/news/national/shyam-benegal-to-head-panel-to-revamp-censorboard/article8054774.ece

Q.24) Chaitra Vaishaka months in traditional Indian calendar corresponds to which of the
following months in English calendar?
a)
b)
c)
d)

March-April
May-June
Jan-Feb
April-May

Q.24) Solution (a)


It refers to March and April
Additional Value Addition
There are 12 months in Hindu lunar Calendar:

Chaitra (30 / 31* Days) Begins March 22 / 21*


Vaisakha (31 Days) Begins April 21
Jyaistha (31 Days) Begins May 22
Asadha (31 Days) Begins June 22
Shravana (31 Days) Begins July 23
Bhadra (31 Days) Begins August 23
Asvina (30 Days) Begins September 23
Kartika (30 Days) Begins October 23
Agrahayana (30 Days) Begins November 22
Pausa (30 Days) Begins December 22
Magha (30 Days) Begins January 21
Phalguna (30 Days) Begins February 20 Leap years

www.iasbaba.com

Page 18

IASbaba 60 Day Plan- Prelims Test 2016 POLITY & CURRENT AFFAIRS [DAY 25]

FESTIVALS/RITUALS

1. Chaitra

The month of Chaitra is also associated with the coming of Spring, since Holi, the spring festival
of colour, is celebrated on the eve of Chaitra (namely, the last day of Phalgun month). Exactly 6
days after which the festival of Chaiti Chhath is observed.
In lunar religious calendars, Chaitra begins with the new moon in March/April and is the first
month of the year. The first of Chaitra is celebrated as New Year's Day, known as Gudi Padwa
in Maharashtra, Chaitrai Vishu and Ugadi in Karnataka and Andhra Pradesh.
Other important festivals in the month are; Ram Navami, the birth anniversary of Lord Ram
celebrated on the 9th day of Chaitra, and Hanuman Jayanti that falls on the last day (purnima)
of Chaitra.

2. Vaikha
The harvest festival of (Baisakhi) is celebrated in this month. Vaisakha Purnima is celebrated as
Buddha Purnima or the birthday of Gautama Buddha amongst southern Buddhists or the
Theravada school. Purnima refers to the Full Moon. Known in Sinhalese as Vesak, it is observed
in the full moon of May

3. Jyaiha

www.iasbaba.com

Page 19

IASbaba 60 Day Plan- Prelims Test 2016 POLITY & CURRENT AFFAIRS [DAY 25]

Vat Pournima is a celebration observed in Maharashtra and Karnataka, India. It is celebrated on


the full moon day (the 15th) of the month of Jyeshtha on the Hindu Calendar, which falls in
June on the Gregorian Calendar. Women pray for their husbands by tying threads around a
banyan tree on this day. It honors Savitri, the legendary wife of Satyavan who escaped death
for her husband's life.

Snana Yatra is a bathing festival celebrated on the Purnima the Hindu month of Jyeshtha. It is
an important festival of the Jagannath Cult. The deities Jagannath, Balabhadra, Subhadra,
Sudarshan, and Madanmohan are brought out from the Jagannath Temple (Puri) and taken in a
procession to the Snana Bedi. They are ceremonially bathed and decorated for a public
audience.
Sitalsasthi Carnival is being conducted in this month on the day of Jyeshtha Shuddha Shashthi in
Odisha for many centuries

4. Asadha

Guru Purnima, a festival dedicated to the Guru, is celebrated on the Purnima (Full Moon) day of
the month. Prior to it Shayani Ekadashi, is observed on the eleventh lunar day (Ekadashi) of the
bright fortnight.

5. Sravana

Shravana(jupaka) is considered to be a holy month in the Hindu calendar due to the many
festivals that are celebrated during this time. Krishna Janmashtami, marking the birth of
Krishna, falls on the 8th day after the full moon. Raksha Bandhan, the festival of brothers and
sisters, is celebrated on Shraavana Poornima (Full Moon). This day in Maharashtra is also
celebrated as Narali Poornima (Naral in Marathi language means coconut). In the coastal
regions of Maharashtra i.e. Konkan, a coconut is offered to the sea for calming it down after the
monsoon season. Fishermen now start fishing in the sea after this ceremony. Nag Panchami is
also celebrated in many parts of India on the fifth day after Amavasya of Shraavana month. The
www.iasbaba.com

Page 20

IASbaba 60 Day Plan- Prelims Test 2016 POLITY & CURRENT AFFAIRS [DAY 25]

snake god Nga is worshiped. The last day of the Shraavana is celebrated as Pola, where the
bull is worshiped by farmers from Maharashtra.

In TamilNadu (& also in Kerala) Aadi Amavasaya is celebrated with great importance in all
temples. It is an equivalent to Mahalaya Amavasaya of north India.In
KarnatakaBasava Panchami is celebrated on 5th day after amavasya.
Shravani Mela is a major festival time at Deoghar in Jharkhand with thousands of saffron-clad
pilgrims bringing holy water around 100 km on foot from the Ganges at Sultanganj.Shravan is
also the time of the annual Kanwar Yatra, the annual pilgrimage of devotees of Shiva, known
as Kanwaria make to Hindu pilgrimage places of Haridwar, Gaumukh and Gangotri in
Uttarakhand to fetch holy waters of Ganges River

6. Bhdrapada or Bhdra also Prohapada

Anant Chaturdashi is a Jain religious observance is performed on the fourteenth day


(Chaturdashi) of the bright fortnight (Shukla paksha) of Bhadrapad month.
Madhu Purnima (Bengali for 'honey full-moon') is a Buddhist festival celebrated in India and
Bangladesh, especially in the region of Chittagong. It occurs on the day of the full moon in the
month of Bhadro (August/September).

7. Asvina

Several major religious holidays take place in Ashvin, including Durga Puja (6-10 Ashvin),
Dasehra (10 Ashvin) and Divali (29 Ashvin), Kojagiri festivals and Kali Puja (new moon of
Ashvin),
8. Kartika

The festival of Kartik Poornima (15th day Full Moon) falls in this month, celebrated asDev
Deepavali in Varanasi. This coincides with the nirvana of the Jain Tirthankara - Mahavira and the
www.iasbaba.com

Page 21

IASbaba 60 Day Plan- Prelims Test 2016 POLITY & CURRENT AFFAIRS [DAY 25]

birth of the Sikh Guru Nanak Guru Nanak Jayanti. And also, the well known festival, for the god
of Sabarimalai, Ayyappan's garland festival.

9. Agrahyaa

Vaikuha Ekda, the Ekda (i.e. 11th lunar day) of this Mrgaa month, is celebrated also
as Mokad Ekda. The 10th Canto, 22nd Chapter of Bhgavata Pura, mentions young
marriageable daughters (gopis) of the cowherd men of Gokula, worshiping Goddess Ktyyan
and taking a vrata or vow, during the entire month of Mrgaa, the first month of the winter
season (iira), to get r Ka as their husband.
Klabhairava Aam (or Klabhairava Jayanti) falls on Ka Paka Aam of this month of
Mrgaa. On this day it is said that Lord iva appeared on earth in the fierce manifestation
(avatra) as r Klabhairava. This day is commemorated with special prayers and rituals.

10. Pausa
The harvest festival of Pongal/Makar Sankranti is celebrated on this month.

11. Magha

Vasant Panchami, sometimes referred to as Saraswati Puja, Shree Panchami, or the Festival of
Kites is a Sikh and Hindu festival held on the fifth day of Magha (in early February) marking the
start of spring and the Holi season. On this day Hindus worship Saraswati, the goddess of
knowledge, music, art and culture.
Ratha Saptami or Rathasapthami is a Hindu festival that falls on the seventh day (Saptami) in
the bright half (Shukla Paksha) of the Hindu month Maagha. It marks the seventh day following
the Suns northerly movement (Uttarayana) of vernal equinox starting from Capricorn (Makara).

12. Phalguna

www.iasbaba.com

Page 22

IASbaba 60 Day Plan- Prelims Test 2016 POLITY & CURRENT AFFAIRS [DAY 25]

Most parts of North India see early celebration of the famous Hindu festival Holi in this month.
Holi is celebrated at the end of the winter season on the last full moon day of the lunar month
Phalguna (Phalguna Purnima), which usually falls in the later part of February or March. The
Hindu festival of Shigmo is also celebrated in Goa and Konkan in the month of Phalguna.
Celebrations can stretch over a month.

Source: Hindu calendar

Q.25) Consider the following regarding The Convention on the Rights of the Child
1. It is a legally binding treaty under the ambit of United Nations.
2. According to the convention a child is any person who has not reached the age of
sixteen unless a different age of maturity is specified in any country's law.
3. It talks about civil, political, economic, social, health and cultural rights of children.
Select the correct code
a)
b)
c)
d)

1 and 2
2 and 3
1 and 3
1, 2 and 3

Q.25) Solution (c)


http://www.thehindu.com/news/national/world-day-against-child-labour-has-there-been-achange/article8720756.ece

http://www.childlineindia.org.in/united-nations-convention-on-the-rights-of-the-child.htm
https://en.wikipedia.org/wiki/Convention_on_the_Rights_of_the_Child
World Day Against Child Labour: 12 June

www.iasbaba.com

Page 23

IASbaba 60 Day Plan- Prelims Test 2016 POLITY & CURRENT AFFAIRS [DAY 25]

www.iasbaba.com

Page 24

IASbaba 60 Day Plan- Prelims Test 2016 POLITY & CURRENT AFFAIRS [DAY 26]

Q.1) Consider the following statements in regard to the conditions of Governors office:
1. Governor is entitled to such emoluments, allowances and privileges as may be
determined by Parliament.
2. However, when the same person is appointed as the governor of two or more states,
the emoluments and allowances are determined by the president.
Which of the statements given above is/are correct?
a) 1 only
b) 2 only
c) Both
d) None
Q.1) Solution (a)
Explanation:
Governor is entitled to such emoluments, allowances and privileges as may be determined
by Parliament.
However, when the same person is appointed as the governor of two or more states, the
emoluments and allowances payable to him are shared by the states in such proportion as
determined by the president.

Q.2) Consider the following statements in regard to the appointment and oath of the
Governor:
1. Governor should be an outsider, that is, he should not belong to the state where he is
appointed.
2. While appointing the governor, the president is required to consult the chief minister of
the state concerned and the latters advice is binding.
3. The oath of office to the governor is administered by the President or some person
appointed in that behalf by him.
Which of the statements given above is/are correct?
a) 1 only
b) 1 and 2 only
c) 1 and 3 only
www.iasbaba.com

Page 1

IASbaba 60 Day Plan- Prelims Test 2016 POLITY & CURRENT AFFAIRS [DAY 26]

d) 3 only

Q.2) Solution (a)


Explanation:
While appointing the governor, the president is required to consult the chief minister of the
state concerned and the latters advice is not binding.
The oath of office to the governor is administered by the chief justice of the concerned
state High Court and in his absence, the senior-most judge of that court available.

Q.3) Consider the following statements:


1. State council of ministers advises the governor with regard to the appointment of
important officials like advocate general, chairman and members of the state public
service commission, state election commissioner, and so on.
2. Presiding officer announces the government policies on the floor of the house.
Which of the statements given above is/are incorrect?
a)
b)
c)
d)

1 only
2 only
Both
None

Q.3) Solution (c)


Explanation:
Chief Minister advises the governor with regard to the appointment of important officials
like advocate general, chairman and members of the state public service commission, state
election commissioner, and so on.
Chief Minister announces the government policies on the floor of the house.

Q.4) Which among the following given statements is/are the constitutional discretion that
Governor exercise?
www.iasbaba.com

Page 2

IASbaba 60 Day Plan- Prelims Test 2016 POLITY & CURRENT AFFAIRS [DAY 26]

1. Determining the amount payable by the Government of Assam, Meghalaya, Tripura and
Mizoram to an autonomous Tribal District Council as royalty accruing from licenses for
mineral exploration
2. Seeking information from the chief minister with regard to the administrative and
legislative matters of the state
3. The governor has constitutional discretion while exercising his functions as the
administrator of an adjoining union territory
Choose the appropriate code:
a) 1 and 2 only
b) 3 only
c) 1 and 3 only
d) All of the above
Q.4) Solution (d)
Explanation:
All the given statements are correct Self explanatory
If any dispute arises as to the share of such royalties to be made over to a district council, it
shall be referred to the governor for determination and the amount determined by the
governor in his discretion shall be deemed to be the amount payable to the district council
and the decision of the governor shall be final
Q.5) Consider the following statements in regard to the Chief Minister of a State:
1. CM is the chief spokesman of the state government
2. He acts as a chairman of the concerned zonal council by rotation, holding office for a
period of one year at a time
3. He is the member of the Inter-State Council and the National Development Council,
both headed by the prime minister
Which of the statements given above is/are correct?
a)
b)
c)
d)

1 and 2 only
1 and 3 only
2 and 3 only
All of the above

www.iasbaba.com

Page 3

IASbaba 60 Day Plan- Prelims Test 2016 POLITY & CURRENT AFFAIRS [DAY 26]

Q.5) Solution (b)


Explanation:
Union home minister is the chairman of all the zonal councils.
CM acts as a vice-chairman of the concerned zonal council by rotation, holding office for a
period of one year at a time

Q.6) Which among the below given statements is/are correct in regard to Governor?
1. The term of the Governor is not fixed by the Constitution and he holds office during the
pleasure of the President
2. Also the Constitution does not lay down any grounds upon which a governor may be
removed by the President
Select the correct code:
a) 1 only
b) 2 only
c) Both
d) None

Q.6) Solution (b)


Explanation:
A governor holds office for a term of five years from the date on which he enters upon his
office. However, this term of five years is subject to the pleasure of the President. Further,
he can resign at any time by addressing a resignation letter to the President.
Q.7) Bicameral legislatures have been provided in some states under the Constitution. In case
of a deadlock between the two Houses in such States
a) Joint sitting is called by the Governor and the decision by the majority is taken as final
decision
b) Decision of the Governor is final
c) The opinion of the Legislative Assembly is taken as final after a lapse of specified period
d) The matter is referred to the President for decision
Q.7) Solution (c)
www.iasbaba.com

Page 4

IASbaba 60 Day Plan- Prelims Test 2016 POLITY & CURRENT AFFAIRS [DAY 26]

Explanation:
Self explanatory
Q.8) Which among the following are the powers of the Governor?
1. He appoints the vice-chancellors of universities in the state
2. He appoints the chief minister and other ministers including Tribal Welfare minister in
the states of Chattisgarh, Jharkhand, Madhya Pradesh and Odisha
3. He appoints the state election commissioner and determines his conditions of service
and tenure of office
4. He decides on the question of removal of members of the state election commission in
consultation with the Election Commission
Choose the appropriate code:
a) 1 and 2 only
b) 1 and 4 only
c) 1, 2 and 3 only
d) All of the above
Q.8) Solution (c)
Explanation:
He appoints the state election commissioner and determines his conditions of service and
tenure of office. However, the state election commissioner can be removed only in like
manner and on the like grounds as a judge of a high court.
Q.9) Consider the following statements in regard to the strength of the state council of
ministers:
1. The total number of ministers, including the chief minister, in the council of ministers in
a state shall not exceed 15 per cent of the total strength of the legislative assembly of
that state.
2. But, the number of ministers, including the chief minister, in a state shall not be less
than 12.
Which of the statements given above is/are not correct?
a) 1 only
b) 2 only
c) Both
www.iasbaba.com

Page 5

IASbaba 60 Day Plan- Prelims Test 2016 POLITY & CURRENT AFFAIRS [DAY 26]

d) None
Q.9) Solution (d)
Explanation:
Both the given statements are correct

Q.10) Governor has certain special responsibilities to discharge according to the directions
issued by the President. In which among the following states he can be directed to discharge
special responsibilities?
1. Karnataka
2. Telangana
3. Gujarat
4. Sikkim
5. Assam
Choose the appropriate code:
a) 1, 3, 4 and 5 only
b) 4 and 5 only
c) 1, 3 and 4 only
d) 1, 2 and 3 only
Q.10) Solution (a)
Explanation:
Governor has certain special responsibilities to discharge according to the directions issued
by the President. They are given below:
1. MaharashtraEstablishment of separate development boards for Vidarbha and
Marathwada.
2. GujaratEstablishment of separate development boards for Saurashtra and Kutch.
3. NagalandWith respect to law and order in the state for so long as the internal disturbance
in the Naga HillsTuensang Area continues.
4. AssamWith respect to the administration of tribal areas.
5. ManipurRegarding the administration of the hill areas in the state.
www.iasbaba.com

Page 6

IASbaba 60 Day Plan- Prelims Test 2016 POLITY & CURRENT AFFAIRS [DAY 26]

6. SikkimFor peace and for ensuring social and economic advancement of the different
sections of the population.
7. Arunachal PradeshWith respect to law and order in the state.
8. Karnataka Establishment of a separate development board for Hyderabad-Karnataka
region
Q.11) Consider the following statements in regard to the Governor:
1. He is the constitutional head of the state as well as the representative of the Centre
2. He is the chairman of the State Planning Board
3. He acts as the chancellor of universities in the state
4. He is the political head of the services
Which of the statements given above is/are correct?
a) All of the above
b) 1, 3 and 4 only
c) 1 and 3 only
d) 1 only
Q.11) Solution (c)
Explanation:
Chief Minister is the chairman of the State Planning Board and also the political head of the
services
Q.12) Which among the below given statements is/are correct with regard to legislative
councils?
1. The Constitution provides for the abolition or creation of legislative councils in states
2. Accordingly, the Parliament can abolish a legislative council or create it, if the legislative
assembly of the concerned state passes a resolution to that effect
3. Such a specific resolution must be passed by the state assembly by a special majority
4. This Act of Parliament is not to be deemed as an amendment of the Constitution for the
purposes of Article 368 and is passed like an ordinary piece of legislation (i.e, by simple
majority)
Select the appropriate code:
a) 1, 3 and 4
b) 2 and 3
www.iasbaba.com

Page 7

IASbaba 60 Day Plan- Prelims Test 2016 POLITY & CURRENT AFFAIRS [DAY 26]

c) 1, 2 and 3
d) All of the above
Q.12) Solution (d)
Explanation:
All the given statements are correct and self-explantory
Q.13) Which among the following statements is/are true in regard to Governor?
1. Governor is consulted by the president while appointing the judges of the concerned
state high court.
2. Governor appoints persons to the judicial service of the state (including district judges)
in consultation with the state high court and the State Public Service Commission.

Select the correct code:


a)
b)
c)
d)

1 only
2 only
Both
None

Q.13) Solution (a)


Explanation:
Governor appoints persons to the judicial service of the state (other than district judges) in
consultation with the state high court and the State Public Service Commission.

Q.14) Which among the below given statements is/are correct in regard to state council of
ministers and central council of ministers?
1. Unlike the Centre, there is no provision in the Constitution for the system of legal
responsibility of the minister in the states.
2. Like at the Centre, in the states too, the council of ministers consists of three categories
of ministers, namely, cabinet ministers, ministers of state, and deputy ministers.

www.iasbaba.com

Page 8

IASbaba 60 Day Plan- Prelims Test 2016 POLITY & CURRENT AFFAIRS [DAY 26]

3. The difference between them lies in their respective ranks, emoluments, and political
importance.
Choose the appropriate code:
a) 1 and 2 only
b) 2 and 3 only
c) 1 and 3 only
d) All of the above
Q.14) Solution (b)
Explanation:
As at the Centre, there is no provision in the Constitution for the system of legal
responsibility of the minister in the states.

Q.15) Consider the following statements in regard to Deputy Ministers:


1. Deputy Ministers can either be given independent charge of departments or can be
attached to cabinet ministers.
2. However, they are not members of the cabinet and do not attend the cabinet meetings
unless specially invited when something related to their departments are considered by
the cabinet.
Which of the statements given above is/are correct?
a)
b)
c)
d)

1 only
2 only
Both
None

Q.15) Solution (d)


Explanation:

www.iasbaba.com

Page 9

IASbaba 60 Day Plan- Prelims Test 2016 POLITY & CURRENT AFFAIRS [DAY 26]

Deputy Ministers are not given independent charge of departments. They are attached to
the cabinet ministers and assist them in their administrative, political and parliamentary
duties. They are not members of the cabinet and do not attend cabinet meetings.

Q.16) In which of the following Union Territories, the administrator of the UT is called a Lt.
Governor?
1.
2.
3.
4.
5.

Andaman and Nicobar Islands


Daman and Diu
Delhi
Pondicherry
Lakshadweep Islands

Select the code from the following:


a)
b)
c)
d)

3 and 4
1,3 and 4
3,4 and 5
All of the above

Q.16) Solution (a)


The Union Territories are administered by the President through an administrator, who is
appointed by him with a suitable designation. This designation is called either Lieutenant
Governor or Chief Commissioner or Administrator. In Andaman & Nicobar Islands, Puducherry
and Delhi, administrator is called Lt. Governor, while in Chandigarh, Dadra & Nagar Haveli,
Daman & Diu and Lakshadweep he/ she is known as Administrator. The President may appoint
a Governor of an adjoining state as administrator of a Union territory. In such case the
Governor works independently with regard to the administration of the Union Territory.
Q.17) Consider the following statements regarding the Writ Jurisdiction of High courts:
1. Writs come under the Original jurisdiction of High Court.
2. High Court has been empowered to issue writs for the enforcement of Fundamental
Rights and for other purposes.

www.iasbaba.com

Page 10

IASbaba 60 Day Plan- Prelims Test 2016 POLITY & CURRENT AFFAIRS [DAY 26]

Which of the above statements are correct?


a)
b)
c)
d)

1 only
2 only
Both 1 and 2
Neither 1 nor 2

Q.17) Solution (c)


Writs come under the original jurisdiction of Supreme court as well as the High Court. There is
something called an original jurisdiction and Original and exclusive jurisdiction. This is original
but not exclusive.
The High Court has been empowered to issue writs of habeas corpus, mandamus, and
prohibition certiorari and quo warranto for the enforcement of the fundamental rights and for
other purposes. The Supreme Court can issue the writs only for the enforcement of
fundamental rights and not for other purposes. The power of the High Court to issue writs in
the nature of habeas corpus cannot be curtailed even during emergency.

Q.18) Consider the following statements:


1. It is constitutionally mandatory for the President to consult the Chief Minister of a
respective state before appointing a Governor.
2. The Governor can be removed before his term by the Legislative Assembly of a state by
the process of impeachment.
3. A governor may be simultaneously assigned more than one states.
Which of the above statements are incorrect?
a)
b)
c)
d)

1 and 2
2 and 3
1 and 3
All of the above

Q.18) Solution (a)

www.iasbaba.com

Page 11

IASbaba 60 Day Plan- Prelims Test 2016 POLITY & CURRENT AFFAIRS [DAY 26]

A Governor is appointed on the advice of the Union Council of Ministers, or in reality on the
advice of the Prime Minister. For the President to consult the Chief Minister of the concerned
state, before the appointment of a Governor is not a constitutional requirement. But a healthy
convention grew up that the Chief Minister was consulted. But in the case of appointment of
several Governors, this convention has not been obeyed.
A Governor may be simultaneously assignee to more than one state. Thus, on many occasions,
the Governor of Assam was also the Governor of Meghalaya, Nagaland, Tripura, etc.
A Governor is appointed for a period of 5 years. The President, if he so pleases may extend his
tenure. A vacancy in the Governors post arises, if:

(a) when the Governor completes his tenure,


(b) resigns, (c) dies in office, and
(d) is removed from office by the President.

The Governors also have often been transferred from one state to another.

Q.19) Consider the following statements:


1. According to the preamble of the Constitution of Kashmir, Kashmir is a temporary part
of India.
2. In case of J and K, the residuary powers belong to the states legislature.
3. According to Indian Constitution, article 370 is a temporary provision granting special
autonomous status to Jammu and Kashmir.
Which of the above statements are correct?
a)
b)
c)
d)

1 and 2
2 and 3
1 and 3
All of the above

Q.19) Solution (b)


According to the constitution of Kashmir, Kashmir is an integral part of India.

www.iasbaba.com

Page 12

IASbaba 60 Day Plan- Prelims Test 2016 POLITY & CURRENT AFFAIRS [DAY 26]

Q.20) Consider the following statements:


1. State Election Commission is responsible for conducting the elections for Legislative
Assemblies.
2. Election Commission appoints tribunals for the decision of doubts and disputes arising
out of or in connection with election to parliament and State Legislatures.
3. Only two Union Territories have Legislative Assemblies.
Which of the above statements are correct?
a)
b)
c)
d)

1 and 2
2 and 3
1 and 3
All of the above

Q.20) Solution (b)


The main duties of the Election Commission are:
(1) To superintend, direct, control and conduct all elections to Parliament and State Legislatures
as also to the office of the President and Vice- President of India.
(2) to lay down general rules for election.
(3) to determine constituencies and to prepare electoral rolls.
(4) to give recognition to political parties.
(5) to allot election symbols to different political parties and individual contestants.
(6) to appoint tribunals for the decision of doubts and disputes arising out of or in connection
with election to parliament and State Legislatures.
Delhi and Pondicherry are the only two Union Territories to have Legislative Assemblies.

www.iasbaba.com

Page 13

IASbaba 60 Day Plan- Prelims Test 2016 POLITY & CURRENT AFFAIRS [DAY 26]

Q.21) Consider the following countries


1. Bulgaria
2. Albania
3. Romania
Which of the above listed country is not bordering Black Sea?
a)
b)
c)
d)

1 and 2 only
2 only
1 and 3 only
2 and 3 only

Q.21) Solution (b)


Albania is not bordering Black Sea

Q.22) Consider the following statements with respect to ParamParagt Krishi Vikas Yojana
1. The scheme follows cluster based approach of 50 farmers to promote organic farming
2. Each farmer will be provided 20000 rupees per acre in three years from seed to
harvesting crops and to transport them to market
Select the correct answer using the codes given below
a)
b)
c)
d)

1 only
2 only
Both
None

Q.22) Solution (c)

Click here

www.iasbaba.com

Page 14

IASbaba 60 Day Plan- Prelims Test 2016 POLITY & CURRENT AFFAIRS [DAY 26]

Q.23) Consider the following statements with respect to Rashtriya Gokul Mission
1. The scheme focuses on conservation and development of indigenous and exotic breeds.
2. Gokul Grams will be established to develop indigenous breeds under Rashtriya Gokul
mission
Select the correct answer using the codes given below
a)
b)
c)
d)

1 only
2 only
Both
None

Q.23) Solution (b)


The scheme focuses on indigenous breeds only

Q.24) Consider the following statements with respect to Jan Aaushadi scheme
1. The schemes aims to make generic medicines available to BPL and APL families only
2. The scheme is spear headed by Ministry of Health and Family Welfare
Select the correct answer
a)
b)
c)
d)

1 only
2 only
Both
None

Q.24) Solution (d)

The schemes aims to make generic medicines available to ALL citizens of India
irrespective of here economic status
The scheme is sphere headed by ministry of chemicals and fertilisers

www.iasbaba.com

Page 15

IASbaba 60 Day Plan- Prelims Test 2016 POLITY & CURRENT AFFAIRS [DAY 26]

Q.25) Consider the following statements with respect to recently announced Price
Stabilization Fund
1. It functions under Consumer Affairs Ministry
2. The Price Stabilization Fund aims to stabilize the prices of tea, coffee, rubber and
tobacco to contain the distress of the farmers
Select the correct answer
a)
b)
c)
d)

1 only
2 only
Both
None

Q.25) Solution (a)


Click here
Initially the fund was created under ministry of agriculture but recently the government will
shift the Rs 500-crore Price stabilization Fund (PSF) to consumer affairs ministry from
agriculture ministry for effective control of price rise in essential commodities and provide relief
to the consumers.
Last year, the PSF was created with a corpus of Rs 500 crore under the agriculture ministry. The
fund was to be used to support market interventions for managing prices of perishable agrihorticultural commodities by procuring directly from farmers and later supplying at reasonable
rates to consumers.
Click here

www.iasbaba.com

Page 16

IASbaba 60 Day Plan- Prelims Test 2016 GEOGRAPHY & CURRENT AFFAIRS [DAY 27]

Q.1) Economy Survey mentions about Rainbow Revolution. Consider the following w.r.t
it
1.
2.
3.
4.
5.
6.

Food Processing
Fisheries
Forestry
Horticulture
Crop Cultivation
Animal Husbandry

Which of the above are included in the concept of Rainbow Revolution?


a)
b)
c)
d)

2, 3 and 5
1, 3, 4 and 5
1, 2, 3, 5 and 6
1, 2, 3, 4, 5 and 6

Q.1) Solution (d)


The various colors of the Rainbow Revolution indicate various farm practices such as Green
Revolution (Foodgrains), White Revolution (Milk), Yellow Revolution (Oil seeds), Blue
Revolution (Fisheries); Golden Revolution (Fruits); Silver Revolution (Eggs), Round
Revolution (Potato), Pink Revolution (Meat), Grey Revolution (Fertilizers) and so on. Thus,
the concept of Rainbow revolution is an integrated development of crop cultivation,
horticulture, forestry, fishery, poultry, animal husbandry and food processing industry.
Economy Survey- Agriculture, More from Less

Q.2) Consider the statements and choose the incorrect from the following:
a)
b)
c)
d)

India is a net importer of water and China is a net exporter of water


Indias has maximum exports in Rice, Sugar, Cotton and Soyabean
National output of pulses comes predominantly from un-irrigated land
A large share of output in wheat, rice and is from irrigated land

Q.2) Solution (a)


India was a net importer of water until around 1980s. With increases in food grain
exports, India has now become a net exporter of water about 1 per cent of total available
water every year. The ratio of export to import of such virtual water is about 4 for India and
www.iasbaba.com

Page 1

IASbaba 60 Day Plan- Prelims Test 2016 GEOGRAPHY & CURRENT AFFAIRS [DAY 27]

0.1 for China. Thus China remains a net importer of water. This is also evident in China and
Indias trade patterns. China imports water-intensive soybeans, cotton, meat and cereal
grains6, while exporting vegetables, fruits and processed food. India, on the other hand,
exports water-intensive rice, cotton, sugar and soybean.

Economy Survey- Agriculture, More from Less

Q.3) Consider the impact of fall in the Crude Oil Price on Indian Economy
1. Exporters of petroleum producers are negatively impacted
2. It will help to narrow down Current Account Deficit
3. The value of Indian Rupee will strengthen
Select the correct code
a)
b)
c)
d)

1 and 2
2 and 3
1 and 3
1, 2 and 3

Q.3) Solution (d)


Economy Survey - Prices, Agriculture and Food Management

Q.4) Arrange the following countries in decreasing order of Average Cereal Yields
1.
2.
3.
4.
5.
6.

China
USA
India
Bangladesh
Brazil
Canada

Select the correct code


a)
b)
c)
d)

2-1-3-5-6-4
1-2-5-6-3-4
2-1-5-4-6-3
2-1-5-3-6-4

www.iasbaba.com

Page 2

IASbaba 60 Day Plan- Prelims Test 2016 GEOGRAPHY & CURRENT AFFAIRS [DAY 27]

Q.4) Solution (c)


Economy Survey - Prices, Agriculture and Food Management, Fig 5.11

Q.5) Jan Aushadhi Scheme is a direct market intervention scheme of


a)
b)
c)
d)

Ministry of Health and Family Welfare


Ministry of Economic Affairs
Ministry of Chemicals and Fertilizers
Ministry of Science and Technology

Q.5) Solution (c)


The Jan Aushadhi Scheme (Public Medicine Scheme) is a direct market intervention scheme
launched by the Department of Pharmaceuticals, Ministry of Chemicals and Fertilizers,
Govt. of India, to make available quality generic medicines.

Q.6) Which of the following pulses are covered under Minimum Support Price (MSP)?
1.
2.
3.
4.

Arhar
Masur
Urad
Moong

Select the correct code


a)
b)
c)
d)

1 and 2
2, 3 and 4
1, 3 and 4
1, 2, 3 and 4

Q.6) Solution (d)


All are covered

Q.7) Consider the following


1. Accelerated Irrigation Benefit Programme- Ministry of Agriculture
2. Integrated Watershed Management Programme- Ministry of Rural Development
3. Soil Health Card- Ministry of Water Resources
Select the incorrect match
www.iasbaba.com

Page 3

IASbaba 60 Day Plan- Prelims Test 2016 GEOGRAPHY & CURRENT AFFAIRS [DAY 27]

a)
b)
c)
d)

Only
1 and 3
2 and 3
Only 2

Q.7) Solution (b)


Accelerated Irrigation Benefit Programme- Ministry of Water Resources
Integrated Watershed Management Programme- Ministry of Rural Development
Soil Health Card- Ministry of Agriculture

Q.8) Ragi, a millet is currently encouraged to include in the diet to supplement nutrients
to counter the malnutrition that is haunting the country. Consider the following
statements.
1. Ragi is rich in iron, calcium, roughage and is mainly grown in dry regions.
2. Among minor producers Sikkim, Arunachal Pradesh, Uttarakhand are included.
3. It is grown as a major crop in Bengaluru-Mysuru table land from where the countrys
majority of Ragi production comes from.
Choose the correct statements from the code below.
a)
b)
c)
d)

1 and 2 only
2 and 3 only
1 and 3 only
All the above

Q.8) Solution (d)


Among minor producers even Jharkhand, Jammu Kashmir are also involved apart from the
states mentioned in the 2nd statement.
Ragi production and consumption is highest in Bengaluru-Mysuru table region.

www.iasbaba.com

Page 4

IASbaba 60 Day Plan- Prelims Test 2016 GEOGRAPHY & CURRENT AFFAIRS [DAY 27]

Q.9) Tea best grows in Assam, Darjeeling and Jalpaiguri districts of West Bengal. What
is/are the uniqueness of this region which suits this crop?
1.
2.
3.
4.

It has warm and humid climate.


Showers are mainly concentrated in 4-5 months ie June to October in any year.
Deep, fertile and well-drained soil.
Rugged topography of the relief near the Himalayan foot hills.

Choose the correct code.


a)
b)
c)
d)

1, 2 and 3 only
1 and 3 only
1, 3 and 4 only
All the above

Q.9) Solution (b)


Showers are almost evenly distributed throughout the year with very low dry spell.
Well drained gentle sloped soils are required and rugged topography is not suitable for tea
plantations.

Q.10) Sugarcane grown in subtropical region has lower productivity than that is grown in
tropical regions.
Consider the following statements about subtropical region i.e Northern India which has
low productivity in sugarcane production.
1.
2.
3.
4.

Frost formations
Floods and water logging
High diurnal temperature
Low humidity

Choose the correct reason/s from the codes below.


a)
b)
c)
d)

1 and 2 only
2 and 3 only
3 and 4 only
1, 2 and 3 only

www.iasbaba.com

Page 5

IASbaba 60 Day Plan- Prelims Test 2016 GEOGRAPHY & CURRENT AFFAIRS [DAY 27]

Q.10) Solution (a)


First two statements are self-explanatory.
A higher diurnal temperature and low humidity are required for a better productivity. Thus
these two factors are not reducing the productivity.

Q.11) Consider the following statements:


1. A short duration growing season between Rabi and Kharif is called Zaid.
2. Zaid crops require warm dry weather for major growth period and longer day length
for flowering.
3. Major crops are seasonal fruits and vegetables like watermelon, musk melon,
cucumber etc.
4. Paddy seeds are sown in this season.
Which of the above statements are correct?
a)
b)
c)
d)

1,2 and 3
2,3 and 4
1 and 3
All of the above

Q.11) Solution (d)


In the Indian sub-continent, the crops grown on irrigated lands which do not have to wait
for monsoons, in the short duration between Rabi and Kharif crop season, mainly from
March to June, are called Zaid crops (also written as Zayad crops). These crops are grown
mainly in the summer season during a period called the "Zayad crop season." They require
warm dry weather for major growth period and longer day length for flowering. The main
produce is seasonal fruits and vegetables.
Rice is a Kharif crop but its seeds are sown in Zaid season. In Zaid season the seeds grow into
saplings. These saplings are plucked and then transplanted in paddy field manually in Kharif
season.
Q.12) Consider the following statements regarding the milk production in India:
1. India is the Worlds leading milk producer.
2. The growth of Indias milk production in year 2015-16 is higher than the average
Worlds growth of milk production.
3. India has amongst the most productive breeds of milch animals.
Which of the above statements are correct?
www.iasbaba.com

Page 6

IASbaba 60 Day Plan- Prelims Test 2016 GEOGRAPHY & CURRENT AFFAIRS [DAY 27]

a)
b)
c)
d)

1 and 2
2 and 3
1 and 3
All of the above

Q.12) Solution (a)


According to economic survey 2015-16, India recorded a growth of 6.26%. Whereas, FAO
has reported an average worldwide growth of 3.1%.
Indian Milch Breeds are amongst the least productive of the world. Programs have been
made to cross breed Indian breeds with foreign breeds to increase their productivity.

Q.13) Sugar is one of the basic human necessities across the world. Sugarcane plantations
became a major driver of large and forced human migrations in 19 th and early 20th
century. Consider the following statements regarding sugarcane and sugar production:
1. It was in India that farmers discovered and began the farming of sugarcane.
2. The process to produce sugar crystals was invented in India.
3. European merchants started trading in sugar, which was a luxury and expensive
spice in Europe.
4. Europeans from India took sugarcane to Europe and it became an important crop to
be grown in England and France.
Which of the above statements are correct?
a)
b)
c)
d)

1,2 and 3
2,3 and 4
1,3 and 4
All of the above

Q.13) Solution (a)


Europeans started growing sugarcane in there colonies in tropical area. Climate of Europe is
not suitable for the growth of sugarcane.
Over 2500 years ago, Indian farmers had discovered and begun farming many spices and
sugarcane. It was in India, between the sixth and fourth centuries BC, that the Persians,
followed by the Greeks, discovered the famous "reeds that produce honey without bees"
being grown. pronounced as saccharum. On their return journey, the Macedonian soldiers
carried the "honey bearing reeds," thus spreading sugar and sugarcane agriculture. People
in India had invented, by about 500 BC, the process to produce sugar crystals. In the local
language, these crystals were called khanda , which is the source of the word candy.
www.iasbaba.com

Page 7

IASbaba 60 Day Plan- Prelims Test 2016 GEOGRAPHY & CURRENT AFFAIRS [DAY 27]

Before the 18th century, cultivation of sugarcane was largely confined to India. A few
merchants began to trade in sugar a luxury and an expensive spice in Europe until the
18th century. Sugar became widely popular in 18th-century Europe, then graduated to
becoming a human necessity in the 19th century all over the world. This evolution of taste
and demand for sugar as an essential food ingredient unleashed major economic and social
changes. Sugarcane does not grow in cold, frost-prone climate; therefore, tropical and
semitropical colonies were sought. Sugarcane plantations, just like cotton farms, became a
major driver of large and forced human migrations in 19th century and early 20th century
of people from Africa and from India, both in millions influencing the ethnic mix, political
conflicts and cultural evolution of Caribbean, South American, Indian Ocean and Pacific
Island nations.

Q.14) Consider the following statements regarding Truck Farming:


1. The crops are grown at the back of truck and directly sold in market.
2. Commercial growing of vegetables and fruits is called truck farming.
Which of the above statements is/are correct?
a)
b)
c)
d)

1 only
2 only
Both 1 and 2
Neither 1 nor 2

Q.14) Solution (d)


Horticultural practice of growing one or more vegetable crops (not fruits) on a large scale
for shipment to distant markets is called truck farming. It got its name as perishable crops
had to be transported quickly to distant markets.

Q.15) European Union has extended ban on import of various food items causing a loss of
approximately $2 million to India. Consider the following statements regarding EU ban on
Indian items:
1. EU has extended its ban indefinitely on the import of Indian mangoes.
2. The ban was imposed as fruit flies were discovered in some of the consignments of
mango.
Which of the above statements are correct?
a) 1 only
www.iasbaba.com

Page 8

IASbaba 60 Day Plan- Prelims Test 2016 GEOGRAPHY & CURRENT AFFAIRS [DAY 27]

b) 2 only
c) Both 1 and 2
d) Neither 1 nor 2
Q.15) Solution (b)
Ban was imposed on mangoes for 20 months in 2014 after the detection of fruit flies in
some of the consignments. However it was exempted last year.
The European Union (EU) has extended the ban on import of four vegetables bitter gourd,
eggplant, taro plant (arbi) and snake gourd from India by another year despite efforts
made by the country to put in place stringent quality control and packaging measures.

Q.16) According to the report of Food and Agriculture Organisation, India is the largest
producer of which of the following agricultural products in the world?
1.
2.
3.
4.
5.

Fresh fruits
Jute
Pulses
Rice
Wheat

Select the code from the following:


a)
b)
c)
d)

All of the above


1,2,3 and 5
1,2 and 3
1 and 3

Q.16) Solution (c)


The Statistics Office of the Food and Agriculture Organisation reported that, per final
numbers for 2009, India had grown to become the world's largest producer of the following
agricultural products:

Fresh Fruit
Lemons and limes
Buffalo milk, whole, fresh
Castor oil seeds
Sunflower seeds
Sorghum
Millet

www.iasbaba.com

Page 9

IASbaba 60 Day Plan- Prelims Test 2016 GEOGRAPHY & CURRENT AFFAIRS [DAY 27]

Spices
Okra
Jute
Beeswax
Bananas
Mangoes, mangosteens, guavas
Pulses
Indigenous buffalo meat
Fruit, tropical
Ginger
Chick peas
Areca nuts
Other bastfibres
Pigeon peas
Papayas
Chillies and peppers, dry
Anise, badian, fennel, coriander
Goat milk, whole, fresh

Per final numbers for 2009, India is the world's second largest producer of the following
agricultural products:

Wheat
Rice
Fresh vegetables
Sugar cane
Groundnuts, with shell
Lentils
Garlic
Cauliflowers and broccoli
Peas, green
Sesame seed
Cashew nuts, with shell
Silk-worm cocoons, reelable
Cow milk, whole, fresh
Tea
Potatoes
Onions
Cotton lint
Cottonseed
Eggplants (aubergines)

www.iasbaba.com

Page 10

IASbaba 60 Day Plan- Prelims Test 2016 GEOGRAPHY & CURRENT AFFAIRS [DAY 27]

Nutmeg, mace and cardamoms


Indigenous goat meat
Cabbages and other brassicas
Pumpkins, squash and gourds

Q.17) Consider the following:


Term
1.
2.
3.
4.

Floriculture
Olericulture
Viticulture
Oenology

Meaning
Production and marketing of Floral crops
Production and marketing of vegetables
Production and marketing of grapes
Study of wine

Which of the above are correctly matched?


a)
b)
c)
d)

1 and 3
2 and 4
1,3 and 4
All of the above

Q.17) Solution (d)


Self-explanatory

Q.18) Between 1970 and 2011, the GDP share of agriculture has fallen from 43% to 16%.
Which of the following statements is the correct reason for this?
1. The agricultural production gradually decreased.
2. There was a large scale rural to urban migration.
3. The secondary and tertiary sectors grew very rapidly.
Select the code from below:
a)
b)
c)
d)

1 and 2
3 only
2 and 3
All of the above

Q.18) Solution (b)


First statement is incorrect as the agricultural production has grown rapidly.
Second statement is correct but is not the correct reason for the fall of share of agriculture
in GDP.
www.iasbaba.com

Page 11

IASbaba 60 Day Plan- Prelims Test 2016 GEOGRAPHY & CURRENT AFFAIRS [DAY 27]

As the share of manufacturing sector and tertiary sector increased, the share of agriculture
relatively decreased.

Q.19) Coconut is an important crop with almost all its parts used for some or the other
purposes. Coconut is widely grown in India and India is the third largest producer of the
world after Indonesia and Philippines. Consider the following statements regarding
coconut:
1.
2.
3.
4.

There are two basic varieties of coconut, the tall and the dwarf.
The plant requires a year round warm and humid climate.
Coarse sand soil is deemed perfect for coconut plantations.
Coconut trees need saline water to grow.

Which of the above statements are correct?


a)
b)
c)
d)

1,2 and 3
2,3 and 4
1,2 and 4
All of the above

Q.19) Solution (a)


There are only two varieties of coconuts; the tall and the dwarf. Tall varieties have a lifespan of around 80-90 years and coconut trees of this variety can grow up to a height of 15
to 18 meters. Whereas short varieties have an average life span of around 40 to 50 years
and they can grow up to a height of 5 to 7 meters. The tall and short varieties also have their
own types which grow in separate parts of the country. Furthermore, there are hybrid
varieties of coconut trees aimed at improved fruit quality.
Large-scale coconut production has its own pre-requisites like apt weather conditions,
proper soil, and proximity to the sea. To begin with, a coarse sand soil is deemed perfect for
coconut plantation. The plant requires a year-round warm and humid climate, and hence, a
close proximity to the sea as well as the equator is required. Lots of sunshine with high
humidity levels are suitable for coconut production. Temperatures above 24C and an
average annual rainfall of around 150 cm a year is also crucial for large-scale coconut
plantation. Coconut trees are rarely seen in low humidity areas and the plant does not
survive for too long in cold conditions. In fact, freezing and frosty conditions prove to be
fatal for the plant. The tree does not bear fruit amidst unsuitable weather conditions. The
coconut tree is a member of a group of plants called halophytes, that are tolerant of salt
water, which would kill most land plants. Scientists have been trying to figure out how the
halophytes extract the desalinated water they need to survive in hopes they could transfer
www.iasbaba.com

Page 12

IASbaba 60 Day Plan- Prelims Test 2016 GEOGRAPHY & CURRENT AFFAIRS [DAY 27]

it to crop plants which cannot do this. Cracking this mystery would let us grow crops in arid
soils where salt is concentrated in the soil and leaches out into irrigation or rain water. For
that reason, they have a good understanding of the two tricks the halophytes use to
desalinate their drinks.

Halophytes have two ways of achieving their salt tolerance. First, their root membranes are
adept at excluding the Na atom of NaCl molecules from their roots. This lets them take in
H2O and other needed minerals but block out the salt. In addition, some halophytes can
route salt to shoots having special compartments within their cells, called vacuoles which
are large storage vats where NaCl can be cordoned off and excluded from the areas on the
plant cell where it would do harm.

Q.20) Indian seeds programme largely adheres to limited generation system for seed
multiplication. The system recognises three kinds of generation, namely breeder,
foundation and certified seeds. Consider the following statements:
1. Breeder seed is the basic seed which is genetically pure and obtained from a healthy
plant.
2. Offsprings of the Breeder seed which can be clearly traced to Breeder seed are
called Foundation Seeds.
3. Certified seed is the last stage, which actually reaches to a farmer. Certified seed is
the progeny of foundation seed and must meet the standards of seed certification
prescribed in the Indian Minimum Seeds Certification Standards, 1988.
Which of the above statements are correct?
a)
b)
c)
d)

1 and 2
2 and 3
1 and 3
All of the above

Q.20) Solution (b)


The Indian seed industry had exhibited impressive growth in the past and is expected to
provide further potential for growth in agricultural production: The role of seed industry is
not only to produce adequate quantity of quality seeds but also to achieve varietal diversity
to suit various agro-climatic zones of the country.

www.iasbaba.com

Page 13

IASbaba 60 Day Plan- Prelims Test 2016 GEOGRAPHY & CURRENT AFFAIRS [DAY 27]

The policy statements are designed towards making available to the Indian farmer,
adequate quantities of seed of superior quality at the appropriate time and place and at an
affordable price so as to meet the countrys food and nutritional security goals.
Indian seeds programme largely adheres to limited generation system for seed
multiplication. The system recognises three kinds of generation, namely breeder,
foundation and certified seeds.
On this basis, there are five different categories viz. Nucleus Seeds, Breeders Seeds,
Foundation Seeds, Registered Seeds and finally certified seeds. The Offspring of breeder
seeds is foundation seeds to registered seeds to certified seeds.
Further, each of the breeders, foundation, registered and certified seeds are certified and
labelled with a different colour tag as per Section 5 of the Seeds Act, 1966. Thus, they are
also called as Labelled Seed. The Breeder seeds have golden yellow tag, Foundation seeds
have white tag, registered seeds have opal blue tag and certified seeds have green tag.
The process of development of certified seeds begins from its distant parent called
Nucleus Seed. The nucleus seed is a genetically pure seed without any impurity. They
are obtained from a handful of healthy plants growing in a plot and then grown
strictly in isolation.
A breeders seed is an offspring of nucleus seed. A breeder is a person (qualified
plant breeder) or organization who raises plant primary for breeding purpose. While
nucleus seed is genetically pure; the Breeder seeds are produced by mutual
multiplication of three different lines which are denoted by A line, B line and R line. A
line is female, while B and R line are male sterile. The seeds from off springs with
best and desired quality are selected and certified as Breeder seeds. Such seeds are
protected by legal rights called as Breeders rights.
Offsprings of the Breeder seed which can be clearly traced to Breeder seed are
called Foundation Seeds. They are further breaded to give rise to certified seeds. The
production of the foundation seeds must be acceptable to a certification agency.
The progeny of the foundation seed approved and certified by a certifying agency
and is suitable to produce certified seeds is called Registered seed.
This is the last stage, which actually reaches to a farmer. Certified seed is the
progeny of foundation seed and must meet the standards of seed certification
prescribed in the Indian Minimum Seeds Certification Standards, 1988.

www.iasbaba.com

Page 14

IASbaba 60 Day Plan- Prelims Test 2016 GEOGRAPHY & CURRENT AFFAIRS [DAY 27]

Q.21) Consider the following


1. Krishna Circuit
2. Buddhist circuit
3. Mahabharat Circuit
Which of the above circuits are included in the Swadesh Darshan scheme?
a)
b)
c)
d)

All
1 and 2 only
2 and 3 only
1 and 3 only

Q.21) Solution (b)


Under the Swadesh Darshan scheme, thirteen thematic circuits have been identified, for
development namely: North-East India Circuit, Buddhist Circuit, Himalayan Circuit, Coastal
Circuit, Krishna Circuit, Desert Circuit, Tribal Circuit, Eco Circuit, Wildlife Circuit, Rural Circuit,
Spiritual Circuit, Ramayana Circuit and Heritage Circuit.

Q.22) Consider the following


1. Lake Tanganyika
2. Lake Volta
3. Lake Victoria
Which of the above lakes are not present in Kenya?
a)
b)
c)
d)

1 and 3 only
2 and 3 only
1 and 2 only
All

Q.22) Solution (c)

Lake Victoria borders Kenya, Tanzania, and Uganda


Lake Volta is present in Ghana
Lake Tanganyika extends to 4 countries Burundi, Tanzania, DR Congo, and Zambia

www.iasbaba.com

Page 15

IASbaba 60 Day Plan- Prelims Test 2016 GEOGRAPHY & CURRENT AFFAIRS [DAY 27]

Q.23) Consider the following statements with respect to Medecins Sans Frontieres
(MSF)
1. It is also called by the name Doctors without Borders
2. It was formed aftermath of World War 2
Select the correct answer
a)
b)
c)
d)

1 only
2 only
Both
None

Q.23) Solution (a)


Mdecins
Sans
Frontires (MSF)
or Doctors
Without
Borders,
is
an
international humanitarian-aid non-governmental organization (NGO) and Nobel Peace
Prize laureate, best known for its projects in war-torn regions and developing
countries facing endemic diseases. It was founded in France in response to the Biafran
War. The organization is known in most of the world by its localized name or simply as MSF;
in Canada and the United States the name Doctors without Borders is commonly used.

Q.24) The method of removing pollutants with the help of living organism like Bacteria is
known as
a)
b)
c)
d)

Bio magnification
Bio Remediation
Leaching
Bio Accumulation

Q.24) Solution (b)


Bio remediation is the use of microorganisms to degrade the environmental contaminants in
to less toxic forms.

Q.25) Consider the following statements with respect to Dispersion of light


1. The sequence VIBGYOR is in the order of increasing frequency.
2. The red light used in the traffic light is visible even in dense fog, because it has
lowest frequency in the VIBGYOR colour spectrum.
www.iasbaba.com

Page 16

IASbaba 60 Day Plan- Prelims Test 2016 GEOGRAPHY & CURRENT AFFAIRS [DAY 27]

Select the correct option from code given below


a)
b)
c)
d)

1 only
2 only
Both
None

Q.25) Solution (b)

The sequence VIBGYOR is in the order of decreasing frequency or increasing wave


length.
The light which has lowest frequency (Red) will bend slightly whereas light which has
more frequency will bend more (Violet)

Source 10th Ncert - 11th chapter

www.iasbaba.com

Page 17

IASbaba 60 Day Plan- Prelims Test 2016 GEOGRAPHY & CURRENT AFFAIRS [Day 28]

Q.1) Consider the following statements regarding Keoladeo National Park:


1.
2.
3.
4.

It was formerly known as Bharatpur Bird Sanctuary.


It is a natural wetland.
It receives thousands of migratory species of birds especially during summer season.
It is a World Heritage Site.

Which of the above statements are correct?


a)
b)
c)
d)

1,2 and 3
2 and 3
1 and 4
1,2 and 4

Q.1) Solution (c)


Keoladeo National Park or Keoladeo Ghana National Park formerly known as the Bharatpur Bird
Sanctuary in Bharatpur, Rajasthan, India is a famous avifauna sanctuary that hosts thousands of
birds, especially during the winter season.
It was declared a protected sanctuary in 1971. It is also a World Heritage Site.
Keoladeo Ghana National Park is a man-made and man-managed wetland and one of the
national parks of India. The reserve protects Bharatpur from frequent floods, provides grazing
grounds for village cattle, and earlier was primarily used as a waterfowl hunting ground.
The sanctuary is one of the richest bird areas in the world and is known for nesting of resident
birds and visiting migratory birds including water birds. The rare Siberian cranes used to winter
in this park but this central population is now extinct. According to founder of the World
Wildlife Fund Peter Scott, Keoladeo National Park is one of the worlds best bird areas.

Q.2) Despite the efforts of the Forest Department and the Customs Department, the
smuggling of Red Sanders is going unabated. Which of the following statements are correct
about Red Sanders?
1.
2.
3.
4.

It is endemic to Southern Eastern Ghats of South India.


The tree has characteristic rich red colour wood.
The wood of the tree is aromatic.
It is listed as an endangered species by IUCN.

Select the correct code from the following:


www.iasbaba.com

Page 1

IASbaba 60 Day Plan- Prelims Test 2016 GEOGRAPHY & CURRENT AFFAIRS [Day 28]

a)
b)
c)
d)

1,3 and 4
1,2 and 3
1,2 and 4
All of the above

Q.2) Solution (c)


Pterocarpus santalinus, with the common names red sanders, is a species of Pterocarpus
endemic to the southern Eastern Ghats mountain range of South India. This tree is valued for
the rich red color of its wood. The wood is not aromatic. The tree is not to be confused with the
aromatic Santalum sandalwood trees that grow natively in South India.
Pterocarpus santalinus is listed as an Endangered species by the IUCN, because of
overexploitation for its timber in South India.

Q.3) Desert vegetation, also called Xerophytes, have adjusted or adapted to its harsh
environment. Which of the following statements are correct adaptations of Xerophytes?
1. Some plants called Succulents, store water in their stems and leaves.
2. Leaves have turned into thorns to reduce water loss and also act as a deterrent to
herbivores.
3. Flowers open at night to lure pollinators.
4. Growth of desert plants is very slow, so that they require less energy.
Select the code from the following:
a)
b)
c)
d)

1,2 and 3
2,3 and 4
1,3 and 4
All of the above

Q.3) Solution (d)


Desert Plant Adaptations

Some plants, called succulents, store water in their stems or leaves;


Some plants have no leaves or small seasonal leaves that only grow after it rains. The
lack of leaves helps reduce water loss during photosynthesis. Leafless plants conduct
photosynthesis in their green stems.
Long root systems spread out wide or go deep into the ground to absorb water;

www.iasbaba.com

Page 2

IASbaba 60 Day Plan- Prelims Test 2016 GEOGRAPHY & CURRENT AFFAIRS [Day 28]

Some plants have a short life cycle, germinating in response to rain, growing, flowering,
and dying within one year. These plants can evade drought.
Leaves with hair help shade the plant, reducing water loss. Other plants have leaves
that turn throughout the day to expose a minimum surface area to the heat.
Spines to discourage animals from eating plants for water;
Waxy coating on stems and leaves help reduce water loss.
Flowers that open at night lure pollinators who are more likely to be active during the
cooler night.
Slower growing requires less energy. The plants don't have to make as much food and
therefore do not lose as much water.

Q.4) Consider the following statements regarding epiphytes:


1. These plants need help of a host plant to grow.
2. These plants are parasites and obtain their nutrients from the host plants.
Which of the above statements are incorrect?
a)
b)
c)
d)

1 only
2 only
Both 1 and 2
Neither 1 nor 2

Q.4) Solution (b)


Epiphytes are the plants growing on host plants but not nourished by the host plants. They do
not draw food from the host plant they only take support in getting access to light. Eg. Orchids

Q.5) Consider the following statements regarding deciduous trees:


1. They lose all their leaves for a part of the year.
2. In cold climates, trees shed their leaves during summers, to reduce loss of water
through transpiration.
Which of the above statements are correct?
a)
b)
c)
d)

1 only
2 only
Both 1 and 2
Neither 1 nor 2

www.iasbaba.com

Page 3

IASbaba 60 Day Plan- Prelims Test 2016 GEOGRAPHY & CURRENT AFFAIRS [Day 28]

Q.5) Solution (a)


In cold climates trees lose their leaves during autumn so that they are bare throughout the
winter.
In hot and dry climates deciduous trees lose their leaves during summer.

Q.6) Sea grasses are specialized angiosperms that resemble grass in appearance. They are the
only group of higher plants adapted to life in the salt water. Consider the following regarding
sea grasses.
1.
2.
3.
4.

They are found on deep sea floors.


There is rich growth of sea grass along Tamilnadu coast and Lakshadweep islands.
They help to stabilize bottom sediments to control erosion.
Dugong is dependent on sea grass for food.

Which of the above statements are correct?


a)
b)
c)
d)

1,2 and 3
2,3 and 4
1,3 and 4
All of the above

Q.6) Solution (b)


They grow in shallow coastal water with sandy or muddy bottoms and require comparatively
calm seas.
IUCN has accorded high priority for conservation of sea grass.
Q.7) Consider the following statements:
1. National parks enjoy a greater degree of protection than wildlife sanctuary.
2. Activities like grazing of livestock, collection of firewood are regulated in a wild life
sanctuary while it is prohibited in a national park.
3. National parks can be created for specific species while wildlife sanctuary is not
particularly focused on any specie.
Which of the above statements are correct?
a) 1 and 2
b) 2 and 3
www.iasbaba.com

Page 4

IASbaba 60 Day Plan- Prelims Test 2016 GEOGRAPHY & CURRENT AFFAIRS [Day 28]

c) 1 and 3
d) All of the above
Q.7) Solution (a)
Wildlife sanctuaries can be created for specific species while National Park is not focused on
any particular specie.
Q.8) Consider the following statements regarding Topical Evergreen forest in India:
1.
2.
3.
4.

These forests grow in areas where rainfall is more than 200cm.


The soil of these forests is highly fertile with large humus content.
Teak and Sal are important tree species.
There is low undergrowth as plant species struggle for sunlight.

Which of the above statements are incorrect?


a)
b)
c)
d)

2,3 and 4
1 only
2 and 3
1 and 4

Q.8) Solution (c)


The soil of tropical rain forest is not fertile as most of the nutrients are leached by heavy rain.
Teak and Sal are important species of deciduous forest.

Q.9) Consider the following statements regarding Onge tribe of India:


1. It is an indigenous tribe of Andaman Islands.
2. They belong to Negrito race.
3. Only less than 100 tribesmen are left.
Which of the above statements are correct?
a)
b)
c)
d)

1 and 2
2 only
2 and 3
All of the above

Q.9) Solution (d)


www.iasbaba.com

Page 5

IASbaba 60 Day Plan- Prelims Test 2016 GEOGRAPHY & CURRENT AFFAIRS [Day 28]

The Onge are one of the Andamanese indigenous peoples of the Andaman Islands. Traditionally
hunter-gatherers, they are a designated Scheduled Tribe of India.
Jarawa and Onge are endangered tribes of Andaman.

Q.10) Consider the following statements:


1. On the basis of endangerment, UNESCO has categorized the languages into two
categories.
2. According to UNESCO, a language becomes extinct when nobody speaks or remembers
the language.
3. Protection and Preservation of Endangered Languages of India is a central
government scheme to save the languages spoken by less than 10,000 people in India.
Which of the above statements are correct?
a)
b)
c)
d)

1 and 2
2 and 3
1 and 3
All of the above

Q.10) Solution (b)


UNESCO has categorised languages on basis of endangerment as: vulnerable, definitely
endangered, severely endangered and critically endangered.

According to the criteria adopted by UNESCO, a language becomes extinct when nobody speaks
or remembers the language.
The government has initiated a scheme to protect, preserve and document endangered Indian
languages - spoken by less than 10,000 people.
Under the scheme "Protection and Preservation of Endangered Languages of India", Mysorebased Central Institute of Indian Languages (CIIL) is working on the protection, preservation and
documentation of endangered languages.

www.iasbaba.com

Page 6

IASbaba 60 Day Plan- Prelims Test 2016 GEOGRAPHY & CURRENT AFFAIRS [Day 28]

Q.11) Consider the statements regarding a particular valley which was in the news recently.
1. It is a part of Gangotri National Park
2. It is a cold desert and looks like a replica of Tibetan plateau
3. It is home to Snow Leopard
Select the correct code
a)
b)
c)
d)

Nellong valley
Kashmir valley
Alaknanda valley
Bhagirathi valley

Q.11) Solution (a)


http://www.hindustantimes.com/travel/nelong-valley-another-ladakh-tucked-in-theuttarakhand-himalayas/story-42e5ckQmuHERYvTJ6SVtmL.html

Q.12) Central Zoo Authority, a body set up in 1992 for the oversight of zoos.
Consider the following statements regarding CZA.
1. It is set up under the Wild Life Protection Act, 1972.
2. It is an affiliated member of World Association of Zoos and Aquarium.
3. It doesnt have the power to regulate the trade of endangered species.
Choose the correct statements from the following code.
a)
b)
c)
d)

1 and 2 only
2 and 3 only
1 and 3 only
All the above

Q.12) Solution (a)


It does have the power to regulate the trade of endangered species. Apart from that it also has
the power to regulate and approve the exchange of animals between Indian and foreign zoos.

www.iasbaba.com

Page 7

IASbaba 60 Day Plan- Prelims Test 2016 GEOGRAPHY & CURRENT AFFAIRS [Day 28]

Q.13) Himalayan yew is a plant found in Himachal Pradesh and Arunachal Pradesh, was in
news recently.
Consider the following statements regarding the plant.
1. It is a medicinal plant.
2. A chemical compound called taxol obtained from the plant is used as an effective anticancer drug.
Choose the incorrect statements from code below.
a)
b)
c)
d)

1 only
2 only
Both
None of the above

Q.13) Solution (d)


A chemical compound called taxol obtained from the plant is used as an effective anti-cancer
drug.
http://www.downtoearth.org.in/news/himalayan-yew-to-fight-cancer-29559

Q.14) Consider the following


1. Trees shed their leaves in the dry season to conserve water
2. These are economically most important forests of India
3. Sal, Teak and Neem are some of the important trees of this forest
Identify the forest?
a)
b)
c)
d)

Deciduous Forests
Evergreen Forests
Tropical Deciduous Forests
Temperate Deciduous Forests

Q.14) Solution (c)

www.iasbaba.com

Page 8

IASbaba 60 Day Plan- Prelims Test 2016 GEOGRAPHY & CURRENT AFFAIRS [Day 28]

Tropical deciduous are the monsoon forests found in the large part of India, northern Australia
and in Central America. These regions experience seasonal changes. Trees shed their leaves in
the dry season to conserve water.
In a large part of our country we have this type of forest. These forests are also called Monsoon
Forests. They are less dense. They shed their leaves at a particular time of the year in spring
and at onset of summer. Important trees of these forests are sal, teak, peepal, neem and
shisham. (The names that we hear often) They are found in Madhya Pradesh, Uttar Pradesh,
Bihar, Jharkhand, Chhattisgarh, Orissa, and in parts of Maharashtra.
Tropical Deciduous Forests are economically most important forests of India.

Q.15) If you are travelling through Eastern Himalayas, which of the following you will
encounter?
1. Golden Langur
2. Takin
3. Black Bear
Select the correct code
a)
b)
c)
d)

1 and 2
2 and 3
Only 2
1, 2 and 3

Q.15) Solution (d)

Q.16) Consider the following statements


1. The leaves of these forest falls in autumn season
2. They are found in Temperate Zone
3. These forests in China and Europe are basically man made
What is the name of this forest?
a) Coniferous Forest
b) Deciduous Forest
www.iasbaba.com

Page 9

IASbaba 60 Day Plan- Prelims Test 2016 GEOGRAPHY & CURRENT AFFAIRS [Day 28]

c) Tropical Rain Forest


d) None

Q.16) Solution (b)


A deciduous forest is one where the leaves fall from the trees in autumn. Deciduous means
"falling out or off at a certain season". he average temperature is 50 degrees and the average
rainfall is 30 to 60 inches per year. These forests are located in the temperate zone above the
tropical forests and below the coniferous forests. These forests have their own specific plants,
trees, animals, and shrubs. Deciduous forests are found in the eastern part of North America
and the middle of Europe. Asia, southwest Russia, Japan, and eastern China also have these
forests. All of the deciduous forests in America are second growth. This means all the original
trees have been cut and the forests have grown from seeds and seedlings. Both China and
Europe have cleared all of their first growth deciduous forests for agriculture. All of the
deciduous forests in Europe and China are man-made.

Q.17) Consider the following statements based on UN report on E-waste


1. The bulk of global e-waste constitutes from mobile phones, calculators, personal
computers, printers, and small information technology equipment.
2. The lowest amount of e-waste per inhabitant was generated in South Asian countries
3. China is the largest producer of e-waste in the world
Select the correct code/s
a)
b)
c)
d)

1, 2 and 3
1 and 2
2 and 3
None

Q.17) Solution (d)


While only 7 per cent of e-waste last year was made up of mobile phones, calculators, personal
computers, printers, and small information technology equipment, almost 60 per cent was a
mix of large and small equipment used in homes and businesses, such as vacuum cleaners,
toasters, electric shavers, video cameras, washing machines, electric stoves, mobile phones,
calculators, personal computers, and lamps.
www.iasbaba.com

Page 10

IASbaba 60 Day Plan- Prelims Test 2016 GEOGRAPHY & CURRENT AFFAIRS [Day 28]

The lowest amount of e-waste per inhabitant was generated in Africa (1.7 kg/inhabitant). The
continent generated 1.9 Mt of e-waste in total.
India is the fifth biggest producer of e-waste in the world, discarding 1.7 million tonnes (Mt) of
electronic and electrical equipment in 2014, a UN report has warned that the volume of global
e-waste is likely to rise by 21 per cent in next three years.
The Global E-Waste Monitor 2014, compiled by U.N.s think tank United Nations University
(UNU), said at 32 per cent, the U.S. and China produced the most e-waste overall in 2014.
India is behind the U.S., China, Japan and Germany.

Q.18) Consider the following with respect to Western Ghat and rainfall pattern along it
1. Karnataka receives less rainfall than Kerala and Maharashtra because of steep slopes
and continuous Mountains
2. The mountain topography in Karnataka is broader than the narrow topography of the
Ghats in Maharashtra.
Select the incorrect statements
a)
b)
c)
d)

Only 1
Only 2
Both
None

Q.18) Solution (a)


Kindly understand that mountains of Karnataka are not steep sloping but gentle sloping. So,
1st statement is incorrect.

The mountain topography in Karnataka is broader than the narrow topography of the Ghats
in Maharashtra. Due to the greater width of the mountains, the rain bearing winds have to
necessarily travel a longer distance and have more time for the drops to coalesce and
precipitate as rainfall, resulting in higher rainfall. In contrast, the narrow width of the Ghats in
Maharashtra allows the rain-bearing wind to cross over to the leeward side rapidly before
precipitation can occur. As for Kerala, the Ghats there are in the form of isolated mountains,
www.iasbaba.com

Page 11

IASbaba 60 Day Plan- Prelims Test 2016 GEOGRAPHY & CURRENT AFFAIRS [Day 28]

where the rain-bearing winds can easily cross over to the leeward side through the gaps in
between without precipitation occurring.
Second, the slope of the mountain has a direct bearing on the possibility of precipitation. This is
borne out by the Ghats of Karnataka where the mountains are gently sloping, compared to the
steep slopes of the Ghats in Maharashtra and Kerala.
The air parcel will retain its energy and speed for a longer time when the slope is gradual. This
will provide sufficient vertical motion to cloud droplets to grow by collisioncoalescence
process and hence form precipitation.
Third, the gentle slope provides a greater area for sunlight absorption and heating leading to
greater convection when compared with an abrupt slope i.e. less Ghat area such as that of the
Maharashtra and Kerala Ghats.
Fourth, the continuous mountain range presents a greater barrier to rain-bearing winds than a
range comprising isolated mountains with gaps in between where the winds can easily pass to
the leeward side. Unlike in the case of Kerala, the Ghats in Maharashtra and Karnataka are
continuous.

Q.19) Indian Green Building Council (IGBC) New Green Buildings rating system addresses
green features under certain categories. What are they?
1.
2.
3.
4.
5.
6.

Site selection and planning


Building Materials and Resources
Innovation and Development
Water Conservation
Noise Conservation
Energy Efficiency

Select the correct order


a)
b)
c)
d)

1, 3, 4 and 5
1, 2, 3, 4, 5 and 6
1, 2, 3 and 4
1, 2, 3, 4 and 6

Q.19) Solution (d)


www.iasbaba.com

Page 12

IASbaba 60 Day Plan- Prelims Test 2016 GEOGRAPHY & CURRENT AFFAIRS [Day 28]

IGBC Green New Buildings rating system addresses green features under the following
categories:

Sustainable Architecture and Design


Site Selection and Planning
Water Conservation
Energy Efficiency
Building Materials and Resources
Indoor Environmental Quality
Innovation and Development

Q.20) What are the factors for extinction of wildlife throughout the world?

1.
2.
3.
4.

Alteration of habitat
Introduction of foreign species
Control of pests and predators
Captive breeding

Select the correct code:


a)
b)
c)
d)

1 and 2 Only
1, 2 and 3 Only
1, 3 and 4 Only
All of the above

Q.20) Solution (b)


Predators or pests are important biotic components of the ecosystem. Natural predators make
the prey population stronger by removing weak members of the prey population but human
beings generally remove the strongest specimens. Captive breeding is for conservation of
troubled species (a measure to safeguard wildlife)

www.iasbaba.com

Page 13

IASbaba 60 Day Plan- Prelims Test 2016 GEOGRAPHY & CURRENT AFFAIRS [Day 28]

SUR IAS ACEDAMY GUJARAT

www.iasbaba.com

Page 14

Q.1) Which among the below given animals/birds cannot walk backwards?
1)
2)
3)
4)

Kangaroos
Emus
Penguins
Alligators

Choose the appropriate answer:


a)
b)
c)
d)

1 only
1 and 2 only
1, 3 and 4 only
1 and 3 only

Q.1) Solution (b)


Explanation:
Only Kangaroos and Emus cannot walk backwards

Q.2) Earth's seasons are caused by which of the following?


a)
b)
c)
d)

The tilt of Earth's axis of rotation relative to the ecliptic as Earth revolves around the Sun
The varying amount of sunspot activity
Earth's orbit about the Sun as an ellipse rather than a circle
The rotation of Earth during a 24-hour day

Q.2) Solution (a)


Explanation:
Seasons are best explained as resulting from Earths axial tilt and not from distance
variations, sunspot activity, atmospheric transparency, or rotation.

Q.3) When cool air flows from a high mountain region to a region of lower elevation, the air
will
a)
b)
c)
d)

increase in moisture content


condense, forming large amounts of dew
undergo adiabatic warming
undergo adiabatic cooling

Q.3) Solution (c)


Explanation:
When cool air flows from a high mountain region to a region of lower elevation, the air
undergoes adiabatic warming.
Adiabatic warming occurs as the pressure of the air is increased as it descends.
Q.4) Match List I with List II and select the correct answer using the code given below the
Lists:
List I

List II

A. Smelting

1. Process in which ore is heated usually in the presence of air, at


temperatures below its melting points
B. Roasting
2. Reduction of oxide ore with carbon at high temperature
C. Calcination 3. Process in which ore is heated, generally in the absence of air, to expel
water from hydrated oxide or carbon dioxide from a carbonate at
temperature below their melting point
D. Corrosion 4. Process of slow conversion of metals into their undesirable compounds by
reaction with moisture and other gases present in the atmosphere

a)
b)
c)
d)

A-B-C-D
1-4-3-2
1-4-2-3
2-1-4-3
2-1-3-4

Q.4) Solution (d)

Q.5) IASbabaji is hiking up Mt. Everest, and recently reached the base camp at 12,000 feet.
While there, babaji decides to make some tea, and puts a thermometer in the boiling water.
To his surprise, the thermometer tells him the water is boiling at 96 C. Why is this
happening?
a) Babaji was surprised to watch water boiling more quickly.
b) The pressure is lower at higher elevations, and results in a lower boiling temperature.
c) The number of solutes in the air is higher at higher elevations, resulting in boiling point
depression.
d) Babaji is suffering from oxygen deprivation, and is reading the thermometer incorrectly.

Q.5) Solution is (b)


Explanation:
At higher elevations, the atmospheric pressure is lower. At 12,000 feet, the atmospheric
pressure is only 0.70 atm, compared to 1 atm at sea level. The reduction in pressure leads
to a reduction in boiling temperature.

Q.6) Polarized sunglasses are used to cut glare from sunlight reflected at a glancing angle off
cars, water, and other surfaces. Such sunglasses are a practical application of which of the
following physical principles?
a)
b)
c)
d)

Brewster's law
Lenz's law
Coulomb's law
Snell's law

Q.6) Solution is (a)


Explanation:
According to Brewsters law, reflected light will always be polarized in a horizontal direction,
parallel to the reflecting surface. Polarized sunglasses are constructed to block this reflected
light and to transmit light polarized only in the vertical direction.
Q.7) Match List I with List II and select the correct answer using the code given below the
Lists:
LIST I
LIST II
A.
B.
C.
D.

Tuber
Bulb
Rhizome
Corm

a)
b)
c)
d)

A-B-C-D
3-4-1-2
2-3-4-1
2-4-3-1
2-3-1-4

1.Saffron
2.Potato
3.Garlic
4.Turmeric

Q.7) Solution is (b)


Explanation:
Underground stem
1. Tuber like potato
2. Corm like Colocasia, Saffron
3. Bulb like Onion, Garlic etc.
4. Rhizome like Turmeric, Ginger etc.

Q.8) Match the following


1. Operation Cactus
2. Operation Pawan
3. Operation Green Hunt

Srilanka
Maldives
China

Select the correct match of Indian Military Operations with other countries
a)
b)
c)
d)

1 and 2
Only 2
2 and 3
None

Q.8) Solution (d)


Operation Cactus was in news recently (It was with Maldives)
http://www.thehindu.com/specials/indiamaldives-relations-at-a-glance/article8460835.ece
Operation Green Hunt- Anti-Naxalite Operation of GoI
Operation Pawan- Srilanka

Q.9) Consider the following


1. Evaporation of gas
2. Compression of the gas

3. Solubility of Gas
4. Expansion of the gas
5. Diffusion of gas
Which of the above phenomenon (in liquid medium) will increase with the increase in
Temperature?
a)
b)
c)
d)

1, 2, 3, 4 and 5
1, 4 and 5
1, 3, 4 and 5
2, 4 and 5

Q.9) Solution (b)


NCERT- Class 9th, Chapter 1
In understanding the effects of temperature on the solubility of gases, it is first important to
remember that temperature is a measure of the average kinetic energy. As temperature
increases, kinetic energy increases. The greater kinetic energy results in greater molecular
motion of the gas particles. As a result, the gas particles dissolved in the liquid are more likely
to escape to the gas phase and the existing gas particles are less likely to be dissolved. The
converse is true as well. The trend is thus as follows: increased temperatures mean lesser
solubility and decreased temperatures mean higher solubility.
Le Chatelier's principle allows better conceptualization of these trends. First, note that the
process of dissolving gas in liquid is usually exothermic. As such, increasing temperatures result
in stress on the product side (because heat is on the product side). In turn, Le
Chatelier's principle predicts that the system shifts towards the reactant side in order to
alleviate this new stress. Consequently, the equilibrium concentration of the gas particles in
gaseous phase increases, resulting in lowered solubility.
Conversely, decreasing temperatures result in stress on the reactant side (because heat is on
the product side). In turn, Le Chtelier's principle predicts that the system shifts toward the
product side in order to compensate for this new stress. Consequently, the equilibrium
concentration of the gas particles in gaseous phase would decrease, resulting
in greater solubility.

Q.10) Which of the following schemes will promote Urban Infrastructure?


1.
2.
3.
4.
5.

Swatch Bharat Abhiyan


Smart City Scheme
Bharat Nirman
Indira Awas Yojana
HRIDAY

Select the correct code


a)
b)
c)
d)

1, 3 and 4
2, 4 and 5
1, 2 and 5
All

Q.10) Solution (c)


Bharat Nirman and IAY- Rural infrastructure schemes

Q.11) Recently scientists have found a way to lock CO 2 by turning it into harmless rock. Which
of the following statements are correct about this process?
a) The CO2 is injected into volcanic bedrock where it will react to form carbonate rocks.
b) CO2 is captured and solidified. This solid CO2 is then compressed to give it permanent
form.
c) CO2 is reacted with amorphous calcium to form calcium carbonate.
d) Carbon di oxide is captured and stored in Ocean trench where it gets solidified due to
high pressure.
Q.11) Solution (a)
The greenhouse gas carbon dioxide (CO2) can be permanently and rapidly locked away from
the atmosphere, by injecting it into volcanic bedrock. The CO2 reacts with the surrounding rock,
forming environmentally benign minerals.

Geoengineers have long explored the possibility of sealing CO2 gas in voids underground, such
as in abandoned oil and gas reservoirs, but these are susceptible to leakage. So attention has
now turned to the mineralisation of carbon to permanently dispose of CO2.

Q.12) Touch screens now have become an important component of many electronic devices
like mobiles, ATM machines, laptops etc. Consider the following statements regarding touch
screens:
1. Commonly there are two types of touch screens- resistive and capacitive.
2. Capacitive touch screens are multilayered with inner most and outer most layers
behaving as a conductor.
3. Resistive screens can detect more than one touch at once at different spots.
Which of the above statements are correct?
a)
b)
c)
d)

1 and 2
2 and 3
1 and 3
All of the above

Q.12) Solution (a)


There are several types of touch screen technologies available, but of these, two are most
common. The first is the Resistive touch screen which works a bit like transparent keyboard
overlaid on top of the screen. There's a flexible upper layer of conducting polyester plastic
bonded to a rigid lower layer of conducting sheet and these are separated by an insulating
membrane. When the screen is pressed at a specific spot, it forces the polyester to touch the
glass and complete a circuit just like pressing the key on a keyboard. A chip inside the screen
figures out the coordinates of the place which is touched.
The other is the capacitive type screen. These screens are made from multiple layers. The
inner layer conducts electricity and so does the outer layer. Effectively the screen behaves like
two electrical conductors separated by an insulator in other words, a capacitor. When the
user brings his finger up to the screen, it alters the electrical field by a certain amount that
varies according to where the finger is. Capacitive screens can be touched in more than one
place at once.

Q.13) Recently Zika virus caused a havoc in South American and central American countries.
Few cases have also been registered in USA. Which of the following statements are correct
about Zika virus?
1. It is disease caused by virus from infected Aedes mosquito.
2. Zika virus can be sexually transmitted.
3. Virus can pass through the fetus of a pregnant woman and can infect the brain cells of
the fetus.
4. The symptoms are similar to other arbovirus infections such as dengue, and include
fever, skin rashes, conjunctivitis, muscle and joint pain, malaise, and headache.
Select the code from the following:
a)
b)
c)
d)

1,3 and 4
1 and 3
3 and 4
All of the above

Q.13) Solution (d)

Zika virus disease is caused by a virus transmitted primarily by Aedes mosquitoes.


People with Zika virus disease can have symptoms including mild fever, skin rash, conjunctivitis,
muscle and joint pain, malaise or headache. These symptoms normally last for 2-7 days.
Sexual transmission of Zika virus has been documented in several different countries. To reduce
the risk of sexual transmission and potential pregnancy complications related to Zika virus
infection, the sexual partners of pregnant women, living in or returning from areas where local
transmission of Zika virus occurs should practice safer sex (including using condoms) or abstain
from sexual activity throughout the pregnancy.

Q.14) A two foot long but discovered in China has been declared as the worlds longest insect.
Which of the following species does this bug belongs to?
a)
b)
c)
d)

A Mantis
A Stick Insect
A centipede
A titan beetle

Q.14) Solution (b)


A bug, measuring over half-a-metre long, discovered in southern China has been declared the
world's longest insect.
A stick insect measuring 62.4 centimetres found two years ago in the southern province of
Guangxi has broken the record for length amongst the world's 807,625 known insects, the
official Xinhua agency said, citing the Insect Museum of West China.

Q.15) Consider the following statements:


1. A black hole is a place in space where gravity pulls so much that even light cannot get
out.
2. Density of black hole is very high.
3. Black holes are made when the fuel of a star gets over and it becomes a white dwarf.
Which of the above statements are correct?
a)
b)
c)
d)

1 and 2
2 and 3
1 and 3
All of the above

Q.15) Solution (a)


A black hole is a place in space where gravity pulls so much that even light cannot get out. The
gravity is so strong because matter has been squeezed into a tiny space. This can happen when
a star is dying.
Because no light can get out, people can't see black holes. They are invisible. Space telescopes
with special tools can help find black holes. The special tools can see how stars that are very
close to black holes act differently than other stars.
Stellar black holes are made when the center of a very big star falls in upon itself, or collapses.
When this happens, it causes a supernova. A supernova is an exploding star that blasts part of
the star into space. Scientists think supermassive black holes were made at the same time as
the galaxy they are in.

Small stars become white dwarfs when there fuel gets over.

Q.16) Hepatitis is the medical term for inflammation of the liver. Consider the following
regarding this
1. There are five types of viruses responsible for Hepatitis
2. Intake of toxic substances, alcohol and certain drugs are some of the causes of Hepatitis
3. Hepatitis B is transmitted through exposure to infective blood, semen, and other body
fluids.

Select the correct code


a) 1 and 2
b) 2 and 3
c) 1 and 3
d) 1, 2 and 3

Q.16) Solution (d)

http://www.thehindu.com/news/cities/bangalore/hepatitis-b-virus-more-prevalent-than-hivin-donor-blood/article8725933.ece
http://www.thehindu.com/news/cities/chennai/its-a-long-road-to-recovery-for-hepatitis-cpatients-at-stanley/article8553352.ece
http://www.who.int/features/qa/76/en/
http://www.thehindu.com/sci-tech/health/two-million-people-coinfected-with-hiv-hepatitis-cglobally/article8341900.ece

Q.17) Consider the following statements with respect to Maglev Technology


1. Maglev trains does not have engine.
2. It does not emit greenhouse gases.
3. It is generally preferred for short distance commutation.

Which of the above given statements is/are correct?


a)
b)
c)
d)

2 only
1 & 2 only
2 & 3 only
None of the above

Q.17) Solution (b)

Maglev Technology is generally used for Mono rails which doesnt operates based upon
engines. Moreover the Magnetic levitation doesnt use any kind of fossil fuels so it wont
release any of the greenhouse gases.
Monorail system is preferred for long distance commutation only metro rail is preferred
for short distance commutation.

Q.18) Consider the following statements about Bio Digester Technology


1. Its a green technology developed by CSIR(Council of Scientific & Industrial Research)
2. It uses anaerobic digestion method.
3. Process is carried out by using Bacteria and Fungi.

Which of the above given statements is/are correct?


a)
b)
c)
d)

2 only
1 & 2 only
2 & 3 only
1,2 & 3

Q.18) Solution (a)

Bio-digester technology is been carried out only with the help of bacteria and not by
using fungi. Moreover the same question can be asked in the perspective of Dark
Fermentation.
It is developed by DRDO

Q.19) Consider the following statements with respect to Green Diesel


1. Green Diesel is generally referred to as Biodiesel.
2. It is produced through Hydro cracking technology.
3. It possesses same chemical properties as petroleum based diesel.

Which of the above given statements is/are not correct?


a)
b)
c)
d)

1 only
2 only
1 & 2 only
1, 2 & 3

Q.19) Solution (a)

Green diesel is entirely different from the Bio diesel.


The major difference between them is the process of making. Biodiesel is processed by
using transesterification, while the green diesel is processed by fractional distillation like
fossil origin by Hydrocracking technology.
Moreover the chemical composition between them is also different.
Green diesel possesses same chemical properties as petroleum based diesel.

Q.20) Consider the following


1. Kolkata Metro Rail Corporation
2. Chennai Metro Rail Limited
3. Rapid Metro Rail Gurgaon

Which of the above metro rail corporation works under Indian Railways?
a)
b)
c)
d)

1 only
2 and 3 only
1 and 2 only
1 and 3 only

Q.20) Solution (a)

Chennai Metro rail is operated by the Ministry of urban development. Rapid metro rail
Gurgaon is Indias first fully privately financed metro and the first metro system in the
country to auction naming rights for its stations.

Q.21) Nirbhaya Fund is set up under which of the following Ministry?


a)
b)
c)
d)

Ministry of Health and Family Welfare


Ministry of Women and Child Development
Ministry of Finance
Minister of Home

Q.21) Solution (c)


Ministry of Finance has constituted Nirbhaya Fund with a corpus of 1000 crores

IASbaba 60 Day Plan- Prelims Test 2016 ENVIRONMENT & CURRENT AFFAIRS [DAY 30]

Q.1) One of the most accepted definitions of sustainable development is Development that
meets the needs of the present without compromising the ability of future generations to
meet their own needs." This definition is given by which of the following commissions?
a)
b)
c)
d)

World Commission on Environment and Development


Environment and energy commission
European Commission on environment
Sustainable and Legacy Commission

Q.1) Solution (a)


The definition was given on 1987 by World Commission on Environment and Development,
popularly known as the Brundtland Commission.

Q.2) Consider the following statements:


1. Agenda 21 is a binding action plan of the United Nations with regard to Sustainable
Development.
2. It is a product of Earth Summit held at Rio de Jeneiro in 1992.
3. 21 in Agenda 21 refers to 21 points of the document that has been pledged for
sustainable development.
Which of the above statements are incorrect?
a)
b)
c)
d)

1 and 2
2 and 3
1 and 3
None of the above

Q.2) Solution (c)


Agenda 21 is a non-binding, voluntarily implemented action plan of the United Nations with
regard to sustainable development. It is a product of the Earth Summit (UN Conference on
Environment and Development) held in Rio de Janeiro, Brazil, in 1992. It is an action agenda for
the UN, other multilateral organizations, and individual governments around the world that can
be executed at local, national, and global levels. The "21" in Agenda 21 refers to the
21st Century. It has been affirmed and had a few modifications at subsequent UN conferences.

www.iasbaba.com

Page 1

IASbaba 60 Day Plan- Prelims Test 2016 ENVIRONMENT & CURRENT AFFAIRS [DAY 30]

Q.3) Heat from the earth can be used as an energy source in many ways, from large and
complex power stations to small and relatively simple pumping systems. This heat energy is
known as geothermal energy. Consider the following statements regarding geo thermal
energy:
1. The predominant source of the Earths heat is the gradual decay of radioactive isotopes.
2. It has extensive global distribution and it is accessible in both developed as well as
under developed nations.
3. It has low emission of sulphur, CO2 and other greenhouse gases.
4. It is independent of external supply and demand effects and fluctuations in exchange
rates.
Which of the above statements are correct?
a)
b)
c)
d)

1,2 and 3
2,3 and 4
1,2 and 4
All of the above

Q.3) Solution (d)


Heat from the earth can be used as an energy source in many ways, from large and complex
power stations to small and relatively simple pumping systems. This heat energy, known as
geothermal energy, can be found almost anywhereas far away as remote deep wells in
Indonesia and as close as the dirt in our backyards.
Many regions of the world are already tapping geothermal energy as an affordable and
sustainable solution to reducing dependence on fossil fuels, and the global warming and public
health risks that result from their use.
The general characteristics of geothermal energy that make it of significant importance for both
electricity production and direct use include:

Extensive global distribution; it is accessible to both developed and developing


countries.

Environmentally friendly nature; it has low emission of sulphur, CO2 and other
greenhouse gases.

Indigenous nature; it is independent of external supply and demand effects and


fluctuations in exchange rates.

www.iasbaba.com

Page 2

IASbaba 60 Day Plan- Prelims Test 2016 ENVIRONMENT & CURRENT AFFAIRS [DAY 30]

Independence of weather and season.

Contribution to the development of diversified power sources.

Q.4) Solar energy is radiant light and heat from the Sun that is harnessed using a range of
ever-evolving technologies. Solar power is the conversion of sunlight into electricity, either
directly using photovoltaics (PV), or indirectly using concentrated solar power (CSP). Consider
the following statements regarding Solar Power Technology:
1. Photovoltaic cells use the ultraviolet radiations to convert solar energy into electric
current.
2. Semiconductors (usually silicon) are used to manufacture solar panels.
3. Concentrated Solar power use infrared radiations to heat the water and rotate turbines
with the generated steam.
4. Concentrated Solar Power systems generally use a huge convex lens to concentrate
energy at its focus.
Which of the above statements are correct?
a)
b)
c)
d)

1 and 2
3 and 4
2 and 3
1,2 and 3

Q.4) Solution (c)


Photovoltaic cells use photons or light radiations to produce electric current.
Concentrated Solar Power systems usually use large set of plane mirrors or concave mirrors to
focus sunlight.

Q.5) Wind energy has a very high potential in India. Indian government has already proposed
a National Wind Mission on the lines of National Solar Mission. Which of the following
statements are correct Regarding Wind energy in India?
1. In 2015-16, Indian Government fell short of achieving its target of wind energy
production.
2. The installed capacity of wind power in India is more than 25000MW.
3. East and North East region of India has no grid connected wind power plant.
www.iasbaba.com

Page 3

IASbaba 60 Day Plan- Prelims Test 2016 ENVIRONMENT & CURRENT AFFAIRS [DAY 30]

4. The highest wind power production in India is in Gujarat.


Select the code from the following:
a)
b)
c)
d)

None of the above


2 and 3
1 and 4
1,2 and 3

Q.5) Solution (b)


As of 31 March 2016 the installed capacity of wind power in India was 26,769 MW, mainly
spread across South, West and North regions. East and North east regions have no grid
connected wind power plant as of March, 2015 end. No offshore wind farm utilizing traditional
fixed-bottom wind turbine technologies in shallow sea areas or floating wind turbine
technologies in deep sea areas are under implementation. However, an Offshore Wind Policy
was announced in 2015 and presently weather stations and LIDARs are being set up by NIWE at
some locations.
Tamil Nadu has the highest installed capacity of wind power in India.

Q.6) Consider the following statements regarding National Mission on Sustainable Habitats:
1. It is one of the missions under National Action Plan on Climate Change.
2. It works for the development and protection of natural habitats, especially of those
species which are at the verge of extinction due to habitat loss.
3. The mission works on development of green corridors to connect protected areas of
India.
Which of the above statements are incorrect?
a)
b)
c)
d)

1 and 2
2 and 3
3 only
None of the above

Q.6) Solution (b)


The national mission on sustainable habitat approved by the Prime Minister. It is one of the
nine missions under National Action Plan on Climate Change and aims to make cities

www.iasbaba.com

Page 4

IASbaba 60 Day Plan- Prelims Test 2016 ENVIRONMENT & CURRENT AFFAIRS [DAY 30]

sustainable through improvements in energy efficiency in buildings, management of solid waste


& shift to public transport.

Q.7) Carbon neutrality refers to achieving net zero carbon emissions by balancing a measured
amount of carbon released with an equivalent amount sequestered or offset, or buying
enough carbon credits to make up the difference. Which of the following countries are carbon
neutral?
a)
b)
c)
d)

Denmark
Bhutan
Iceland
Sweden

Q.7) Solution (b)


Two countries have achieved carbon neutrality: Vatican city and Bhutan
Several countries have pledged carbon neutrality, including: British Columbia (Canadian
province), Costa Rica, Iceland, Maldives, New Zealand, Norway, Tuvalu and Sweden.

Q.8) Buildings contribute maximum to the green house gas emissions in the world. To reduce
the carbon foot print of buildings, the concept of green buildings was given. Consider the
following regarding Green Buildings:
1. Green building refers to both a structure and the using of processes that are
environmentally responsible and resource-efficient throughout a building's life-cycle.
2. GRIHA, an acronym for Green Rating for Integrated Habitat Assessment, is the National
Green Building Rating System of India.
3. GRIHA has been developed by TERI.
Which of the above statements are correct?
a)
b)
c)
d)

1 and 2
2 and 3
1 and 3
All of the above

Q.8) Solution (d)


www.iasbaba.com

Page 5

IASbaba 60 Day Plan- Prelims Test 2016 ENVIRONMENT & CURRENT AFFAIRS [DAY 30]

Green building (also known as green construction or sustainable building) refers to both a
structure and the using of processes that are environmentally responsible and resourceefficient throughout a building's life-cycle: from siting to design, construction, operation,
maintenance, renovation, and demolition. In other words, green building design involves
finding the balance between homebuilding and the sustainable environment. This requires
close cooperation of the design team, the architects, the engineers, and the client at all project
stages. The Green Building practice expands and complements the classical building design
concerns of economy, utility, durability, and comfort.
Leadership in Energy and Environmental Design (LEED) is a set of rating systems for the design,
construction, operation, and maintenance of green buildings which was Developed by the U.S.
Green Building Council.
GRIHA, an acronym for Green Rating for Integrated Habitat Assessment, is the National Rating
System of India. TERI took the responsibility of popularizing green building by developing a tool
for measuring and rating a building's environmental performance in the context of India's
varied climate and building practices.

Q.9) Which of the following are the result of modern agriculture?


1.
2.
3.
4.
5.

Nitrate pollution
Eutrophication
Biomagnification
Ozone Depletion
Salinisation

Select the code from the following:


a)
b)
c)
d)

1,2 and 3
1,2,3 and 5
2,3,4 and 5
All of the above

Q.9) Solution (b)


Direct question.
Ozone depletion is not the result of agricultural activities.

www.iasbaba.com

Page 6

IASbaba 60 Day Plan- Prelims Test 2016 ENVIRONMENT & CURRENT AFFAIRS [DAY 30]

Q.10) Which of the following statements correctly explains an Urban Heat Island?
a) A city or a metropolitan area, which is significantly warmer than the surrounding rural
area, due to anthropogenic activities.
b) An island with sudden increase in urbanization activity, increasing its average
temperature.
c) A volcanic island where people have settled due to population pressure on the
mainland.
d) None of the above
Q.10) Solution (a)
An urban heat island (UHI) is a city or metropolitan area that is significantly warmer than its
surrounding rural areas due to human activities. The temperature difference usually is larger at
night than during the day, and is most apparent when winds are weak. The main cause of the
urban heat island effect is from the modification of land surfaces. Waste heat generated by
energy usage is a secondary contributor. As a population center grows, it tends to expand its
area and increase its average temperature. The less-used term heat island refers to any area,
populated or not, which is consistently hotter than the surrounding area.

Q.11) Consider the following


1. Mosses
2. Litchens
3. Microbes
Which among the above is not a pioneer species?
a)
b)
c)
d)

1 and 2 only
2 and 3 only
3 only
None

Q.11) Solution (d)


Lichens, mosses, microbes are some of the pioneer species in primary succession. In primary
succession on a terrestrial site the new site is first colonised by a few hardy pioneer species that
are often mosses, lichens, microbes.

www.iasbaba.com

Page 7

IASbaba 60 Day Plan- Prelims Test 2016 ENVIRONMENT & CURRENT AFFAIRS [DAY 30]

These new conditions may be conductive to the establishment of additional organisms that may
subsequently arrive at the site.

Q.12) A Biotic interaction in which one species is harmed and other is unaffected is
a)
b)
c)
d)

Competition
Mutualism
Commensalism
Amensalism

Q.12) Solution (d)


Amensalism is a biotic interaction in which one species is harmed and other is unaffected. For
ex- a large tree shades a small plant, retarding the growth of small pants. The small plants have
no effect on the large tree.

Q.13) Consider the following


1. Durand line separates India and Pakistan
2. Siegfried line separates Germany and France
3. Mannerheim line separates Russia and Finland
Which of the above are correctly matched?
a)
b)
c)
d)

1 and 3 only
1 and 2 only
2 and 3 only
All

Q.13) Solution (c)


Durand line is between Pakistan and Afghanistan

Q.14) Consider the following with respect to Upgrading Skills and Training in Traditional
Arts/Crafts Development (USTTAD) Scheme
www.iasbaba.com

Page 8

IASbaba 60 Day Plan- Prelims Test 2016 ENVIRONMENT & CURRENT AFFAIRS [DAY 30]

1. It aims for capacity building of traditional artisans and craftsman belonging to minorities
community
2. The scheme is implemented by Ministry of Skill Development and Entrepreneurship
Select the correct answer
a)
b)
c)
d)

1 only
2 only
Both
None

Q.14) Solution (a)


It aims for capacity building of traditional artisans and craftsman belonging to minorities
community, it is implemented by Ministry of Minorities Affairs

Q.15) Consider the following statements with respect to first Jain Council
1. It was convened at Pataliputra in 3rd century BC by Sthalabahu
2. The first Jain council resulted in the compilation of 12 Angas
Select the correct answer
a)
b)
c)
d)

1 only
2 only
Both
None

Q.15) Solution (c)


-The first Jain Council was convened at Pataliputra by Sthalabahu,in the beginning of the 3rd
century B.C. It resulted into the compilation of 12 Angas replacing the lost 14 purvas.
-The second Jain Council was held at Valabhi in 5th century A.D. It resulted into he final
compilation of 12 Angas and 12 Upangas.

www.iasbaba.com

Page 9

IASbaba 60 Day Plan- Prelims Test 2016 ENVIRONMENT & CURRENT AFFAIRS [DAY 30]

Q.16) Refer the diagrams below. Which diagram best represents the average monthly
temperature changes during a year for a tundra in the Northern Hemisphere?

Q.16) Solution (a)


Explanation:
Diagram (a) best represents the average monthly temperature changes during a year for a
tundra in the Northern Hemisphere.
A tundra is a level or undulating, treeless plain characteristic of arctic and subarctic regions
with long, cold winters and short, cool summers.

Q.17) Where do the fungi obtain nutrients and energy?


a) The Sun
b) The primary producers only
c) The consumers only
d) All members of the community
Q.17) Solution (d)
Explanation:
www.iasbaba.com

Page 10

IASbaba 60 Day Plan- Prelims Test 2016 ENVIRONMENT & CURRENT AFFAIRS [DAY 30]

Fungi, as a group, are heterotrophic organisms with an absorptive mode of nutrition. They
can obtain energy and nutrients from all members of the community.

Q.18) A lichen is best characterized as a symbiosis between which of the following?


a) A liverwort and a fungus
b) A moss and a liverwort
c) An alga and a moss
d) An alga and a fungus
Q.18) Solution (d)
Explanation:
A lichen represents a symbiotic relationship between an alga and a fungus.

Q.19) Of the following, the major cause of infant mortality worldwide is


a) starvation
b) waterborne diseases
c) toxic chemicals
d) nuclear radiation
Q.19) Solution (b)
Explanation:
Water is used for sewage systems, drinking, washing activities, and food production. Cross
contamination of drinking and washing water with sewage is a serious problem worldwide
that leads to the spread of waterborne diseases and infant mortality.

Q.20) All of the following are likely to increase after large areas of tropical rain forests are cut
down EXCEPT the
a) species diversity of the areas
b) erosion by rivers fiowing through the areas
c) rate of nutrient loss from the areas
d) average surface temperature of the soil in the areas
Q.20) Solution (a)
Explantion:
www.iasbaba.com

Page 11

IASbaba 60 Day Plan- Prelims Test 2016 ENVIRONMENT & CURRENT AFFAIRS [DAY 30]

The correct answerthe exceptionis (a). The continuing loss of the diverse habitats found
in tropical rain forests will cause a decrease in species diversity, not an increase.

Q.21) Observations of lakes in areas with granitic bedrock indicate that the lakes are
becoming depleted of living organisms. The primary cause is considered to be
a)
b)
c)
d)

nuclear waste
lowered water levels
acid rain
garbage dumping

Q.21) Solution (c)


Explanation:
Rain combines with various types of air pollution to form acid rain.
Lakes formed in limestone bedrock can neutralize the acidity of the rain, but lakes with
granitic bedrock have limited capacity to do so.
The increasing acidity of such lakes has had a negative impact on the organisms normally
inhabiting them.

Q.22) Which of the following is a true statement about the flow of energy in an ecosystem?
a)
b)
c)
d)

Smaller organisms need less energy per gram of body weight than do larger organisms.
Energy transfer between organisms normally involves conservation of heat energy.
Energy flow between trophic levels is inefficient.
Chemical energy is converted into radiant energy, which is then converted into chemical
energy at the next trophic level.

Q.22) Solution (c)


Explanation:
Biologists estimate that approximately 90 percent of the energy is lost between levels of an
energy pyramid. Thus, it is true that energy is transferred between trophic levels
inefficiently.

www.iasbaba.com

Page 12

IASbaba 60 Day Plan- Prelims Test 2016 ENVIRONMENT & CURRENT AFFAIRS [DAY 30]

Q.23) Which of the following is most likely the result of decreasing levels of ozone in the
stratosphere?
a)
b)
c)
d)

A decrease in the levels of smog in major cities


A decrease in the rate of global warming
An increase in the occurrence of skin cancer in humans
An increase in photosynthetic activity of phytoplankton

Q.23) Solution (c)


Explanation:
Ground-level ozone is involved with smog formation, not ozone in the stratosphere.
While tropospheric ozone is considered a greenhouse gas, stratospheric ozone is not
thought to have a significant effect on global warming.
Decreasing levels of stratospheric ozone result in increased levels of ultraviolet radiation
reaching the ocean, which may inhibit phytoplankton photosynthetic activity in surface
waters.
The increased levels of ultraviolet radiation can also negatively affect human health.

Q.24) Weather events characterized by heavy rain, thunderstorms, and a sharp temperature
drop followed by clearing are most likely related to which of the following?
a)
b)
c)
d)

The passage of a cold front


The passage of a warm front
The action of a stationary front
The heat island effect

Q.24) Solution (a)


Explanation:
The passage of a cold front is often accompanied by rain and sometimes thunder.
There is generally a sharp decrease in temperature while the front is passing and a
continuing decrease in temperature after it has passed.
Passage of the front also typically brings a decrease in precipitation followed by clearing.
The other choices are not consistent with this set of conditions.

www.iasbaba.com

Page 13

IASbaba 60 Day Plan- Prelims Test 2016 ENVIRONMENT & CURRENT AFFAIRS [DAY 30]

Q.25) Which of the following substances is found in the runoff of chemically fertilized lawns
and is most likely to cause eutrophication in streams and lakes?
a) Sodium
b) Carbonate
c) Phosphate
d) Sulfate
Q.25) Solution (c)
Explanation:
Excess phosphates from lawn fertilizers can be carried by runoff into streams and lakes,
causing rapid plant growth and algal blooms. Sodium, carbonate, and sulfate are less likely
to be limiting nutrients and supplied by chemical lawn fertilizers.

Q.26) Methane, a gas that can be trapped and used for energy, is most likely to be produced
by which of the following?
a) Decomposition of organic materials in landfills
b) Burning of organic materials in incinerators
c) Heating of inorganic materials in a high oxygen atmosphere
d) Recycling of metals in recycling plants
Q.26) Solution (a)
Explanation:
Methanogenic microorganisms can produce methane from organic material in landfills.
Burning of organic material will not produce methane. Heating of inorganic materials in a
high-oxygen atmosphere or the recycling of metals are not likely to produce significant
methane.

Q.27) In the discussion of the nature of and solution to environmental problems, it is most
important for students to understand that
a) science is constantly advancing, and technology will be developed to solve current
problems
b) changes in the abiotic portion of the biosphere are self-correcting and result in
evolution of the biotic portion of the biosphere
c) once pollution has been stopped, communities will revert to their preindustrial status
d) cultural, political, and economic issues must all be considered, in addition to scientific
issues
www.iasbaba.com

Page 14

IASbaba 60 Day Plan- Prelims Test 2016 ENVIRONMENT & CURRENT AFFAIRS [DAY 30]

Q.27) Solution (d)


Explanation:
Understanding and finding solutions to environmental problems requires a consideration of
scientific, economic, political, and cultural issues.
Humans cannot assume that advances in science and technology alone will solve current
environmental problems.
There is no evidence that organisms will be able to evolve and self-correct in response to
the rapidly changing environment.
Since a number of organisms have gone extinct and large amounts of habitat have been
destroyed, simply stopping pollution cannot guarantee that communities will revert to their
preindustrial status.

Q.28) Which of the following hypotheses is LEAST amenable to verification by


experimentation?
a) Using compound X to control insect pests that damage crops also reduces the
population levels of pollinator insects.
b) Building a new factory in a town with high unemployment is more important than
preserving the habitat the factory will destroy.
c) Applying chemical fertilizers above recommended amounts increases the concentration
of inorganic nutrients in runoff.
d) Humans and chimpanzees share some of the same genes.
Q.28) Solution (b)
Explanation:
The correct answerthe LEAST amenable to verification by experimentationis choice (B).
The hypothesis that jobs are more important than habitat cannot be tested by
experimentation.
The effects of chemicals on insects and of fertilizers on nutrients in runoff, as well as the
incidence of the same genes occurring in different species, can all be measured and tested
by experimentation.

www.iasbaba.com

Page 15

IASbaba 60 Day Plan- Prelims Test 2016 HISTORY & CURRENT AFFAIRS [DAY 31]

Q.1) Although modern educated class, including Congress leadership was not happy with the
provisions of the Indian Councils Act of 1892, it was considered as a major achievement of
moderate INC. Which of the following statements are correct about Indian Councils Act 1892?
1. The Act was passed by the Central Legislative Council of India.
2. The idea of election was first time introduced through this Act.
3. The Act provided for additional members in Central as well as Provincial Legislative
Council.
Select the code from the following:
a)
b)
c)
d)

1 and 2
2 and 3
1 and 3
All of the above

Q.1) Solution (b)


The Act was passed by the parliament of Britain.
A system of indirect elections was introduced to elect the members of the councils. The
universities, district board, municipalities, zamindars and chambers of commerce were
empowered to recommend members to provincial councils.

Q.2) Consider the following statements:


1. Kaiser I Hind medal was awarded by the Indian Government for exceptional Public
service.
2. Mahatma Gandhi was awarded Kaiser I Hind for recruiting volunteers for British Indian
Army during World War I.
3. Mahatma Gandhi returned his award to protest against the draconian Rowlatt Act.
Which of the above statements are correct?
a)
b)
c)
d)

1 and 2
2 and 3
1 and 3
None of the above

www.iasbaba.com

Page 1

IASbaba 60 Day Plan- Prelims Test 2016 HISTORY & CURRENT AFFAIRS [DAY 31]

Q.2) Solution (d)


Kaiser I hind was awarded by British Monarch for exceptional public service.
Mahatma Gandhi got this award for his ambulance service in South Africa during Boer War.
He returned his medal to protest against Jalianwala Bagh Massacre

Q.3) Abhinav Bharat Society was one of the most significant of the secret societies of
Maharashtra. Consider the following statements regarding Abhinava Bharat society:
1. It was initially started by Savarkar Brothers as Mitra Mela in Nasik.
2. It believed in overthrow of British government through armed rebellion.
3. It was involved in the killing of A.M.T. Jackson, the district magistrate of Nasik, the case
popularly known as The Nasik Conspiracy case.
Which of the following statements are correct?
a)
b)
c)
d)

1 and 2
2 and 3
1 and 3
All of the above

Q.3) Solution (d)


Vinayak Savarkar and Ganesh Savarkar started Mitra Mela, a revolutionary secret society in
Nasik in 1903. It was one among several such melas (revolutionary societies) functioning in
Maharashtra at that time, which believed in the overthrow of British rule through armed
rebellion. In 1904, in a meeting attended by 200 members from various towns in Maharashtra,
Vinayak Savarkar renamed it Abhinav Bharat, taking after Giuseppe Mazzini's Young Italy.
A. M. T. Jackson, the district magistrate of Nasik, was assassinated in India by Anant Laxman
Kanhare in 1909 in the historic "Nasik Conspiracy Case".
The investigation into the Jackson assassination revealed the existence of the Abhinav Bharat
Society and the role of the Savarkar brothers in leading it. Vinayak Savarkar was found to have
dispatched twenty Browning pistols to India, one of which was used in the Jackson
assassination. He was charged in the Jackson murder and sentenced to "transportation" for life.
Savarkar was imprisoned in the Cellular Jail in the Andaman Islands in 1910.
www.iasbaba.com

Page 2

IASbaba 60 Day Plan- Prelims Test 2016 HISTORY & CURRENT AFFAIRS [DAY 31]

Q.4) Consider the following statements regarding Sir William Jones:


1. He founded the Asiatic Society of Bengal.
2. The Gentoo Code or the legal code was translated by him from Sanskrit to English.
Which of the above statements are correct?
a)
b)
c)
d)

1 only
2 only
Both 1 and 2
Neither 1 nor 2

Q.4) Solution (a)


His most famous accomplishment in India was establishing the Asiatic Society of Bengal, in
January of 1784. The founding of the Society grew out of Jones's love for India, its people and
its culture, as well as his abhorrence of oppression, nationalism and imperialism.
The Gentoo Code is a legal code translated from Sanskrit (in which it was known as
vivdravasetu) into Persian by Brahmin scholars; and then from Persian into English by
Nathaniel Brassey Halhed, a British grammarian working for the East India Company. The
translation was funded and encouraged by Warren Hastings as a method of increasing colonial
hold over the Indies.

Q.5) Which of the following is the oldest Women University of India?


a)
b)
c)
d)

Shreemati Nathibai Damodar Thackersey Women's University (SNDT)- Mumbai


Jesus and Mary College- Delhi
Lady Sri Ram College for Women- Delhi
Sri Padmavati Mahila Viswavidyalayam Tamil Nadu

Q.5) Solution (a)


SNDT Women's University is the first Women's university in India as well as in South-East Asia.
It was founded in 1916.

www.iasbaba.com

Page 3

IASbaba 60 Day Plan- Prelims Test 2016 HISTORY & CURRENT AFFAIRS [DAY 31]

Q.6) Consider the following statements:


1. A servant of India Society was founded by Gopal Krishna Gokhale in London.
2. The aim of the society was to create awareness about imperialist plans and exploitation
of India by Britain.
Which of the above statements are correct?
a)
b)
c)
d)

1 only
2 only
Both 1 and 2
Neither 1 nor 2

Q.6) Solution (d)


The Servants of India Society was formed in Pune, Maharashtra, on June 12, 1905 by Gopal
Krishna Gokhale. The Society organized many campaigns to promote education, sanitation,
health care and fight the social evils of untouchability and discrimination, alcoholism, poverty,
oppression of women and domestic abuse. The publication of The Hitavada, the organ of the
Society in English from Nagpur commenced in 1911.

Q.7) The Deobandi movement was started in 1866 by orthodox Sunni Muslims. Which of the
following statements are incorrect about Deobandi Movement?
1. It was a revivalist Islamic movement to propagate pure teachings of Quran and Hadith
and to keep alive the spirit of Jihad again the foreign invader.
2. They were against the Aligarh movement and Sir Syed Ahmad.
3. They gave full support to formation of Indian National Congress.
Select the code from the following:
a)
b)
c)
d)

1 and 2
1 and 3
2 and 3
None of the above

Q.7) Solution (d)


www.iasbaba.com

Page 4

IASbaba 60 Day Plan- Prelims Test 2016 HISTORY & CURRENT AFFAIRS [DAY 31]

Factual question.
All the above statements are correct about Deobandi movement. Hence none of the above
statement is incorrect.

Q.8) Consider the following statements:


1. Lucknow Pact refers to an agreement between The Indian National Congress and The
Muslim League.
2. Jinnah, member of Congress as well as league, was the mastermind and architect of this
pact.
3. Same resolutions were passed by the Congress and the Muslim league in a joint session.
4. Lucknow session also established cordial relations between the moderate and extremist
factions within Congress.
Which of the above statements are correct?
a)
b)
c)
d)

1,2 and 3
2,3 and 4
1,3 and 4
All of the above

Q.8) Solution (d)


The Lucknow Session 1916 {presided by Ambica Charan Majumdar) was special in many
respects. Firstly, this session brought the moderates and extremists in Congress on common
platform again after nearly a decade, particularly due to efforts of Annie Besant.
Secondly, Congress and All India Muslim League signed the historic Lucknow Pact.
The idea was that such joint demand would give an impression of Hindu-Muslim unity.
kindly read the points of the joint resolution as a question can be asked on that. (100 years)

www.iasbaba.com

Page 5

IASbaba 60 Day Plan- Prelims Test 2016 HISTORY & CURRENT AFFAIRS [DAY 31]

Q.9) The Bardoli Satyagraha was a major event of civil disobedience and revolt in the Indian
Independence Movement. Which of the following statements are correct about this
movement?
1. The taluka Bardoli in Gujrat had suffered huge loss due to floods and famine. Despite
this fact, the Bombay presidency raised the tax by 30%.
2. The Satyagraha was led by Congress under the leadership of Mahatma Gandhi.
3. The farmers refused the payment of taxes. In retaliation, government forcefully ceased
all the property including cattle.
Select the code from the following:
a)
b)
c)
d)

1 and 2
2 and 3
1 and 3
All of the above

Q.9) Solution (c)


Bardoli Satyagraha was led by Vallabh Bhai Patel. Gandhi and Patel agreed that neither the
Congress nor Gandhi would directly involve themselves, and the struggle left entirely to the
people of Bardoli taluka.

Q.10) Who of the following eminent personalities gave the title Mahatma to Gandhiji?
a)
b)
c)
d)

Sardar Vallabbhai Patel


B.R.Ambedkar
Dr.Rajendra Prasad
Rabindranath Tagore

Q.10) Solution (d)


Recently a controversy has been going on regarding who gave the title to Mahatma Gandhi.
Since the History books tell us it was given by Rabindranath Tagore. We will stick to this answer
only.

www.iasbaba.com

Page 6

IASbaba 60 Day Plan- Prelims Test 2016 HISTORY & CURRENT AFFAIRS [DAY 31]

Q.11) Consider the following statements regarding Ahmadabad Mill strike:


1. There was a conflict between the mill owners and the workers on the issue of plague
bonus and wage hike.
2. This was the first time Mahatma Gandhi used the method of Satyagraha (non-violent
protest) in India.
3. The workers wanted a wage hike of 50% while Gandhiji asked them to settle at 35%.
Which of the above statements are correct?
a)
b)
c)
d)

1 and 2
2 and 3
1 and 3
All of the above

Q.11) Solution (c)


This was the first time Mahatma Gandhi went on a Hunger Strike.

Q.12) After sudden suspension of the non-cooperation movement the youth of the country
was not satisfied. They wanted to overthrow the British rule by violence and revolutionary
activities. One of the most prominent organizations that emerged during this time was
Hindustan Republican Association (HRA). Which of the following statements are correct
about HRA?
1. It was organized by Chandra Shekhar Azad and Bhagat Singh.
2. The aim was to establish Federal Republic of the United States of India by violent
revolution.
3. The manifesto of HRA was called The Revolutionary.
Select the code from the following:
a)
b)
c)
d)

1 and 2
2 and 3
1 and 3
All of the above

www.iasbaba.com

Page 7

IASbaba 60 Day Plan- Prelims Test 2016 HISTORY & CURRENT AFFAIRS [DAY 31]

Q.12) Solution (b)


Sachindra Sanyal, Narendra Mohan Sen and Pratul Ganguly organized HRA in East Bengal.
The name was chosen as Hindustan Republican Association on the lines of Irish Republican
Army.
The manifesto of this revolutionary organization was The Revolutionary.
The Objective of the HRA was to establish Federated Republic of the United States of India
through an organized armed revolution.
The tactics of HRA were killing the officials; organize political dacoties to raise funds, terrorism
among the British and British loyalists and strikes against the raj. However, it is incorrect to say
that these revolutionaries were terrorists by attitude.
The perspective of the HRA was socialistic and it wanted to establish a United States of India by
deposing the British. The idea attracted the young champions; some immediately joined the
organization were Bhagat Singh, ChandraShekhar Azad, Sukhdev, Ram Prasad Bismil, Roshan
Singh, Ashfaqulla Khan, Rajendra Lahiri and many others.
The first organized crime of this group was the Kakori Train Conspiracy.

Q.13) Consider the following statements regarding the Self-Respect Movement:


1. It was founded by E.V. Ramaswamy against Brahmanism in Kerala.
2. It aimed to achieve a society where backward classes have equal rights.
3. The movement promoted inter-caste marriages which were not officiated by a Brahmin
Priest.
Which of the above statements are correct?
a)
b)
c)
d)

1 only
2 and 3
1 and 2
None of the above

Q.13) Solution (b)


It was started by E.V. Ramaswamy (popularky known as Periyar) in Tamil Nadu.

www.iasbaba.com

Page 8

IASbaba 60 Day Plan- Prelims Test 2016 HISTORY & CURRENT AFFAIRS [DAY 31]

Q.14) First Factory Act was introduced by Lord Ripon in 1881 to improve the condition of
factory workers in India. Which of the following were not the provisions of this Act?
1. The Act banned the appointment of children below 10 years of age.
2. The working hours for women were reduced to 8 hours a day.
3. It made compulsory for all dangerous machines in the factories to be properly fenced to
ensure security to the workers.
Select the code from the following:
a)
b)
c)
d)

1 and 2
2 and 3
1 and 3
None of the above

Q.14) Solution (a)


Lord Ripon introduced the Factory Act of 1881 to improve the service condition of the factory
workers in India. The Act banned the appointment of children below the age of seven in
factories. It reduced the working hours for children. It made compulsory for all dangerous
machines in the factories to be properly fenced to ensure security to the workers.
Read Factories Act II as well. The first bill focused on children while the second bill focused on
the working condition of women.

Q.15) Consider the following statements regarding Rani Gadinliu:


1. Gaidinliu was a Naga spiritual and political leader who led a violent revolt against the
British at the age of 13.
2. She came to be considered an incarnation of the goddess Cherachamdinliu.
3. The title Rani was given to her by Pt. Jawaharlal Nehru.
Which of the above statements are correct?
a)
b)
c)
d)

1 and 2
2 and 3
1 and 3
All of the above

www.iasbaba.com

Page 9

IASbaba 60 Day Plan- Prelims Test 2016 HISTORY & CURRENT AFFAIRS [DAY 31]

Q.15) Solution (d)


Factual question. Self Explanatory.

Q.16) Consider the following


1. Jasrota
2. Kishtwar
3. Dachigam
Which of the above wild life sanctuary/national park is/are found in Kashmir?
a)
b)
c)
d)

1&2 only
2&3 only
1&3 only
All

Q.16) Solution (d)


All are found in the state of Kashmir
Click here

Q.17) Consider the following


1. Uttarakhand
2. Himachal Pradesh
3. Kerala
Montane type of climate is found in which of the following states?
a)
b)
c)
d)

1 and 2
2 and 3
1 and 3
All

www.iasbaba.com

Page 10

IASbaba 60 Day Plan- Prelims Test 2016 HISTORY & CURRENT AFFAIRS [DAY 31]

Q.17) Solution (a)

The elevation at which one habitat changes to another varies across the globe,
particularly by latitude. The upper limit of montane forests, the forest line or timberline,
is often marked by a change to hardier species that occur in less dense stands.
Uttarakhand, Arunachal Pradesh, Himachal Pradesh, Jammu and Kashmir are some of
the states where montane climate is observed

Q.18) Consider the following


1. Spoon billed sandpiper
2. Ganges shark
3. Red panda
Which of the listed above species are critically endangered?
a)
b)
c)
d)

1 and 2
2 and 3
1 and 3
All

Q.18) Solution (a)

Spoon billed sandpiper, Ganges shark, large tooth saw fish, hawksbill turtle are some of
the critically endangered species.
Red panda which is found in the state of Sikkim is endangered
Click here

Q.19) Global investment trend monitor report is released by which of the following
international organisations United nation conference on trade and development
a)
b)
c)
d)

WTO
UNCTAD
IMF
ADB

UNCTAD, which is governed by its 194 member States, is the United Nations
body responsible for dealing with development issues, particularly international
trade the main driver of development

Q.19) Solution (b)


www.iasbaba.com

Page 11

IASbaba 60 Day Plan- Prelims Test 2016 HISTORY & CURRENT AFFAIRS [DAY 31]

UNCTAD releases global investment trend monitor report


Click here

Q.20) Consider the following statement


1. Green climate fund was created under United Nations framework convention on
climate change
2. Global environment facility is a financial mechanism to help developing countries to
mitigate climate change and was created during recently held COP 21
Select the correct answer using the codes given below
a)
b)
c)
d)

1 only
2 only
Both
None

Q.20) Solution (a)

The Global Environment Facility (GEF) was established as a pilot programme for
environmental protection. The current project cycle is GEF-6 over the years 2014-18. In
1992, when the Biodiversity and Climate Change Conventions were adopted at Rio de
Janeiro, the GEF was adopted as a financial mechanism for helping developing countries
meet their financing needs for achieving their climate change goals. As of November
2015, the GEF has directly invested a total of US$14.5 billion in 3946 projects in 167
countries, out of which US$4.2 billion is in 1010 projects for climate change mitigation.
Till date, India has received US$516.6 million of GEF grant, of which US$324.69 million is
for climate change mitigation projects while US$10 million is for climate change
adaptation projects.
The GCF was established as an operating entity of the financial mechanism of the
UNFCCC in 2011 and is expected to be a major channel for climate finance from
developed to developing countries.
Source Economic survey

www.iasbaba.com

Page 12

IASbaba 60 Day Plan- Prelims Test 2016 HISTORY & CURRENT AFFAIRS [DAY 31]

Q.21) Which of the following are the views of the Orientalists, in regard to strengthen the
hold of British rule in India?
1.
2.
3.
4.

Intellectual curiosity and romanticizing of India


Need of reform imposed from outside
Wanted gradual change via reason
Wanted immediate change via force if necessary

Select the correct answer from the following


a)
b)
c)
d)

1 only
1 and 3 only
1, 2 and 3 only
2, 3 and 4 only

Q.21) Solution (b)


Explanation:

The British devised several strategies to make their rule effective. The early British
administrators in India like Warren Hastings, William Jones, Jonathan Duncan and others
glorified Indias ancient past (romanticizing of India). These scholars and administrators
were called Orientalists.
They thought that a better understanding of Indian languages, literature and culture would
make it easier for them to rule India.
The first orientalists were 19th century scholars who translated the writings of the orient
into English, based on the assumption that a truly effective colonial conquest required
knowledge of the conquered peoples. By knowing the orient, the west came to own it. The
orient became the studied, the seen, the observed, and the object.
William Jones (17461794), the founder of the Asiatic Society of Bengal, who, with his vast
knowledge of Oriental peoples was the undisputed founder of scholarly Orientalism.
Most Orientalists had a kind of dual purpose of improving the quality of life of Indian
peoples and advancing arts and knowledge back in the heart of the Empire.
It should be noted here that the Orientalists never tried to bring reforms imposed from
outside nor wanted immediate change. They wanted to strengthen their hold by brining
gradual change without any force or violent means.

www.iasbaba.com

Page 13

IASbaba 60 Day Plan- Prelims Test 2016 HISTORY & CURRENT AFFAIRS [DAY 31]

Q.22) Which of the following initiatives were the ones which worked on the idea of a
common political organisation for the whole India?
1.
2.
3.
4.
5.

The East India Association


The Poona Sarvajanik Sabha
The Indian Association of Calcatta
The Madras Mahajana Sabha
Federation of the Native Pres

Select the correct answer from the options given below:


a)
b)
c)
d)

1, 2, 3 and 4 are correct


1, 2, 3 and 5 are correct
1, 4 and 5 are correct
All are correct

Q.22) Solution (b)


Explanation:

Self-explanatory. Except, the Madras Mahajana Sabha, all other initiatives worked on the
idea of a common political organization for the whole India
Madras Mahajana Sabha was considered to be a unique and holy organization which has
paved the way for India's national freedom by the South Indians.

Q.23) The moderate leaders political outlook was a happy combination of liberalism and
moderation. And hence they worked to procure for Indian:
1.
2.
3.
4.

Freedom from race and creed prejudice


Equality before law
Extension of civil liberties
Extension of representative institutions

Select the correct answer from the following?


a) 1 only
b) 1 and 2 only
c) 1, 2 and 3 only
www.iasbaba.com

Page 14

IASbaba 60 Day Plan- Prelims Test 2016 HISTORY & CURRENT AFFAIRS [DAY 31]

d) 1, 2, 3 and 4

Q.23) Solution (d)


Explanation:

All the statements are correct and self-explanatory

Q.24) Which of the following were the beliefs of revolutionaries, in regard to Indian national
movements?
1. The alien rule was destructive of all that is worthwhile in national life political liberties,
religious freedom, morality and culture.
2. The Western imperialism could only be ended by Western method of violence
Select the correct answer from the following?
a)
b)
c)
d)

Only 1 is correct
Only 2 is correct
Both are correct
Neither 1 nor 2 is correct

Q.24) Solution (c)


Explanation:

The Revolutionary movement for Indian independence is a part of the Indian independence
movement comprising the actions of the underground revolutionary factions.
Groups believing in armed revolution against the ruling British fall into this category, as
opposed to the generally peaceful civil disobedience movement spearheaded by Mohandas
Karamchand Gandhi.
The revolutionaries doubted the efficacy of peaceful methods and stressed the need of
forceful constitutional agitations.
Both the given statements are correct

www.iasbaba.com

Page 15

IASbaba 60 Day Plan- Prelims Test 2016 HISTORY & CURRENT AFFAIRS [DAY 31]

Q.25) Consider the following statements:


1. The INC was under the control of the Extremist when the I World war broke out in 1914
2. Still, the INC decided to support the British war efforts, both as a matter of duty and in a
spirit of bargaining to bet concession
Select the correct answer from the following?
a)
b)
c)
d)

Only 1 is correct
Only 2 is correct
Both are correct
Neither 1 nor 2 is correct

Q.25) Solution (b)


Explanation:

INC was under the control of the Moderates after the Surat Split of 1907
When the 1st World War broke out in 1914, INC was dominated by Moderates and
Extremists had lost their influence or control.

www.iasbaba.com

Page 16

IASbaba 60 Day Plan- Prelims Test 2016 HISTORY & CURRENT AFFAIRS [DAY 32]
Q.1) The script of Indus Valley civilization is considered to be the oldest script of the Indian
Sub continent. Which of the following statements are correct about this script?
1. Despite the efforts of linguists and symbologists, it is yet undeciphered.
2. The script is written in Boustrophedon fashion.
3. Square stamp seals are the dominant form of Indus writing media.
Select the code from the following:
a)
b)
c)
d)

1 and 2
2 and 3
1 and 3
All of the above

Q.1) Solution (d)


Boustrophedon is the writing style in which script is written from left to right in one line and
right to left in other.

Q.2) Which of the following statements are correct about the town planning of Indus Valley
Civilisation?
1. The towns had well laid covered drains and houses had toilets.
2. Bricks were used for the first time in the history by habitants of Indus Valley civilization.
3. All houses were single storied and with standard size showing equal economic status of
everyone.
4. Proper granaries were built to store surplus grains.
Select the code from the following:
a)
b)
c)
d)

1,2 and 4
1 and 4
1,3 and 4
All of the above

Q.2) Solution (b)


Burnt bricks were used for the first time.
www.iasbaba.com

Page 1

IASbaba 60 Day Plan- Prelims Test 2016 HISTORY & CURRENT AFFAIRS [DAY 32]
Houses were both single storied and multi-storied showing different economic status of people.

Q.3) Consider the following statements regarding Yoga:


1. It is amongst the six ideologies of Hinduism.
2. We have received the knowledge of Yoga from ancient written accounts of Patanjali.
3. Yoga is covered under AYUSH program as an indigenous therapy.
Which of the above statements are correct regarding Yoga?
a)
b)
c)
d)

1 and 3
1 and 2
2 and 3
All of the above

Q.3) Solution (d)


Self explanatory.

Q.4) Consider the following statements regarding Early Vedic Period:


1.
2.
3.
4.

Aryans referred to Indus people as Dasyus.


Early Aryans made large wooden castles to protect themselves from foreign attacks.
Instead of land, the tribes owned cattle as private property.
Aryans used Chariots instead of riding a horse.

Which of the above statements are correct?


a)
b)
c)
d)

1,2 and 3
2,3 and 4
1,3 and 4
All of the above

Q.4) Solution (c)


Early Aryans were nomads and did not make permanent settlements.
www.iasbaba.com

Page 2

IASbaba 60 Day Plan- Prelims Test 2016 HISTORY & CURRENT AFFAIRS [DAY 32]
Wealth was owned in terms of cattle and not land.
Horse riding was introduced in India by Kushans. Before them only chariots were used.

Q.5) Out of Six Astikas of Hindu philosophy, which of the following is the oldest?
a)
b)
c)
d)

Mimansa
Sankhya
Vedanta
Nyaya

Q.5) Solution (b)


Many Hindu intellectual traditions were classified during the medieval period of BrahmanicSanskritic scholasticism into a standard list of six orthodox (astika) schools (darshanas), the "Six
Philosophies" (a-darana), all of which accept the testimony of the Vedas.

Samkhya, the enumeration school


Yoga, the school of Patanjali (which provisionally asserts the metaphysics of Samkhya)
Nyaya, the school of logic
Vaisheshika, the atomist school
Purva Mimamsa (or simply Mimamsa), the tradition of Vedic exegesis, with emphasis on
Vedic ritual, and
Vedanta (also called Uttara Mimamsa), the Upanishadic tradition, with emphasis on
Vedic philosophy.

Q.6) Consider the following statements regarding the Prayag Prashasti:


1. It contains inscriptions praising the political and military achievement of Chandra Gupta
II.
2. The inscription were written by Harisena, the court poet and minister in Gupta Court.
3. The inscriptions are written in Pali language under Ashokan edict.
4. The edict contains the inscriptions of Ashoka, the Guptas and Jehangir.
Which of the above statements are incorrect?

www.iasbaba.com

Page 3

IASbaba 60 Day Plan- Prelims Test 2016 HISTORY & CURRENT AFFAIRS [DAY 32]
a)
b)
c)
d)

1 and 3
2 and 4
1,3 and 4
None of the above

Q.6) Solution (a)


Prayag Prashasti is attributed to the 4th century CE Gupta emperor, Samudragupta, follows
immediately below the edicts of Ashoka. It is considered "the most important historical
document of the classical Gupta age". It is in excellent Sanskrit, written in the more refined
Gupta script (a later version of Brahmi) by the poet and minister, Harishena. The inscription is a
panegyric praising Samudragupta and lists the political and military achievements of his reign
including his expeditions to the south. It provides a unique snapshot of the Gupta empire and
its neighbours and is the source of much of what is known of the geopolitical landscape of that
era.

Q.7) Current the Hindu calendar that is being used is modified Shaka Samvat- Saka calendar.
Consider the following statements:
1. Saka era was started by Kanishka in AD 78.
2. Kanishka was the founder of Saka dynasty in India.
Which of the above statements are correct?
a)
b)
c)
d)

1 only
2 only
Both 1 and 2
Neither 1 nor 2

Q.7) Solution (a)


Kanishka was a Kushan ruler.

Q.8) Which of the following Tamil Sangam literary piece of art are correctly matched with
their content:
www.iasbaba.com

Page 4

IASbaba 60 Day Plan- Prelims Test 2016 HISTORY & CURRENT AFFAIRS [DAY 32]
1.
2.
3.
4.

Thirukullar
Tolakappiam
Ettutukai
Silappadikkaram

Tamil grammar treatise


Philosophical work
Anthology
Epic

Select the code from following:


a)
b)
c)
d)

All of the above


1 and 2
3 and 4
2,3 and 4

Q.8) Solution (c)


Thirukullar is a philosophical work(couplets) written by Thiruvalluvar.
Tolakappiam is a tamil grammar treatise written by Tolakappiyar.

Q.9) Consider the following statements:


1. Jainism predates Buddhism, while Buddha was older than Mahavira.
2. Right faith, Right knowledge and Right Conduct are the Triratnas (three gems) of
Jainism.
3. To remove all sufferings Mahavira gave an eight fold path to be followed.
Which of the above statements are correct?
a)
b)
c)
d)

2 only
1 and 2
2 and 3
All of the above

Q.9) Solution (b)


Although Buddha was older than Mahavir, Jainism is believed to be started by Rishabh dev, the
first Tirthankar of Jainism. Mahavir is considered to be 24th and the last Tirthankar.
The three principles of Jainism, also known as Triratnas (threegems), are:
www.iasbaba.com

Page 5

IASbaba 60 Day Plan- Prelims Test 2016 HISTORY & CURRENT AFFAIRS [DAY 32]

right faith
right knowledge
right conduct.

The Four Noble Truths of Buddha are:


The world is full of suffering.
The cause of suffering is desire.
If desires are get rid off, suffering can be removed.
This can be done by following the Eightfold Path.
The Eightfold Path consists of right view, right resolve, rightspeech, right conduct, right
livelihood, right effort, right mindfulness and right concentration.

Q.10) Consider the following statements regarding Mudrarakshasa:


1. It is a historical play written by Kalidasa that narrates the ascent of king Chandragupta
Maurya to power in India.
2. Chanakya Neeti of Saam, Daam, Dand, Bhed is mentioned in Mudrarakshasa.
3. Mudrarakshasa is written in Sanskrit.
Which of the above statements are incorrect?
a)
b)
c)
d)

1 only
1 and 2
All of the above
None of the above

Q.10) Solution (a)


Mudrarakshasa was written by Vishakhadatta.

Q.11) Many new creeds and faiths emerged in the 6th century BC against the contemporary
Brahmanical society. One of the most staunch amongst them was the Ajivika Sect. Which of
the following statements are correct about the Ajivikas?
1. Ajivika was one of the Nastika school of Indian philosophy.
2. It was founded by Makkhali Gosala.
www.iasbaba.com

Page 6

IASbaba 60 Day Plan- Prelims Test 2016 HISTORY & CURRENT AFFAIRS [DAY 32]
3. Ajivikas do not believe in Karma philosophy and believe that whatever happens is
preordained by the cosmic order.
4. Bindusara was a follower of Ajivika Sect.
Select the code from the following:
a)
b)
c)
d)

1 and 4
2 and 3
1,3 and 4
All of the above

Q.11) Solution (d)


Ajivika is one of the nstika or "heterodox" schools of Indian philosophy. Purportedly founded
in the 5th century BCE by Makkhali Gosala, it was a ramaa movement and a major rival of
early Buddhism and Jainism. jvikas were organised renunciates who formed discrete
communities.
The jvika school is known for its Niyati doctrine of absolute determinism, the premise that
there is no free will, that everything that has happened, is happening and will happen is entirely
preordained and a function of cosmic principles. jvika considered the karma doctrine as a
fallacy. Ajivika metaphysics included a theory of atoms similar to the Vaisheshika school, where
everything was composed of atoms, qualities emerged from aggregates of atoms, but the
aggregation and nature of these atoms was predetermined by cosmic forces. jvikas were
atheists and rejected the authority of the Vedas, but they believed that in every living being is
an tman a central premise of Hinduism and Jainism.

Q.12) Milind or Menander-I was one of the most famous Indo-Bactrian King. The most
important source of information about him is Milind Panho written by Nagasena.
Which of the following statements about Milind Panho are correct?
1. It contains religious dialogues of Milind with a Buddhist monk.
2. It is written in Sanskrit.
Select the code from the following:
a) 1 only
b) 2 only
www.iasbaba.com

Page 7

IASbaba 60 Day Plan- Prelims Test 2016 HISTORY & CURRENT AFFAIRS [DAY 32]
c) Both 1 and 2
d) Neither 1 nor 2

Q.12) Solution (a)


Milind panho is written in Pali.

Q.13) Which of the following statements are correct about PanchaSiddhanta?


1. It was written by Aryabhatta.
2. It is a detailed account of Greek and Roman Astrology.
Select the correct code from the following:
a)
b)
c)
d)

1 only
2 only
Both 1 and 2
Neither 1 nor 2

Q.13) Solution (d)


Panchasiddhanta was written by Vrahamihira. It is a treatise on mathematical astronomy.

Q.14) Consider the following statements regarding the society in Chola kingdom:
1.
2.
3.
4.

Cholas were Shaivites and they patronized only Shiva temples.


The Devadasi system emerged during this period.
Practice of Sati was absent in the society.
The inscriptions during Chola rule, mentions about 6 broad caste divisions.

Which of the above statements are incorrect?


a)
b)
c)
d)

1,2 and 3
1,3 and 4
2,3 and 4
All of the above

www.iasbaba.com

Page 8

IASbaba 60 Day Plan- Prelims Test 2016 HISTORY & CURRENT AFFAIRS [DAY 32]

Q.14) Solution (b)


Caste system was widely prevalent during the Chola period. Brahmins and Kshatriyas enjoyed
special privileges. The inscriptions of the later period of the Chola rule mention about two
major
divisions among the castes Valangai and Idangai castes. However, there was cooperation
among various castes and sub-castes in social and religious life. The position of women did not
improve. The practice of sati was prevalent among the royal families. The devadasi system or
dancing girls attached to temples emerged during this period.
Both Saivism and Vaishnavism continued to flourish during the Chola period. A number of
temples were built with the patronage of Chola kings and queens.

Q.15) Which of the following inscription provide detailed information about the formation
and functioning of village assemblies during Chola period?
a) Tanjore Inscription
b) Mammalapuram inscription
c) Uttiramerur Inscription
d) Gangaikondacholapuram inscription
Q.15) Solution (c)
The system of village autonomy with sabhas and their committees developed through the ages
and reached its culmination during the Chola rule. Two inscriptions belonging to the period of
Parantaka I found at Uttiramerur provide details of the formation and functions of village
councils. That village was divided into thirty wards and each was to nominate its members to
the village council.
The qualifications to become a ward member were:
a. Ownership of at least one fourth veli of land.
b. Own residence.
c. Above thirty years and below seventy years of age.
d. Knowledge of Vedas.
However, certain norms of disqualification were also mentioned in the inscriptions. They
were:

www.iasbaba.com

Page 9

IASbaba 60 Day Plan- Prelims Test 2016 HISTORY & CURRENT AFFAIRS [DAY 32]
a. Those who had been members of the committees for the past three years.
b. Those who had failed to submit accounts as committee members.
c. Those who had committed sins.
d. Those who had stolen the property of others.
From the persons duly nominated, one was to be chosen for each ward by kudavolai system for
a year. The names of eligible persons were written on palm-leaves and put into a pot. A young
boy or girl would take out thirty names each for one ward. They were divided into six variyams
such as samvatsaravariyam, erivariyam, thotta variyam, pancha variyam, pon variyam and
puravuvari variyam to take up six different functions of the village administration. The
committee members were called variyapperumakkal. They usually met in the temple or under a
tree and passed resolutions. The number of committees and ward members varied from village
to village.
Q.16) Palmyra, the ancient city was recently in news. In which of the following countries is it
located?
a)
b)
c)
d)

Syria
Iraq
Saudi Arabia
Israel

Q.16) Solution (a)


Factual question.
A questions is expected. Do read about it.

Q.17) International Intellectual Property Index is compiled by which of the following


organizations?
a)
b)
c)
d)

IMF
WTO
World Bank
US Chamber of Commerce

Q.17) Solution (d)


www.iasbaba.com

Page 10

IASbaba 60 Day Plan- Prelims Test 2016 HISTORY & CURRENT AFFAIRS [DAY 32]

International Intellectual Property Index is compiled by US Chamber of Commerce.


Click here

Q.18) Consider the following statements with respect to Nai Manzil


1. The scheme is launched by Skill Development Ministry
2. The scheme focuses on education and skill development of BPL and APL families

Select the correct option


a)
b)
c)
d)

1 only
2 only
Both
None

Q.18) Solution (d)

The scheme is launched by Ministry of Minorities and it focuses on skill development of


minorities communities.
Click here

Q.19) Shishu, Kishore, Tarun are terms associated with which of the following government of
India scheme
a)
b)
c)
d)

Loans issued under Start Up India


Development stages of Smart City Mission
Loans issued under MUDRA Scheme
Age based vaccines under Mission Indradhanush

Q.19) Solution (c)

The above stages are under MUDRA Scheme


Click here

Q.20) Global financial literacy survey is a report released by which of the following
organization?
www.iasbaba.com

Page 11

IASbaba 60 Day Plan- Prelims Test 2016 HISTORY & CURRENT AFFAIRS [DAY 32]
a)
b)
c)
d)

WTO
IMF
ADB
S&P rating services

Q.20) Solution (d)

It is released by S&P rating services


Click here

Q.21) Gender Development Index is a report released by which of the following


organizations?
a)
b)
c)
d)

UNDP
UNICEF
IMF
ILO

Q.21) Solution (a)

It is released by UNDP
Click here

Q.22) Consider the following statements with respect to Bilateral Totalization Agreement
(BTA)
1. It ensures that the tax charged for the exclusive purpose of social security in one
country is excluded from taxation
2. India has a BTA with USA but not Double Taxation Avoidance Agreement (DTAA)
3. USA has a BTA with only two Asian countries Japan and South Korea
Which of the following statements is/are correct?
a)
b)
c)
d)

Only 1
1 and 2
1 and 3
Only 3

www.iasbaba.com

Page 12

IASbaba 60 Day Plan- Prelims Test 2016 HISTORY & CURRENT AFFAIRS [DAY 32]

Q.22) Solution (c)


A Totalisation agreement is a bilateral treaty between two countries that integrates the social
security laws of two countries. The purpose of the totalisation agreements is to eliminate
double social security taxation on citizen/residents of India who are posted to foreign countries
like US and second, to allow workers who divide their careers between India and a foreign
country to continue to be covered under Indias social security system.
India has a DTAA with USA but not BTA.
The US currently has Totalisation Agreements with 24 countries, the only two Asian countries
being South Korea and Japan.
India has signed social security agreements with Belgium, France, Germany, Switzerland,
Luxembourg and the Netherlands in the recent past.

Q.23) Match List I with List II and select the correct answer using the code given below the
Lists:
List I
A.
B.
C.
D.

Sardar Vallabhbhai Patel


Pandit Deendayal Upadhyaya
Atal Bihari Vajpayee
Dr. B R Ambedkar

List II
1. Good Governance Day
2. Antyodaya Diwas
3. National Unity Day

A-B-C-D
a)
b)
c)
d)

1-( )-3-2
3-2-1-( )
( )-3-2-1
2-3-( )-1

Q.23) Solution (b)


Sardar Vallabhbhai Patel National Unity Day
www.iasbaba.com

Page 13

IASbaba 60 Day Plan- Prelims Test 2016 HISTORY & CURRENT AFFAIRS [DAY 32]
Pandit Deendayal Upadhyaya Antyodaya Diwas
Atal Bihari Vajpayee Good Governance Day

Q.24) Consider the following statements with respect to Rat Hole Mining.
1. It is mainly used for iron ore and coal mining in India
2. It is more prevalent in the state of Jharkhand
3. National Green Tribunal has issued a blanket ban on Rat Hole Mining
Which of the statements is/are incorrect?
a)
b)
c)
d)

Only 2
1 and 2
2 and 3
All of the above

Q.24) Solution (b)


Rat Hole Mining is concerned with only Coal Mining in the state of Meghalaya. NGT has issued a
blanket ban on the same.
Know More Click Here
In News Click Here

Q.25) Which of the following statements is not true about Mural Painting?
1) Murals are large works executed on the walls of solid structure
2) Murals are mainly found in natural caves and rock-cut chambers
3) Themes of Murals - Buddhist, Jain and Hindu religions
Select the correct code
a)
b)
c)
d)

1 only
1 and 3 only
2 only
None of the above

www.iasbaba.com

Page 14

IASbaba 60 Day Plan- Prelims Test 2016 HISTORY & CURRENT AFFAIRS [DAY 32]

Q.25) Solution (d)


Explanation:
Recently, INTACH (Indian National Trust for Art, Culture and Heritage) found 400yr old
murals belonging to Nayaka period in a mantapa called Chitra Sabhai (painted using plant
pigments and natural dyes)
Nayaka Paintings (extension of Vijayanagara style) depicts episodes of Mahabharata and
Ramayana and also Krishna-leela (some stories also include Shiva and Vishnu)
Excerpt from The Hindu article
Above all the given statements are correct

Q.26) The Fourth Buddhist Council was held in Kashmir under the leadership of
a)
b)
c)
d)

Bindusara
Ashoka
Kunal
Kanishka

Q.26) Solution (d)


Explanation:

Q.27) Which of the following statement is/are correct about archaeological survey of India?

www.iasbaba.com

Page 15

IASbaba 60 Day Plan- Prelims Test 2016 HISTORY & CURRENT AFFAIRS [DAY 32]
1) ASI is responsible for the maintenance, restoration and discovery of ancient monument,
archeological site, horticulture site and museums.
2) Archeological and historical pursuits in India started with the effort of Charles Wilkins.
3) ASI is also responsible for epigraphical and numismatic study.
4) It is also responsible for archeological expedition outside India.
Select the appropriate code:
a)
b)
c)
d)

1 and 2 only
1, 2 and 3 only
1, 3 and 4 only
Al of the above

Q.27) Solution (c)


Explanation:

Archaeological and historical pursuits in India started with the efforts of Sir William Jones,
who put together a group of antiquarians to form the Asiatic Society on 15th January 1784
in Calcutta.
Rest all statements are correct with regard to ASIs functions.

Q.28) Consider the following statements with regard to Sarnath:


1) Sarnath, an important seat of Buddhist pilgrimage is located just a few kilometers from
Varanasi near the confluence of the Ganga and the Gomti rivers.
2) World Bank to fund for Sarnath redevelopment
3) Lord Buddha gave his 1st sermon here
4) Shreyamshanath (11th Thirthankara) of Jainism was born here
Which of the above statement is/are not correct?
a)
b)
c)
d)

1 only
2 only
None
All of the above

www.iasbaba.com

Page 16

IASbaba 60 Day Plan- Prelims Test 2016 HISTORY & CURRENT AFFAIRS [DAY 32]
Q.28) Solution (c)
Explanation:
Factual question self explanatory
Excerpt from The Hindu article

Q.29) The famous phrase Vasudhaiva Kutumbakam belongs to which Upanishad?


a)
b)
c)
d)

Mundaka Upanishad
Maho Upanishad
Brihadaranyako Upanishad
Kena Upanishad

Q.29) Solution (b)


Explanation:
Factual question

Q.30) Consider the following pairs:


(Foreign Travellers) (Visited during)
1) Marco polo : Pandya Kingdom
2) Bernier : : Shah Jahan
3) Ralph Fisch : : Akbar
Which of the pairs given above are matched correctly?
a)
b)
c)
d)

1, 2 and 3 only
2 and 3 only
1 and 3 only
1 and 2 only

Q.30) Solution (a)


Explanation:
www.iasbaba.com

Page 17

IASbaba 60 Day Plan- Prelims Test 2016 HISTORY & CURRENT AFFAIRS [DAY 32]

All the given pairs are matched correctly


Excerpt from The Hindu article
To read more: http://www.geocities.ws/rezuj/ati.html

Q.31) Match the incorrect pairs:


Renowned works : : Artists
1)
2)
3)
4)
5)

Landscape : : Ram Kumar


Tandava : : Swaminathan
Peace : : K.K. Hebbar
Monkey God : : M.F.Husain
Indian traditional Girl : : I.Jayachandran

Choose the answer from the code below


a)
b)
c)
d)

1 and 2
2 and 3
1 and 3
None of the above

Q.31) Solution (d)


Explanation:
All the given works and artists are correctly matched and self-explanatory
Excerpt from The Hindu Article

www.iasbaba.com

Page 18

IASbaba 60 Day Plan- Prelims Test 2016 ECONOMICS & CURRENT AFFAIRS [DAY 33]

Q.1) Which of the following is incorrectly matched?


a)
b)
c)
d)

Tax administration reforms commission Justice Ashok Kumar Mathur


Expenditure Management Commission Bimal Jalan
Committee to simplify Income Tax laws Justice R V Easwar
FRBM review panel N K Singh

Q.1) Solution (a)


Tax administration reforms commission is headed by Sri Parthasarathi Shome and not Justic
A K Mathur. Justice Mathur is head of 7th Pay Commission.

Q.2) The GST Council will be the body that decides which taxes levied by the Centre,
States and local bodies will go into the GST. Who among the following are members of
GST Council?
1.
2.
3.
4.
5.

Union Minister of Finance


Minister of Finance of each state
Union Minister of state for Finance
Chief Economic Advisor to Minister of Finance
Governor of RBI

Select the correct answer


a)
b)
c)
d)

1, 2 and 4 only
1, 2, and 3 only
1, 2, 3 and 5 only
All of the above

Q.2) Solution (b)


The 122nd Constitution Amendment Bill seeks to establish a GST Council tasked with
optimising tax collection for goods and services by the State and Centre. The Council will
consist of the Union Finance Minister (as Chairman), the Union Minister of State in charge of
revenue or Finance, and the Minister in charge of Finance or Taxation or any other,
nominated by each State government.
The GST Council will be the body that decides which taxes levied by the Centre, States and
local bodies will go into the GST; which goods and services will be subjected to GST; and the
basis and the rates at which GST will be applied.

Q.3) Consider the following about Gold Monetization Scheme.


www.iasbaba.com

Page 1

IASbaba 60 Day Plan- Prelims Test 2016 ECONOMICS & CURRENT AFFAIRS [DAY 33]

1. The designated banks will accept gold deposits under the Short Term (1-3 years)
Bank Deposit as well as Medium (5-7 years) and Long (12-15 years) Term
Government Deposit Schemes.
2. Both principal and interest to be paid to the depositors of gold, will be valued in
gold.
3. Customer will have the choice to take cash or gold on redemption, but the
preference has to be stated at the time of deposit.
4. The interest rate is decided by the banks concerned.
Select the correct answer
a)
b)
c)
d)

1 and 2 only
1, 2 and 3 only
1, 3 and 4 only
All of the above

Q.3) Solution (d)


Its an important scheme of the government and is expected as a question. All the above are
right. Yes interest rates are decided by the banks.
Further reading: http://finmin.nic.in/swarnabharat/gold-monetisation.html

Q.4) Which of following qualify as measures taken to curb tax evasion and black money by
government?
1.
2.
3.
4.

Income Disclosure Scheme of 2016


Amendment to DTAA with Mauritius
Place of Effective Management Rule to applicable
Undisclosed foreign income and assets act 2015

Select the correct answer


a)
b)
c)
d)

1, 2 and 3 only
1, 2 and 4 only
1, 3 and 4 only
All of the above

Q.4) Solution (d)

www.iasbaba.com

Page 2

IASbaba 60 Day Plan- Prelims Test 2016 ECONOMICS & CURRENT AFFAIRS [DAY 33]

All the above are right and crucial measures taken up by the government in last 2 financial
years. Place of Effective Management Rule may be raising eyebrows. So let us understand it.
http://www2.deloitte.com/in/en/pages/tax/articles/place-of-effective-management.html
Q.5) Consider the following.
1. Controller General of Accounts is part of Department of Revenue and prepares the
appropriation bill.
2. CGA is head of Indian Civil Accounts Service and administers it.
Select the correct answer:
a)
b)
c)
d)

1 only
2 only
Both 1 and 2
None of the above

Q.5) Solution (b)


No it is not Department of Revenue but Expenditure. Now that is nail biting. Isnt it?
Indian Civil Accounts Organisation performs a key role in delivery of financial management
services for the Government of India. The organisation provides payment services, supports
the tax collection system, performs government wide accounting, financial reporting
functions, preparation of budget estimates and carries out Internal Audit in civil ministries
of the Union Government. Controller General of Accounts (CGA) in Ministry of Finance
heads the organisation and is responsible for administering this system.
Q.6) Consider the following.
1. Ideally custom duties should be high on raw material import and low on export for a
country like India.
2. Similarly custom duties should be high on finished goods exports and low on import
in line with Make in India initiative.
Select the correct answer:
a)
b)
c)
d)

1 only
2 only
Both 1 and 2
None of the above

www.iasbaba.com

Page 3

IASbaba 60 Day Plan- Prelims Test 2016 ECONOMICS & CURRENT AFFAIRS [DAY 33]

Q.6) Solution (d)


Just the reverse of both statements isnt it? Ideally custom duties should be low on raw
material import and high on export for a country like India to encourage manufacturing
within India.
Q.7) Consider the following.
1. The Krishi Kalyan Cess and Swachch Bharat Cess are applicable on RBI and Postal
services but not on government insurance schemes for social sector.
2. Krishi Kalyan Cess and Swachch Bharat Cess proceeds are part of Consolidated Fund
of India and will be part of the shareable tax pool as per the recommendation of the
Tax Administration reform Council 2015-16.
Select the correct answer:
a)
b)
c)
d)

1 only
2 only
Both 1 and 2
None of the above

Q.7) Solution (d)


We know any Cess is not part of sharable tax pool and there has been no such
recommendation. 17 types of services including RBI, Postal Education, insurance have been
kept out of the above service tax net.

Q.8) Consider the following about revenue and capital receipts.


1. Interest receipts, dividends and profits are parts of revenue receipts.
2. Article 112 of the constitution mandates expenditure is shown in revenue and other
categories.
3. Plan- Non Plan expenditure segregation has been constitutionally mandated and
needs an amendment to be removed.
4. Disinvestment proceeds are part of capital receipts.
Select the correct answer
a) 1 and 2 only
b) 1, 2 and 3 only
www.iasbaba.com

Page 4

IASbaba 60 Day Plan- Prelims Test 2016 ECONOMICS & CURRENT AFFAIRS [DAY 33]

c) 1, 2 and 4 only
d) All of the above
Q.8) Solution (c)
Non plan expenditure is not a constitutional term. Rest should be self explainatory.
Further reading: http://pib.nic.in/budget2015/book/Budgetary.pdf

Q.9) Which of the following is incorrect w.r.t Ways and Means Advances (WMAs)?
a) RBI provides 2 types of WMAs to state governments one with and another without
collaterals.
b) WMAs given by RBI to government of India do not require any collateral.
c) Replacement of adhoc bills with WMAs represents an advance in fiscal discipline and
harmonization of fiscal and monetary policies.
d) WMAs are made at Repo rate.

Q.9) Solution (d)


The interest rate charged on WMA and overdrafts at present are the Bank Rate (9 per cent)
and the Bank Rate plus two percentage points (11 per cent), respectively.
Further reading: https://www.rbi.org.in/Scripts/PublicationReportDetails.aspx?ID=6

Q.10) Which of the following is correct debt?


a) The union government borrows directly from international capital markets with the
help of debt management agencies.
b) Small savings and provident funds are part of other liabilities.
c) Dated securities as part of market borrowings are issued by union governments as Tbills with 14 days, 91days, 182days maturity.
d) Public debt means only internal debt and other liabilities but not external debt.

Q.10) Solution (b)


The union government doesnt borrow directly from international capital markets. 14 day
treasury bills are only issued by state governments. Public debt includes both internal and
external debt.
Source: India Year Book Chapter : Finance

www.iasbaba.com

Page 5

IASbaba 60 Day Plan- Prelims Test 2016 ECONOMICS & CURRENT AFFAIRS [DAY 33]

Q.11) Which of the following recommendations of 14 th Finance Commission regarding tax


Devolution are accepted by the Government?
1. StatesShare in the net proceeds of Union tax revenues increased to 42% from 32%
earlier.
2. Sharing pattern under various CSS to undergo a change, with States sharing higher
fiscal responsibility for scheme implementation.
3. Distribution of grants to States for local bodies based on 2011 population data
(90%weight) and area (10%weight).
4. Revenue compensation to States under GST should be for five years;100% in first
three years, 75% in fourth year and 50% in fifth year.
Select the code from the following:
a)
b)
c)
d)

1,2 and 3
2,3 and 4
1,3 and 4
All of the above

Q.11) Solution (a)


The last recommendation has not been accepted. It is still under consideration. Refer: 14 th
Finance Commission.

Q.12) Consider the following statements regarding Financial Stability:


1. Financial stability could be defined as a situation in which the financial sector
provides critical services to the real economy without any discontinuity.
2. Financial Stability report is published every year by Ministry of Finance.
3. Financial stability is an important aspect of the Monetary Policy of India.
Which of the above statements are correct?
a)
b)
c)
d)

1 and 2
2 and 3
1 and 3
All of the above

Q.12) Solution (c)


www.iasbaba.com

Page 6

IASbaba 60 Day Plan- Prelims Test 2016 ECONOMICS & CURRENT AFFAIRS [DAY 33]

Financial stability means financial institutions individually and collectively are being able to
deliver their functions properly, withstanding external shocks and avoiding internal
weaknesses. India Financial Stability Report published by the Reserve Bank of India (March
2010), defines financial stability: From a macro prudential perspective, financial stability
could be defined as a situation in which the financial sector provides critical services to the
real economy without any discontinuity.
During the time of the global financial crisis, RBI has made many unconventional measures
to protect the banking system. Liquidity support was provided abundantly so that no banks
should face stress. The RBI since 2010 is publishing India Financial Stability Report to assess
financial stability scenario in the country. Financial stability is now one of the three
important objectives of monetary policy besides price stability and credit support.

Q.13) Volume two of Economic Survey talks about a Chakravyuh problem of Indian
economy. Which of the following does Chakravyuh refers to?
a)
b)
c)
d)

Vicious cycle of poverty and lack of capital


Over dependence of economy on Foreign Direct investment
Lack of clear exit policy for companies
Conflict between economic growth and inflation

Q.13) Solution (c)


The Chapter 2 titled Chakravyuha Challenge of the Indian Economy.

Q.14) Consider the following Statements regarding Gyan Sangam summit:


1. It was a meeting organized by Ministry of Finance and RBI with central banks of Tax
haven countries to find a method to recover black money which is tapped outside.
2. It is a forum where the highest officials from public sector banks, the government
and the Reserve Bank of India, meet to discuss issues faced by the sector.
3. Focus was discussion on bank consolidation, NPAs and carrying out changes to
recovery laws.
Which of the above statements are correct?
a)
b)
c)
d)

1 only
2 and 3
1 and 3
None of the above

www.iasbaba.com

Page 7

IASbaba 60 Day Plan- Prelims Test 2016 ECONOMICS & CURRENT AFFAIRS [DAY 33]

Q.14) Solution (b)


Second Edition of Gyan Sangam was held this year to discuss consolidation of banks, NPAs,
changes to recovery laws and giving out employee stock options.

Q.15) SARFAESI Act refers to Securitisation and Reconstruction of Financial Assets and
Enforcement of Security Interest Act, 2002. Banks utilize this act as an effective tool for
bad loans (NPA) recovery. Which of the following statements are correct about this act?
1. Upon loan default, banks can seize the securities (except agricultural land) without
intervention of the court.
2. Court intervention is not required only in case of secured loans where bank can
enforce the underlying security eg hypothecation, pledge and mortgages.
3. If the asset in question is an unsecured asset, the bank would have to move the
court to file civil case against the defaulters.
Select the code from the following:
a)
b)
c)
d)

1 and 2
2 and 3
1 and 3
All of the above

Q.15) Solution (d)


SARFAESI Act as we know is Securitisation and Reconstruction of Financial Assets and
Enforcement of Security Interest Act, 2002. Banks utilize this act as an effective tool for bad
loans ( NPA) recovery.
It is possible where non-performing assets are backed by securities charged to the Bank by
way of hypothecation or mortgage or assignment. Upon loan default, banks can seize the
securities (except agricultural land) without intervention of the court.
SARFAESI is effective only for secured loans where bank can enforce the underlying security
eg hypothecation, pledge and mortgages. In such cases, court intervention is not necessary,
unless the security is invalid or fraudulent.
However, if the asset in question is an unsecured asset, the bank would have to move the
court to file civil case against the defaulters.

Q.16) Which of the following regarding Tax Buoyancy are correct:


www.iasbaba.com

Page 8

IASbaba 60 Day Plan- Prelims Test 2016 ECONOMICS & CURRENT AFFAIRS [DAY 33]

1. Tax buoyancy explains this relationship between the changes in governments tax
revenue growth and the changes in GDP.
2. When a tax is buoyant, its revenue increases without increasing the tax rate
Select the code from below:
a)
b)
c)
d)

1 only
2 only
Both 1 and 2
Neither 1 nor 2

Q.16) Solution (c)


There is a strong connection between the governments tax revenue earnings and economic
growth. The simple fact is that as the economy achieves faster growth, the tax revenue of
the government also goes up.
Tax buoyancy explains this relationship between the changes in governments tax revenue
growth and the changes in GDP. It refers to the responsiveness of tax revenue growth to
changes in GDP. When a tax is buoyant, its revenue increases without increasing the tax
rate.

Q.17) Which of the following statements are correct regarding Pigovian Tax?
1. A Pigovian tax is a tax levied on any market activity that generates negative

externalities.
2. It is often levied on companies that pollute the environment or create excess social
costs.
3. A type of a Pigovian tax is a "sin tax", which is a special tax on tobacco products and
alcohol.
Select the code from the following:
a)
b)
c)
d)

1 and 2
2 and 3
1 and 3
All of the above

Q.17) Solution (d)


www.iasbaba.com

Page 9

IASbaba 60 Day Plan- Prelims Test 2016 ECONOMICS & CURRENT AFFAIRS [DAY 33]

A Pigovian tax is a special tax that is often levied on companies that pollute the environment
or create excess social costs, called negative externalities, through business practices. In a
true market economy, a Pigovian tax is the most efficient and effective way to correct
negative externalities.
A type of a Pigovian tax is a "sin tax", which is a special tax on tobacco products and alcohol.
Q.18) NITI Aayog has replaced the older planning commission. Which of the following
points differentiates NITI aayog from Planning Commission?
1. NITI Aayog is an advisory body while Planning Commissions recommendations were
binding.
2. State representatives were not there in Planning Commission while NITI Aayog has
included Chief Ministers of all states as well as administrators of UTs.
3. The provision of regional council is there in Niti Ayog to address local / regional
development issues.
Select the code from the following:
a)
b)
c)
d)

1 and 2
2 and 3
1 and 3
All of the above

Q.18) Solution (b)


Both NITI Aayog and Planning Commission were advisory bodies.

Q.19) Consider the following statements regarding subsidies in current fiscal year:
1. Subsidies on food, fertiliser and petroleum have been pegged higher by over 4 per
cent.
2. The deregulation of diesel price, along with the introduction of direct benefit
(subsidy) transfer into the bank accounts of domestic LPG consumers, coupled with a
sharp decline in global crude oil prices has helped containing the petroleum subsidy
bill.
Which of the above statements are correct?
a) 1 only
b) 2 only
www.iasbaba.com

Page 10

IASbaba 60 Day Plan- Prelims Test 2016 ECONOMICS & CURRENT AFFAIRS [DAY 33]

c) Both 1 and 2
d) Neither 1 nor 2

Q.19) Solution (b)


Subsidies on food, fertiliser and petroleum have been pegged lower by over 4 per cent. This
is one of the biggest drop in subsidy expenditure. (Economic Survey- Public Finance).

Q.20) Which of the following statement are the recommendation of Tax Administration
Reforms Commission?
1. A minimum of 10% of the tax administrations budget must be spent on taxpayer
services.
2. Pre-filled tax returns should be provided to all individuals. The taxpayer will have
the option of accepting the tax return or modifying it.
3. CBDT and CBEC should be fully integrated under Central Board of Direct and Indirect
Taxes.
4. The Permanent Account Number (PAN) should be developed as a Common Business
Identification Number (CBIN), to be used by other departments such as customs,
excise, etc.
Select the code from the following:
a)
b)
c)
d)

1,3 and 4
3 and 4
1,2 and 3
All of the above

Q.20) Solution (d)


TARC made the following recommendations.
Consumer Focus:

There should be a separate vertical for delivery of taxpayer services in each Board. A
minimum of 10% of the tax administrations budget must be spent on taxpayer
services.

www.iasbaba.com

Page 11

IASbaba 60 Day Plan- Prelims Test 2016 ECONOMICS & CURRENT AFFAIRS [DAY 33]

The decision of the Ombudsman with regard to redressing taxpayer grievances


should be binding on tax officers.

Pre-filled tax returns should be provided to all individuals. The taxpayer will have
the option of accepting the tax return or modifying it.

Structure and Governance:

CBDT and CBEC should be fully integrated in 10 years. Within the next 5 years, they
should move towards a unified management structure under the Central Board of
Direct and Indirect Taxes.

The post of Revenue Secretary should be abolished and its functions should be
assigned to the two Boards. A Governing Council to oversee the working of the two
Boards, and a Tax Council to suggest policy and legislation should be set up.

Human Resource Development:

There should be a focus on specialisation, including lateral entry of specialists in the


Boards. Indian Revenue Service (IRS) officers should specialise in a particular tax
administration areas.

The Central Vigilance Commission should have a Member who has been an IRS
officer. The policy of not taking cognizance of anonymous complaints should be
strictly followed.

Dispute Resolution and Management:

Retrospective legislation should be avoided.

Both Boards should start a special drive for review and liquidation of cases currently
clogging the system by setting up dedicated task forces.

A separate dispute management vertical should be set up in each Board. In addition,


the process of pre-dispute consultation before issuing a tax demand notice should be
put into practice.

Internal Processes: The Permanent Account Number (PAN) should be developed as a


Common Business Identification Number (CBIN), to be used by other departments such as
customs, excise, etc.

Q.21) Consider the following statements with respect to cryogenic engine


www.iasbaba.com

Page 12

IASbaba 60 Day Plan- Prelims Test 2016 ECONOMICS & CURRENT AFFAIRS [DAY 33]

1. In Cryogenic engine its fuel or oxidizer (or both) are gases liquefied and are stored at
very low temperatures.
2. Combination of liquid hydrogen (LH2) fuel and the liquid oxygen (LOX) oxidizer is one
of the most widely used fuels in the rocket
Select the correct option
a)
b)
c)
d)

1 only
2 only
Both
None

Q.21) Solution (c)


Click here

Q.22) Consider the following statements with respect to GAGAN satellite


a)
b)
c)
d)

It is used for telemedicine in rural areas


It is used to provide live streaming of classes from top Indian universities
It is used to provide navigation facilities for civil aviation
It is used for rain and whether forecasting

Q.22) Solution (c)


Click here

Q.23) Consider the following


1. Chandrayaan 1
2. Mars orbiter mission
3. Space capsule recovery experiment
Which of the above space missions are launched using PSLV launch vehicle?
a)
b)
c)
d)

1 &3 only
1&2 only
2&3 only
All

www.iasbaba.com

Page 13

IASbaba 60 Day Plan- Prelims Test 2016 ECONOMICS & CURRENT AFFAIRS [DAY 33]

Q.23) Solution (d)

The PSLV is one of world's most reliable launch vehicles. It has been in service for
over twenty years and has launched various satellites for historic missions like
Chandrayaan-1, Mars Orbiter Mission, Space Capsule Recovery Experiment, Indian
Regional Navigation Satellite System (IRNSS) etc. PSLV remains a favourite among
various organisations as a launch service provider and has launched over 40 satellites
for 19 countries. In 2008 it created a record for most number of satellites placed in
orbit in one launch by launching 10 satellites into various Low Earth Orbits.
Click here

Q.24) Consider the following statements with respect to IRNSS (Indian regional navigation
satellite system)
1. It provide accurate position information service to users in India as well as the region
extending up to 1500 km from its boundary
2. IRNSS will provide two types of services namely, Standard Positioning Service (SPS)
and Restricted Service (RS) to all users
Select the correct answer
a)
b)
c)
d)

1 only
2 only
Both
None

Q.24) Solution (a)

Restricted area service is for military and other defence related purposes
IRNSS is an independent regional navigation satellite system being developed by
India. It is designed to provide accurate position information service to users in India
as well as the region extending up to 1500 km from its boundary, which is its primary
service area. An Extended Service Area lies between primary service area and area
enclosed by the rectangle from Latitude 30 deg South to 50 deg North, Longitude 30
deg East to 130 deg East.
IRNSS will provide two types of services, namely, Standard Positioning Service (SPS)
which is provided to all the users and Restricted Service (RS), which is an encrypted
service provided only to the authorised users. The IRNSS System is expected to
provide a position accuracy of better than 20 m in the primary service area.
Click here

www.iasbaba.com

Page 14

IASbaba 60 Day Plan- Prelims Test 2016 ECONOMICS & CURRENT AFFAIRS [DAY 33]

Q.25) Consider the following statements with respect to ISRO (Indian space research
organisation)
1. ISRO is the space agency of the Indian government headquartered at Sri Hari Kota
2. "Space technology in the Service of humankind." is the motto of ISRO
Select the correct option
a)
b)
c)
d)

1 only
2 only
Both
None

Q.25) Solution (b)

ISRO is headquartered at Bengaluru


"Space technology in the Service of humankind." is the motto of ISRO
Click here

www.iasbaba.com

Page 15

IASbaba 60 Day Plan- Prelims Test 2016 ECONOMICS [DAY 34]


Q.1) Consider the following w.r.t. Securities and Exchange Board of India (SEBI).
1. SEBI was established under the Securities and Exchange Board of India act 1992.
2. The Securities Laws (Amendment) Act, 2014 provides for search and seizure after
obtaining approval from Union Minister of Finance.
3. National Strategy of Financial Education was initiated by SEBI with a vision of a
financially and empowered India.
4. SEBI is part of Financial Stability and Development Council (FSDC).
Select the correct answer
a) 1, 2 and 4 only
b) 1 and 3 only
c) 1, 3 and 4 only
d) 2, 3 and 4 only

Q.1) Solution (c)


It isnt MoF but a magistrate or Judge of a designated court in Mumbai. Rest of the statements
are correct. Yes SEBI also has a role in financial inclusion.
Refer Chapter Finance, India Year Book 2016.

Q.2) In 2012 SEBI brought out regulations for Alternative Investment Funds (AIFs). Which of
the following is incorrect?

1. Category I AIFs are those who invest in start-ups or early stage ventures or social
ventures that government considers socially or economically viable.
2. Category II AIFs are those funds are allowed to invest anywhere in any combination, but
cannot take debts, except for day-to-day operation purposes.
3. Category III AIFs are funds that make short-term investments and then sell, like hedge
funds, come under this.
4. Union Budget 2015-16 tax pass through has been given for all 3 categories.
www.iasbaba.com

Page 1

IASbaba 60 Day Plan- Prelims Test 2016 ECONOMICS [DAY 34]

Select the correct answer


a) 1, 2 and 3 only
b) 1, 2 and 4 only
c) 1, 3 and 4 only
d) All of the above

Q.2) Solution (a)


Tax pass through has been given for only categories I and II. Rest are correct.
Further reading:
http://www.arthapedia.in/index.php%3Ftitle%3DAlternative_Investment_Funds_(AIFs)

Q.3) Consider the following about Securities Appellate Tribunal (SAT)

1. SAT consists of a presiding officer who is a deputy governor of RBI concerned with
money market and capital market.
2. SAT operates under Civil Procedure Code 1908.

Select the correct answer:

a) 1 only
b) 2 only
c) Both 1 and 2
d) None of the above

Q.3) Solution (d)

www.iasbaba.com

Page 2

IASbaba 60 Day Plan- Prelims Test 2016 ECONOMICS [DAY 34]


The presiding officer of SAT is a sitting/retired judge of apex court or sitting/retired chief justice
of high court or sitting/retired judge of high court who has completed not less than 7 years of
service in office.
SAT is guided by principles of natural justice and is not bound by any penal codes.
Securities Appellate Tribunal is a statutory body established under the provisions of Section 15K
of the Securities and Exchange Board of India Act, 1992 to hear and dispose of appeals against
orders passed by the Securities and Exchange Board of India or by an adjudicating officer under
the Act and to exercise jurisdiction, powers and authority conferred on the Tribunal by or under
this Act or any other law for the time being in force.

Q.4) Which of the following are part of the Foreign Investment Promotion Board (FIPB)?

1. Secretary Department of Economic Affairs.


2. Secretary Department of Revenue.
3. Secretary DIPP
4. Principal Secretary to PM
5. Secretary Ministry of Small Scale Industries.

Select the correct answer


a) 1, 2, 3 and 4 only
b) 1, 2, 3 and 5 only
c) 1, 2, 4 and 5 only
d) All of the above

Q.4) Solution (b)


Principal Secretary to PM is not a part.
The permanent members of the Board are the following:
1. Secretary to the Government of India, DEA, Ministry of Finance - Chairman

www.iasbaba.com

Page 3

IASbaba 60 Day Plan- Prelims Test 2016 ECONOMICS [DAY 34]


2. Secretary to the Government of India, Department of Industrial Policy and Promotion
(DIPP).
3. Secretary to the Government of India, Department of Commerce (DoC).
4. Secretary to the Government of India (Economic Relations), Ministry of External Affairs
(MEA).
5. Secretary to the Government of India, Ministry of Overseas Indian Affairs (MOIA)
6. Secretary, Department of Revenue (DoR), Ministry of Finance (co-opted permanently).
7. Secretary, Ministry of Small and Medium & Micro Enterprises (co-opted permanently).

The Board may co-opt other Secretaries to the Government of India and officials of financial
institutions, banks and professional experts in industry and commerce, when required. The
Secretary to the Government of India, Ministry of Small, Medium and Micro Enterprises and the
Secretary to the Government of India, Department of Revenue have been co-opted on the
Board.

Q.5) FDI is prohibited in which of the following?

1. Lottery business
2. Business of Chit funds.
3. Nidhi Company
4. Manufacturing of Cigars, Cheroots and cigarettes of tobacco
5. Gambling and betting.

Select the correct answer


a) 1, 2 and 4 only
b) 1, 3 and 5 only
c) 1, 2 and 5 only
d) All of the above
www.iasbaba.com

Page 4

IASbaba 60 Day Plan- Prelims Test 2016 ECONOMICS [DAY 34]

Q.5) Solution (d)


SECTORS WHERE FOREIGN DIRECT INVESTMENT IS PROHIBITED:

Lottery Business including Government /private lottery, online lotteries, etc.

Gambling and Betting including casinos etc.

Chit funds

Nidhi company-(borrowing from members and lending to members only).

Trading in Transferable Development Rights (TDRs)

Real Estate Business (other than construction development) or Construction of Farm


Houses

Manufacturing of Cigars, cheroots, cigarillos and cigarettes, of tobacco or of tobacco


substitutes

Activities / sectors not open to private sector investment e.g. Atomic Energy and
Railway Transport (other than construction, operation and maintenance of
(i) Suburban corridor projects through PPP,
(ii) High speed train projects,
(iii) Dedicated freight lines,
(iv) Rolling stock including train sets, and locomotives/coaches manufacturing and
maintenance facilities,
(v) Railway Electrification,
(vi) Signaling systems,
(vii) Freight terminals,
(viii) Passenger terminals,
(ix) Infrastructure in industrial park pertaining to railway line/sidings including electrified
railway lines and connectivities to main railway line and
(x) Mass Rapid Transport Systems.)

Services like legal, book keeping, accounting & auditing.

Further reading: http://www.makeinindia.com/policy/foreign-direct-investment

www.iasbaba.com

Page 5

IASbaba 60 Day Plan- Prelims Test 2016 ECONOMICS [DAY 34]


Q.6) Which of the following is incorrect about FIPB?

a) The Foreign Investment Promotion Board (FIPB), presently is housed in the Department
of Economic Affairs, Ministry of Finance, is an inter-ministerial body.
b) The FIPB was initially constituted under the Prime Minister's Office (PMO) in the wake of
the economic liberalization drive of the early 1990s.
c) The Board was reconstituted in 1996 with transfer of the FIPB to DIPP
d) The approval limit for FIPB is Rs. 2000crore beyond which it needs approval of Union
Cabinet.

Q.6) Solution (d)


http://fipb.gov.in/AboutUs.aspx
The information here with approval limits is a little outdated. The government recently raised it
to 3000 crores and beyond which it is approved by CCEA and not cabinet.

Q.7) Which of the following is correct about National Investment and Infrastructure Fund
(NIIF)?

a) NIIF was set up as a NBFC, to raise debt to invest in the equity of infrastructure finance
companies such as Indian Rail Finance Corporation (IRFC) and National Housing Bank
(NHB).
b) The objective of NIIF would be to maximize economic impact mainly through
infrastructure development in commercially viable projects, only greenfield and not
brownfield, but including stalled projects.
c) The NIIF will be established as one or more Alternate Investment Funds (AIF) under the
SEBI Regulations.
d) The Government had approved the creation of NIIF with the aim to attract investment
from international sources and not domestic for maximizing economic impact.

www.iasbaba.com

Page 6

IASbaba 60 Day Plan- Prelims Test 2016 ECONOMICS [DAY 34]


Q.7) Solution (c)
NIIF is set up as a trust and not NBFC. It is for both brown and Greenfield projects. NIIF is
applicable for both domestic and international sources.
Further reading:
http://arthapedia.in/index.php%3Ftitle%3DNational_Investment_and_Infrastructure_Fund_(NII
F)

Q.8) Consider the following about buyback of shares.

1. A buyback is the repurchase of outstanding shares (repurchase) by a company in order


to reduce the number of shares on the market.
2. Companies will buy back shares either to increase the value of shares still available
(reducing supply), or to eliminate any threats by shareholders who may be looking for a
controlling stake.

Select the correct answer:

a) 1 only
b) 2 only
c) Both 1 and 2
d) None of the above

Q.8) Solution (c)


A buyback allows companies to invest in themselves. By reducing the number of shares
outstanding on the market, buybacks increase the proportion of shares a company owns.
Buybacks can be carried out in two ways:
1. Shareholders may be presented with a tender offer whereby they have the option to submit
(or tender) a portion or all of their shares within a certain time frame and at a premium to the

www.iasbaba.com

Page 7

IASbaba 60 Day Plan- Prelims Test 2016 ECONOMICS [DAY 34]


current market price. This premium compensates investors for tendering their shares rather
than holding on to them.
2. Companies buy back shares on the open market over an extended period of time.
Further

reading:

http://www.yourarticlelibrary.com/accounting/buy-back-shares/buy-back-

shares-meaning-reasons-aspects-and-other-details/62687/

Q.9) Recently, the market regulator Securities and Exchange Board of India (SEBI) barred 13
entities from the securities market for five years. They were accused of manipulating Global
Depository Receipts (GDRs) issued by Indian companies. Which of the following are correct
about GDRs?

1. Global Depository Receipts (GDRs) is a mechanism which allows one to buy and sell
shares of a foreign company without having to bother about opening a foreign
brokerage account.
2. Global Depository Receipts (GDRs) is a mechanism which allows one to buy and sell
shares of a foreign company without having to bother about opening a foreign
brokerage account.
3. GDRs are usually backed by sovereign governments of that country that provide
companies, investors and traders opportunities to make global investments.
4. Countrys prone to political unrest or geopolitical conflicts can severely impact the
valuations of a company, resulting in GDR valuations losses for an investor who may
have been betting on that particular company.

Select the correct answer


a) 1 and 2 only
b) 1, 2 and 3 only
c) 1, 2 and 4 only
d) All of the above

www.iasbaba.com

Page 8

IASbaba 60 Day Plan- Prelims Test 2016 ECONOMICS [DAY 34]


Q.9) Solution (c)
It is not the governments that back the GDRs. GDRs are usually backed by depository banks that
provide companies, investors and traders opportunities to make global investments. These are
banks whose primary task is to hold shares of companies based in another country. Such banks
essentially sell the GDRs. They also ensure that investors receive their dividends and capital
gains. Depository banks also handle all tax-related issues in the companys home country. Since
all GDR transactions have to go through a depository bank, investments made in them are safe.
However, their valuations are always associated with normal market risks.
Further

reading:

https://in.finance.yahoo.com/news/all-you-need-to-know-about-global-

depository-receipts-043746355.html

Q.10) Consider the following about Participatory Notes (P-Notes).

1. "P-notes," are financial instruments used by investors or hedge funds that are not
registered with the Securities and Exchange Board of India (SEBI) to invest in Indian
securities.
2. India has permitted P Notes recently under the expert suggestion of top economist
Subramaniam Swamy who considers P Notes will bring back all black money stashed
away.

Select the correct answer:

a) 1 only
b) 2 only
c) Both 1 and 2
d) None of the above

Q.10) Solution (a)

www.iasbaba.com

Page 9

IASbaba 60 Day Plan- Prelims Test 2016 ECONOMICS [DAY 34]


Who doesnt know Mr. Swamy!!! There is no such... P Notes are considered dangerous and are
not allowed.
Participatory notes, also referred to as "P-notes," are financial instruments used by investors
or hedge funds that are not registered with the Securities and Exchange Board of India (SEBI) to
invest in Indian securities. Any dividends or capital gains collected from the underlying
securities go back to the investors. Indian regulators are against participatory notes because
they fear that hedge funds acting through participatory notes will cause economic volatility in
India's exchanges.
Further reading: http://www.thehindubusinessline.com/todays-paper/why-participatory-notesare-dangerous/article1672845.ece

Q.11) Consider the following statements:


1. Capital market loans are used by industries for fixed investment.
2. In principle Capital market is a short term loanable fund.
Which of the above statements are correct?
a)
b)
c)
d)

1 only
2 only
Both 1 and 2
Neither 1 nor 2

Q.11) Solution (a)


The Indian capital market is the market for long term loanable funds as distinct from money
market which deals with short term funds.
It refers to the facilities and institutional arrangements for borrowing and lending term fundsmedium and long term funds. In principle capital market loans are used by industries mainly for
fixed investments. It does not deal in capital goods but is concerned with raising money capital.

Q.12) Gilt-Edged securities are the most sought after type securities. Which of the following
statements are correct about Gilt-Edged securities?
www.iasbaba.com

Page 10

IASbaba 60 Day Plan- Prelims Test 2016 ECONOMICS [DAY 34]


1. Gilt-edged securities are a high-grade investment with very low risk with confirmed
dividend or interest returns.
2. They are based on platinum, gold and silver.
Select the code from following:
a)
b)
c)
d)

1 only
2 only
Both 1 and 2
Neither 1 nor 2

Q.12) Solution (a)


Government securities are instruments issued by the government to borrow money from the
market. They are also known as gilts or gilt edged securities
Government security means a security created and issued by the Government for the
purpose of raising a public loan or for any other purpose as may be notified by the Government
in the Official Gazette and having one of the forms mentioned in the Government Securities
Act, 2006.

Q.13) Indian companies are allowed to access global finance market and benefit from the
lower cost of funds. They have been permitted to raise resources through which of the
following means?
1.
2.
3.
4.

American Depository Receipts (ADRs)


Global Depository Receipts (GDRs)
Foreign Currency Convertible Bonds (FCCBs)
External Commercial Borrowings (ECBs)

Select the code from below:


a)
b)
c)
d)

1 and 2
3 and 4
1,2 and 4
All of the above

www.iasbaba.com

Page 11

IASbaba 60 Day Plan- Prelims Test 2016 ECONOMICS [DAY 34]


Q.13) Solution (d)
Indian companies are allowed to access global finance market and benefit from the lower cost
of funds. They have been permitted to raise resources through issue of American Depository
Receipts (ADRs), Global Depository Receipts (GDRs), Foreign Currency Convertible Bonds
(FCCBs) and External Commercial Borrowings (ECBs). Further Indian financial system is opened
up for investments of foreign funds through Non-Resident Indians (NRIs), Foreign Institutional
investors (FIls), and Overseas Corporate Bodies (OCBs).
A foreign currency convertible bond (FCCB) is a type of convertible bond issued in a currency
different than the issuer's domestic currency. In other words, the money being raised by the
issuing company is in the form of a foreign currency. A convertible bond is a mix between a
debt and equity instrument. It acts like a bond by making regular coupon and principal
payments, but these bonds also give the bondholder the option to convert the bond into stock.

Q.14) Buy Back of shares is a method of financial engineering. It is process which enables the
company to go back to its share holders and offer to purchase the shares by them. Which of
the following statements are correct about the process of buy-back of shares?
1. When a company faces a cash crunch it buys back its shares.
2. Buy Back is used as a defence mechanism when there is a threat of corporate takeover.
It provides a safeguard against hostile takeover by increasing promoters holding.
3. Buy Back can divert away the companys funds from productive investments.
4. A company can buy its shares every year with subject to the satisfaction of other
conditions like debt-equity ratio, limits of buy back etc.
Select the code from the following:
a)
b)
c)
d)

1,2 and 3
2,3 and 4
1,3 and 4
All of the above

Q.14) Solution (b)


A company buys back its shares when it has access of cash.

www.iasbaba.com

Page 12

IASbaba 60 Day Plan- Prelims Test 2016 ECONOMICS [DAY 34]


One of the negative effects of buy back can be that it might divert away the companys funds
from productive investments.

Q.15) A Commercial Paper is an important instrument in the money market. Which of the
following statements are correct about Commercial Papers?
1. Commercial paper is an unsecured, short-term debt instrument issued by a corporation,
typically for the financing of accounts receivable, inventories and meeting short-term
liabilities.
2. Commercial papers are usually sold at a discount from the face value.
3. It is not backed by collateral, only firms with excellent credit ratings from a recognized
credit rating agency will be able to sell their commercial paper at a reasonable price.
Select the code from the following:
a)
b)
c)
d)

1 and 2
2 and 3
1 and 3
All of the above

Q.15) Solution (d)


Commercial paper is a money-market security issued (sold) by large corporations to obtain
funds to meet short-term debt obligations (for example, payroll), and is backed only by an
issuing bank or company promise to pay the face amount on the maturity date specified on the
note. Since it is not backed by collateral, only firms with excellent credit ratings from a
recognized credit rating agency will be able to sell their commercial paper at a reasonable price.
Commercial paper is usually sold at a discount from face value, and generally carries lower
interest repayment rates than bonds due to the shorter maturities of commercial paper.
Typically, the longer the maturity on a note, the higher the interest rate the issuing institution
pays. Interest rates fluctuate with market conditions, but are typically lower than banks' rates.

Q.16) Which of the following statements regarding Certificate of Deposits are correct?
1. Whenever a big corporation wants to open an account in RBI, it needs to take a prior
permission from the Governor. After assessing the performance of the company and its

www.iasbaba.com

Page 13

IASbaba 60 Day Plan- Prelims Test 2016 ECONOMICS [DAY 34]


market rating, the corporation is issued a certificate by the bank, called Certificate of
Deposit.
2. Certificate of Deposit (CD) refers to a money market instrument, which is negotiable and
equivalent to a promissory note.
Select the code from below:
a)
b)
c)
d)

1 only
2 only
Both 1 and 2
Neither 1 nor 2

Q.16) Solution (b)


Certificate of Deposit (CD) refers to a money market instrument, which is negotiable and
equivalent to a promissory note. All scheduled commercial banks excluding Regional Rural
Banks (RRBs) and Local Area Banks (LABs) and Select All India Financial Institutions permitted by
RBI are eligible to issue certificates of deposit.
Statement (1) is totally absurd. Nothing like that exist.

Q.17) SEBI has established a Foreign Portfolio Investor Regulatory Framework. Which of the
following statements are correct about FPIs according to this framework?
1. All existing Foreign Institutional Investors (FIIs) and QFIs are to be merged into one
category called FPI.
2. An applicant desirous of FPI registration should not be a resident in India or a NonResident Indian.
3. FPI registration is to be undertaken and granted by Designated Depository Participants
(DDPs) on behalf of SEBI.
4. The registration of FPI is temporary and has to be renewed yearly.

Which of the above statements are correct?


a) 1,2 and 3
b) 2,3 and 4
c) 1,3 and 4
www.iasbaba.com

Page 14

IASbaba 60 Day Plan- Prelims Test 2016 ECONOMICS [DAY 34]


d) All of the above

Q.17) Solution (a)


An FPI has been defined to mean a person who satisfies the prescribed eligibility criteria and
has been registered under the FPI Regulations. All existing Foreign Institutional Investors (FIIs)
and QFIs are to be merged into one category called FPI.
Registration of a FPI
FPI registration is to be undertaken and granted by Designated Depository Participants (DDPs)
on behalf of SEBI.
Registration is to be granted within 30 days of application, subject to requisite information
being provided.
Registration will be permanent unless suspended or cancelled.

Q.18) Consider the following statements regarding Treasury bills:


1.
2.
3.
4.

Treasury bills are issued by the RBI on behalf of the government of India.
Treasury bills are issued through auctions and for a period of 91 days only.
State governments are also allowed to issue Treasury bills.
Treasury bills are available for a minimum amount of 1 lakh and then in multiples of
25000.

Which of the above statements are incorrect?


a)
b)
c)
d)

1 only
2,3 and 4
1,2 and 3
None of the above

Q.18) Solution (b)


Treasury Bill Market (T - Bills):This market deals in Treasury Bills of short term duration issued by RBI on behalf of
Government of India. At present three types of treasury bills are issued through auctions,
namely 91 day, 182 day and364day treasury bills. State government does not issue any treasury
www.iasbaba.com

Page 15

IASbaba 60 Day Plan- Prelims Test 2016 ECONOMICS [DAY 34]


bills. Interest is determined by market forces. Treasury bills are available for a minimum
amount of Rs. 25,000 and in multiples of Rs. 25,000. Periodic auctions are held for their Issue.
T-bills are highly liquid, readily available; there is absence of risk of default. In India T-bills have
narrow market and are undeveloped. Commercial Banks, Primary Dealers, Mutual Funds,
Corporates, Financial Institutions, Provident or Pension Funds and Insurance Companies can
participate in T-bills market.
Q.19) Most of the trading in the Indian stock market takes place on its two stock exchanges:
the Bombay Stock Exchange (BSE) and the National Stock Exchange (NSE). The BSE has been
in existence since 1875. The NSE, on the other hand, was founded in 1992 and started trading
in 1994. Which of the following statements are correct about the stock exchanges of India?
1. Both exchanges follow the same trading mechanism, trading hours and settlement
process.
2. The number of firms listed in NSE is almost three times than they are listed in BSE.
3. NSE enjoys almost a complete monopoly in Derivative trading.
Select the code from following:
a)
b)
c)
d)

1 and 2
2 and 3
1 and 3
All of the above

Q.19) Solution (c)


The number of firms listed in NSE is almost one third than that of BSE.

Q.20) Which of the following are the characteristics of a Bull Market?


1. In a bull market, there is a weak demand and strong supply of securities.
2. Share prices will rise as investors compete to obtain available equity.
Select the code from the following:
a) 1 only
b) 2 only
c) Both 1 and 2
www.iasbaba.com

Page 16

IASbaba 60 Day Plan- Prelims Test 2016 ECONOMICS [DAY 34]


d) Neither 1 nor 2

Q.20) Solution (b)


In a bull market, we see strong demand and weak supply for securities. In other words, many
investors are wishing to buy securities while few are willing to sell. As a result, share prices will
rise as investors compete to obtain available equity. In a bear market, the opposite is true as
more people are looking to sell than buy. The demand is significantly lower than supply and, as
a result, share prices drop.

www.iasbaba.com

Page 17

IASbaba 60 Day Plan- Prelims Test 2016 SCIENCE AND TECHNOLOGY & CURRENT AFFAIRS [DAY 35]

Q.1) Consider the following w.r.t Semiconductors


1. Semiconducting materials are always crystalline solids
2. Semiconductors are always manmade and properties are tweaked by Doping
3. A semiconductor is a material whose electrical conductivity decreases as the
temperature increases
Select the incorrect code
a)
b)
c)
d)

1 and 2 only
2 and 3 only
1 and 3 Only
All

Q.1) Solution (d)


Hint for this question- http://www.thehindu.com/todays-paper/tp-business/globalsemiconductor-capital-spending-to-decline-2-in-2016/article8578400.ece

Semiconductors are crystalline or amorphous solids with distinct electrical characteristics.


Semiconductors are substances that only conduct electricity under certain conditions and
include Silicon, Germanium, and sometimes tin. They are semiconductors that occur
naturally and do not require any sort of chemical doping and can often be recognized by
characteristic crystal lattice structures that they form.
A semiconductor is a material whose electrical conductivity increases with increasing
temperature. This broad definition distinguishes semiconductors from metals, whose
electrical conductivity decreases as the temperature increases. In general, there are two
basic classifications of semiconductors: intrinsic semiconductors and extrinsic
semiconductors. Unlike extrinsic semiconductors, intrinsic semiconductors are naturally
occurring elements within nature. The defining characteristic of these intrinsic
semiconductor elements is their four valence electrons each occupying a different orbital.

Q.2) TERAHERTZ imaging has recently attracted attention for its ability to see through
everyday objects. It exists between
a) Microwave and Radio waves wavelengths
b) Radio waves and Infrared wavelengths
c) Microwaves and Infrared wavelengths
www.iasbaba.com

Page 1

IASbaba 60 Day Plan- Prelims Test 2016 SCIENCE AND TECHNOLOGY & CURRENT AFFAIRS [DAY 35]

d) Infrared and Visible wavelengths

Q.2) Solution (c)


TERAHERTZ imaging, which exists between the microwave and infrared wavelengths, has
recently attracted attention for its ability to see through everyday objects. Among other
things, it has been used to detect defects in space shuttle panels and uncover the material
composition and substructure of paintings and murals. Current terahertz devices remain
inherently slow, invasive, and better suited to non-biological materials. Besides, they are
extremely expensive to manufacture.
Hinthttp://www.frontline.in/science-and-technology/advances-in-terahertzimaging/article8745879.ece

Q.3) There is a decline in the population of pollinators as per recent research and findings.
What could be the possible reasons for this?
1.
2.
3.
4.
5.

Use of pathogens
Use of street lights
Air pollution
Misuse of pesticides
Change in seasonal behaviour due to global warming

Select the appropriate code


a)
b)
c)
d)

1, 2, 4 and 5
1, 3, 4 and 5
1, 3 and 4
1, 2, 3, 4 and 5

Q.3) Solution (d)


Hinthttp://www.frontline.in/science-and-technology/moths-streetlighting/article8745865.ece
https://en.wikipedia.org/wiki/Pollinator_decline#Consequences

Q.4) Project Baseline is associated with which of the following?


www.iasbaba.com

Page 2

IASbaba 60 Day Plan- Prelims Test 2016 SCIENCE AND TECHNOLOGY & CURRENT AFFAIRS [DAY 35]

a)
b)
c)
d)

Seeds
Invasive Species
Drugs
Microchips

Q.4) Solution (a)


Hintcapsule/article8466314.ece

http://www.frontline.in/science-and-technology/seed-time-

Q.5) 3D Printing has gained lot of attention in recent times. Consider the following
statements regarding 3D Printing
1. 3D printing is a process of making three dimensional solid objects from anything
2. It is also known as Additive Manufacturing (AM)
3. 3D printing can create in a wide range of materials that include thermoplastics,
thermoplastic composites, pure metals, metal alloys, ceramics and various forms of
food.
Select the incorrect code
a)
b)
c)
d)

Only 1
Only 2
2 and 3
1 and 2

Q.5) Solution (a)


3D printing is a process of making three dimensional solid objects from a digital file not
ANYTHING
Hint
http://www.thehindu.com/news/international/3d-printing-could-help-fix-damagedcartilage-in-knees/article8365269.ece
http://www.thehindu.com/news/cities/Visakhapatnam/3d-printing-to-make-healthcareaffordable/article8231694.ece

www.iasbaba.com

Page 3

IASbaba 60 Day Plan- Prelims Test 2016 SCIENCE AND TECHNOLOGY & CURRENT AFFAIRS [DAY 35]

Q.6) Pick the odd one out in regard to its stem type:
a)
b)
c)
d)

Bamboo
Sugar cane
Corn
Banana

Q.6) Solution (d)


Explanation:
Different types of stems
(a) Monocots: Secure stems of several
plants such as bamboo, sugar cane and
corn. If we cut each of the stems
crosswise with a very sharp knife or
razor blade, we can observe the
similarities in the cut cross sections.
Especially we can notice that the tubes
or fibro-vascular bundles are scattered
throughout the pith on the inside of the stem.
(b) Dicers/Dicots: Secure the stems of several plants or small trees such as willow, tomato,
mango, banana etc. Cut across each of these stems with a sharp knife or razor blade, we can
observe that just under the outside layer of the stem there is a bright green layer. This is the
cambium layer. Also we can observe that the tubes or fibro- vascular bundles are arranged
in a ring about the central, or woody portion of the stem.

Q.7) Match List I with List II and select the correct answer using the code given below the
Lists:

A.
B.
C.
D.
E.

List I
Magnetic Flux
Magnetic Induction
Luminous intensity
Electric Charge
Inductance

List II
1. Tesla
2. Candela
3. Coulomb
4. Weber
5. Henry

A-B-C-D-E
a) 4-3-5-2-1
www.iasbaba.com

Page 4

IASbaba 60 Day Plan- Prelims Test 2016 SCIENCE AND TECHNOLOGY & CURRENT AFFAIRS [DAY 35]

b) 4-1-2-3-5
c) 1-4-2-3-5
d) 1-4-2-5-3

Q.7) Solution (b)


Explanation:
A.
B.
C.
D.
E.

Magnetic Flux
Magnetic Induction
Luminous intensity
Electric Charge
Inductance

1. Weber
2. Tesla
3. Candela
4. Coulomb
5. Henry

Q.8) Connecting link between ape and man is


a)
b)
c)
d)

Cro Magnon Man


Australopithecus
Neanderthal Man
Lemur

Q.8) Solution (b)


Explanation:
Australopithecus shows characters of both apes and man.

Q.9) Haemophilic man marries a normal woman. Their offsprings will be


a)
b)
c)
d)

all girls haemophilic


all normal
all haemophilic
all boys haemophilic

Q.9) Solution (b)


Explanation:
www.iasbaba.com

Page 5

IASbaba 60 Day Plan- Prelims Test 2016 SCIENCE AND TECHNOLOGY & CURRENT AFFAIRS [DAY 35]

Daughters and sons both will be phenotypically normal.

Q.10) Consider the following in regard to Dolphins and Porpoise:


1) Both are cetaceans and closely related to whales, but both belong to different
families.
2) Both are highly intelligent, they have large complex brains and are self-aware like
humans
3) Dolphins tend to have prominent, elongated beaks and cone-shaped teeth, while
porpoises have smaller mouths and spade-shaped teeth.
4) Dolphins have hooked or curved dorsal fin (the one in the middle of the animal's
back), while the porpoises have triangular dorsal fin.
Which of the statements given above is/are correct?
a)
b)
c)
d)

3 and 4 only
1, 2 and 3 only
1 and 2 only
All of the above

Q.10) Solution (d)


Explanation:
Factual question All given statements are correct and self-explanatory.

www.iasbaba.com

Page 6

IASbaba 60 Day Plan- Prelims Test 2016 SCIENCE AND TECHNOLOGY & CURRENT AFFAIRS [DAY 35]

Q.11) Consider the following statements regarding project Loon:


1. It is an ambitious project of Facebook to provide network to rural and remote
areas.
2. Helium balloons are used to carry electronic payloads and are placed in atmosphere
over a place where network towers are not available.
3. The balloons will be placed in Stratosphere.
Which of the above statements are correct?
a)
b)
c)
d)

1 only
2 and 3
1 and 2 only
All of the above

Q.11) Solution (b)

www.iasbaba.com

Page 7

IASbaba 60 Day Plan- Prelims Test 2016 SCIENCE AND TECHNOLOGY & CURRENT AFFAIRS [DAY 35]

Project Loon the project of Google to provide network connection in rural and remote
areas where network connection is not available.
Facebook is going to start a similar project, where it will be using drones rather than helium
balloons.

Q.12) The use of nanobots in healthcare, a fictional theory, is a reality now. Consider the
following statements:
1. Nanobots are tiny robots that can be directly injected in the blood stream and act
like white blood cells and destroy bacteria and other pathogen.
2. Nanobots will have their own sensors and propulsion system.
3. They can be used to deliver targeted medicine with minimum side effect.
Select the code from the following:
a)
b)
c)
d)

1 only
1 and 2
2 and 3
All of the above

Q.12) Solution (d)


Although nanobots are far from being utilized today, but the future is coming where these
tiny robots can function like our own white blood cells and destroy bacteria and other
pathogens.

www.iasbaba.com

Page 8

IASbaba 60 Day Plan- Prelims Test 2016 SCIENCE AND TECHNOLOGY & CURRENT AFFAIRS [DAY 35]

These miniature robots would function like their full-size equivalents with their own
sensors, and propulsion systems and could perform small tasks like delivering chemotherapy
1000 times more powerful than using drugs and would not cause as many side-effects to
patients like the current treatments do.
Other specific types of nanobots that are being developed are Microbivore, Respirocyte,
Clottocyte, and Cellular repair nanobots that can destroy bacteria, carry oxygen, create
blood clots for wounds, and repair cells.

Q.13) Consider the following statements regarding Impacting Research Innovation and
Technology (IMPRINT) Project?
1. It is an initiative of Ministry of Human Resource Development and Pan IIT + IISc.
2. The project aims at finding engineering and scientific solutions to major social and
developmental needs like healthcare, energy, ICT, sustainable habitat etc.
Which of the above statements are incorrect?
www.iasbaba.com

Page 9

IASbaba 60 Day Plan- Prelims Test 2016 SCIENCE AND TECHNOLOGY & CURRENT AFFAIRS [DAY 35]

a)
b)
c)
d)

1 only
2 only
Both 1 and 2
Neither 1 nor 2

Q.13) Solution (d)


http://imprint-india.org/

Q.14) India will be getting its first Bullet Train from a Spanish Company called Taglo. The
company is globally known for faster and lighter trains. Consider the following
statements:
1. The train will be tested on Delhi-Mumbai Railway line.
2. It can be run at a speed of around 300 km/hr without any significant upgrade of
railway tracks.
3. Currently the fastest train of India is Gatiman Express which runs between Delhi and
Agra.
Which of the above statements are correct?
a)
b)
c)
d)

1 and 2
2 and 3
1 and 3
All of the above

Q.14) Solution (c)


The company has offered lighter and safer trains, which can be run at speeds of about 160
km an hour and without any major upgrade of existing tracks. The fastest train on Indian
Railways system, Gatimaan Express, runs at 160 km an hour between Delhi and Agra.

Q.15) C. V. Raman, was the first Indian Scientist to receive a Nobel Price. He published his
theory on the Raman Effect in 1928. Raman Effect is associated with which of the
following?
a)
b)
c)
d)

Scattering of light
Total internal reflection
Atomic structure
Semi-conductors

www.iasbaba.com

Page 10

IASbaba 60 Day Plan- Prelims Test 2016 SCIENCE AND TECHNOLOGY & CURRENT AFFAIRS [DAY 35]

Q.15) Solution (a)


Raman Effect, change in the wavelength of light that occurs when a light beam is deflected
by molecules.
When a beam of light traverses a dust-free, transparent sample of a chemical compound, a
small fraction of the light emerges in directions other than that of the incident (incoming)
beam. Most of this scattered light is of unchanged wavelength. A small part, however, has
wavelengths different from that of the incident light; its presence is a result of the Raman
Effect.

Q.16) Consider the following statements regarding microwave ovens:


1. Instead of generating heat that warms the food from outside, the microwaves
penetrate food and create the heat within.
2. Microwaves are electromagnetic waves that are created by a component called
Megatron in the oven. Megatron has vacuum tubes which creates the microwave.
3. Microwaves are known to destroy the nutrients in the food.
Which of the above statements are correct?
a)
b)
c)
d)

1 and 2
2 and 3
1 and 3
All of the above

Q.16) Solution (a)


The key component of a microwave oven is the magnetron. Although the name conjures up
hardware from a questionable science-fiction movie, the sophisticated vacuum tube
generates microwaves powerful enough for military radars (for which it was originally
developed). Instead of a flame or electric coil generating heat that warms food from the
outside, the microwaves penetrate food and create heat from within.

Q.17) Early Career Research Award scheme aims to provide quick research support to the
young researchers who are in their early career for pursuing exciting and innovative
research in frontier areas of science and engineering. Consider the following statements
regarding this scheme:
www.iasbaba.com

Page 11

IASbaba 60 Day Plan- Prelims Test 2016 SCIENCE AND TECHNOLOGY & CURRENT AFFAIRS [DAY 35]

1. The research grant is 50 Lakhs, including overhead charges for travelling, manpower,
research equipment etc.
2. The award is given in three installments for three years.
3. The award is only for Indian citizens.
Which of the above statements regarding the scheme are incorrect?
a)
b)
c)
d)

1 and 2
3 only
1 and 3
None of the above

Q.17) Solution (a)

The Early Career Research for the purpose of the program refers to the first
assignment of the applicant in a regular capacity in a recognized academic institution
or national laboratory or any other recognized R & D institution in India.
The Early Career Research Award is a one-time award and carries a research grant up
to Rs. 50 Lakhs (excluding overheads) for a period of three years.
The research grant covers equipment, manpower, consumables, manpower, travel
and contingency apart from overheads.

Eligibility :

The applicant should be an Indian citizen.


The applicant should hold Ph.D. degree in Science or Engineering or M.D or M.S
degree in any area of medicine.
The scheme encourages young researchers at the start of their independent
research careers. The applicant, therefore, must hold a regular academic/research
position in a recognized academic institution/ or national laboratories or any other
recognized R&D institutions and must apply not later than the first two years of their
regular service.

Q.18) Japanese Encephalitis is a notifiable disease in India. Which of the following


statements are correct about JE?
1. Japanese Encephalitis is a caused by flavivirus transmitted through infected Culex
mosquitoes.
2. JE virus is neurotorpic and arbovirus and primarily affects central nervous system.
3. India has launched JENVAC, its first indigenous vaccine to protect children from JEV.
www.iasbaba.com

Page 12

IASbaba 60 Day Plan- Prelims Test 2016 SCIENCE AND TECHNOLOGY & CURRENT AFFAIRS [DAY 35]

4. JANVAC has been induced in National Immunisation Program.


Which of the above statements are correct?
a)
b)
c)
d)

1 and 2
1,2 and 3
2,3 and 4
All of the above

Q.18) Solution (d)


Factual question. Self Expalnatory.
Note: Read about National Immunisation Program and diseases covered in it. A direct
question can be asked.

Q.19) Which of the following forms of Carbon, is a good conductor of electricity?


1.
2.
3.
4.

Diamond
Graphite
Graphene
Charcoal

Select the code from the following:


a)
b)
c)
d)

2 and 3
1 and 2
3 and 4
2 only

Q.19) Solution (a)


Graphene is a 2D nanostructure of carbon which is capable of conducting electricity and
offers very less resistance. Research is being going on to use graphene in solar panels and
touch screens to increase efficiency.

Q.20) Rustom II developed by DRDO is an important aeronautical achievement of India.


Which of the following statements regarding Rustom II are correct?
1. It is remotely piloted UAV in the category of Medium Altitude, Long Endurance
(MALE) category.
www.iasbaba.com

Page 13

IASbaba 60 Day Plan- Prelims Test 2016 SCIENCE AND TECHNOLOGY & CURRENT AFFAIRS [DAY 35]

2. Rustom II is a fully featured, Combat capable drone built on the lines of US predator
drones.
Select the code from the following:
a)
b)
c)
d)

1 only
2 only
Both 1 and 2
Neither 1 nor 2

Q.20) Solution (c)


Both the options are correct regarding Rustom II.
http://www.militaryfactory.com/aircraft/detail.asp?aircraft_id=985
You can read about Rustom II from any other source as well.

Q.21) Consider the following statements with respect to Colombia


1. Quito is the capital of Colombia
2. Quito is one of the closest places on earth where equator passes through
Select the correct answer
a)
b)
c)
d)

1 only
2 only
Both
None

Q.21) Solution (b)

Bogota is the capital of Colombia


Click here
Click here

Q.22) 38th parallel is found between which of the following two countries
a) North Korea and South Korea
b) South Korea and China
www.iasbaba.com

Page 14

IASbaba 60 Day Plan- Prelims Test 2016 SCIENCE AND TECHNOLOGY & CURRENT AFFAIRS [DAY 35]

c) North Korea and China


d) China and Japan

Q.22) Solution (a)

38th parallel is between North Korea and South Korea.


Click here

Q.23) Consider the following statements with respect to Shanghai Cooperation


Organisation
1. It consists of all members of Central Asia including China and Russia
2. It is headquartered at Shanghai
Select the correct answer
a)
b)
c)
d)

1 only
2 only
Both
None

Q.23) Solution (d)

Turkmenistan is not the member of central Asia


The headquarters of SCO is located in Beijing
Click here

Q.24) Consider the following statements with respect to Net Neutrality


1. It is the principle that ISPs and governments should treat all data on the Internet
equally not discriminating or charging differently by user, content site platform
application, type of attached equipment and modes of communication
2. It creates a level playing field for all web services and websites, having access to the
same connection speed
Select the incorrect option
a) 1 only
b) 2 only
c) Both
www.iasbaba.com

Page 15

IASbaba 60 Day Plan- Prelims Test 2016 SCIENCE AND TECHNOLOGY & CURRENT AFFAIRS [DAY 35]

d) None

Q.24) Solution (d)

Click here

Q.25) Consider the following statements about Rajiv Rinn Yojana


1. The RRY is a central sector scheme which provides an interest subsidy on housing
loans to EWS and LIG households
2. It replaces the Interest Subsidy Scheme for Housing the Urban Poor (ISHUP)
Select the correct option
a)
b)
c)
d)

1 only
2 only
Both
None

Q.25) Solution (c)

Rajiv Rinn Yojana (RRY) is an instrument to address the housing needs of the
EWS/LIG segments in urban areas, through enhanced credit flow. It is also
formulated to channelize institutional credit to the poorer segments of the society
and increasing home ownership in the country along with addressing housing
shortage. RRY has been formulated by modifying the Interest Subsidy Scheme for
Housing the Urban Poor (ISHUP) piloted in the 11th Plan period with enhanced scope
and coverage. RRY is a Central Sector Scheme applicable in all the urban areas of the
Country.
Click here

www.iasbaba.com

Page 16

Вам также может понравиться